Download Onco-Nephrology Curriculum - American Society of Nephrology

Document related concepts
no text concepts found
Transcript
Chapter 1: Onco-Nephrology: Growth of the
Kidney–Cancer Connection
Mark A. Perazella, MD,* and Mitchell H. Rosner, MD†
*Section of Nephrology, Department of Medicine, Yale University School of Medicine, New Haven, Connecticut; and †Division
of Nephrology, Department of Medicine, University of Virginia Health System, Charlottesville, Virginia
The question has been asked of many of us interested in the kidney–cancer connection; Why
onco-nephrology? Nephrologists have traditionally treated cancer patients with various forms of
kidney disease. However, although typical AKI and
electrolyte/acid–base disturbances can be handled
by the practicing clinical nephrologist, increasingly
it has become clear that many of the issues are more
complex and highly specialized. For example,
many nephrologists were not trained in the era of
bone marrow/hematopoietic stem cell transplant,
which has a number of unusual and complicated
forms of kidney injury. In addition, the number of
anticancer drugs with various types of nephrotoxicity has increased dramatically, and their entry
into clinical practice continues at a fast pace. These
and other issues have led to a burgeoning interest
in a more specific focus of nephrologists on the
patient with cancer.
Recognizing this changing landscape, nephrologists at various large cancer centers began a
discussion about the need to address this rapidly
growing area of nephrology. In 2009/2010, Abdulla
Salahudeen recruited leaders from these cancer
centers and asked the American Society of Nephrology (ASN) to sponsor a “forum” where this
emerging area of nephrology could be discussed. In
2012, the ASN had its first official meeting of what
was later named the “Onco-Nephrology Forum.”
Dr. Salahudeen chaired the group, pushed forward
the kidney–cancer agenda with the forum members, and put the fledgling area of onco-nephrology
on the nephrology map. The description of the
Onco-Nephrology Forum on the ASN website is
noted below:
“Why onco-nephrology? While all nephrologists address
nephrology problems in cancer patients, many of these
problems are increasingly complex. To provide the best
nephrology care for cancer patients, we must understand
rapidly changing protocols and therapies.
American Society of Nephrology
Emerging kidney toxicities associated with drugs targeting VEGF and TKIs and other signaling pathways,
tumor lysis syndrome, cytotoxic chemotherapy-induced
kidney toxicities, kidney problems in myeloma, tumor or
treatment-related microangiopathies and glomerulonephritis, stem cell transplant–associated acute and
chronic kidney injuries, obstructive uropathies, severe
fluid and electrolytes abnormalities, and dosing and
timing of chemotherapy in CKD and ESRD patients:
these and other complex problems, and their increasing
frequency and severity, provide a unique and unprecedented opportunity for nephrologists to improve
treatment for cancer patients worldwide.
Onco-nephrologists help cancer care teams prevent
kidney problems or resolve them as they arise and improve patient outcomes. Research in cancer nephrology is
already improving kidney care in cancer patients. A
more focused approach to cancer nephrology may also
help address challenges like renal cell carcinoma in
ESRD.
The American Society of Nephrology believes onconephrology represents an emerging frontier in the fight
against kidney disease.”
The ASN Onco-Nephrology Forum has worked
hard to spread the word and “metastasize” into
multiple journal, meetings, and conferences. One
very helpful contribution to this endeavor included
the appearance of publications on the onconephrology topic in several high-level journals visible
to many nephrologists. Entire issues dedicated to
Correspondence: Mark A. Perazella, Section of Nephrology,
Department of Medicine, Yale University School of Medicine, BB
114, 330 Cedar Street, New Haven, Connecticut.
Copyright © 2016 by the American Society of Nephrology
Onco-Nephrology Curriculum
1
Table 1. Onco-Nephrology Curriculum committee members
Figure 1. Timeline of the birth and growth of onco-nephrology.
Formation of the ASN Onco-Nephrology Forum, numerous conference publications, and dedicated journal publications characterize and highlight the process. ASN, American Society of
Nephrology; ONF, Onco-Nephrology Forum; NKF, National
Kidney Foundation; JCO, Journal of Clinical Oncology; ON, OncoNephrology; ACKD, Advances in Chronic Kidney Disease; CJASN,
Clinical Journal of the American Society of Nephrology; KI, Kidney
International; JASN, Journal of the American Society of Nephrology;
Sem Nephrol, Seminar in Nephrology.
onco-nephrology appeared in Seminars in Nephrology and Advances in Chronic Kidney Disease, while a series of articles on this
subject was published in the Clinical Journal of the American
Society of Nephrology Moving Points in Nephrology feature.
In 2011, the ASN had its first Kidney Week Early Program dedicated to onco-nephrology. This Early Program continues on an
every other year schedule. Many of the Kidney Week Clinical
Nephrology Conferences included sessions covering various
onco-nephrology topics. The National Kidney Foundation
Annual Spring Meeting similarly dedicated a session to onconephrology (Figure 1). In addition, editorials describing the importance of onco-nephrology, some suggesting the need for a
“new subspecialty” appeared in the Journal of the American Society of Nephrology, Kidney International, and the Journal of Clinical Oncology, authored by members of the ASN Onco-Nephrology
Forum.
With these important accomplishments, the OncoNephrology Forum with Mark Perazella as the new Onco-
2
American Society of Nephrology
Mark A. Perazella (ONF Chair, Lead Editor)
Mitchell H. Rosner (Lead Editor)
Kevin W. Finkel (Section Editor)
Ilya Glezerman (Section Editor)
Susie L. Hu (Section Editor)
Kenar D. Jhaveri (Section Editor)
Amit Lahoti (Section Editor)
Anushree C. Shirali (Section Editor)
Ala Abudayyeh
Joseph R. Angelo
Joseph V. Bonventre
Anthony Chang
Eric P. Cohen
Farhad R. Danesh
Mona D. Doshi
Amaka Edeani
Carlos Flombaum
Sangeeta R. Hingorani
Benjamin Humphreys
Divya Monga
Abdulla K. Salahudeen
Nephrology Forum Chair continued to forge ahead and felt the
time was ripe for the creation of an Onco-Nephrology
Curriculum. After creation of an outline of topics and
discussion by the advisory group (Table 1), the core curriculum was submitted to the ASN Education Committee for review. The curriculum was subsequently approved and the
Onco-Nephrology Forum group, under the direction of the
curriculum committee co-chairs (Perazella and Rosner), put
together the ultimate plan for creation of the curriculum document. The lead editors, section editors, and chapter authors
(Tables 1 and 2) were identified and the writing began. The
chapters are truly an example of outstanding contributions by
experts in the subfield of onco-nephrology. All of the authors
are to be congratulated on their fine work and keeping to the
originally planned timeline for completion. The product of
this work will appear on the ASN’s website and will be available to the ASN membership, the nephrology training programs, and all other interested health care providers. We are
confident that this curriculum will strengthen the teaching of
onco-nephrology and further expand all practitioners’ knowledge of the subject. We hope the readers enjoy the document.
Onco-Nephrology Curriculum
Table 2. Onco-Nephrology Curriculum chapters and authors
1) Onco-Nephrology: Growth of the Kidney-Cancer Connection
2) Why do we need an Onco-Nephrology Curriculum?
3) AKI associated with Malignancies
4) Tumor Lysis Syndrome
5) Electrolyte and Acid-Base Disorders and Cancer
6) Glomerular Disease and Cancer
7) Hematologic Diseases and Kidney Disease
8) Clinical tests for Monoclonal Proteins
9) Hematopoietic Stem Cell Transplant-related Kidney Disease
10) Radiation-associated Kidney Injury
11) Chemotherapy and Kidney injury
12) Pharmacokinetics of Chemotherapeutic Agents in Kidney Disease
13) CKD as a Complication of Cancer
14) Hereditary Renal Cancer Syndromes
15) Work-up and Management of Small Renal Masses
16) Cancer in Solid Organ Transplantation
17) Cancer Screening in ESRD
18) Ethics of RRT, Initiation and Withdrawal, in Cancer Patients
19) Palliative Care in Patients with Kidney Disease and Cancer
American Society of Nephrology
Mark Perazella, Mitchell Rosner
Mark Perazella, Mitchell Rosner
Amit Lahoti, Benjamin Humphreys
Amaka Edeani, Anushree Shirali
Anushree Shirali
Divya Monga, Kenar Jhaveri
Ala Abudayyeh, Kevin Finkel
Nelson Leung
Sangeeta Hingorani, Joseph Angelo
Amaka Edeani, Eric Cohen
Ilya Glezerman, Edgar Jaimes
Sheron Latcha
Maurizio Gallieni, Camillo Porta, and Laura Cosmai
Katherine Nathanson
Susie Hu, Anthony Chang
Mona Doshi
Jean Holley
Michael Germain
Alvin Moss
Onco-Nephrology Curriculum
3
AUTHOR QUERIES
AUTHOR PLEASE ANSWER ALL QUERIES
There are no queries in this article.
Chapter 2: Why Do We Need an Onco-Nephrology
Curriculum?
Mark A. Perazella, MD,* and Mitchell H. Rosner, MD†
*Section of Nephrology, Department of Medicine, Yale University School of Medicine, New Haven, Connecticut; and
†
Division of Nephrology, Department of Medicine, University of Virginia Health System, Charlottesville, Virginia
As health care providers, we are acutely aware of the
National Vital Statistics Report describing the
significant toll cancer, as the second leading cause
of death, has on our patients (1). Importantly, cancer incidence rates are highest in the elderly (2). At
the same time, the US Renal Data System (USRDS)
notes that AKI rates are increasing in the elderly, with
rates 10-fold higher than the nonelderly population
(3). Importantly, both AKI and CKD are highly
prevalent in cancer patients, in particular renal cell
cancer, liver cancer, multiple myeloma, leukemias,
and lymphomas (4,5). The Belgian Renal Insufficiency and Anticancer Medication (BIRMA) study
noted the frequent occurrence of kidney disease in
five major cancers (Figure 1) (6). Most concerning is
the increased mortality noted in patients with AKI/
CKD compared with those without kidney disease.
For instance, the development of AKI can be associated with cessation of effective chemotherapeutic
regimens, or the presence of preexisting CKD may
limit the use of otherwise active regimens that may
be curative. This combination of cancer, kidney disease, and mortality has led to the recognition that
nephrology and oncology are intricately linked and
require our full attention as a subspecialty (Figure
2). Hence, “onco-nephrology” was born in a few
large centers but has steadily grown to include
many medical centers, hospitals, and clinics.
What exactly is onco-nephrology? It is a rapidly
growing area of nephrology where kidney disease in
cancer patients has become an important source of
consultations, with the trend occurring over the last
10–15 years. Oncology patients now make up a significant number of the patients that nephrologists see
for kidney-related problems in the outpatient clinic,
on the inpatient floors, and in the medical intensive
care unit (ICU). There is an increase in the number of
patients with kidney disease, in part related to high
incidence rates for many malignancies, as well as improvement in the cancer death rates due to more
American Society of Nephrology
effective chemotherapeutic agents, including biologics, and stem cell therapies. However, this has
led to an increase in the number of cancer survivors
that often develop acute and/or CKD due to their
malignancy and/or its associated treatment. The
best example of the bidirectionality of cancer and
kidney disease is seen between renal cancer and
CKD (Figure 3 ).
Cancer can directly injure the kidneys through
tumor infiltration or production of nephrotoxic
(paraneoplastic) substances. Any one of the growing
numbers of therapeutic agents that extend patient
lives can cause various types of acute or CKD, along
with serious electrolyte and acid–base abnormalities.
In addition, patients may develop multiorgan illness
requiring ICU-level care and RRT. Certain malignancies are more likely to cause this severe form of
multiorgan dysfunction and may be associated with
higher mortality rates. When this type of critical
illness occurs in the setting of advanced malignancy,
it raises questions about the appropriateness of aggressive care in “futile situations” and the role of
palliation. Thus, care for oncology patients has become more specialized and complicated, requiring
collaboration between nephrologists, oncologists,
intensivists, and palliative care specialists.
The remarkable advances in cancer management
present both new opportunities and complex challenges for the oncology and nephrology communities. It is essential for nephrologists to be informed
and actively involved in certain facets of cancer care.
A better understanding of the rapidly evolving field
of cancer biology and its therapy is required for
nephrologists to become valuable members of the
Correspondence: Mark A. Perazella, Section of Nephrology,
Department of Medicine, Yale University School of Medicine, BB
114, 330 Cedar Street, New Haven, Connecticut.
Copyright © 2016 by the American Society of Nephrology
Onco-Nephrology Curriculum
1
Figure 1. Kidney injury associated with five different cancers in the BIRMA study. The percentage of patients with kidney injury as
defined by SCR, GFR ,90, or GFR ,60 is noted both for the individual cancers and all cancers lumped together. BIRMA, Belgian Renal
Insufficiency and Anticancer Medication study; SCR, serum creatinine; aMDRD, abbreviated MDRD. Adapted with permission from
reference 6.
cancer care team and to provide the best nephrology care
possible. The goal of this American Society of Nephrology
(ASN) sponsored Onco-Nephrology core curriculum is to
provide the ASN membership including veteran nephrologists,
newly minted nephro-clinicians, and fellowship trainees with
the building blocks on which further information can be added
as technology advances. This educational venue will be available outside the ASN membership as well.
Nephrologists must be well prepared to care for patients
with cancer and its associated renal complications. The renal
manifestations of cancer have many unique features, and these
conditions often require specialized approaches to manage
fluid, electrolyte, and acid–base disturbances, as well as acute
and chronic kidney injury. Furthermore, the ever-evolving
field of cancer therapy demands a comprehensive team approach with the nephrologist as one of the critically important
care providers. As such, it is essential for nephrologists to
develop expertise in the practice of onco-nephrology. We
hope this curriculum provides the initial framework to
achieve this goal.
TAKE HOME POINTS
c Kidney disease is a frequent and increasing complication of cancer.
c There is a bidirectional relationship between cancer and kidney disease.
Figure 2. The relationship between cancer and AKI and CKD. Cancer, AKI, and CKD are linked by various exposures and
pathways.
2
Onco-Nephrology Curriculum
American Society of Nephrology
Figure 3. The bidirectionality between renal cancer and CKD. Common exposures that can cause both renal cell cancer and CKD
are noted in the middle bidirectional arrow.
c Onco-nephrology is a growing area of nephrology that requires clinicians
to have a better understanding of the renal complications of cancer including electrolyte/acid–base disturbances, AKI, and CKD.
c The Onco-Nephrology Curriculum is an educational tool created
by ASN Onco-Nephrology Forum members and other expert nephrologists.
REFERENCES
1. Hoybert DL, Xu J. Deaths: Preliminary data for 2011. Natl Vital Stat Rep
61: 2012
2. National Cancer Institute. Age-adjusted SEER incidence rates, 2007–
2011 (Table 2.7). SEER cancer statistics review (CSR) 1975–2011.
Surveillance, epidemiology, and end results program. Available at:
http://seer.cancer.gov/csr/1975_2011/browse_csr.php?sectionSEL52&
pageSEL5sect_02_table.07.html. Accessed March 1, 2015
American Society of Nephrology
3. USRDS. Percent of Medicare patients aged 661 (a) with at least one AKI
hospitalization, and (b) with an AKI hospitalization that had dialysis by
year, 2003–2012 (Figure 5.1). Chapter 5: Acute kidney injury. Available
at: http://www.usrds.org/2014/view/v1_05.aspx. Accessed March 1,
2015
4. Christiansen CF, Johansen MB, Langeberg WJ, Fryzek JP, Sørensen HT.
Incidence of acute kidney injury in cancer patients: A Danish populationbased cohort study. Eur J Intern Med 22: 399–406, 2011
5. Schmid M, Abd-El-Barr AE, Gandaglia G, Sood A, Olugbade K Jr,
Ruhotina N, Sammon JD, Varda B, Chang SL, Kibel AS, Chun FK, Menon
M, Fisch M, Trinh QD. Predictors of 30-day acute kidney injury following
radical and partial nephrectomy for renal cell carcinoma. Urol Cancer 32:
1285–1291, 2014
6. Janus N, Launay-Vacher V, Byloos E, Machiels JP, Duck L, Kerger J,
Wynendaele W, Canon JL, Lybaert W, Nortier J, Deray G, Wildiers H.
Cancer and renal insufficiency results of the BIRMA study. Br J Cancer
103: 1815–1821, 2010
Onco-Nephrology Curriculum
3
REVIEW QUESTIONS
1. Which of the following malignancies has the highest 1-year
risk for AKI?
a.
b.
c.
d.
e.
Multiple myeloma
Lymphoma
Renal cell cancer
Liver cancer
Leukemia
Answer: c is correct. Although all of these cancers are
associated with increased AKI risk, renal cell cancer was
found to have the highest 1-year risk in a cohort study examining the incidence of AKI in cancer patients (4).
4
Onco-Nephrology Curriculum
2. In a patient with a recent diagnosis of cancer, which of the
following complications are increased in the setting of the
cancer diagnosis?
a.
b.
c.
d.
AKI
CKD
Mortality
All of the above
Answer: d is correct. Cancer is associated with an increased
incidence of AKI, CKD, and overall mortality. These complications are the result of the tumor itself (infiltration or tumor
products), drug nephrotoxicity, comorbid diseases, or all of
the above.
American Society of Nephrology
Chapter 3: AKI Associated With Malignancies
Amit Lahoti, MD,* and Benjamin D. Humphreys, MD, PhD†
*Division of Internal Medicine, Section of Nephrology, The University of Texas MD Anderson Cancer Center, Houston,
Texas; and †Division of Nephrology, Washington University School of Medicine, St. Louis, Missouri
INTRODUCTION
Advances in treatment, risk stratification, and
supportive care have improved survival of patients
with cancer over the last two decades (1). AKI may
result from the cancer itself (e.g., infiltration or obstruction), the treatment of cancer (e.g., chemotherapy toxicity), or associated complications
(e.g., sepsis). Cancer, by itself, is not a contraindication for starting RRT, even in the setting of multiorgan failure (2–4). However, decision-making is
complex and requires a multidisciplinary approach
between the oncologist, intensivist, and nephrologist.
The development of AKI may lead to longer length of
hospital stay, decreased functional status and quality
of life, and exclusion from further cancer therapy.
AKI and RRTmay lead to unpredictable levels of chemotherapeutic agents and anti-infective drugs. AKI
may also increase inflammatory cytokines in the
lung, leading to increased vascular permeability (5)
and the need for mechanical ventilation (6). Therefore, early detection and prevention of AKI is crucial
in patients with cancer.
DEFINITION
More than 35 different definitions for AKI have been
used in the literature, which has made crosscomparisons between studies difficult. This led to
the development of the RIFLE classification, which
defined three stages of AKI (risk, injury, and failure)
and two stages of renal failure requiring dialysis (loss
and ESRD) (7). Stages for AKI are determined by
the percent rise in serum creatinine relative to baseline, decreased urine output, or the need for dialysis. It is unclear whether the criteria are well
balanced in respect to urine output and serum
creatinine, as most studies have not utilized the
urine output component. The RIFLE classification
has been validated in numerous patient populations and has highlighted the significant effect of
American Society of Nephrology
mild degrees of renal injury on mortality. Significant renal injury may occur without elevation in
serum creatinine, and an elevation of 0.3 mg/dL
has been associated with increased mortality in
hospitalized patients.
The Acute Kidney Injury Network (AKIN) proposed modifications to the RIFLE criteria with three
stages of AKI corresponding to the risk, injury, and
failure categories (8). Patients with an absolute rise in
serum creatinine of 0.3 mg/dL are included into the
least severe category (stage 1). The loss and ESRD
categories were eliminated, and all patients requiring
dialysis were classified into the most severe category
(stage 3). Last, a time constraint of 48 hours to reach
stage 1 was also included in the AKIN definition.
Whether the AKIN modifications to the RIFLE
criteria have led to improvements in classification
has yet to be determined (9). Recently, The Kidney
Disease Improving Global Outcomes (KDIGO)
work group combined elements of the RIFLE and
AKIN classifications to define AKI as 1) an increase
in serum creatinine (SCr) $0.3 mg/dL within 48
hours, 2) an increase in SCr to $1.5 times baseline
within the prior 7 days, or 3) a urine volume of
,0.5 mL/kg/h for 6 hours. Severity of AKI is staged
similar to the AKIN criteria. Several studies have
correlated AKI as defined by these criteria with increased mortality, length of stay, and hospital costs
in patients with cancer (10–13).
EPIDEMIOLOGY AND PROGNOSIS
AKI is common in hospitalized patients with cancer
and is associated with increased length of stay and
hospital costs. In a Danish population-based study of
1.2 million cancer patients, the incidence of AKI
defined by the RIFLE criteria was highest in patients
Correspondence: Amit Lahoti, UT MD Anderson, Cancer Center,
Unit 1468, PO Box 301402, Houston, Texas 77230.
Copyright © 2016 by the American Society of Nephrology
Onco-Nephrology Curriculum
1
with renal cell cancer (44%), multiple myeloma (33%), liver
cancer (32%), and leukemia (28%) (14). Compared with patients without cancer, critically ill patients with cancer have a
higher incidence of AKI requiring RRT. Depending on the definition of AKI and the underlying case mix, it has been reported
that 13%–42% of critically ill patients with cancer develop AKI
and 8%–60% require RRT (15). The incidence is highest
for those patients with hematologic malignancies,
multiple myeloma, and septic shock.
The 28-day mortality of patients with cancer who require RRT
is 66%–88% (16). In one study of critically ill patients with
cancer, the odds ratio for 30-day mortality was increased twofold in patients with AKI. However, approximately one-half of
the patients with AKI survived to day 30 after admission (17). In
one study of AKI in critically ill patients, there was complete
recovery of renal function in 82% and partial recovery in 12%,
and chronic dialysis was needed in only 6% of patients (18).
Overall severity of illness, age, and functional status may have
more of an impact on prognosis than underlying malignancy,
and the presence of cancer may not be an absolute exclusion
criterion for withholding RRT. However, the prognosis of critically ill recipients of stem cell transplants who develop AKI
remains poor, with mortality exceeding 80%. A team-based
approach between the oncologist, critical care physician, and
nephrologist is necessary to identify patients who are most suitable for initiation of RRT.
kidney injury molecule 1 (KIM-1), neutrophil gelatinaseassociated lipocalin (NGAL), N-acetyl-b-D-glucosaminidase
(NAG), interleukin 18 (IL-18), and matrix metalloproteinase
9 (MMP-9). The accuracy and reliability of these markers varies across individual studies. An assay for serum and urinary
NGAL levels has become recently available but is not routinely
used in the clinical setting at this time.
EPIDEMIOLOGY OF AKI IN CANCER PATIENTS
The overall incidence of AKI among cancer patients was recently
defined in a large Danish study. Among 1.2 million people
followed between 1999 and 2006, there were 37,267 incident
cancer patients with a baseline creatinine measurement. The
1-year risk of AKI in this population (defined as a .50% rise in
serum creatinine) was 17.5%, with a 27% risk over 5 years (14).
Patients with distant metastases were at the highest risk of AKI.
More severe AKI, defined as a doubling of serum creatinine
(injury in the RIFLE criteria) (20), had an 8.8% and 14.6%
risk at 1 and 5 years, respectively. Even more severe AKI, corresponding to failure in RIFLE criteria and reflecting a tripling of
serum creatinine or absolute rise .4 mg/dL, was seen in 4.5%
and 7.6% of patients at 1 and 5 years, respectively. Among cancer
patients with any stage of AKI (9,613 total), 5.1% required
dialysis within 1 year of AKI onset. Older patients were most
heavily represented in this analysis.
ASSESSMENT OF KIDNEY FUNCTION
Cancers with highest AKI risk
The ideal marker of kidney function would be a substance that is
freely filtered, neither secreted nor reabsorbed, and is solely
eliminated by the kidney. Although inulin and radiolabeled EDTA
and iothalomate demonstrate many of these characteristics, their
complexity and cost of measurement have precluded use in daily
practice. Serum creatinine has been traditionally used as a marker
of kidney function, but when used in isolation, it is not an adequate
measure. Serum creatinine values are altered by many other factors
including muscle mass, diet, sex, and tubular secretion. Patients
with cancer may present with spuriously low serum creatinine
levels secondary to cachexia. However, estimating equations for
GFR, which factor other variables such as age, sex, and race along
with serum creatinine, provide a reasonable estimate of renal
function in most patients. The most commonly used estimating
equations are the Cockcroft-Gault, the Modification of Diet in
Renal Disease (MDRD), and the Chronic Kidney Disease Epidemiology Collaboration (CKD-EPI) formulas. Among patients
with cancer who have serum creatinine values within the normal
range, 20% of patients have unsuspected CKD when the GFR is
estimated by Cockcroft-Gault formula (19).
It is well understood that elevation in serum creatinine is a
relatively late marker of renal injury, as a significant amount of
kidney function may be lost before a rise in serum creatinine is
apparent. Several urinary biomarkers of AKI that have greater
sensitivity for acute renal injury have been proposed, including
Certain cancers carry a much higher risk of AKI than others. In
the Danish study above, kidney cancer, multiple myeloma, and
liver cancer had the highest 1-year risk of AKI at 44.0%, 33.0%,
and 31.8%, respectively. After diagnosis of renal cell carcinoma,
many patients still undergo radical nephrectomy, and this
procedure itself is associated with a 33.7% risk of AKI and
predicts the future development of CKD at 1 year (21).
Patients with acute lymphoma or leukemia undergoing
induction chemotherapy are also at an especially high risk of
AKI. In a series of 537 patients with either acute myelogenous
leukemia or high-risk myelodysplastic syndrome undergoing
induction, 36% developed AKI. Even among patients with mild
AKI (defined as RIFLE risk), 8-week mortality was 13.6% (95%
confidence interval, 7.8%–23%) compared with patients with no
AKI whose 8-week mortality was 3.8% (95% confidence interval,
2.2%–6.4%). Patients requiring RRT experienced mortality of
61.7% (95% confidence interval, 50%–74%) over the same
time frame (12).
AKI is common in hospitalized cancer patients and also
correlates with increased length of stay, cost, and mortality.
Candrilli and colleagues analyzed the 2004 Nationwide Inpatient
Sample for patients with hematologic malignancies. They
identified 350,601 patients without AKI, 27,654 patients with
mild or moderate AKI (not requiring dialysis), and 5,148 patients
with severe AKI (requiring dialysis). The average length of stay
and costs among these groups were 7.4, 12.2, and 17.6 days, and
2
Onco-Nephrology Curriculum
American Society of Nephrology
Table 1. Cancer-specific risk factors for AKI
Table 2. Common causes of AKI in patients with cancer
Age .65 years
Congestive heart failure (i.e., exposure to anthracyclines, trastuzumab)
CKD
Hypovolemia (i.e., chemotherapy-related nausea and vomiting, acute
graft-versus-host disease)
Distant metastases
Multiple myeloma
Liver cancer
Nephrectomy for renal cell carcinoma
Induction chemotherapy for acute lymphoma or leukemia
Prerenal azotemia
Volume depletion
Nausea, vomiting, diarrhea
Decreased oral intake owing to mucositis (5-fluorouracil,
methotrexate, taxanes)
Polyuria caused by hyperglycemia (steroids) or diabetes
insipidus (pituitary tumor)
“Third spacing” (hypoalbuminemia, liver or peritoneal
metastases, interleukin-2)
Insensible loss of fluid from skin lesions (mycosis fungoides)
Hemodynamic-mediated
Sepsis
Renal arteriolar vasoconstriction (nonsteroidal antiinflammatory drugs [NSAIDs], calcineurin inhibitors,
hypercalcemia)
Congestive heart failure
Hepatorenal syndrome/hepatic sinusoidal obstruction
syndrome
Budd-Chiari syndrome
Intrahepatic inferior vena cava compression or thrombosis
caused by hepatomegaly or a tumor
Intravenous iodinated contrast agent
Abdominal compartment syndrome
Intrinsic renal disease
Acute tubular necrosis
Chemotherapy (cisplatin, ifosfamide)
Anti-infectives (amphotericin B, foscarnet, cidofovir,
aminoglycosides, vancomycin)
Bisphosphonates
Sepsis
Prolonged prerenal azotemia
Allergic interstitial nephritis (penicillins, cephalosporins,
fluoroquinolones, NSAIDs)
Crystal nephropathy (methotrexate, acyclovir, ciprofloxacin,
sulfonamides, rifampin)
Osmotic nephrosis (IV immunoglobulin, mannitol, starch)
Thrombotic microangiopathy (post-hematopoietic stem cell
transplant, gemcitabine, prior
radiation therapy)
Myeloma-related kidney disease
Postrenal obstruction
Bladder outlet obstruction (malignancy of cervix, prostate,
bladder, or uterus)
Retroperitoneal disease (metastasis, lymphadenopathy, fibrosis)
Hemorrhagic cystitis (cyclophosphamide, BK virus)
Ureteral strictures (prior radiation therapy, BK virus)
$13,947, $25,638, and $44,619, respectively (22). Cancer-specific
risk factors for AKI are summarized in Table 1.
ETIOLOGY OF AKI
The causes of AKI in patients with cancer are numerous (Table
2). The sites along the nephron at which some of these syndromes act are depicted in Figure 1. The specific diagnoses
will be discussed in detail elsewhere in the core curriculum,
but some notable causes are highlighted in this chapter.
Sepsis
Sepsis is the most common cause of AKI in patients with cancer.
In population-based studies, approximately 15% of critically ill
patients with sepsis have underlying cancer (23). Acute tubular
necrosis secondary to sepsis remains the leading cause of AKI in
critically ill patients with cancer. Patients with hematologic malignancies are especially prone to the development of bacterial
infections and sepsis secondary to prolonged neutropenia.
Nearly half of patients admitted to the intensive care unit
(ICU) with hematologic malignancies have underlying sepsis
compared with 12%–25% of patients with solid tumors (24).
Studies have demonstrated improved survival of cancer patients with sepsis over the last decade, except in patients that
require RRT, where hospital mortality approaches 80%
(25,26).
Sepsis causes AKI by systemic vasodilation, leading to decreased
effective circulating volume, cytokine activation, endothelial
damage, and microthrombi formation. The use of vasoconstricting pressor agents further exacerbates an effective prerenal state.
Anti-infectives
The high incidence of sepsis in critically ill cancer patients
necessitates the use of nephrotoxic antibacterial and antifungal
agents. Aminoglycosides may cause nephrotoxicity after 5–7
days of therapy, and patients present with nonoliguric AKI,
hypokalemia, hypomagnesemia, and hypocalcemia. The risk
of renal toxicity may be minimized with once daily dosing.
Several alternative drugs to aminoglycosides that do not cause
AKI have become available in the treatment of neutropenic
fever. Amphotericin B deoxycholate may cause tubular
American Society of Nephrology
toxicity and vasoconstriction, leading to nonoliguric AKI,
hypokalemia, hypomagnesemia, and distal renal tubular acidosis. Newer liposomal and lipid formulations are less nephrotoxic with comparable efficacy. Other novel antifungal
agents, caspofungin and voriconizole, are also less nephrotoxic and are often used as first-line therapy. Several studies
have reported on the nephrotoxicity of vancomycin, although
the biological mechanism remains undefined. Reported risk
factors for AKI are higher trough levels (.15 mg/dL) and
Onco-Nephrology Curriculum
3
Figure 1. Sites of injury in AKI syndromes. TMA, thrombotic microangiopathy; ATN, acute tubular necrosis.
higher daily doses (.4 g/day) (27,28). Patients present with
nonoliguric AKI and bland urine sediment, and most patients
recover renal function after discontinuation of the drug.
Chemotherapy
Cisplatin is a DNA alkylating agent used to treat a variety of
tumors including sarcomas, small cell lung cancer, ovarian
cancer, and germ cell tumors. It is directly tubular toxic
and leads to salt wasting, hyponatremia, hypomagnesemia, and
AKI. A low chloride environment enhances toxicity, and
concurrent saline administration to achieve urine output
.3 L/day is the mainstay of prevention. Approximately onethird of patients will experience AKI within days after treatment,
and episodes worsen with repeated dosing. Tubular injury
may be permanent with doses .100 mg/m2. Amifostine, a
free radical scavenger, has been shown to ameliorate cisplatin
nephrotoxicity. Newer platinum agents such as carboplatin
and oxaliplatin appear to cause less tubular injury. Ifosfamide
is an alkylating agent commonly used in treating sarcomas and
metastatic germ cell turmors, which may cause AKI in up to
30% of patients. Proximal tubular injury may also lead to
glucosuria, hypokalemia, hypophophatemia, and proximal
renal tubular acidosis. Severe cases may present with Fanconi’s
syndrome. Cumulative doses .100 g/m2 are associated with
moderate to severe tubular injury. Risk factors for AKI include
prior cisplatin therapy, tumor infiltration of the kidney, and
underlying CKD. Mesna protects against bladder toxicity
4
Onco-Nephrology Curriculum
from metabolites excreted in the urine, which helps prevent
hemorrhagic cystitis.
Methotrexate is an antifolate and antimetabolite commonly
used in the treatment of leukemia, lymphoma, and sarcoma.
High-dose methotrexate (.1 g/m2) may cause AKI by forming
intratubular crystals leading to obstruction and direct tubular
cell toxicity. Patients generally present with nonoliguric AKI
with a subsequent rapid rise in serum creatinine. Intravenous
hydration and urinary alkalinization prevent the precipitation
of methotrexate crystals. In the setting of AKI, methotrexate
may accumulate and cause neutropenia, hepatitis, mucositis,
and neurologic impairment. Folinic acid may be given concurrently to replete folic acid stores and minimize toxicities.
Dialysis can acutely clear methotrexate from the blood, but
levels quickly rebound after discontinuation of treatment.
Carboxypeptidase G2 can rapidly convert methotrexate to
an inactive metabolite and recently became commercially
available. This therapy also suffers from rebound in plasma
levels, but to a lesser degree than high-flux dialysis.
Targeted therapy
Targeted therapy against vascular endothelial growth factor
(VEGF) has advanced the treatment of certain tumors including
colorectal and renal cell carcinoma. Monoclonal antibody to
VEGF (bevacizumab) and tyrosine kinase inhibitors of the VEGF
pathway (sunitinib, sorafenib, pazopanib, axitinib, and regorafenib)
have been associated with the development of hypertension and
American Society of Nephrology
proteinuria (29). Rare cases of thrombotic microangiopathy
(TMA) have also been reported (30). Symptoms generally
resolve with discontinuation of the drug.
Multiple myeloma
Multiple myeloma involves the clonal proliferation of malignant plasma cells and is the second most common hematologic
malignancy after non-Hodgkin lymphoma. Approximately
one-half of patients with multiple myeloma present with AKI,
and 10% require dialysis on initial presentation (31). The
common mechanisms of injury include cast nephropathy,
light chain deposition disease, light chain amyloidosis, hypercalcemia, and acute tubular necrosis (ATN) from sepsis. Suppression of normal hematopoiesis predisposes patients to
infections and sepsis, which often requires ICU admission.
Initial management consists of saline hydration, correction
of hypercalcemia, alkalinization of urine, and avoidance of
nonsteroidal anti-inflammatory drugs and iodinated contrast.
Renal recovery occurs in up to one-half of patients, except in
patients who require dialysis, where recovery rates are ,25%.
In a randomized controlled trial, the use of plasma exchange
did not significantly decrease the composite end point of
death, dialysis dependence, or GFR ,30 mL/min (32). With
concurrent chemotherapy, the use of high cut-off filters with
extended daily dialysis may help to decrease circulating monoclonal light chains. Multicenter randomized controlled trials
studying the utility of high cut-off hemofilters are currently
ongoing.
Hematopoietic cell transplant
The number of hematopoietic cell transplants (HCTs) performed has dramatically increased over the last three decades.
Refinement in techniques has permitted transplants in older
patients with more comorbidities. All patients, regardless of the
type of transplant, are susceptible to infection after transplant
until engraftment is complete. During this period, patients are
at most risk of developing AKI from ischemic and toxic ATN in
the setting of sepsis. Patients who receive allogeneic transplants
require calcineurin inhibitors to prevent graft-versus-host
disease (GVHD), which further increases the risk of AKI. The
need for RRT after HCT increases mortality more than 70%
(33,34).
Engraftment syndrome may occur within days after autologous HCT and is a common reason for ICU admission. It is
associated with cytokine release in association with rapid
neutrophil recovery after HCT. Patients develop fever, noncardiogenic pulmonary edema, erythrodermatous skin rash,
and peripheral edema. Often these patients develop nonoliguric AKI with relatively bland urine sediment. The
mainstay of treatment is corticosteroids and diuretics, and
most patients will recover renal function without the need for
RRT.
Hepatic sinusoidal obstruction syndrome (HSOS), formerly termed veno-occlusive disease, is associated with AKI
within the first month after allogeneic HCT. Damage to the
American Society of Nephrology
hepatic sinusoidal endothelium from the pretransplant conditioning regimen leads to sloughing of the endothelium,
collagen deposition, fibrosis, and liver failure. In severe cases,
patients may subsequently develop AKI from hepatorenal
syndrome. Presentation includes right upper quadrant abdominal pain, ascites, edema, and elevated bilirubin. Treatment includes salt restriction, diuretics, and RRT if needed.
Severe HSOS, defined as severe liver injury unresponsive to
supportive care, often requires ICU admission and is historically associated with near 100% mortality. Defibrotide, an
oligonucleotide that has antithrombotic and profibrinolytic
properties with minimal anticoagulant effects, has shown
promise in patients with severe HSOS. Several clinical trials
using defibrotide for treatment of severe HSOS have demonstrated improvement in complete response rates and overall
survival, and the drug is currently commercially available in
Europe (35,36). A new drug application (NDA) for defibrotide
was submitted to the Food and Drug Administration in 2014
and has been granted Fast Track Designation.
TMA occurs in approximately 2%–21% of patients after
allogeneic stem cell transplant (37). In one study, 3% of all
cancer patients admitted with AKI to the ICU had underlying
TMA (4). Patients often present with progressive AKI, anemia
out of proportion to underlying renal function, and hypertension. Risk factors for transplant-associated TMA (TA-TMA)
are acute GVHD, recipient/donor mismatch, total body
irradiation .1,200 cGy, and adenovirus infection (37).
TA-TMA is not associated with ADAMTS-13 deficiency and is
poorly responsive to plasmapheresis. Calcineurin inhibitors are
also associated with TMA and should be withheld or decreased
in dose if possible.
Contrast-induced nephropathy
Intravascular administration of iodinated contrast is associated
with contrast-induced nephropathy (CIN). Risk factors include underlying CKD, diabetes mellitus, volume depletion,
and coadministration of other nephrotoxins. Intra-arterial
injection is considered to be more nephrotoxic compared
with intravenous administration. In addition, high osmolar
(.1400 mOsm/kg) and low osmolar (600–800 mOsm/kg)
contrast agents are associated with a higher incidence of
AKI in comparison to iso-osmolar (300 mOsm/kg) contrast.
Preventive measures should be taken in patients with
GFR ,60 mL/min including limiting contrast volume, using
iso-osmolar contrast, prehydration with normal saline, and
discontinuation of concurrent nephrotoxic agents. Several
meta-analyses have examined the use of N-acetylcysteine in
the prevention of CIN but results remain inconclusive, as is
the use of bicarbonate (38). There is insufficient evidence to
recommend hemodialysis or hemofiltration for the prevention or treatment of CIN.
Abdominal compartment syndrome
Abdominal compartment syndrome (ACS) is most commonly
defined as an intra-abdominal pressure (IAP) .10 and clearly
Onco-Nephrology Curriculum
5
.20 mmHg with evidence of organ dysfunction that improves
with abdominal decompression. Patients may present with
tachypnea with high ventilatory pressures, liver dysfunction, intestinal ischemia, and oliguric AKI. In patients with cancer, common causes include malignant ascites, urinary leak from a recent
urologic procedure, and colonic dilatation. The IAP, which is
measured by transducing a foley catheter filled with saline with a
pressure monitoring system, is normally 0–10 mmHg. Values
between 12 and 20 mmHg are classified as intra-abdominal
hypertension and are not generally associated with organ dysfunction. Depending on the etiology, treatment may involve
diuretics, paracentesis, colonic decompression with nasogastric suction, and decompression laparotomy. Generally, urine
output and renal function markedly improve with therapy.
CONCLUSION
AKI is a common complication of cancer or its treatment.
Advances in supportive care including RRT have improved
outcomes in critically ill patients with cancer, with the
exception of patients with allogeneic stem cell transplants. A
joint decision-making process between the oncologist, intensivist, and nephrologist is vital to determine which patients are
best suited for RRT. Identification of risk factors for AKI, as well
as the development of biomarkers of kidney injury, may lead to
earlier intervention.
TAKE HOME POINTS
c The selection of patients best suited for RRT requires a team-based
approach between the oncologist, intensivist, and nephrologist.
c Manifestations of kidney disease from chemotherapy and targeted
therapy include AKI, proteinuria, electrolytes derangements, and TMA.
c Nearly one-half of patients with multiple myeloma have evidence of AKI
on initial presentation, and 10% require dialysis.
c Engraftment syndrome, HSOS, and TMA are unique causes of AKI in
patients after stem cell transplant. The mortality of patients that require
dialysis after stem cell transplant remains high.
REFERENCES
1. Brenner H. Long-term survival rates of cancer patients achieved by the
end of the 20th century: A period analysis. Lancet 360: 1131–1135,
2002
2. Benoit DD, Hoste EA, Depuydt PO, Offner FC, Lameire NH,
Vandewoude KH, Dhondt AW, Noens LA, Decruyenaere JM. Outcome
in critically ill medical patients treated with renal replacement therapy
for acute renal failure: comparison between patients with and those
without haematological malignancies. Nephrol Dial Transplant 20:
552–558, 2005
3. Berghmans T, Meert AP, Markiewicz E, Sculier JP. Continuous venovenous haemofiltration in cancer patients with renal failure: A singlecentre experience. Support Care Cancer 12: 306–311, 2004
6
Onco-Nephrology Curriculum
4. Darmon M, Thiery G, Ciroldi M, Porcher R, Schlemmer B, Azoulay E.
Should dialysis be offered to cancer patients with acute kidney injury?
Intensive Care Med 33: 765–772, 2007
5. Kramer AA, Postler G, Salhab KF, Mendez C, Carey LC, Rabb H. Renal
ischemia/reperfusion leads to macrophage-mediated increase in pulmonary vascular permeability. Kidney Int 55: 2362–2367, 1999
6. Vieira JM, Jr., Castro I, Curvello-Neto A, Demarzo S, Caruso P, Pastore
L, Jr., Imanishe MH, Abdulkader RC, Deheinzelin D. Effect of acute
kidney injury on weaning from mechanical ventilation in critically ill
patients. Crit Care Med 35: 184–191, 2007
7. Bellomo R, Ronco C, Kellum JA, Mehta RL, Palevsky P. Acute renal
failure: Definition, outcome measures, animal models, fluid therapy and
information technology needs: the Second International Consensus
Conference of the Acute Dialysis Quality Initiative (ADQI) Group. Crit
Care 8: R204–R212, 2004
8. Mehta RL, Kellum JA, Shah SV, Molitoris BA, Ronco C, Warnock
DG, Levin A. Acute Kidney Injury Network: Report of an initiative to
improve outcomes in acute kidney injury. Crit Care 11: R31, 2007
9. Kellum JA. Defining and classifying AKI: One set of criteria. Nephrol
Dial Transplant 23: 1471–1472, 2008
10. Lahoti A, Nates JL, Wakefield CD, Price KJ, Salahudeen AK. Costs and
outcomes of acute kidney injury in critically ill patients with cancer. J
Support Oncol 9: 149–155, 2011
11. Salahudeen AK, Doshi SM, Pawar T, Nowshad G, Lahoti A, Shah P. Incidence rate, clinical correlates, and outcomes of AKI in patients admitted
to a comprehensive cancer center. Clin J Am Soc Nephrol 8: 347–354, 2013
12. Lahoti A, Kantarjian H, Salahudeen AK, Ravandi F, Cortes JE, Faderl S,
O’Brien S, Wierda W, Mattiuzzi GN. Predictors and outcome of acute
kidney injury in patients with acute myelogenous leukemia or high-risk
myelodysplastic syndrome. Cancer 116: 4063–4068, 2010
13. Kim CS, Oak CY, Kim HY, Kang YU, Choi JS, Bae EH, Ma SK, Kweon SS, Kim
SW. Incidence, predictive factors, and clinical outcomes of acute kidney
injury after gastric surgery for gastric cancer. PLoS One 8: e82289, 2013
14. Christiansen CF, Johansen MB, Langeberg WJ, Fryzek JP, Sorensen
HT. Incidence of acute kidney injury in cancer patients: A Danish
population-based cohort study. Eur J Intern Med 22: 399–406, 2011
15. Darmon M, Ciroldi M, Thiery G, Schlemmer B, Azoulay E. Clinical review: Specific aspects of acute renal failure in cancer patients. Crit Care
10: 211, 2006
16. Benoit DD, Hoste EA. Acute kidney injury in critically ill patients with
cancer. Crit Care Clin 26: 151–179, 2009
17. Darmon M, Thiery G, Ciroldi M, de Miranda S, Galicier L, Raffoux E, Le
Gall JR, Schlemmer B, Azoulay E. Intensive care in patients with newly
diagnosed malignancies and a need for cancer chemotherapy. Crit
Care Med 33: 2488–2493, 2005
18. Soares M, Salluh JI, Carvalho MS, Darmon M, Rocco JR, Spector N.
Prognosis of critically ill patients with cancer and acute renal dysfunction. J Clin Oncol 24: 4003–4010, 2006
19. Dogan E, Izmirli M, Ceylan K, Erkoc R, Sayarlioglu H, Begenik H, Alici S.
Incidence of renal insufficiency in cancer patients. Adv Ther 22: 357–
362, 2005
20. Eheman C, Henley SJ, Ballard-Barbash R, Jacobs EJ, Schymura MJ,
Noone AM, Pan L, Anderson RN, Fulton JE, Kohler BA, Jemal A, Ward
E, Plescia M, Ries LA, Edwards BK. Annual Report to the Nation on the
status of cancer, 1975-2008, featuring cancers associated with excess
weight and lack of sufficient physical activity. Cancer 118: 2338–2366,
2012
21. Cho A, Lee JE, Kwon GY, Huh W, Lee HM, Kim YG, Kim DJ, Oh HY, Choi
HY. Post-operative acute kidney injury in patients with renal cell carcinoma is a potent risk factor for new-onset chronic kidney disease after
radical nephrectomy. Nephrol Dial Transplant 26: 3496–3501, 2011
22. Candrilli S, Bell T, Irish W, Morris E, Goldman S, Cairo MS. A comparison
of inpatient length of stay and costs among patients with hematologic
malignancies (excluding hodgkin disease) associated with and without
acute renal failure. Clin Lymphoma Myeloma 8: 44–51, 2008
American Society of Nephrology
23. Lameire N, Van Biesen W, Vanholder R. Acute renal problems in the
critically ill cancer patient. Curr Opin Crit Care 14: 635–646, 2008
24. Taccone FS, Artigas AA, Sprung CL, Moreno R, Sakr Y, Vincent JL.
Characteristics and outcomes of cancer patients in European ICUs. Crit
Care (London, England) 13: R15, 2009
25. Larche J, Azoulay E, Fieux F, Mesnard L, Moreau D, Thiery G, Darmon
M, Le Gall JR, Schlemmer B. Improved survival of critically ill cancer
patients with septic shock. Intensive Care Med 29: 1688–1695, 2003
26. Pene F, Percheron S, Lemiale V, Viallon V, Claessens YE, Marque S,
Charpentier J, Angus DC, Cariou A, Chiche JD, Mira JP. Temporal
changes in management and outcome of septic shock in patients with
malignancies in the intensive care unit. Critical Care Med 36: 690–696,
2008
27. Hidayat LK, Hsu DI, Quist R, Shriner KA, Wong-Beringer A. High-dose
vancomycin therapy for methicillin-resistant Staphylococcus aureus
infections: Efficacy and toxicity. Arch Intern Med 166: 2138–2144,
2006
28. Lodise TP, Lomaestro B, Graves J, Drusano GL. Larger vancomycin
doses (at least four grams per day) are associated with an increased
incidence of nephrotoxicity. Antimicrob Agents Chemother 52: 1330–
1336, 2008
29. Izzedine H, Rixe O, Billemont B, Baumelou A, Deray G. Angiogenesis
inhibitor therapies: Focus on kidney toxicity and hypertension. Am J
Kidney Dis 50: 203–218, 2007
30. Eremina V, Jefferson JA, Kowalewska J, Hochster H, Haas M, Weisstuch
J, Richardson C, Kopp JB, Kabir MG, Backx PH, Gerber HP, Ferrara N,
Barisoni L, Alpers CE, Quaggin SE. VEGF inhibition and renal thrombotic microangiopathy. N Engl J Med 358: 1129–1136, 2008
31. Kyle RA, Gertz MA, Witzig TE, Lust JA, Lacy MQ, Dispenzieri A, Fonseca
R, Rajkumar SV, Offord JR, Larson DR, Plevak ME, Therneau TM, Greipp
PR. Review of 1027 patients with newly diagnosed multiple myeloma.
Mayo Clin Proc 78: 21–33, 2003
American Society of Nephrology
32. Clark WF, Stewart AK, Rock GA, Sternbach M, Sutton DM, Barrett BJ,
Heidenheim AP, Garg AX, Churchill DN. Plasma exchange when myeloma presents as acute renal failure: A randomized, controlled trial.
Ann Intern Med 143: 777–784, 2005
33. Lopes JA, Jorge S. Acute kidney injury following HCT: Incidence, risk
factors and outcome. Bone Marrow Transplantation 46: 1399–1408,
2011
34. Humphreys BD. Onco-nephrology: Kidney disease in the cancer patient: Introduction. Semin Nephrol 30: 531–533, 2010
35. Richardson PG, Murakami C, Jin Z, Warren D, Momtaz P, Hoppensteadt
D, Elias AD, Antin JH, Soiffer R, Spitzer T, Avigan D, Bearman SI, Martin
PL, Kurtzberg J, Vredenburgh J, Chen AR, Arai S, Vogelsang G,
McDonald GB, Guinan EC. Multi-institutional use of defibrotide in 88
patients after stem cell transplantation with severe veno-occlusive
disease and multisystem organ failure: response without significant
toxicity in a high-risk population and factors predictive of outcome.
Blood 100: 4337–4343, 2002
36. Richardson PG, Soiffer RJ, Antin JH, Uno H, Jin Z, Kurtzberg J, Martin
PL, Steinbach G, Murray KF, Vogelsang GB, Chen AR, Krishnan A,
Kernan NA, Avigan DE, Spitzer TR, Shulman HM, Di Salvo DN, Revta C,
Warren D, Momtaz P, Bradwin G, Wei LJ, Iacobelli M, McDonald GB,
Guinan EC. Defibrotide for the treatment of severe hepatic venoocclusive disease and multiorgan failure after stem cell transplantation:
a multicenter, randomized, dose-finding trial. Biol Blood Marrow
Transplant 16: 1005–1017, 2010
37. Changsirikulchai S, Myerson D, Guthrie KA, McDonald GB, Alpers
CE, Hingorani SR. Renal thrombotic microangiopathy after hematopoietic cell transplant: Role of GVHD in pathogenesis. Clin J Am Soc
Nephrol 4: 345–353, 2009
38. Vaitkus PT, Brar C. N-acetylcysteine in the prevention of contrastinduced nephropathy: Publication bias perpetuated by meta-analyses.
Am Heart J 153: 275–280, 2007
Onco-Nephrology Curriculum
7
REVIEW QUESTIONS
1. Criteria for AKI as defined by the KDIGO classification
include the following except:
a. A rise in SCr $0.3 mg/dL within 48 hours
b. An increase in SCr to $1.5 times baseline within the prior
7 days
c. A urine volume of ,0.5 mL/kg/h for 6 hours
d. An increase in SCr to $1.5 times the upper limit of the
“normal” range as listed in the laboratory reference values
Answer: d is correct. The KDIGO classification defines AKI
as 1) an increase in SCr $0.3 mg/dL within 48 hours; 2) an
increase in SCr to $1.5 times baseline within the prior 7 days,
or 3) a urine volume of ,0.5 mL/kg/h for 6 hours. The upper
limit of normal from a reference range should not be used in
diagnosing AKI if the patient’s baseline SCr level is known.
2. Common manifestations of myeloma-related kidney disease include all of the following except:
a.
b.
c.
d.
8
Cast nephropathy
Light chain deposition disease
Thrombotic microangiopathy (TMA)
Light chain amyloidosis
Onco-Nephrology Curriculum
Answer: c is correct. The three most common manifestations of myeloma-related kidney disease include cast
nephropathy, light chain deposition disease, and light
chain amyloidosis. Other less common manifestations
include heavy chain deposition disease, membranoproliferative glomerulonephritis from cryoglobulinemia,
and fibrillary glomerulonephritis. TMA is not a common
presentation.
3. Which of the following therapies has shown efficacy in the
treatment of HSOS after stem cell transplant?
a.
b.
c.
d.
Heparin
Defibrotide
Tissue plasminogen activator (tPA)
Plasmapheresis
Answer: b is correct. Heparin has been used for prophylaxis
of HSOS with mixed results. Both heparin and tPA have unacceptable bleeding risks when used for treatment of HSOS.
Defibrotide, an oligonucleotide that has antithrombotic and
profibrinolytic properties with minimal anticoagulant effects, has shown promise in the treatment of patients with
severe HSOS. Plasmapheresis has no role in the treatment
of HSOS.
American Society of Nephrology
Chapter 4: Tumor Lysis Syndrome
Amaka Edeani, MD,* and Anushree Shirali, MD†
*Kidney Diseases Branch, National Institute of Diabetes and Digestive and Kidney Diseases, National Institutes of
Health, Bethesda, Maryland; and †Section of Nephrology, Yale University School of Medicine, New Haven, Connecticut
INTRODUCTION
Tumor lysis syndrome (TLS) is a constellation of
metabolic abnormalities resulting from either spontaneous or chemotherapy-induced tumor cell death.
Tumor cytotoxicity releases intracellular contents,
including nucleic acids, proteins, and electrolytes
into the systemic circulation and may lead to development of hyperuricemia, hyperphosphatemia, hypocalcemia, and hyperkalemia. Clinically, this results
in multiorgan effects such as AKI, cardiac arrhythmias, and seizures (1,2). TLS is the most common
oncologic emergency (3), and without prompt recognition and early therapeutic intervention, morbidity and mortality is high.
DEFINITION
Hande and Garrow (4) first initiated a definition of
the clinical and pathologic characteristics of patients
at risk for developing TLS. Based on a retrospective
analysis of 102 patients with non-Hodgkin lymphoma (NHL), they classified TLS as laboratory
TLS (LTLS) or clinical TLS (CTLS). Cairo and Bishop
(1) modified these criteria to formulate a commonly
used classification system for TLS. This system (Table
1) defines LTLS when two or more of the following
abnormalities are met within 3 days before or 7 days
after the initiation of chemotherapy: 1) 25% decrease
from baseline in serum calcium, and/or 2) 25% increase from baseline in the serum values of uric acid,
potassium, or phosphorous.
The Cairo and Bishop definition assumes adequate
volume expansion and prophylaxis with a hypouricemic agent. LTLS is defined as CTLS (Table 1) when
LTLS is accompanied by one or more clinical manifestations such as cardiac arrhythmia, death, seizure,
or AKI with an elevated serum creatinine .1.5 times
upper limit of normal. Additionally, this definition of
CTLS assumes that the clinical manifestations are not
caused directly by the therapeutic agent. Last, a third
American Society of Nephrology
class specifies patients with normal laboratory and
clinical parameters as having no LTLS or CTLS.
Cairo and Bishop also proposed a grading system
combining the definitions of no TLS, LTLS, and CTLS,
with the maximal clinical manifestations in each
affected organ defining the grade of TLS (1). Although
this grading system attempts to provide uniform definitions to TLS severity, it is not widely used in clinical
practice.
The Cairo-Bishop classification is not immune to
critique despite its common use. Specifically, patients
with TLS may not always have two or more abnormalities present at once, but one metabolic derangement may precede another (2). Furthermore, a 25%
change from baseline may not always be significant if
it does not result in a value outside the normal range
(2). From a renal standpoint, Wilson and Berns (5)
have noted that defining AKI on the basis of a creatinine value .1.5 times the upper limit of normal
does not clearly distinguish CKD from AKI. Thus,
they propose using established definitions of AKI in
CTLS such as an absolute 0.3 mg/dL increase or relative 50% increase in creatinine over baseline. Finally,
they point out that the Cairo-Bishop classifications
cannot be applied to spontaneous TLS, which is common with high-risk malignancies, as chemotherapy
is a required criterion for LTLS and CTLS.
EPIDEMIOLOGY AND RISK FACTORS
TLS is most commonly described in NHL, particularly
Burkitt-type lymphoma (BTL), as well as other hematologic malignancies, such as acute lymphocytic
and lymphoblastic leukemia (ALL) and acute myeloid
leukemia (AML) (6–8), and less commonly in chronic
Correspondence: Anushree Shirali, Section of Nephrology, Yale
University School of Medicine, PO Box 208029, New Haven,
Connecticut 06520-8029.
Copyright © 2016 by the American Society of Nephrology
Onco-Nephrology Curriculum
1
Table 1. Cairo-Bishop definition of laboratory tumor lysis syndrome and clinical tumor lysis syndrome
Laboratory Tumor Lysis Syndrome
Metabolite or electrolyte
Uric acid
Potassium
Phosphorus
Calcium
Criterion for diagnosis
$8 mg/dL or 25% increase from baseline
$6 mEq/L or 25% increase from baseline
$6.5 mg/dL (children), $4.5 mg/dL (adults), or 25% increase from baseline
$25% decrease from baseline
Clinical Tumor Lysis Syndrome
LTLS and one or more of the following: 1) creatinine 3 $1.5 ULN (age .12 years of age or age adjusted); 2) cardiac arrhythmia or sudden death;
3) seizure
LTLS, laboratory tumor lysis syndrome; ULN, upper limit of normal.
leukemias (9–11) and multiple myeloma (12,13). More rarely,
TLS has also been described with solid malignancies (14,15) with
particular features, including large tumor burden, metastatic disease, specifically in the liver, short doubling time, increased chemosensitivity, and elevated uric acid and lactate dehydrogenase
(LDH) (15). Among solid tumors, small-cell carcinoma of the
lung, germ cell tumors, neuroblastoma, and breast carcinoma
have all been linked to development of TLS (8). TLS is usually
associated with cytotoxic chemotherapy but reports have also
linked it to the use of imatinib (11), bortezomib (12), corticosteroids (16,17), rituximab (18), methotrexate (19), and thalidomide (13,20). There are also case reports of TLS following total
body irradiation (21) and chemoembolization (22). Last, TLS
may also be spontaneous, i.e., not requiring initiation of cytotoxic
therapy. This has been most frequently described in BTL (23–25).
The incidence of TLS varies based on the underlying
malignancy and the definition of TLS. Most incidence data
are from older, retrospective studies that precede the CairoBishop classification, so there is considerable heterogeneity in
the data. In a review of 102 patients with high-grade NHL and
using the Hande-Garrow classification, LTLS was seen in 42%
of patients, with CTLS occurring only in 6% (4). In BTL,
however, 56% and 11% of patients met criteria for LTLS and
CTLS, respectively. Mato et al. (26) studied 194 patients receiving induction therapy for AML and found a TLS incidence
of 9.8%. In a mixed adult and pediatric study of 788 European
patients with acute leukemia or NHL (27), the overall incidence of LTLS and CTLS was 18.9% and 5%, respectively.
When classified by tumor type, LTLS and CTLS incidence rates
of 14.7% and 3.4% were seen in AML patients, respectively;
21.4% and 5.2% in ALL patients, respectively; and 19.6% and
6.1% in patients with NHL, respectively (27). Wössman et al.
(28) reviewed the incidence and complications of 1,791 children with NHL and reported an overall incidence of 4.4%, of
which 26% had B-cell ALL (B-ALL).
Risk stratification
Risk factors (2,29) for TLS include cancer and patient-specific
factors. Increased tumor burden is the most cancer-specific
risk factor and is demonstrated by elevated LDH (28), white
blood cell count .50,000/mm3, massive liver metastasis (14),
bone marrow involvement (2), cancer stage, proliferation rate
2
Onco-Nephrology Curriculum
of cancer cells, and cell sensitivity to cytotoxic therapy. Patient-related factors include age, volume depletion, preexisting CKD, hyperuricemia, and hyponatremia. Recognition of
these high-risk factors is an important step in the management
of TLS. In 2008, an expert panel (7) developed a TLS risk
classification system, based on published evidence and expert
opinion, in which malignancies as were described as low
(,1% chance), intermediate (1%–5% chance), or high risk
(.5% chance) for developing TLS. Classification into these
risk groups incorporates type of histology, extent of disease,
renal involvement or dysfunction, and type of induction therapy (Table 2).
Other factors that have been shown to be predictive of TLS
include male sex and presence of splenomegaly (26,28,30). Certain cytogenetic shifts may also portend greater risk for TLS.
Specifically, MYCN gene mutation in neuroblastoma (31), t
(8;14)(q24;q32) in L3 type of acute lymphoblastic leukemia
(32), and inv(16)(p13;q22) in acute myelocytic leukemia (33)
are all linked to more aggressive disease and greater risk for TLS.
PATHOPHYSIOLOGY AND CLINICAL
MANIFESTATIONS
TLS is a direct consequence of cell lysis and release of
intracellular products. When clearance of these products, by
excretion (renal or hepatic excretion or phagocytosis by the
reticuloendothelial system) (23), is impaired and their serum
burden increases, the clinical sequelae of TLS may occur. Of
these cellular products, nucleic acids (converted to uric acid),
potassium, and phosphorus are particularly important in the
pathophysiology of TLS.
Hyperuricemia
The nucleic acids adenine and guanine are metabolized to
xanthine, which is further metabolized by xanthine oxidase to
the water-insoluble uric acid (5) (Figure 1). Because humans
lack a functional gene for urate oxidase (uricase), which further metabolizes uric acid to the freely soluble and excretable
allantoin, patients with high-risk malignancy are susceptible
to rapid increases in serum uric acid. Uric acid is freely filtered
at the glomerulus, and handling in the renal proximal tubule
American Society of Nephrology
Table 2. Risk classification of TLS according to type of malignancy, extent of disease, and presence or absence of renal
dysfunction
Type of malignancy
Solid tumor
Myeloma
Chronic leukemia
Lymphoma: Burkitt type
Lymphoma: non-Burkitt type
Anaplastic large cell
Lymphoblastic lymphoma
Hodgkin, small lymphocytic, follicular, marginal zone B cell,
MALT, nonblastoid mantle cell, cutaneous T cell
Adult T-cell lymphoma, diffuse large B cell, peripheral T cell,
transformed, or blastoid mantle cell
Leukemia:-Burkitt type
Leukemia: non-Burkitt type; acute myeloid leukemia (AML);
acute lymphoblastic Lleukemia (ALL)
Renal dysfunction
Absent
Present
Risk
Low
Low
CML: low
CLL w/alkylating agents: low
CLL w/targeted or biological agents: intermediate
Early stage and LDH ,2 3 ULN: intermediate
Early stage and LDH .2 3 ULN: high
Advanced stage: high
Child with stage III/IV disease: intermediate
All others: low
Early stage and LDH ,2 3 ULN: intermediate
Early stage and LDH .2 3 ULN: high
Advanced stage: high
Low
Adult with normal LDH: low
Child with stage I/II disease: low
Adult with LDH . ULN and nonbulky disease: intermediate
Adult with LDH . ULN and bulky disease: high
Child with stage III/IV disease and LDH ,2 3 ULN: intermediate
Child with stage III/IV disease and LDH .2 3 ULN: high
High
AML with WBC ,25 3 109/L and LDH ,2 3 ULN: low
AML with WBC ,25 3 109/L and LDH .2 3 ULN: intermediate
AML with WBC 5 25–100 3 109/L: intermediate
ALL with WBC ,100 3 109/L and LDH ,2 3 ULN: intermediate
ALL with WBC ,100 3 109/L and LDH .2 3 ULN: high
ALL with WBC .100 3 109/L: high
Risk
If low risk disease, no change
If intermediate risk disease and normal UA, phosphorus,
and potassium, no change
If UA, phosphorus, or potassium . ULN, intermediate risk disease
becomes high risk
Low risk disease become intermediate risk
Intermediate risk disease becomes high risk
CML, chronic myeloid leukemia; CLL, chronic lymphocytic leukemia; MALT, mucosa-associated lymphoid tissue; LDH, lactate dehydrogenase; AML, acute myeloid
leukemia; WBC, white blood cell count; ALL, acute lymphocytic and lymphoblastic leukemia; UA, urinalysis; ULN, upper limit of normal.
is a combination of reabsorption and secretion via the luminal
urate/anion exchanger urate transporter 1 (URAT-1) and the
basolateral organic anion transporter (OAT) (34). URAT-1 is
an apical membrane transporter and exchanges anions for urate
absorption from the tubular lumen. It is critical in regulating
urate levels and is targeted by uricosuric and antiuricosuric
agents (34). When the capacity to transport luminal uric acid
is overwhelmed, there is potential for uric acid to crystallize
within the tubular lumen. An acidic urine pH favors this process.
Uric acid crystals can cause direct tubular injury by
obstruction, but other pathways for injury include induction
of chemokine-mediated inflammation from monocyte
chemoattractant protein-1 (MCP-1) (35) and macrophage
migration inhibition factor (MIF) (36). There are also
American Society of Nephrology
crystal-independent mechanisms which target hemodynamics.
These include increased peritubular capillary pressures, increased vasoconstriction, and decreased blood flow (5,37–39).
Uric acid may also prevent recovery from AKI in TLS, as it has
been shown to inhibit proximal tubule cell proliferation (38).
These diverse mechanisms are united in their propensity to
cause AKI. Clinically, hyperuricemia is unlikely to cause symptoms because urinary crystallization of uric acid does not result
in the renal colic, which is typical of uric acid nephrolithiasis.
Hyperkalemia
Massive tumor cell lysis releases potassium into the extracellular environment, leading to severe hyperkalemia when
uptake capacity by muscle and liver is exceeded, especially in
Onco-Nephrology Curriculum
3
Figure 1. Schematic of purine metabolism. Allopurinol acts an inhibitor of xanthine oxidase via its active metabolite, oxypurinol.
Dashed arrow and box indicate arm of metabolism not constitutively present in humans; this conversion of uric acid to water-soluble
allantoin is stimulated clinically by the administration of rasburicase (recombinant urate oxidase). Black arrows denote enzyme stimulation; red lines denote inhibition.
the setting of CKD or AKI. Muscle weakness may be the initial
symptom, but cardiac arrhythmia, manifested initially by
peaked Twaves, widened QRS complexes, and sine waves, is the
feared complication.
population (42). Although the data were not broken down into
cause of AKI, the incidence of TLS was similar at 17%, suggesting
that AKI and TLS were also linked in this population. AKI due to
TLS may be asymptomatic or include symptoms of uremia, including nausea, vomiting, and lethargy.
Hyperphosphatemia and hypocalcemia
Because phosphate is an intracellular electrolyte, cell lysis releases
significant amounts of it. However, malignant hematologic cells
may contain four times more intracellular phosphate in comparison to normal mature lymphoid cells (3), making
hyperphosphatemia a particular issue with tumor cell lysis. Because phosphorus excretion is tied to kidney function, hyperphosphatemia occurs when the kidney’s excretory capacity is
overwhelmed. Thus, preexisting CKD or AKI enhances risk
for hyperphosphatemia with TLS. Spontaneous tumor lysis,
however, is less commonly associated with hyperphosphatemia
and may be due to rapid uptake of extracellular phosphate by
residual highly metabolically active tumor cells (5). Hyperphosphatemia may cause nausea, vomiting, diarrhea, or lethargy, but
it exerts its predominant toxicity by binding to calcium cations.
This results in secondary hypocalcemia and its downstream neuromuscular and cardiovascular effects such as cramps, hypotension, tetany, and arrhythmias. Additionally, calcium–phosphate
precipitates may deposit in tissues, as seen in nephrocalcinosis,
including the renal interstitium.
AKI
AKI in TLS may be either due to the aforementioned effects of
acute urate nephropathy or hyperphosphatemic nephrocalcinosis affecting the renal tubulointerstitium or a combination of
the two. Some studies have suggested that a urine uric acid to
creatinine ratio of .1 may be specific to uric acid nephropathy
(40), but another study has noted high uric acid to creatinine
ratios in AKI from other etiologies (41).
The association between AKI and TLS has been demonstrated
across various populations and tumor subtypes (5). Annemans
et al. (27) found that in patients with leukemia and NHL who
had TLS, 45% had AKI. A smaller pediatric cohort of B-cell NHL
or ALL noted renal insufficiency in 20% percent of the study
4
Onco-Nephrology Curriculum
MANAGEMENT
Prophylaxis and monitoring
Prevention of TLS begins with recognition of risk factors and
close laboratory and clinical monitoring. Patients at highest
risk of developing TLS (Table 2) require intensified monitoring
with more frequent electrolyte checks. Patients with high-risk
disease may be prone to lactic acidosis from massive tumor cell
necrosis. Because acidosis inhibits uric acid excretion (43),
prompt recognition and correct of acidosis may prevent or
ameliorate uric acid nephropathy. Additionally, nonsteroidal
anti-inflammatory drugs, iodinated radiocontrast dye, and
other potentially nephrotoxic therapeutic agents should be
avoided to abrogate the risk of AKI from TLS.
Volume expansion
Delivery of crystalloid intravenous fluids (IVFs) is recommended
for all patients and is essential for those with higher TLS risk.
Volume expansion supports adequate intravascular volume and
renal blood flow, which maintain glomerular filtration. This is the
cornerstone of uric acid, potassium, and phosphate excretion and
may delay and prevent the need for renal replacement measures
(2,6,44). High-dose IVFs up to 3 L have been recommended (2),
for a target urine output of $2 mL/kg/h. Diuretics may be necessary if patients develop volume overload, but routine use is not
recommended to avoid volume depletion.
Urinary alkalinization
Alkalinization makes physiologic sense, as increasing urine pH
from 5 to 7 can increase the solubility of uric acid .10-fold
(28). However, urinary alkalinization decreases calcium–
phosphate solubility (2), thereby exacerbating its precipitation
American Society of Nephrology
and deposition. Furthermore, if urinary alkalinization results
in rising serum pH, free calcium may bind albumin more
avidly and further exacerbate hypocalcemia (45). Thus, urinary alkalinization is not recommended in the management of
TLS (2,6,45).
Allopurinol
Allopurinol is converted in vivo to oxypurinol and as a xanthine
analog acts as a competitive inhibitor of xanthine oxidase and
blocks the conversion of purines to uric acid (6,46) (Figure 1).
This prevents hyperuricemia but does not treat preexisting hyperuricemia (6). Furthermore, because oxypurinol also inhibits
the conversion of xanthine to uric acid, serum and urine xanthine levels may rise and precipitate xanthine crystal deposition
in the renal tubules and xanthine-induced obstructive nephropathy (47). Administration of allopurinol is recommended for
prophylaxis in patients with low and intermediate risk of developing TLS (2,6). Smalley et al. (48) studied 1,172 patients to
evaluate the efficacy and safety of intravenous allopurinol in
patients with hyperuricemia. They noted reduced uric acid levels
in 57% of adults and 88% of children. When used as prophylactic
therapy, allopurinol prevented an increase in uric acid levels in
93% of adults and 92% of children.
Because oxypurinol excretion is by the kidney, dose
adjustments are necessary for patients with CKD and AKI.
Allopurinol has been associated with a hypersensitivity syndrome with rash, acute hepatitis, and eosinophilia (45,49).
Allopurinol reduces the clearance of purine-based chemotherapeutic agents such as 6-mercaptopurine and azathioprine
(6). It may also interact with azathioprine and cyclophosphamide in potentiating severe bone marrow suppression (6,45).
Febuxostat
Febuxostat is a novel xanthine oxidase inhibitor lacking the
hypersensitivity profile of allopurinol. Because it is metabolized to
inactive metabolites by the liver, adjustment for reduced GFR is not
necessary. It has been proposed as a viable alternative to allopurinol
in TLS prophylaxis for patients with allopurinol hypersensitivity or
renal dysfunction (45). A recently completed phase III study of
febuxostat versus allopurinol in TLS prevention found significantly lower serum uric acid in the febuxostat but found no significant difference in serum creatinine change compared with
allopurinol (50). Febuxostat use has been limited by its significant
cost compared with generically available allopurinol (45).
to FDA approval, 1,069 adult and pediatric patients received
rasburicase on a compassionate use basis (53). Decreased serum
uric acid levels were observed in 99% of children and 100% of
adults. Hemodialysis was performed in only 2.8% of patients.
In a study of 131 patients with newly diagnosed leukemia or
lymphoma, Pui et al. (54) reported a decrease in plasma uric
acid concentrations from 9.7 to 1 mg/dL (P 5 0.0001) in 65
patients who presented with hyperuricemia and a decrease
from 4.3 to 0.5 mg/dL (P 5 0.0001) in the remaining patients.
There was negligible toxicity, and no patients required dialysis.
Cortes et al. (55) compared response rates in dosing rasburicase alone versus rasburicase followed by allopurinol versus allopurinol alone. They reported a plasma uric acid response rate
of 87% in the rasburicase group, 78% in the rasburicase followed
by allopurinol group, and 66% in the allopurinol group, with a
significantly greater response for rasburicase compared with allopurinol in the overall study population (P 5 0.001), in patients
at high risk for TLS (89% versus 68%; P 5 0.012), and in those
with baseline hyperuricemia (90% versus 53%; P 5 0.015). Of
note, there are no prospective studies to date that have examined
the impact of rasburicase on relevant clinical end points such as
morbidity from AKI. Nonetheless, rasburicase should be used
for prophylaxis in patients with high risk of developing TLS (7).
The FDA-approved dosing guidelines recommend 0.2 mg/kg in
50 mL normal saline as a 30-minute intravenous infusion once
daily for up to 5 days (51). Length of treatment is related to
control of plasma uric acid levels, but use of rasburicase for
.5 days is rarely needed (6,51). In comparison with generically
available allopurinol, rasburicase is significantly more expensive
(up to $3,600 per 7.5-mg vial) (45), and in most published studies, one-time dosing was sufficient to suppress hyperuricemia.
Rasburicase does not require dosing adjustment for GFR
and is not known to have any known clinically relevant drug–
drug interactions (51,56). Adverse reactions are rare but may
include rash, increased liver enzyme levels, headaches, fever,
vomiting, and nausea (56).
Rasburicase is active ex vivo, so blood samples for serum
uric acid levels must be stored on ice to avoid erroneously low
results (45). Patients with glucose 6-phosphate dehydrogenase
(G6PD) deficiency can develop significant methemoglobinemia and hemolysis due to oxidative stress triggered by hydrogen peroxide (57,58). Accordingly, patients should have G6PD
status tested prior to starting rasburicase.
RRT
Rasburicase
Rasburicase (Elitek) is an Aspergillus-derived recombinant urate
oxidase approved by the US Food and Drug Administration
(FDA) in 2002 for the initial management of hyperuricemia in
pediatric patients with leukemia, lymphoma, and solid tumor
malignancies receiving anticancer therapy (51). It was subsequently approved for use in adults in 2009 (51). Rasburicase
catalyzes the conversion of uric acid to allantoin, carbon dioxide,
and hydrogen peroxide (Figure 1). Allantoin is 5- to 10-fold
more soluble than uric acid (52) and is readily excreted. Prior
American Society of Nephrology
The need for renal replacement has significantly reduced since
the advent of rasburicase, but about 1.5% of children and 5% of
adults require dialysis during induction therapies (53). Indications for RRT are similar to those for AKI from other causes,
but due to the rapid onset of the clinical manifestations of TLS,
the threshold for initiating dialytic therapies is lower than in
other situations. Although intermittent hemodialysis (IHD)
may be sufficient for most patients, continuous RRT (CRRT)
at high dialysate or replacement fluid flow rates (.3-4 L/h)
may be necessary in those patients with severe TLS who
Onco-Nephrology Curriculum
5
experience rebound in serum potassium and phosphorous
levels with IHD (45,59,60).
PROGNOSIS
There are many confounding factors that impact clinical
outcomes in patients with malignancies, particularly in those
who have TLS, but AKI appears to be a significant predictor of
short- and long-term mortality from TLS. A study comparing
hematologic cancer patients without AKI to patients with AKI
(61) showed significantly lower hospital mortality (7% and
21%, respectively) and 6-month mortality (51% and 66%, respectively) in patients without AKI. TLS is most common
during initial presentation of disease because relapsed malignancies are significantly more chemoresistant (5). There are
fewer case reports of TLS in recurrent disease (62).
CONCLUSIONS
TLS is a common oncologic emergency that requires immediate diagnosis and prompt treatment to avoid morbidity and
mortality. Understanding the diagnostic criteria for TLS,
knowing the tumor types at high risk for TLS, and instituting
prophylactic and treatment measures are essential for the
nephrologist who treats patients with malignant diseases.
TAKE HOME POINTS
c TLS is the most common oncologic emergency.
c The risk of TLS depends on tumor type but is also influenced by other
factors.
c There is a high burden of AKI in patients with TLS.
c Prophylaxis with volume expansion is the mainstay of preventing TLS in
any patient-risk category.
c Patients at high risk for TLS should receive rasburicase for initial treat-
ment of hyperuricemia.
ACKNOWLEDGMENTS
Dr. Edeani’s work is supported by the intramural Research Program
of the National Institute of Diabetes and Digestive and Kidney Diseases, National Institutes of Health.
REFERENCES
1. Cairo MS, Bishop M. Tumor lysis syndrome: New therapeutic strategies
and classification. Br J Haematol 127: 3–11, 2004
2. Howard SC, Jones DP, Pui C. The tumor lysis syndrome. N Engl J Med
364: 1844–1854, 2011
3. Flombaum CD. Metabolic emergencies in the cancer patient. Semin
Oncol 27: 322–334, 2000
4. Hande KR, Garrow GC. Acute tumor lysis syndrome in patients with
high-grade non-Hodgkin’s lymphoma. Am J Med 94: 133–139, 1993
6
Onco-Nephrology Curriculum
5. Perry Wilson F, Berns JS. Onco-nephrology: Tumor lysis syndrome. Clin
J Am Soc Nephrol 7: 1730–1739, 2012
6. Coiffier B, Altman A, Pui C, Younes A, Cairo MS. Guidelines for the
Management of Pediatric and Adult Tumor Lysis Syndrome: An
evidence-based review. J Clin Oncol 26: 2767–2778, 2008
7. Cairo MS, Coiffier B, Reiter A, Younes A, Baruchel A, Bosly A, Goldman
SC, Leverger G, Ohyashiki K, Panagiotidis P, Pession A, Pui CH, Ribera
JM, Rosti G, Rule S, Tsukimoto I, Zinzani PL. Recommendations for the
evaluation of risk and prophylaxis of tumour lysis syndrome (TLS) in
adults and children with malignant diseases: An expert TLS panel
consensus. Br J Haematol 149: 578–586, 2010
8. Baeksgaard L, Sørenson JB. Acute tumor lysis syndrome in solid tumors:
A case report and review of the literature. Cancer Chemother Pharmacol 51: 187–192, 2003
9. Jensen M, Winkler U, Manzke O, Diehl V, Engert A. Rapid tumor lysis
in a patient with B-cell chronic lymphocytic leukemia and lymphocytosis
treated with an anti-CD20 monoclonal antibody (IDEC-C2B8, rituximab). Ann Hematol 77: 89–91, 1998
10. Cech P, Block JB, Cone LA, Stone A. Tumor lysis syndrome after tamoxifen flare. N Engl J Med 315: 263–264, 1986
11. Al-Kali A, Farooq S, Tfayli A. Tumor lysis syndrome after starting
treatment with Gleevec in a patient with chronic myelogenous leukemia. J Clin Pharm Ther 34: 607–610, 2009
12. Sezer O, Vesole DH, Singhal S, Richardson P, Stadtmauer E, Jakob C, Boral
AL, Esseltine DL, Mehta J. Bortezomib-induced tumor lysis syndrome in
multiple myeloma. Clin Lymphoma Myeloma 7: 233–235, 2006
13. Fuente N, Mane JM, Barcelo R, Muñoz A, Perez-Hoyos T, LopezVivanco G. Tumor lysis syndrome in a multiple myeloma patient treated
with thalidomide. Ann Oncol 15: 537, 2004
14. Mirrakhimov AE, Ali AM, Khan MK, Barbaryan A. Tumor lysis syndrome
in solid tumors: An up to date review of the literature. Rare Tumors 6:
68–76, 2014
15. Gemici C. Tumour lysis syndrome in solid tumors. Clin Oncol 18: 773–
780, 2006
16. Lerza R, Botta M, Barsotti B, Schenone E, Mencoboni M, Bogliolo G,
Pannacciulli I, Arboscello E. Dexamethasone-induced acute tumor lysis
syndrome in a T-cell malignant lymphoma. Leuk Lymphoma 43: 1129–
1132, 2002
17. Tiley C, Grimwade D, Findlay M, Treleaven J, Height S, Catalano J,
Powles R. Tumour lysis following hydrocortisone prior to a blood
product transfusion in T-cell acute lymphoblastic leukemia. Leuk
Lymphoma 8: 143–146, 1992
18. Jabr F. Acute tumor lysis syndrome induced by rituximab in diffuse
large B-cell lymphoma. Int J Hematol 82: 312–314, 2005
19. Simmons ED, Somberg KA. Acute tumor lysis syndrome after intrathecal methotrexate administration. Cancer 67: 2062–2065, 1991
20. Lee CC, Wu YH, Chung SH, et al. Acute tumor lysis syndrome after
thalidomide therapy in advanced hepatocellular carcinoma. Oncologist 11: 87–88, 2006
21. Linck D, Basara N, Tran V, Chen WJ. Peracute onset of severe tumor
lysis syndrome immediately after 4 Gy fractionated TBI as part of reduced intensity preparative regimen in a patient with T-ALL with high
tumor burden. Bone Marrow Transplant. 31: 935–937, 2003
22. Hsieh PM, Hung KC, Chen YS. Tumor lysis syndrome after transarterial
chemoembolization of hepatocellular carcinoma: Case reports and
literature review. World J Gastroenterol 15: 4726–4728, 2009
23. Cohen LF, Balow JE, Magrath IT, Poplack DG, Zeigler JL. Acute tumor
lysis syndrome: A review of 37 patients with Burkitt’s Lymphoma. Am J
Med 68: 486–491, 1980
24. Iversen U, Iversen OH, Bluming AZ, Zeigler JL, Kyalwasi S. Cell kinetics
of African cases of Burkitt lymphoma. A preliminary report. Eur J Cancer
8: 305–308, 1972
25. Jasek AM, Day HJ. Acute spontaneous tumor lysis syndrome. Am J
Hematol 47: 129–131, 1994
26. Mato AR, Riccio BE, Qin L, Heitjan DF, Carroll M, Loren A, Porter DL,
Perl A, Stadtmauer E, Tsai D, Gewirtz A, Luger SM. A predictive model
American Society of Nephrology
27.
28.
29.
30.
31.
32.
33.
34.
35.
36.
37.
38.
39.
40.
41.
42.
for the detection of tumor lysis syndrome during AML induction therapy. Leuk Lymphoma 47: 877–883, 2006
Annemans L, Moeremans K, Lamotte M, Garcia Conde J, van den Berg
H, Myint H, Pieters R, Uyttebroeck A. Incidence, medical resource utilization and costs of hyperuricemia and tumour lysis syndrome in patients with acute leukemia and non-Hogkin’s lymphoma in four
European countries. Leuk Lymphoma 44: 77–83, 2003
Wössman W, Schrappe M, Meyer U, Zimmermann M, Reiter A. Incidence of tumor lysis syndrome in children with advanced stage
Burkitt’s lymphoma/leukemia before and after introduction of prophylactic use of urate oxidase. Ann Hematol 82: 160–165, 2003
Tosi P, Barosi G, Lazzaro C, Lis V, Marchetti M, Morra E, Pession A,
Rosti G, Santoro A, Zinzani PL, Tura S. Consensus conference on the
management of tumor lysis syndrome. Haematologica 93: 1877–
1885, 2008
Montesinos P, Lorenzo I, Martín G, Sanz J, Pérez-Sirvent ML, Martínez
D, Ortí G, Algarra L, Martínez J, Moscardó F, de la Rubia J, Jarque I,
Sanz G, Sanz MA. Tumor lysis syndrome in patients with acute myeloid
leukemia: Identification of risk factors and development of a predictive
model. Haematologica 93: 67–74, 2008
Kushner BH, LaQuaglia MP, Modak S, Cheung NK. Tumor lysis
syndrome, neuroblastoma, and correlation between lactate dehydrogenase levels and MYCN-amplification. Med Pediatr Oncol 41:
80–82, 2003
Fenaux P, Lai JL, Miaux O, Zandecki M, Jouet JP, Bauters F. Burkitt cell
acute leukemia (L3 ALL) in adults: A report of 18 cases. Br J Haematol
71: 371–376, 1989
Seftel MD, Bruyere H, Copland M, Hogge DE, Horsman DE, Nantel SH,
Shepherd JD, Lavoie JC, Le A, Sutherland HJ, Toze CL, Nevill TJ.
Fulminant tumour lysis syndrome in acute myelogenous leukemia with
inv(16)(p13;q22). Eur J Haematol 69: 193–199, 2002
Enomoto A, Kimura H, Chairoungdua A, Shigeta Y, Jutabha P, Cha SH,
Takeda M, Sekine T, Igarashi T, Matsuo H, Kikuchi Y, Oda T, Ichida K,
Hosoya T, Shimokata K, Niwa T, Kanai Y, Endou H. Molecular identification of a renal urate-anion exchanger that regulates blood urate
levels. Nature 417: 447–452, 2002
Umekawa T, Chegini N, Khan SR. Increased expression of monocyte
chemoattractant protein-1 (MCP-1) by renal epithelial cells in culture on
exposure to calcium oxalate, phosphate and uric acid crystals. Nephrol
Dial Transplant 18: 664–669, 2003
Kim YG, Huang XR, Suga S, Mazzali M, Tang D, Metz C, Bucala R,
Kivlighn S, Johnson RJ, Lan HY. Involvement of macrophage migration
inhibitory factor (MIF) in experimental uric acid nephropathy. Mol Med
6: 837–848, 2000
Kang DH, Park SK, Lee IK, Johnson RJ. Uric acid-induced C-reactive
protein expression: Implication on cell proliferation and nitric acid production of human vascular cells. J Am Soc Nephrol 16: 3553–3562, 2005
Han HJ, Lim MJ, Lee YJ, Lee JH, Yang IS, Taub M. Uric acid inhibits renal
proximal tubule cell proliferation via at least two signaling pathways
involving PKC, MAPK, cPLA2 and NF-kappaB. Am J Physiol Renal
Physiol 292: F373–F381, 2007
Feig DI, Kang DH, Johnson RJ. Uric acid and cardiovascular risk. N Engl
J Med 359: 1811–1821, 2008
Davidson MB, Thakkar S, Hix JK, Bhandarkar ND, Wong A, Schreiber
MJ. Pathophysiology, clinical consequences, and treatment of tumor
lysis syndrome. Am J Med 116: 546–554, 2004
Tungsanga K, Boonwichit D, Lekhakula A, Setprija V. Urine uric acid and
urine creatinine ratio in acute renal failure. Arch Intern Med 144: 934–
937, 1984
Cairo MS, Gerrard M, Sposto R, Auperin A, Pinkerton CR, Michon J,
Weston C, Perkins SL, Raphael M, McCarthy K, Patte C; FAB LMB96
International Study Committee. Results of a randomized international
study of high-risk central nervous system B non-Hodgkin lymphoma
and B acute lymphoblastic leukemia in children and adolescents. Blood
109: 2736–2743, 2007
American Society of Nephrology
43. Lieber C, Jones D, Losowsky M, Davidson CS. Interrelation of uric acid
and ethanol metabolism in man. J Clin Invest 41: 1863–1870, 1962
44. Jones DP, Mahmoud H, Chesney W. Tumor lysis syndrome: Pathogenesis and management. Pediatr Nephrol 9: 206–212, 1995
45. Perry Wilson F, Berns JS. Tumor lysis syndrome: New challenges and
recent advances. Adv Chronic Kidney Dis 21: 18–26, 2014
46. Krakoff IH, Meyer RL. Prevention of hyperuricemia in leukemia and
lymphoma: Use of allopurinol, a xanthine oxidase inhibitor. JAMA 193:
1–6, 1965
47. LaRosa C, McMullen L, Bakdash S, Ellis D, Krishnamurti L, Wu HY, Moritz
ML. Acute renal failure from xanthine nephropathy during management of acute leukemia. Pediatr Nephrol 22: 132–135, 2007
48. Smalley RV, Guaspari A, Haase-Statz S, Anderson SA, Cederberg D,
Hohneker JA. Allopurinol: Intravenous use for prevention and treatment of hyperuricemia. J Clin Oncol 18: 1758–1763, 2000
49. Arellano F, Sacristan JA. Allopurinol hypersensitivity syndrome: A review. Ann Pharmacother 27: 337–343, 1993
50. Spina M, Nagy Z, Ribera JM, Spina M, Nagy Z, Ribera FM, Federico M,
Aurer I, Jordan K, Borsaru G, Pristupa AS, Bosi A, Grosicki S, Glushko
NL, Ristic D, Jakucs J, Montesinos P, Mayer J, Rego EM, Baldini S,
Scartoni S, Capriati A, Maggi CA, Simonelli C. A randomized doubleblind phase III pivotal study of febuxostat versus allopurinol in the prevention of tumor lysis syndrome: Florence study. J Clin Oncol 32: 5s, 2014
51. Elitek (Rasburicase) Package Label. Available at: http://www.accessdata.
fda.gov/drugsatfda_docs/label/2009/103946s5083lbl.pdf. Accessed
April 30, 2015
52. Brogard JM, Coumaros D, Franckhauser J, Stahl A, Stahl J. Enzymatic
uricolysis: A study of the effect of a fungal urate-oxydase. Rev Eur
Etudes Clin Biol 17: 890–895, 1972
53. Jeha S, Kantarjian H, Irwin D, Shen V, Shenoy S, Blaney S, Camitta B, Pui
CH. Efficacy and safety of rasburicase (ElitekTM), in the management of a
malignancy-associated hyperuricemia in pediatric and adult patients: Final
results of a multicenter compassionate use trial. Leukemia 19: 34–38, 2005
54. Pui C, Mahmoud HH, Wiley JM, Woods GM, Leverger G, Camitta B,
Hastings C, Blaney SM, Relling MV, Reaman GH. Recombinant urate
oxidase for the prophylaxis or treatment of hyperuricemia in patients
with leukemia or lymphoma. J Clin Oncol 19: 697–704, 2001
55. Cortes J, Moore JO, Maziarz RT, Wetzler M, Craig M, Matous J, Luger S,
Dey BR, Schiller GJ, Pham D, Abboud CN, Krishnamurthy M, Brown A Jr,
Laadem A, Seiter K. Control of plasma uric acid in adults at risk for tumor
lysis syndrome: Efficacy and safety of rasburicase alone and rasburicase
followed by allopurinol compared with allopurinol alone—Results of a
multicenter phase III study. J Clin Oncol 28: 4207–4213, 2010
56. Sood AR, Burry LD, Cheng DKF. Clarifying the role of rasburicase in
tumor lysis syndrome. Pharmacotherapy 27: 111–121, 2007
57. Sonbol MS, Yadav H, Vaidya R, Rana V, Witzig TE. Methemoglobinemia
and hemolysis in a patient with G6PD deficiency treated with rasburicase. Am J Hematol 88: 152–154, 2013
58. Bontant T, Le Garrac S, Avran D, Dauger S. Methaemoglobinaemia in a
G6PD-deficient child treated with rasburicase. BMJ Case Rep 2014
59. Agha-Razii M, Amyot SL, Pichette V, Cardinal J, Ouimet D, Leblanc M.
Continuous veno-venous hemodiafiltration for the treatment of spontaneous tumor lysis syndrome complicated by acute renal failure and
severe hyperuricemia. Clin Nephrol 54: 59–63, 2000
60. Sakarcan A, Quigley R. Hyperphosphatemia in tumor lysis syndrome:
The role of hemodialysis and continuous veno-venous hemofiltration.
Pediatr Nephrol 3: 351–353, 1994
61. Darmon M, Guichard I, Vincent F, Schlemmer B, Azoulay E. Prognostic
significance of acute renal injury in acute tumor lysis syndrome. Leuk
Lymphoma 51: 221–227, 2010
62. Hummel M, Buchheidt D, Reiter S, Bergmann J, Adam K, Hehlmann R.
Recurrent chemotherapy-induced tumor lysis lysis syndrome (TLS) with
renal failure in a patient with chronic lymphocytic leukemia: Successful
treatment and prevention of TLS with low-dose rasburicase. Eur J
Haematol 75: 518–521, 2005
Onco-Nephrology Curriculum
7
REVIEW QUESTIONS
1. Which of the following cancers are considered high risk for
tumor lysis syndrome?
a.
b.
c.
d.
e.
Lung cancer
Lung cancer and patient has AKI
Burkitt-type lymphoma, advanced stage
Adult T-cell lymphoma and normal LDH
ALL with WBC ,100 3 109/L and LDH ,2 3 ULN
Answer: c is correct. As shown in Table 2, the risk of TLS
depends on type of malignancy, stage or extent of disease, and
presence/absence of renal disease. Burkitt-type lymphoma
that is in an advanced stage confers a high risk of TLS. Solid
tumors such as lung cancer are considered low risk, and the
presence of renal failure raises that to intermediate risk. Thus,
answers a and b are incorrect. Adult T-cell lymphoma is considered low risk if LDH is normal and acute lymphoblastic
leukemia with WBC ,100 3 109/L is considered intermediate
risk if LDH ,2 3 ULN.
2. Which of the following electrolyte abnormalities define
laboratory TLS?
a. Hypokalemia
b. Hypercalcemia
c. Hypophosphatemia
8
Onco-Nephrology Curriculum
d. Hypernatremia
e. Hypocalcemia
Answer: e is correct. As show in Table 2, laboratory TLS is
defined by two or more abnormalities in serum electrolytes.
These include a 25% increase from baseline in phosphorous,
potassium, or uric acid or a 25% decrease from baseline in calcium. Thus, answers a, b, and c are incorrect. Serum sodium
concentration is not directly affected in TLS; therefore, answer
d is incorrect.
3. Rasburicase is part of the treatment regimen for tumor lysis
syndrome because
a. It increases urinary alkalinization
b. It improves the ability of proximal tubular cells to recover
from AKI
c. It stimulates the URAT1 transporter to increase uptake of
uric acid from the tubular lumen
d. It catalyzes the conversion of uric acid into allantoin
e. It prevents xanthine crystal deposition in tubular lumens
Answer: d is correct. Rasburicase, as shown in Figure 3, is
recombinant urate oxidase that enzymatically transforms uric
acid into allantoin. It has no known effect on urine pH, renal
tubular cells, URAT1 transporters, or xanthine crystals. Thus,
answers a–c and e are incorrect.
American Society of Nephrology
Chapter 5: Electrolyte and Acid–Base Disorders in
Malignancy
Anushree C. Shirali, MD
Section of Nephrology, Yale University School of Medicine, New Haven, Connecticut
INTRODUCTION
Renal complications in cancer patients include AKI,
hypertension, or electrolyte and acid–base disorders.
Of the latter, there are various types that share the
ability to increase morbidity and mortality, delay
treatment, and decrease quality of life. Understanding the etiology of electrolyte and acid–base abnormalities in cancer patients is critical to prompt
recognition and appropriate treatment so that these
complications may be avoided. This section of the
Onco-Nephrology Curriculum will review the pathophysiology, clinical presentation, and management
of electrolyte and acid–base abnormalities in patients
with malignancies. Specifically, disturbances in the
following will be reviewed: 1) electrolytes: disorders
of sodium, potassium, calcium, magnesium, and
phosphorous; and 2) acid–base: metabolic acidosis.
BACKGROUND, EPIDEMIOLOGY, AND
SPECIFIC DISORDERS
Electrolyte and acid–base disturbances are common in cancer patients, either due to the malignancy or treatment of the malignancy. For
example, a patient may develop metabolic acidosis
from lactate produced by disseminated lymphoma or from chemotherapy-induced diarrhea.
Published statistics are not robust for each type of
electrolyte or acid–base disorder, but there are data
on those associated with greater morbidity or mortality, such as hyponatremia. In one such analysis of
cancer-related admissions, 47% of patients with
mostly solid tumors had hyponatremia, and of
these, 11% had moderate (sodium [Na] 120–129
mEq/L) to severe (Na , 120 mEq/L) hyponatremia
(1). This is disproportionately higher than the
hyponatremia prevalence rates of 15%–30%
reported for general medicine admissions (2). In
cancer patients with nonsolid tumors, rates of
hyponatremia are less. For example, in acute
American Society of Nephrology
leukemia, the prevalence of hyponatremia is only
10%, whereas the prevalence of hypokalemia
ranges between 43% and 64% (3). This suggests
that differences in pathophysiologic mechanisms
may drive unique electrolyte disorders in different
malignancies. In the next sections, the clinical features, pathophysiology, and treatment of the most
common electrolyte and acid–base disorders in cancer patients will be considered.
Hyponatremia
Cancer is a common etiology for hyponatremia in the
hospitalized patient, accounting for 14% of cases in a
prospective observational cohort (4). Similar to reports on hyponatremia in the general population,
lower serum sodium concentration is associated
with increased hospital length of stay and 90-day
mortality (1). In patients with small-cell lung cancer
(SCLC), in those who had hyponatremia prior to chemotherapy initiation, failure to achieve normonatremia
within the first two cycles of chemotherapy was a
predictive marker for decreased survival (5).
Hyponatremia associated with cancer may have
several potential etiologies (Table 1). Regardless of the
etiology, patients may be asymptomatic with mild to
moderate disease but may experience headache, fatigue, and mental status changes with moderate to
severe hyponatremia. Examination findings such as
frank or orthostatic hypotension in volume depletion
or edema in the third-spacing states of cirrhosis may
point to potential causes. In conjunction with examination data, urine studies are indispensable, with
urine sodium ,20 mEq/L reflecting the sodium avidity of volume depletion and urine sodium .40 mEq/L
suggesting the syndrome of inappropriate antidiuretic
Correspondence: Anushree Shirali, Section of Nephrology, Yale
University School of Medicine, PO Box 208029, New Haven,
Connecticut 06520-8029.
Copyright © 2016 by the American Society of Nephrology
Onco-Nephrology Curriculum
1
Table 1. Common mechanisms for hyponatremia in the cancer patient
Etiology of hyponatremia
Pseudohyponatremia
Reduced water excretion
Decreased circulating volume
Decreased effective circulating volume
SIADH
Nonosmotic stimuli for ADH
Salt wasting
Clinical examples specific to the cancer patient
Paraproteinemias
Underlying CKD or AKI
Nausea, vomiting, nasogastric suctioning, and diarrhea;
hematemesis or hematochezia (gastrointestinal malignancies
or steroid-induced ulcer disease)
Underlying or new onset of CHF, cirrhosis, ascites, severe
hypoalbuminemia, veno-occlusive disease
Tumor release of ADH: SCLC and head and neck cancer
Chemotherapy: cyclophosphamide, cisplatin/carboplatin, vincristine,
vinblastine
Other drugs: SSRIs, NSAIDs
Pain, nausea, vomiting
Cisplatin
ADH, antidiuretic hormone; CHF, congestive heart failure; SIADH, syndrome of inappropriate ADH secretion; NSAIDs, nonsteroidal anti-inflammatory drugs; SCLC,
small cell lung cancer; SSRI, selective serotonin reuptake inhibitors.
hormone secretion (SIADH) in a euvolemic patient or rarely salt
wasting due to cisplatin therapy.
Although patients with cancer have hyponatremia due to
many etiologies (Table 1), SIADH is the most common etiology that is directly attributable to cancer. This is because cancer patients have nonvolume and nonosmotic stimuli for
antidiuretic hormone (ADH) release, such as nausea/vomiting
and pain or medications such as cyclosphosphamide. An additional factor is the paraneoplastic release of ADH from tumor subtypes, notably SCLC and head and neck cancer (6).
Finally, several chemotherapy drugs have been linked to
hyponatremia via potentiation of ADH release or action, including vinblastine, vincristine, and cyclophosphamide (7).
Cisplatin is another antineoplastic agent linked to
hyponatremia through a mechanism that involves salt wasting
at the loop of Henle (8). Ten percent of patients on cisplatin
therapy at a single center developed hyponatremia with a high
urine sodium concentration but with profound volume depletion. Thus, these patients required volume expansion with
saline to correct their hyponatremia, in contrast to patients
with SIADH who are volume replete. This underscores the
need for correct etiologic diagnosis of hyponatremia to provide appropriate therapy. Therapy must be tailored for the
patient, the underlying diagnosis, and the severity of hyponatremia. Severe hyponatremia with serum Na concentration
,110 mEq/L and neurologic symptoms may need 3% hypertonic saline for acute management. Fluid restriction is the
mainstay of treatment for SIADH, with salt tablets and loop
diuretics as adjunctive therapy. However, these measures can
hinder quality of life in the cancer patient, and thus, aquaretics
that inhibit the vasopressin type 2 receptor (V2-R) to inhibit
water reabsorption in the collecting duct are suggested for
management of hyponatremia secondary to SIADH in cancer
patients if other therapies are not feasible or effective. However, the data on their clinical utility in cancer patients are sparse.
In a small, single center safety and efficacy study, tolvaptan, a
V2-R antagonist, was superior to placebo in the correction of
hyponatremia but did not decrease hospital length of stay (LOS)
2
Onco-Nephrology Curriculum
or improve cognitive testing (9). In addition, chronic tolvaptan
use may be limited by expense and cumulative dose-dependent
hepatotoxicity (10).
Hypokalemia
Similar to hyponatremia, hypokalemia is commonly encountered in cancer patients, resulting from cancer-distinct and
cancer-specific causes (Table 2) and, more commonly, from a
combination of the two. Proper diagnosis starts with excluding
pseudohypokalemia from postphlebotomy transcellular
shifts, which is seen in patients with profound leukocytosis
whose blood samples are not refrigerated or immediately analyzed. Once true hypokalemia is confirmed, measurement of
urine potassium and the trans-tubular potassium gradient can
be helpful in analyzing renal potassium wasting (11).
In cancer-distinct causes, chemotherapy leads to hypokalemia either indirectly via side effects of decreased appetite/
intake, vomiting, and diarrhea or directly via renal tubular
effects. For example, ifosfamide causes renal potassium
wasting, either as an isolated proximal tubulopathy or Fanconi
syndrome (FS), which may persist after treatment. Fifteen
percent of pediatric cancer patients who received ifosfamide
therapy exhibited persistent hypokalemia months to years after
the end of treatment (12).
Cancer-specific causes of hypokalemia include tumors that
secrete ectopic adrenocorticotropin hormone (ACTH) such as
SCLC, thymus or bronchial carcinoid, thyroid medullary
carcinoma, or neuroendocrine tumors (13). Although uncommon, these tumors stimulate renal potassium wasting
via excessive cortisol release that activates the mineralocorticoid
pathway. Accordingly, other features of hypercortisolemia
are also present including pigmented skin, diabetes, and hypertension (13). Another cancer-specific etiology for hypokalemia is evident in acute myeloid leukemia (AML), M4 and M5
subtypes, which has been long associated with hypokalemia
(14,15). These malignancies increase serum lysozyme
and lysozymuria, leading to the hypothesis that lysozymemediated tubular injury leaks potassium (and other electrolytes)
American Society of Nephrology
Table 2. Cancer-distinct and cancer-specific causes of hypokalemia in the patient with malignancy
Etiology of hyponatremia
Pseuodohypokalemia
Redistribution into cells
Poor intake
Extrarenal losses
Renal losses
Cancer distinct
Cancer specific
Phlebotomy error with tight tourniquet
Use of GM-CSF, vitamin B12
Anorexia, nausea, mucositis
Vomiting, diarrhea from chemotherapy or radiation enteritis
Hypomagnesemia, Fanconi syndrome with chemotherapy
Clonal leukocytosis with leukemias
Blast crisis with leukemias
Tumor-induced dysphagia
VIPoma, villous adenoma (rare)
Lysozymuria, Fanconi syndrome from light chain
injury (myeloma), ectopic ACTH production
ACTH, adrenocorticotropin hormone; GM-CSF, granulocyte macrophage colony stimulating factor; VIP, vasoactive intestinal peptide.
into urine. Another putative mechanism may be renin-like
activity by AML blast cells stimulating the mineralocorticoid
pathway (16).
The potassium losses in these cases may be profound and
require aggressive replacement. The choices for replacement
are the same as those utilized for hypokalemia in the noncancer
patient (11), but it should be noted that given the difficulty
cancer patients may have with oral intake due to nausea,
mucositis, etc., intravenous dosing is often necessary. Hypokalemia treatment is also ineffective if hypomagnesemia remains uncorrected, due to unchecked potassium losses via the
renal outer medullary K1 channel (ROMK) channel in distal
nephron tubular cells (17).
panitumumab, display tumoricidal activity against a variety of
cancers, but they also prevent the insertion of a magnesium
channel, transient receptor potential M6 (TRPM6), into the
apical membrane of distal tubular cells (Figure 1) (18). As a result, magnesium cannot be reabsorbed from the tubular lumen
and serum magnesium levels fall, affecting 10%–36% of patients
in early clinical trials of cetuximab (7). A fractional excretion of
magnesium .15% in a hypomagnesemic patient indicates renal
wasting. Treatment involves replacing magnesium, and intravenous dosing is usually needed because diarrhea is a dose-limiting
adverse effect of oral magnesium. Fortunately, renal magnesium
wasting subsides over time following discontinuation of the
EGFR antagonist. However, this is not the case with the platin
drugs, where renal magnesium wasting can be permanent.
Hypomagnesemia in cancer patients
Hypomagnesemia in the cancer patient may be due to decreased
intake or from renal magnesium wasting. Renal losses of
magnesium are principally due to chemotherapy-mediated
injury to the distal nephron, the site of active magnesium
reabsorption in the nephron. This has been noted with cisplatin,
but a rising number of cases are being attributed to drugs that
target the epidermal growth factor receptor (EGFR) pathway.
Monoclonal antibodies against EGFR, such as cetuximab and
Hypercalcemia
Twenty percent to 30% of cancer patients experience hypercalcemia during the course of their malignancy (19), and this is
predictive of poor prognosis (20). Hypercalcemia of malignancy
uses one of two mechanisms: 1) osteolytic release of local calcium from bone directly involved by cancer cells or 2) stimulation of osteoclast activity by release of the tumor-derived
endocrine factors. Although these mechanisms are distinct,
Figure 1. Absorption of magnesium from the tubular lumen is via an EGFR-dependent apical channel, TRPM6. This pathway is
inhibited by use of anti-EGFR monoclonal antibodies such cetuximab. EGF, epidermal growth factor; EGFR, epidermal growth factor
receptor; TRPM6, transient receptor potential M6. Red line denotes inhibition of interaction.
American Society of Nephrology
Onco-Nephrology Curriculum
3
the resultant hypercalcemia in either case may be mild and
asymptomatic, moderate and accompanied by nausea/vomiting,
constipation, bone pain, and fatigue, or severe and manifested by
confusion and coma (21). It is important to correct serum calcium concentration for hypoalbuminemia so that hypercalcemia
levels can be properly graded.
Among solid tumors, primary bone cancers and metastatic
breast or prostate cancer stimulate osteolysis, which correlates
with the overall tumor burden. Although metastases do not
occur in nonsolid tumors, osteolysis may be stimulated by a
variety of immune and nonimmune pathways in multiple
myeloma. Both result in release of sequestered calcium from
bone with the common pathway centered on the interaction
between receptor activator of nuclear factor-kB (RANK),
which is present on osteoclasts and their precursors, and
RANK ligand (RANKL), which is present on osteoblast and
bone marrow stromal cell surfaces (22). The putative mechanism involves RANKL binding to its cognate receptor RANK
through the influence of parathyroid hormone (PTH) and
PTH-related peptide (PTHRP), which subsequently increases
osteoclastic activity and release of local calcium (21).
Tumor-derived endocrine factors are responsible for the
humoral hypercalcemia of malignancy, including PTHRP and
1,25-dihydroxyvitamin D [1,25(OH)2D]. More rarely, there is
PTH release from primary parathyroid carcinoma (23) or ovarian
cancer (24). PTHRP is most commonly secreted by squamous cell
carcinoma of the lung or head and neck, but renal cell, ovarian,
breast, and esophageal cancers have all been associated with hypercalcemia from PTHRP release (21). 1,25(OH)2D, however, is
more likely to be secreted by lymphoma cells or tumor-associated
macrophages that possess inherent 1-a-hydroxylase activity that
is not subject to regulation by PTH (25–27).
Treatment of hypercalcemia of malignancy is focused on
increasing urinary calcium excretion and suppression of the
calcium source. The first objective is achieved by volume
expansion with saline to drive urinary calcium excretion.
Furosemide, once routinely touted as an adjunct to saline, has
no proven benefit and should only be reserved for cases of volume
overload (28). The second objective may be fulfilled by suppressing
osteoclast activity through use of bisphosphonates such as
zoledronate or pamidronate. The former causes acute renal
tubular injury, and the latter has been linked to a collapsing
variant of FSGS, and high intravenous (IV) dosing should be
used with caution, particularly with preexisting CKD. An
emerging option that directly targets a pathway in hypercalcemia of malignancy is the use of RANKL inhibitors such as
denosumab. These agents have shown to be superior to
bisphosphonates in the treatment of skeletal related events in
cancers with bony metastases (29,30), and early evidence suggests they are useful in the treatment of hypercalcemia of
malignancy, particularly in bisphosphonate-resistant cases (31).
Hypophosphatemia in cancer patients
Cancer patients usually experience hypophosphatemia as a consequence of chemotherapy. This may be due to generalized
4
Onco-Nephrology Curriculum
malnutrition from anorexia or malnutrition causing poor intake,
or it may be the result of renal phosphate wasting from druginduced proximal tubulopathy and FS. As mentioned previously,
FS is common with ifosfamide, but has also been associated with
cisplatin and imatinib use (32,33). A fractional excretion of phosphate that is .5% in the setting of hypophosphatemia is diagnostic of renal phosphate wasting. Treatment of hypophosphatemia
centers on phosphate replacement, which may approach several
grams per day in cases of renal phosphate wasting.
A rare cause of hypophosphatemia is tumor-induced osteomalacia, whose mechanism is dependent on the phosphatonin,
fibroblast growth factor 23 (FGF-23). The role of the FGF-23
pathway has been detailed previously (6,34). Briefly, FGF-23 is a
phosphaturic agent whose expression is tightly regulated by
phosphate, 1,25(OH)2D, and other factors. In tumor-induced
osteomalacia, constitutive release of FGF-23 without usual
regulatory checkpoints leads to persistent FGF-23 activation,
resulting in severe phosphaturia, hypophosphatemia, and osteomalacia. Several malignancies are associated with this syndrome
including hemangiopericytomas, giant cell tumors, and osteoblastomas (34). Definitive treatment is surgical resection, as the
phosphate wasting may be so profound that medical management may not be sufficient. Thus, functional imaging such as
F-18 fluorodeoxyglucose positron emission tomography is
suggested for diagnosis, which has high sensitivity for these
tumors but may not be specific (34).
Metabolic acidosis in cancer patients
Anion gap (AG) acidosis and non–anion gap (NAG) acidosis is
prevalent in cancer patients. Among the various AG acidosis
disorders, lactic acidosis (LA) is the most cancer specific. Cancer patients may have type A LA due to tissue hypoxia from
sepsis or cardiac failure, but they may also have type B LA with
no evidence for tissue ischemia. Type B LA is well described in
leukemias and lymphomas (35), but other reported cases include multiple myeloma, gastric cancer, and breast cancer
(36–38). The pathophysiology of malignancy-associated LA
is unclear, but speculative mechanisms include anaerobic
glycolysis by tumor cells, stimulation of lactate production by
tumor-derived cytokines, and thiamine deficiency (36). Treatment involves control of tumor burden. Bicarbonate infusion
may be necessary for critical drops in serum pH, but paradoxically may stimulate more lactate production. Dialysis is often
requested for lactic acidosis, but clearance with either intermittent or continuous dialysis modalities is insufficient to
overcome ongoing production.
Non-AG acidosis in cancer patients is most likely related
to infection or therapy-related diarrhea, but renal tubular
acidosis (RTA) should be considered. Tubular injury from
chemotherapy can cause RTA either in isolation or as part of
the FS. Light chain–associated tubular injury in multiple myeloma is another cause of FS and can present with RTA. Bicarbonate supplementation is sometimes necessary in patients
with RTAs, and its success depends on the degree of renal bicarbonate wasting.
American Society of Nephrology
Other disorders
Cancer patients may have electrolyte and acid–base abnormalities beyond those reviewed in this chapter. In particular,
hyperkalemia, hyperphosphatemia, and hypocalcemia are diagnostic criteria for tumor lysis syndrome, which is detailed
in Chapter 4 of the ASN Onco-Nephrology Curriculum. In
acid–base disorders, metabolic alkalosis may accompany the
rare renin-producing tumor but is more common with persistent vomiting or diuretic use. Respiratory alkalosis may
occur with pontine tumor or infection-associated stimulation
of central respiratory centers.
10.
11.
12.
13.
14.
CONCLUSION
15.
A myriad of cancer and chemotherapy-related electrolyte
and acid-base disorders can affect cancer patients. Although
most patients develop mild disease, some patients may
experience significant morbidity. Diagnosing electrolyte
and acid–base abnormalities and initiating treatment quickly
is essential for the nephrologist seeking to improve the outcomes of cancer patients.
16.
17.
18.
TAKE HOME POINTS
c Electrolyte and acid–base abnormalities occur frequently in the cancer pa-
tient and contribute to poor quality of life.
19.
c Disturbances in electrolyte and acid–base homeostasis may occur due to the
cancer itself or due to adverse effects of therapy.
20.
c Treatment of electrolyte and acid–base disorders in cancer should be etiology
specific and patient centered.
21.
REFERENCES
Q:1
1. Doshi SM, Shah P, Lei X, Lahoti A, Salahudeen AK. Hyponatremia in
hospitalized cancer patients and its impact on clinical outcomes. Am J
Kidney Dis 59: 222–228, 2012
2. Verbalis JG, Goldsmith SR, Greenberg A, Korzelius C, Schrier RW,
Sterns RH, Thompson CJ. Diagnosis, evaluation, and treatment of
hyponatremia: expert panel recommendations. Am J Med 126[Suppl 1]:
S1–S42, 2013
3. Filippatos TD, Milionis HJ, Elisaf MS. Alterations in electrolyte equilibrium in patients with acute leukemia. Eur J Haematol 75: 449–460,
2005
4. Gill G, Huda B, Boyd A, Skagen K, Wile D, Watson I, van Heyningen C.
Characteristics and mortality of severe hyponatraemia–a hospitalbased study. Clin Endocrinol 65: 246–249, 2006
5. Hansen O, Sorensen P, Hansen KH. The occurrence of hyponatremia in SCLC
and the influence on prognosis: a retrospective study of 453 patients treated
in a single institution in a 10-year period. Lung cancer 68: 111–114, 2010
6. Rosner MH, Dalkin AC. Electrolyte disorders associated with cancer.
Adv Chronic Kidney Dis 21: 7–17, 2014
7. Shirali AC, Perazella MA. Tubulointerstitial injury associated with chemotherapeutic agents. Adv Chronic Kidney Dis 21: 56–63, 2014
8. Hutchison FN, Perez EA, Gandara DR, Lawrence HJ, Kaysen GA. Renal
salt wasting in patients treated with cisplatin. Ann Intern Med 108:
21–25, 1988
9. Salahudeen AK, Ali N, George M, Lahoti A, Palla S. Tolvaptan in hospitalized cancer patients with hyponatremia: a double-blind, randomized,
American Society of Nephrology
22.
23.
24.
25.
26.
27.
28.
placebo-controlled clinical trial on efficacy and safety. Cancer 120:
744–751, 2014
Administration USFaD. FDA Drug Safety Communication: FDA limits
duration and usage of Samsca (tolvaptan) due to possible liver injury
leading to organ transplant or death 2013. Available from: http://www.
fda.gov/Drugs/DrugSafety/ucm350062.htm
Unwin RJ, Luft FC, Shirley DG. Pathophysiology and management of
hypokalemia: a clinical perspective. Nat Rev Nephrol 7: 75–84, 2011
Skinner R, Cotterill SJ, Stevens MC. Risk factors for nephrotoxicity after
ifosfamide treatment in children: a UKCCSG Late Effects Group study.
United Kingdom Children’s Cancer Study Group. Br J Cancer 82:
1636–1645, 2000
Alexandraki KI, Grossman AB. The ectopic ACTH syndrome. Rev
Endocr Metab Disord 11: 117–126, 2010
Perazella MA, Eisen RN, Frederick WG, Brown E. Renal failure and severe hypokalemia associated with acute myelomonocytic leukemia.
Am J Kidney Dis 22: 462–467, 1993
Muggia FM, Heinemann HO, Farhangi M, Osserman EF. Lysozymuria
and renal tubular dysfunction in monocytic and myelomonocytic
leukemia. Am J Med 47: 351–366, 1969
Wulf GG, Jahns-Streubel G, Strutz F, Basenau D, Hufner M, Buske C,
Wormann B, Hiddemann W. Paraneoplastic hypokalemia in acute myeloid
leukemia: a case of renin activity in AML blast cells. Ann Hematol 73:
139–141, 1996
Huang CL, Kuo E. Mechanism of hypokalemia in magnesium deficiency. JASN 18: 2649–2652, 2007
Groenestege WM, Thebault S, van der Wijst J, van den Berg D,
Janssen R, Tejpar S, van den Heuvel LP, van Cutsem E, Hoenderop JG,
Knoers NV, Bindels RJ. Impaired basolateral sorting of pro-EGF causes
isolated recessive renal hypomagnesemia. J Clin Invest 117:
2260–2270, 2007
Stewart AF. Clinical practice. Hypercalcemia associated with cancer.
N Engl J Med 352: 373–379, 2005
Ralston SH, Gallacher SJ, Patel U, Campbell J, Boyle IT. Cancerassociated hypercalcemia: morbidity and mortality. Clinical experience
in 126 treated patients. Ann Intern Med 112: 499–504, 1990
Rosner MH, Dalkin AC. Onco-nephrology: the pathophysiology
and treatment of malignancy-associated hypercalcemia. CJASN 7:
1722–1729, 2012
Papadopoulou EC, Batzios SP, Dimitriadou M, Perifanis V, Garipidou V.
Multiple myeloma and bone disease: pathogenesis and current therapeutic approaches. Hippokratia 14: 76–81, 2010
Collins MT, Skarulis MC, Bilezikian JP, Silverberg SJ, Spiegel AM, Marx
SJ. Treatment of hypercalcemia secondary to parathyroid carcinoma
with a novel calcimimetic agent. J Clin Endocrinol Metab 83:
1083–1088, 1998
Nussbaum SR, Gaz RD, Arnold A. Hypercalcemia and ectopic secretion
of parathyroid hormone by an ovarian carcinoma with rearrangement
of the gene for parathyroid hormone. N Engl J Med 323: 1324–1328,
1990
Maletkovic J, Isorena JP, Palma Diaz MF, Korenman SG, Yeh MW.
Multifactorial hypercalcemia and literature review on primary hyperparathyroidism associated with lymphoma. Case Rep Endocrinol 2014:
893134, 2014
Evans KN, Taylor H, Zehnder D, Kilby MD, Bulmer JN, Shah F, Adams
JS, Hewison M. Increased expression of 25-hydroxyvitamin D-1alphahydroxylase in dysgerminomas: a novel form of humoral hypercalcemia
of malignancy. Am J Pathol 165: 807–813, 2004
Hewison M, Kantorovich V, Liker HR, Van Herle AJ, Cohan P, Zehnder
D, Adams JS. Vitamin D-mediated hypercalcemia in lymphoma: evidence for hormone production by tumor-adjacent macrophages.
J Bone Miner Res 18: 579–582, 2003
LeGrand SB, Leskuski D, Zama I. Narrative review: furosemide for hypercalcemia: an unproven yet common practice. Ann Intern Med 149:
259–263, 2008
Onco-Nephrology Curriculum
5
29. Fizazi K, Carducci M, Smith M, Damiao R, Brown J, Karsh L, Milecki P,
Shore N, Rader M, Wang H, Tadros S, Dansey R, Goessl C. Denosumab
versus zoledronic acid for treatment of bone metastases in men with
castration-resistant prostate cancer: a randomised, double-blind study.
Lancet 377: 1813–822, 2011
30. Stopeck AT, Lipton A, Body JJ, Steger GG, Tonkin K, de Boer RH,
Lichinitser M, Fujiwara Y, Yardley DA, Viniegra M, Fan M, Jiang Q,
Dansey R, Jun S, Braun A. Denosumab compared with zoledronic acid
for the treatment of bone metastases in patients with advanced breast
cancer: a randomized, double-blind study. J Clin Oncol 28: 5132–5739,
2010
31. Hu MI, Glezerman I, Leboulleux S, Insogna K, Gucalp R, Misiorowski W,
Yu B, Ying W, Jain RK. Denosumab for patients with persistent or relapsed hypercalcemia of malignancy despite recent bisphosphonate
treatment. J Natl Cancer Inst 105: 1417–1420, 2013
32. Hall AM, Bass P, Unwin RJ. Drug-induced renal Fanconi syndrome. QJM
107: 261–269, 2014
6
Onco-Nephrology Curriculum
33. Francois H, Coppo P, Hayman JP, Fouqueray B, Mougenot B, Ronco P.
Partial fanconi syndrome induced by imatinib therapy: a novel cause of
urinary phosphate loss. Am J Kidney Dis 51: 298–301, 2008
34. Chong WH, Molinolo AA, Chen CC, Collins MT. Tumor-induced
osteomalacia. Endocr Relat Cancer 18: R53–R77, 2011
35. Friedenberg AS, Brandoff DE, Schiffman FJ. Type B lactic acidosis as a
severe metabolic complication in lymphoma and leukemia: a case series
from a single institution and literature review. Medicine 86: 225–232, 2007
36. Sia P, Plumb TJ, Fillaus JA. Type B lactic acidosis associated with
multiple myeloma. Am J Kidney Dis 62: 633–637, 2013
37. de Groot R, Sprenger RA, Imholz AL, Gerding MN. Type B lactic acidosis as a
severe metabolic complication in lymphoma and leukemia: a case series
from a single institution and literature review.nger RA, Imholz AL, Gerding
MN. Type B lactic acidosis in solid malignancies. Neth J Med 69: 120–123,
2011
38. Hashemi-Sadraei N, Machicado JD, Gupta R, Huapaya JA. Lactic acidosis
in gastric cancer. J Gastrointest Cancer 45[Suppl 1]: 192–194, 2014
American Society of Nephrology
REVIEW QUESTIONS
1. Which of the following tumors is most likely to be associated with SIADH?
a.
b.
c.
d.
e.
Non–small-cell lung cancer
Acute myeloid leukemia, M4 type
Small-cell lung cancer
Breast cancer
Bronchial carcinoid
Answer: c is correct. Hyponatremia in cancer patients from
persistent ADH stimulation or potentiation may be cancer
distinct, e.g., from pain, nausea, or chemotherapy. Certain
cancers, however, are known to release ectopic ADH, including small cell lung cancer and head and neck cancer. Non–
small cell lung cancer and breast cancer are not associated
with SIADH from ectopic ADH release. AML-M4 is associated with lysozyme-mediated renal potassium wasting,
whereas bronchial carcinoid secretes ectopic adrenocorticotropin hormone.
2. Which of the following laboratory abnormalities are seen
in Fanconi syndrome?
a.
b.
c.
d.
e.
Hyperkalemia
Hypomagnesemia
Hypophosphatemia
Metabolic alkalosis
Hyperglycemia
Answer: c is correct. Fanconi syndrome is a constellation of
metabolic abnormalities, which result following proximal tubule
injury. As the proximal tubule is the primary site for
American Society of Nephrology
reabsorption of bicarbonate, ammonia, amino acids, glucose,
and most electrolytes including sodium, potassium, and phosphorus, this syndrome typically results in potassium, phosphorus, and bicarbonate wasting. Patients may exhibit hypokalemia,
hypophosphatemia, metabolic acidosis due to renal tubular acidosis, and glucosuria. Because active magnesium reabsorption
takes place at the distal tubule, magnesium levels are not affected.
3. Which of the following is true of the lactic acidosis of
malignancy?
a. Levels of lactic acid decrease with bicarbonate infusion
b. Continuous RRT is recommended for control of lactic
acid levels
c. Measurement of the urinary anion gap can aid in diagnosis
d. Control of tumor burden is not necessary in improving
lactic acid levels
e. It may be the result of anaerobic glycolysis and lactate
production by tumor cells
Answer: e is correct. Type B lactic acidosis is the production
of lactic acid in the absence of ischemia. It is linked with certain malignancies, particularly lymphoma, and hypotheses
for its pathophysiology include tumor-induced anaerobic glycolysis and lactate production. Lactic acidosis of malignancy
correlates well with tumor burden, and levels improve with
control of that burden. The urinary anion gap is not useful in
the diagnosis of this disorder. Although acute treatment of acidosis may require bicarbonate, levels of lactic acid may rise with
sustained bicarbonate infusion. Last, hemodialysis is an inefficient modality for lactic acid clearance, as production is higher
than clearance rates, even in continuous RRT.
Onco-Nephrology Curriculum
7
AUTHOR QUERIES
AUTHOR PLEASE ANSWER ALL QUERIES
Q1: Please verify corrections, especially to references. The end of the references were changed to match
style, so please make sure the edits are correct.
Chapter 6: Glomerular Diseases and Cancer
Divya Monga* and Kenar D. Jhaveri†
*Nephrology Division, University of Mississippi Medical Center, Jackson, Mississippi; and †Nephrology Division,
Northwell Health, Hofstra Northwell School of Medicine, Great Neck, New York
INTRODUCTION
Glomerular diseases are associated with many solid
and hematologic malignancies. Additionally, many chemotherapeutic agents and post–stem cell transplant–
associated glomerular lesions have been described.
These glomerular lesions are most likely due to
abnormal products produced by tumor cells, although the exact pathogenesis is unclear. The treatment of these cancer-associated glomerulopathies
is primarily targeted at treating the underlying
malignancy. This chapter will review glomerular
diseases associated with cancer, chemotherapy,
and hematopoietic stem cell transplantation
(HSCT).
SOLID TUMOR–ASSOCIATED
GLOMERULAR DISEASES
Membranous nephropathy
Membranous nephropathy (MN) is the most common glomerular pathology (Figure 1, A and B)
described in patients with solid tumors (1,2).
In a review of 240 patients with biopsy proven
MN, Lefaucheur et al. (3) reported a prevalence
of malignancy of 10%. Only about half of these
patients had symptoms related to cancer at the
time of their kidney biopsy. Also, most of these
patients were diagnosed with malignancy within
a year of MN diagnoses. Review of case series
shows a reported prevalence of as low as 1% to as
high as 22% (2).
Classically, the solid tumors most commonly
associated with MN are lung, bronchus, and gastric
cancers, followed by renal cell, prostate, and
thymoma (2). Other cancers reported with MN
are colorectal, pancreatic, esophageal, and hepatic
carcinomas.
Differentiating primary MN from secondary
MN associated with malignancy can be difficult.
Our suspicion for a secondary glomerular disease
American Society of Nephrology
should be high in a patient with known cancer who
has presence of proteinuria or nephrotic syndrome.
Also development of proteinuria within a year of
diagnosis of cancer should raise the suspicion of
secondary form of glomerulopathy. Studies have
reported risk factors like age .65 years and history
of smoking for .20 pack-years for paraneoplastic
MN (3). Review of relevant studies (3–7) has suggested certain parameters, which can help differentiate primary from secondary MN, the latter being
associated with cancer. These features are summarized in Table 1 (8).
In addition to these findings, one should have a
high index of suspicion for malignancy when a
patient with MN is evaluated. It is reasonable to
perform routine age- and sex-appropriate screening for malignancy, once other known causes of
secondary MN have been excluded. In patients with
high risk of lung cancer, low-dose chest computed
tomography should be considered. The risk of
cancer persists for $5 years from the time of kidney biopsy (9). This is most likely due to slowgrowing malignancy, use of cytotoxic therapy for
MN, or increased surveillance. Therefore, close
medical follow-up is needed even if the cancer is
not detected on initial screening at the time of
MN diagnosis.
The possible mechanisms by which solid tumors
may be associated with MN include the following
(10):
1) In situ immune complex formation: Antibodies
are formed against a tumor antigen, which is
localized in the sub epithelial location or to a
podocyte antigen that is identical or similar to
the tumor antigen.
Correspondence: Divya Monga, Division of Nephrology, University of Mississippi Medical Center, 2500 N. State St., Jackson,
Mississippi 39216.
Copyright © 2016 by the American Society of Nephrology
Onco-Nephrology Curriculum
1
Figure 1. Membranous nephropathy. (A) Light microscopy showing immune complex deposits. Note the thickened basement
membrane, which stains black while deposits within it stain pink, giving a variegated appearance to the capillary wall. Silver [periodic
acid silver methamine (PASM)] stain; 603, original magnification. (B) Electron microscopy showing immune complex deposits in a
subepithelial location, between effaced podocyte foot processes (top) and the basement membrane (bottom).
2) Tumor antigens may form circulating immune complexes that are subsequently trapped in glomerular
capillaries.
3) External factors such as infections with oncogenic viruses or
altered immune function that can cause both the malignancy and MN.
Other glomerular diseases
Minimal change disease (MCD) has been reported in association with solid tumors like lung, colorectal, renal cell cancers,
and thymoma. Rarely, pancreatic, bladder, breast, and ovarian
cancers have also been associated (2). Focal segmental glomerular sclerosis (FSGS) has been observed with renal cell carcinoma, thymoma, and rarely with lung, breast, and esophageal
cancers (2). A membranoproliferative glomerular nephritis
(MPGN) pattern of injury has been described with lung, kidney, and stomach cancer (2).
Mustonen et al. (11) reported the first known association
between IgA nephropathy and solid tumors of the respiratory
tract, buccal mucosa, and nasopharynx. Renal cell carcinoma
is the most frequently reported solid malignancy associated
with IgA nephropathy (12). Treatment of underlying cancer
improved the IgA nephropathy (11).
Rarely, both solid and hematologic malignancies have been
associated with adult Henoch-Schönlein purpura (HSP)
(13,14). Endocapillary glomerulonephritis is the most commonly seen lesion on kidney biopsy in adults with HSP (15).
Older age and male sex were identified risk factors for cancerassociated HSP (14).
Crescentic glomerulonephritis (CGN) has been associated
with renal cell, gastric, and lung cancers (2).
Thrombotic microangiopathy (TMA) has been associated
with mucin-producing gastric, lung, and breast cancers (16).
In these patients, ADAMTS13 activity is not impaired, and
they respond poorly to plasmapharesis (17).
The exact mechanism of these solid tumor malignancy
associations with glomerular disease is poorly understood.
There have been animal studies (18) done to help us understand the pathomechanisms involved.
This animal study suggested that T-cell response might be
critical in the development of paraneoplastic glomerular
disease. Th1 (T-helper type 1)-predominant responses have
been associated with proliferative and crescentic forms of GN
and Th2 (T-helper type 2) type responses with a membranous
pattern of injury (19). Cancer-associated MCD might be related
to vascular endothelial growth factor (VEGF) production
(20). Overexpression of VEGF leads to a collapsing variant
of FSGS, and underexpression is associated with a TMA pattern
of injury (21,22).
Thymoma-associated glomerular disease
MCD is the most common glomerular disease associated
with thymoma (23). The prevalence of thymoma associated
glomerulopathy is ;2% (23). Other glomerular lesions
Table 1. Differences between primary and tumor-associated secondary MN
Compared feature
History
Serologic markers
Histopathologic clues on kidney
biopsy
Primary MN
Younger age, no history of smoking
Presence of circulating anti-PLA2R autoantibodies
in serum
Predominance of glomerular IgG4 deposition
Enhanced glomerular PLA2R staining
Presence of less than eight inflammatory cells
per glomeruli
Solid tumor–associated MN
Age .65 years, smoking .20 pack-years
Absence of anti-PLA2R autoantibodies
Predominance of glomerular IgG1/IgG2 deposition
Normal glomerular PLA2R staining
Presence of greater than eight inflammatory cells
per glomeruli
IgG, immunoglobulin G; MN, membranous nephropathy; PLA2 R phospholipase A2. Reprinted with permission from reference 73.
2
Onco-Nephrology Curriculum
American Society of Nephrology
Table 2. Glomerular diseases associated with solid tumors
and hematologic malignancies (23)
Malignancy
Lung cancer
a
Colon cancer
Stomach cancer
Pancreas cancer
Bladder cancer
Renal cell cancer
Prostate cancer
Breast cancer
Esophageal cancer
Gastrointestinal stromal
tumor
Gastric cancer
Spleen sarcoma
Head and neck cancer
Wilms’ tumor
Teratoma
Ovarian cancer
Cervical cancer
Endometrial cancer
Tongue cancer
Mesothelioma
Melanoma
Skin cancers (basal and
squamous cell)
Pheochromocytoma
Thymoma
Hodgkin disease
Non-Hodgkin’s disease
CLL
AML
CML
MGUS
T-cell leukemia
HEMATOLOGIC MALIGNANCIES–ASSOCIATED
GLOMERULAR DISEASES
Glomerular diseases reported
MN, MCD, MPGN, IgAN, FSGS, CGN,
TMA
MN, MCD, CGN
MN
MN, MCD, IgAN
MCD
AAA, CGN, IgAN, MCD, FSGS, MPGN
MN, CGN
MN, FSGS, MPGN, TMA
MPGN, FSGS
AAA
MPGN, CGN, TMA
AAA
MN, IgAN
MN, MPGN
MN
MN, MCD
MN
MN
IgAN
MCD
MN, MPGN
MN
MN
MCD, FSGS, CGN, MPGN
MCD, MN, MPGN, IgAN, FSGS, CGN,
AAA, Anti-GBM
MN, MCD, MPGN, IgAN, FSGS
MN, MCD, MPGN, FSGS, CGN
MN, FSGS
MN, MCD, MPGN
MPGN
FSGS
a
Includes small-cell, non–small-cell, squamous cell, and bronchogenic cancers.
AAA, AA amyloidosis; AML, acute myelogenous leukemia; CGN, Crescentic
glomerulonephritis; CLL, chronic lymphocytic leukemia; CML, chronic myelogenous leukemia; FSGS, focal segmental global sclerosis; GBM, glomerular basement membrane; IgAN, IgA nephropathy; MCD, minimal change
disease; MGUS, monoclonal gammopathy of unclear significance; MN,
membranous nephropathy; MPGN, membranoproliferative glomerular nephritis; TMA, thrombotic microangiopathy. Reprinted with permission from
reference 23.
described are MN, FSGS, CGN, and lupus-like nephritis
(24). MN is associated with thymoma of epithelial origin.
MCD is associated with thymoma with lymphocyte predominance. The pathogenesis of thymoma-associated MN seems
to be similar to solid tumor–associated MN, and thymomaassociated MN is likely related to T-cell dysfunction (24).
Studies (25–27) have suggested a major role of T cells, especially the Th2 subtype, in thymoma-associated nephrotic
syndromes.
Table 2 summarizes the various solid tumors seen with
solid tumors.
American Society of Nephrology
Minimal change disease
MCD is classically associated with Hodgkin lymphoma (HL),
more so in the mixed cellularity and nodular sclerosing types.
MCD usually presents around the time of diagnosis of the
malignancy (28). One case series does report diagnosis of
MCD preceding the diagnosis of lymphoma by several months
(29). A poor response to the treatment of MCD with corticosteroids should raise suspicion of underlying lymphoma. In
the case series by Audard et al., (29), the simultaneous diagnosis of HL and MCD was associated with the remission of
proteinuria in response to chemotherapy.
Th2-related cytokines such as interleukin (IL)-13 have been
reported to cause inflammatory response in Hodgkin disease
(30), and rat studies have shown that overexpression of IL-13
induces proteinuria, hypoalbuminemia, and hypercholesterolemia (31). These kidney biopsies showed fusion of foot
processes, suggesting MCD like pathology.
Membranoproliferative glomerulonephritis
Da’as et al. (32) reviewed 42 cases of glomerular disease with
chronic lymphocytic leukemia (CLL); of these, 36 had nephrotic syndrome. The most common glomerular lesion was
MPGN, followed by MN. Another case series of 13 patients
with glomerular disease and either CLL or well-differentiated
lymphocytic lymphoma (33) showed that the majority had an
MPGN pattern of injury. Most MPGN patients had an associated cryoglobulinemia.
An MPGN pattern on kidney biopsy (Figure 2) may also be a clue
to a developing of undiagnosed lymphoplasmacytic malignancy
(8). Sethi et al. (34) reported an association between MPGN and
monoclonal gammopathy of uncertain significance. They showed
that patients with monoclonal gammopathy with normal bone
marrow biopsies had granular immune deposits on their kidney
biopsy, which correlated with their serum and urine monoclonal
proteins. This study (34) also demonstrated that monoclonal
gammopathy can be seen in the setting of other lymphoplasmacytic
diseases, including low-grade B-cell lymphoma, CLL, and multiple
myeloma. Although this direct relationship is not proven, the current observation suggests this possibility (34). More of this is discussed in the paraproteinemia chapter of the curriculum.
Glomerular diseases associated with myeloproliferative
disorders
Myeloproliferative disorders include chronic myelogenous
leukemia (CML), polycythemia Vera (PCV), and essential
thrombocythemia. A recent study (35) of 11 patients with
myeloproliferative disorders with proteinuria and renal failure
showed mesangial sclerosis with hypercellularity in all patients, segmental sclerosis in eight patients, features of TMA
in eight patients, and intracapillary hematopoietic cells in four
patients. Glomerular disorders associated with myeloproliferative disorders are usually late complications and tend to
Onco-Nephrology Curriculum
3
Figure 2. Membranoproliferative glomerulonephritis. Light
microscopy showing basement membrane duplication and increased cells in capillary lumens. Silver (PASM) stain; 603, original
magnification.
have a poor renal prognosis, with progressive kidney injury
occurring in most patients (35).
Essential thrombocythemia and PCV have been associated with FSGS and mesangial proliferative glomerular
disease. The prevalence of glomerular disease in PCV and
essential thrombocythemia is approximately 3%–4% (36).
CML is least likely to have an association with glomerular
pathology (36).
FSGS has also been reported with Hodgkin’s lymphoma
(28) with good response to chemotherapy.
Other glomerular diseases associated with
lymphoproliferative disorders
MN has also been reported with CLL, but less commonly
compared with MPGN (32). A case of IgA nephropathy has
been described with cutaneous T-cell lymphoma (37).
Fibrillary glomerulonephritis (FGN) and Immunotactoid
glomerulopathy (ITG) are rare groups of disorders characterized by formation of organized glomerular deposits (Figure 3,
A and B). These diseases can either occur as primary condition
or be secondary to systemic diseases, mainly lymphoproliferative
disorders. ITG is more strongly associated with neoplasms,
typically paraproteinemias and CLL, compared with FGN
(38). ITG on kidney biopsy should warrant an investigation
of an underlying hematologic malignancy. Treatment is directed toward underlying malignancy.
Glomerular diseases have also been associated with hemophagocytic syndrome. This syndrome is most commonly associated with Epstein-Barr virus; however, it has also been
described with T-cell lymphoma (39,40). Thaunat et al. (40)
described 11 patients with hemophagocytic syndrome who
developed nephrotic syndrome. Renal biopsy showed glomerular lesions consisting of MCD, FSGS, and TMA. In the absence of a causative viral infection, hemophagocytic syndrome
is often treated with immunosuppressive therapy with uncertain renal outcomes.
HEMATOPOIETIC STEM CELL TRANSPLANT–
RELATED GLOMERULAR DISEASES
In HSCT patients, the kidney biopsy findings in patients
with nephrotic range proteinuria include MN, MCD, and
FSGS (41). Although we discuss briefly here, an entire
chapter is devoted to HSCT-related kidney disease in the
Curriculum.
Chronic graft-versus-host disease
MN accounts for a majority of cases of HSCT-associated
glomerular disease, followed by MCD (41). When MCD occurs
in such patients, it is prudent to rule out recurrence of the
primary hematologic malignancy.
Figure 3. Fibrillary and immunotactoid glomerulonephritis. (A) Electron microscopy view of fibrillary glomerulonephritis. (B) Electron
microscopy view of immunotactoid glomerulonephritis.
4
Onco-Nephrology Curriculum
American Society of Nephrology
A review of literature by Brukamp et al. (41) showed a close
temporal relationship between development of nephrotic syndrome shortly after cessation of immunosuppression and the
diagnosis of chronic graft-versus-host disease (GVHD). Luo
et al. (42) investigated the etiology and pathogenesis of nephrotic syndrome after allogenic HSCT. Nephrotic syndrome
after allogenic SCT was associated with the occurrence of
chronic GVHD.
Autologous HSCT can also develop glomerular diseases
(43), although in these patients, GVHD does not occur. It is
possible that there is an immune dysregulation that might be
causing nephrotic syndrome secondary to induction agents or
that these glomerular diseases are de novo. T cell–depleted
HSCT recipients are highly unlikely to develop glomerular
diseases. However, our knowledge about glomerular diseases
in HSCT patients is incomplete, and more research is needed
for complete understanding.
Thrombotic microangiopathy after HSCT
TMA after HSCT is also known as bone marrow transplant
nephropathy or, in some specific cases, radiation nephropathy.
A diagnosis criteria for HSCT-related TMA included .4%
schistocytes in blood, de novo prolonged or progressive
thrombocytopenia, sudden persistent increase in lactate dehydrogenase, and a decrease in serum haptoglobin (44). Studies have suggested that acute GVHD grade 2–4, hepatic
GVHD, veno-occlusive disease, adenovirus infection, older
age, being female, and total body irradiation of .12 Gy are
risk factors for the development of TMA (45,46). TMA can also
occur in T cell–depleted group of patients where calcineurin
inhibitors (CNIs) and GVHD do not exist (47). Treatment
of HSCT-related TMA is usually supportive, with control of
hypertension and proteinuria. Plasma exchange has not proven
to be effective.
CHEMOTHERAPY-ASSOCIATED GLOMERULAR
DISEASE
Thrombotic microangiopathy
Mitomycin C, an alkylating agent, used to treat breast, gastric,
and pancreatic cancer, can cause TMA-like syndrome. Its
nephrotoxicity is dose dependent and usually appears after a
cumulative dose of .40–60 mg/m2 given over a period of
several months (48).
Gemcitabine, commonly used for pancreas, urothelial, and
ovarian cancers, has been shown to cause TMA (49). Cessation
of these medications is shown to improve TMA. Carfilzomib
is a second-generation proteasome inhibitor used for the
treatment of relapsed or refractory multiple myeloma. There
has been recent case reports (50,51) that reported TMA associated with the use of this agent. One of them (51) had kidney
biopsy evidence of TMA (Figure 4). Treatment options include
cessation of the drug with uncertain importance of therapeutic
plasma exchange. Kidney biopsy–proven renal TMA has been
American Society of Nephrology
Figure 4. Thrombotic microangiopathy. Light microscopy view
showing red cell thrombi in the afferent arteriole and two glomerular capillaries. Some basement membrane duplication, but
without increased intracapillary cells, is also visible. Silver (PASM)
stain; 603, original magnification.
reported by Kwa et al. (52) in patients receiving years of
pegylated liposomal doxorubicin for recurrent ovarian cancer.
Bisphosphonate-induced glomerular injury
Pamidronate is used to treat malignancy associated bone
disorders in myeloma. Markowitz et al. (53) showed that
pamidronate causes biopsy-proven collapsing FSGS. MCD
has also been reported with this agent (54).
Interferon-induced glomerular injury
Interferons (IFN)-a, -b, and -g have been associated with moderate proteinuria (55). Markowitz et al. (56) reported 11 cases of
collapsing FSGS that developed during treatment with IFN. IFNa developed significant proteinuria and renal failure after a short
duration of treatment. Patients treated with IFN-b developed
proteinuria after a prolonged course of treatment. The authors
(56) also reviewed 21 additional cases of IFN-associated glomerular disease. Thirteen of these patients had FSGS, and the rest had
MCD. The mechanism of this injury is not fully understood.
There is a direct effect of IFN on the podocyte by altering the
cellular proliferation and cell metabolism (56). The indirect effects
of IFN might be due to adaptive immune mechanism that increase
macrophage activation or via 1L-6 or 1L-13 production (56).
IFN-a, when used for treatment of CML, has been reported
to be associated with TMA (57,58).
Calcineurin and mammalian target of rapamycin
inhibitors
CNIs can cause a rare manifestation of TMA with glomerular
changes. The histology is indistinguishable from other causes
of TMA (59). The only consensus on treatment is to withdraw
the CNIs (60).
Mammalian target of rapamycin (mTOR) inhibitors such
as sirolimus, tensirolimus, and everolimus can develop
Onco-Nephrology Curriculum
5
Table 3. Glomerular toxicity associated with chemotherapeutic agents
Type of cells involved
Glomerular epithelial cells (podocytes)
Minimal change disease
Focal segmental glomerular sclerosis
Other glomerular diseases
Glomerular endothelial cells
Thrombotic microangiopathy
Chemotherapy agents
Interferon-a and -b, pamidronate, tyrosine kinase inhibitors,
anthracyclines, mTOR inhibitors
Interferon-a and -g, pamidronate, tyrosine kinase inhibitors, clofarabine,
anthracyclines, mTOR inhibitors
Ipilimumab, mTOR inhibitors
Mitomycin-c, gemcitabine, cisplatin, carboplatin, cytarabine, lomustine,
tamoxifen, bleomycin, bortezomib, carfilzomib, anthracyclines, hydroxyurea
complications including TMA and FSGS in renal transplant
patients (61–63). MCD, MN, FSGS, MPGN, and IgA nephropathy have also been associated with sirolimus in the
kidney transplant literature (64–66). There is speculation that
sirolimus-induced proteinuria is related to collapsing FSGS associated with VEGF overexpression in podocytes.
Antiangiogenesis agents
Antiangiogenic agents are used primarily for advanced stage solid
tumors, including renal cell carcinoma, non–small cell lung carcinoma, colorectal carcinoma, and gastrointestinal stromal tumors. Monoclonal antibodies against VEGF and tyrosine kinase
inhibitors (TKIs) (67,68) have been observed to cause hypertension, proteinuria, and renal vascular injury, manifested by proteinuria and TMA (69). VEGF maintains normal functioning of
glomerular endothelial cells, podocytes, mesangium, and
peritubular capillaries. Hence, inhibition of VEGF can lead to
dose-dependent proteinuria, swelling and detachment of glomerular endothelial cells, vacuolization of endothelial cells,
disruption of slit diaphragms, and down-regulation of nephrin
(70). Examples of anti-VEGF therapy include bevacizumab,
and TKIs include sunitinib and sorafenib. In the majority of
cases, proteinuria and hypertension resolve or significantly improve with cessation of this therapy (69). VEGF inhibitors are
more likely to present as TMA or renal-limited TMA and TKIs
as MCD or MCD/FSGS on kidney biopsy (71).
New chemotherapeutic agent–associated glomerular
disease
Several new chemotherapies are now available in clinical practice. Renal toxicity of these novel agents has been increasingly
reported in the last decade. Clofaribine is a purine nucleoside
analog used to treat relapsed or refractory pediatric acute lymphoblastic leukemia and adult acute myelogenous leukemia.
Nephrotoxicity most commonly manifests as elevation in serum
creatinine. Kintzel et al. (72) reported AKI following exposure of
clofaribine along with nephrotic range proteinuria. Unfortunately
a kidney biopsy was not available. Extrapolating from animal
studies, Jhaveri et al. (73) postulated that inhibition of ribonucleotide reductase by clofarabine might be the cause of collapsing
glomerulopathy and/or kidney injury seen with this agent.
Ipilimumab is a monoclonal antibody against human
cytotoxic T-lymphocyte antigen 4. It is US Food and Drug
6
Onco-Nephrology Curriculum
Administration (FDA) approved for unresectable or metastatic
melanoma. Renal biopsy in a patient with ipilimumabassociated AKI with nephrotic range proteinuria revealed
lupus nephritis with positive anti–double-stranded DNA antibodies (74). There are also case reports of acute granulomatous interstitial nephritis by this agent (75).
Anthracyclines like daunorubicin and doxorubicin have been
known to cause nephrotic syndrome with renal lesions consistent with MCD, FSGS not otherwise specified (NOS), or collapsing glomerulopathy (76).
Table 3 summarizes the glomerular toxicities associated
with chemotherapy agents.
Ongoing education and heightened physician awareness
regarding these negative associations is central to early
recognition and their successful management.
CONCLUSION
Several cancers are associated with various glomerular diseases.
Membranous nephropathy remains the most common glomerular pathology reported in patients with solid tumors.
Although MCD disease has been classically associated with HL,
MPGN has been recognized in patients with CLL. Several
reports and studies in the literature suggest that treating the
cancer leads to resolution of the glomerular disease.
TAKE HOME POINTS
c Many solid and hematologic malignancies are associated with different
glomerular diseases.
c Several case reports and case series of cancer-associated glomerular
diseases have shown that treating the cancer may lead to resolution of
the glomerular process.
c
Although membranous nephropathy has been classically associated with
solid malignancies, minimal change disease has been commonly described
with hematologic malignancies, especially Hodgkin lymphoma.
c Membranoproliferative glomerulonephritis is increasingly being
recognized to be associated with chronic hematologic malignancies
such as chronic lymphocytic leukemia.
c Chemotherapy agents can also lead to glomerular diseases, the most
common being TMA associated with targeted therapies.
American Society of Nephrology
ACKNOWLEDGMENTS
Pathology images are courtesy of James Pullman, Albert Einstein
Medical Center, NY.
19.
20.
REFERENCES
1. Ronco PM. Paraneoplastic glomerulopathies: New insights into an old
entity. Kidney Int 56: 355–377, 1999
2. Bacchetta J, Juillard L, Cochat P, Droz JP. Paraneoplastic glomerular
diseases and malignancies. Crit Rev Oncol Hematol 70: 39–58, 2009
3. Lefaucheur C, Stengel B, Nochy D, Martel P, Hill GS, Jacquot C, Rossert
J, Group G-PS. Membranous nephropathy and cancer: Epidemiologic
evidence and determinants of high-risk cancer association. Kidney Int
70: 1510–1517, 2006
4. Beck LH, Jr., Bonegio RG, Lambeau G, Beck DM, Powell DW, Cummins
TD, Klein JB, Salant DJ. M-type phospholipase A2 receptor as target
antigen in idiopathic membranous nephropathy. New Engl J Med 361:
11–21, 2009
5. Qin W, Beck LH, Jr., Zeng C, Chen Z, Li S, Zuo K, Salant DJ, Liu Z. Antiphospholipase A2 receptor antibody in membranous nephropathy. J
Am Soc Nephrol 22: 1137–1143, 2011
6. Hoxha E, Kneissler U, Stege G, Zahner G, Thiele I, Panzer U, Harendza S,
Helmchen UM, Stahl RA. Enhanced expression of the M-type phospholipase A2 receptor in glomeruli correlates with serum receptor antibodies in primary membranous nephropathy. Kidney Int 82: 797–804, 2012
7. Ohtani H, Wakui H, Komatsuda A, Okuyama S, Masai R, Maki N, Kigawa
A, Sawada K, Imai H. Distribution of glomerular IgG subclass deposits in
malignancy-associated membranous nephropathy. Nephrol Dialysis
Transplant 19: 574–579, 2004
8. Jhaveri KD, Shah HH, Patel C, Kadiyala A, Stokes MB, Radhakrishnan J.
Glomerular diseases associated with cancer, chemotherapy, and hematopoietic stem cell transplantation. Adv Chronic Kidney Dis 21: 48–
55, 2014
9. Bjorneklett R, Vikse BE, Svarstad E, Aasarod K, Bostad L, Langmark F,
Iversen BM. Long-term risk of cancer in membranous nephropathy patients. Am J Kidney Dis 50: 396–403, 2007
10. Beck LH Jr. Membranous nephropathy and malignancy. Semin Nephrol
30: 635–644, 2010
11. Mustonen J, Pasternack A, Helin H. IgA mesangial nephropathy in
neoplastic diseases. Contribut Nephrol 40: 283–291, 1984
12. Magyarlaki T, Kiss B, Buzogany I, Fazekas A, Sukosd F, Nagy J. Renal
cell carcinoma and paraneoplastic IgA nephropathy. Nephron 82: 127–
130, 1999
13. Pertuiset E, Liote F, Launay-Russ E, Kemiche F, Cerf-Payrastre I,
Chesneau AM. Adult Henoch-Schonlein purpura associated with malignancy. Semin Arthrit Rheum 29: 360–367, 2000
14. Zurada JM, Ward KM, Grossman ME. Henoch-Schonlein purpura associated with malignancy in adults. J Am Acad Dermatol 55[5 Suppl]:
S65–S70, 2006
15. Pillebout E, Thervet E, Hill G, Alberti C, Vanhille P, Nochy D. HenochSchonlein Purpura in adults: Outcome and prognostic factors. J Am Soc
Nephrol 13: 1271–1278, 2001
16. Werner TL, Agarwal N, Carney HM, Rodgers GM. Management of
cancer-associated thrombotic microangiopathy: What is the right approach? Am J Hematol 82: 295–298, 2007
17. Francis KK, Kalyanam N, Terrell DR, Vesely SK, George JN. Disseminated malignancy misdiagnosed as thrombotic thrombocytopenic
purpura: A report of 10 patients and a systematic review of published
cases. Oncologist 12: 11–19, 2007
18. Takeda S, Chinda J, Murakami T, Numata A, Iwazu Y, Akimoto T,
Hamano Y, Muto S, Takahashi M, Kusano E. Development of features
of glomerulopathy in tumor-bearing rats: A potential model for
American Society of Nephrology
21.
22.
23.
24.
25.
26.
27.
28.
29.
30.
31.
32.
33.
34.
35.
36.
37.
paraneoplastic glomerulopathy. Nephrol Dialysis Transplant 27: 1786–
1792, 2012
Holdsworth SR, Kitching AR, Tipping PG. Th1 and Th2 T helper cell
subsets affect patterns of injury and outcomes in glomerulonephritis.
Kidney Int 55: 1198–1216, 1999
Taniguchi K, Fujioka H, Torashima Y, Yamaguchi J, Izawa K, Kanematsu
T. Rectal cancer with paraneoplastic nephropathy: Association of vascular endothelial growth factor. Digestive Surg 21: 455–457, 2004
Eremina V, Sood M, Haigh J, Nagy A, Lajoie G, Ferrara N, Gerber HP,
Kikkawa Y, Miner JH, Quaggin SE. Glomerular-specific alterations of
VEGF-A expression lead to distinct congenital and acquired renal diseases. J Clin Invest 111: 707–716, 2003
Eremina V, Jefferson JA, Kowalewska J, Hochster H, Haas M, Weisstuch J,
Richardson C, Kopp JB, Kabir MG, Backx PH. VEGF inhibition and renal
thrombotic microangiopathy. New Engl J Med 358: 1129–1136, 2008
Jhaveri KD, Shah HH, Calderon K, Campenot ES, Radhakrishnan J.
Glomerular diseases seen with cancer and chemotherapy: A narrative
review. Kidney Int 84: 34–44, 2013
Karras A, de Montpreville V, Fakhouri F, Grunfeld JP, Lesavre P, Groupe
d’Etudes des Nephropathies Associees aux T. Renal and thymic pathology in thymoma-associated nephropathy: Report of 21 cases and review
of the literature. Nephrol Dialysis Transplant 20: 1075–1082, 2005
Hirokawa K, Utsuyama M, Kasai M, Konno A, Kurashima C, Moriizumi E.
Age-related hyperplasia of the thymus and T-cell system in the Buffalo
rat. Immunological and immunohistological studies. Virchows Arch B
59: 38–47, 1990
Le Berre L, Herve C, Buzelin F, Usal C, Soulillou JP, Dantal J. Renal
macrophage activation and Th2 polarization precedes the development of nephrotic syndrome in Buffalo/Mna rats. Kidney Int 68: 2079–
2090, 2005
Le Berre L, Bruneau S, Naulet J, Renaudin K, Buzelin F, Usal C, Smit H,
Condamine T, Soulillou JP, Dantal J. Induction of T regulatory cells
attenuates idiopathic nephrotic syndrome. J Am Soc Nephrol 20: 57–
67, 2009
Mallouk A, Pham PT, Pham PC. Concurrent FSGS and Hodgkin’s
lymphoma: Case report and literature review on the link between nephrotic glomerulopathies and hematological malignancies. Clin Exper
Nephrol 10: 284–289, 2006
Audard V, Larousserie F, Grimbert P, Abtahi M, Sotto JJ, Delmer A,
Boue F, Nochy D, Brousse N, Delarue R. Minimal change nephrotic
syndrome and classical Hodgkin’s lymphoma: Report of 21 cases and
review of the literature. Kidney Int 69: 2251–2260, 2006
Kuppers R, Schwering I, Brauninger A, Rajewsky K, Hansmann ML.
Biology of Hodgkin’s lymphoma. Ann Oncol 13[Suppl 1]: 11–18, 2002
Lai KW, Wei CL, Tan LK, Tan PH, Chiang GS, Lee CG, Jordan SC, Yap
HK. Overexpression of interleukin-13 induces minimal-change-like
nephropathy in rats. J Am Soc Nephrol 18: 1476–1485, 2007
Da’as N, Polliack A, Cohen Y, Amir G, Darmon D, Kleinman Y, Goldfarb
AW, Ben-Yehuda D. Kidney involvement and renal manifestations in nonHodgkin’s lymphoma and lymphocytic leukemia: A retrospective study in
700 patients. Eur J Haematol 67: 158–164, 2001
Moulin B, Ronco PM, Mougenot B, Francois A, Fillastre JP, Mignon F.
Glomerulonephritis in chronic lymphocytic leukemia and related B-cell
lymphomas. Kidney Int 42: 127–135, 1992
Sethi S, Zand L, Leung N, Smith RJ, Jevremonic D, Herrmann SS,
Fervenza FC. Membranoproliferative glomerulonephritis secondary to
monoclonal gammopathy. Clin J Am Soc Nephrol 5: 770–782, 2010
Said SM, Leung N, Sethi S, Cornell LD, Fidler ME, Grande JP, Herrmann
S, Tefferi A, D’Agati VD, Nasr SH. Myeloproliferative neoplasms cause
glomerulopathy. Kidney Int 80: 753–759, 2011
Au WY, Chan KW, Lui SL, Lam CC, Kwong YL. Focal segmental
glomerulosclerosis and mesangial sclerosis associated with myeloproliferative disorders. Am J Kidney Dis 34: 889–893, 1999
Bajel A, Yin Lin M, Hill PA, Goodman D, McCormack C, Foley P, Davison
J, Honemann D, Kenealy M, Lade S. IgA nephropathy associated with
cutaneous T cell lymphoma. Leukemia lymphoma 50: 2083–2085, 2009
Onco-Nephrology Curriculum
7
38. Rosenstock JL, Markowitz GS, Valeri AM, Sacchi G, Appel GB, D’Agati
VD. Fibrillary and immunotactoid glomerulonephritis: Distinct entities
with different clinical and pathologic features. Kidney Int 63: 1450–
1461, 2003
39. Chang CS, Wang CH, Su IJ, Chen YC, Shen MC. Hematophagic
histiocytosis: A clinicopathologic analysis of 23 cases with special reference to the association with peripheral T-cell lymphoma. J. Formosan
Med Assoc 93: 421–428, 1994
40. Thaunat O, Delahousse M, Fakhouri F, Martinez F, Stephan JL, Noel LH,
Karras A. Nephrotic syndrome associated with hemophagocytic syndrome. Kidney Int 69: 1892–1898, 2006
41. Brukamp K, Doyle AM, Bloom RD, Bunin N, Tomaszewski JE, Cizman B.
Nephrotic syndrome after hematopoietic cell transplantation: Do glomerular lesions represent renal graft-versus-host disease? Clin J Am
Soc Nephrol 1: 685–694, 2006
42. Luo XD, Liu QF, Zhang Y, Sun J, Wang GB, Fan ZP, Yi ZS, Ling YW, Wei
YQ, Liu XL. Nephrotic syndrome after allogeneic hematopoietic stem
cell transplantation: Etiology and pathogenesis. Blood Cells Molecules
Dis 46: 182–187, 2011
43. Troxell ML, Pilapil M, Miklos DB, Higgins JP, Kambham N. Renal
pathology in hematopoietic cell transplantation recipients. Modern
Pathol 21: 396–406, 2008
44. Ruutu T, Barosi G, Benjamin RJ, Clark RE, George JN, Gratwohl A,
Holler E, Iacobelli M, Kentouche K, Lammle B. Diagnostic criteria for
hematopoietic stem cell transplant-associated microangiopathy: Results of a consensus process by an International Working Group.
Haematologica 92: 95–100, 2007
45. Chang A, Hingorani S, Kowalewska J, Flowers ME, Aneja T, Smith KD,
Meehan SM, Nicosia RF, Alpers CE. Spectrum of renal pathology in
hematopoietic cell transplantation: A series of 20 patients and review of
the literature. Clin J Am Soc Nephrol 2: 1014–1023, 2007
46. Changsirikulchai S, Myerson D, Guthrie KA, McDonald GB, Alpers CE,
Hingorani SR. Renal thrombotic microangiopathy after hematopoietic
cell transplant: Role of GVHD in pathogenesis. Clin J Am Soc Nephrol
4: 345–353, 2009
47. Glezerman IG, Jhaveri KD, Watson TH, Edwards AM, Papadopoulos
EB, Young JW, Flombaum CD, Jakubowski AA. Chronic kidney disease,
thrombotic microangiopathy, and hypertension following T celldepleted hematopoietic stem cell transplantation. Biol Blood Marrow
Transplant 16: 976–984, 2010
48. Lesesne JB, Rothschild N, Erickson B, Korec S, Sisk R, Keller J, Arbus M,
Woolley PV, Chiazze L, Schein PS. Cancer-associated hemolytic-uremic
syndrome: analysis of 85 cases from a national registry. J Clin Oncol 7:
781–789, 1989
49. Glezerman I, Kris MG, Miller V, Seshan S, Flombaum CD. Gemcitabine
nephrotoxicity and hemolytic uremic syndrome: Report of 29 cases
from a single institution. Clin Nephrol 71: 130–139, 2009
50. Sullivan MR, Danilov AV, Lansigan F, Dunbar NM. Carfilzomib associated thrombotic microangiopathy initially treated with therapeutic
plasma exchange. J Clin Apheresis 2014
51. Hobeika L, Self SE, Velez JC. Renal thrombotic microangiopathy and
podocytopathy associated with the use of carfilzomib in a patient with
multiple myeloma. BMC Nephrol 15: 156, 2014
52. Kwa M, Baumgartner R, Shavit L, Barash I, Michael J, Puzanov I,
Kopolovic J, Rosengarten O, Blank S, Curtin JP. Is renal thrombotic
angiopathy an emerging problem in the treatment of ovarian cancer
recurrences? Oncologist 17: 1534–1540, 2012
53. Markowitz GS, Appel GB, Fine PL, Fenves AZ, Loon NR, Jagannath S,
Kuhn JA, Dratch AD, D’Agati VD. Collapsing focal segmental glomerulosclerosis following treatment with high-dose pamidronate. J Am Soc
Nephrol 12: 1164–1172, 2001
54. Barri YM, Munshi NC, Sukumalchantra S, Abulezz SR, Bonsib SM, Wallach
J, Walker PD. Podocyte injury associated glomerulopathies induced by
pamidronate. Kidney Int 65: 634–641, 2004
8
Onco-Nephrology Curriculum
55. Quesada JR, Talpaz M, Rios A, Kurzrock R, Gutterman JU. Clinical toxicity
of interferons in cancer patients: a review. J Clin Oncol 4: 234–243, 1986
56. Markowitz GS, Nasr SH, Stokes MB, D’Agati VD. Treatment with
IFN-{alpha}, -{beta}, or -{gamma} is associated with collapsing focal
segmental glomerulosclerosis. Clin J Am Soc Nephrol 5: 607–615, 2010
57. Badid C, McGregor B, Faivre JM, Guerard A, Juillard L, Fouque D,
Laville M. Renal thrombotic microangiopathy induced by interferonalpha. Nephrol Dialysis Transplant 16: 846–848, 2001
58. Ohashi N, Yonemura K, Sugiura T, Isozaki T, Togawa A, Fujigaki Y,
Yamamoto T, Hishida A. Withdrawal of interferon-alpha results in
prompt resolution of thrombocytopenia and hemolysis but not renal
failure in hemolytic uremic syndrome caused by interferon-alpha. Am J
Kidney Dis 41: E10, 2003
59. Liptak P, Ivanyi B. Primer: Histopathology of calcineurin-inhibitor toxicity in renal allografts. Nature Clin Pract Nephrol 2: 398–404, 2006
60. Ho VT, Cutler C, Carter S, Martin P, Adams R, Horowitz M, Ferrara J,
Soiffer R, Giralt S. Blood and marrow transplant clinical trials network
toxicity committee consensus summary: Thrombotic microangiopathy
after hematopoietic stem cell transplantation. Biol Blood Marrow
Transplant 11: 571–575, 2005
61. Jhaveri KD, Schatz JH, Young JW, Flombaum C. Sirolimus (rapamycin)
induced proteinuria in a patient undergoing allogeneic hematopoietic
stem cell transplant. Transplantation 86: 180–181, 2008
62. Hochegger K, Wurz E, Nachbaur D, Rosenkranz AR, Clausen J. Rapamycininduced proteinuria following allogeneic hematopoietic stem cell transplantation. Bone Marrow Transplant 44: 63–65, 2009
63. Izzedine H, Boostandoot E, Spano JP, Bardier A, Khayat D. Temsirolimusinduced glomerulopathy. Oncology 76: 170–172, 2009
64. Mainra R, Mulay A, Bell R, Karpinski J, Hoar S, Knoll G, Robertson S,
Wang D. Sirolimus use and de novo minimal change nephropathy following renal transplantation. Transplantation 80:1816, 2005
65. Franco AF, Martini D, Abensur H, Noronha IL. Proteinuria in transplant
patients associated with sirolimus. Transplant Proc 39: 449–452, 2007
66. Letavernier E, Bruneval P, Mandet C, Duong Van Huyen JP, Peraldi MN,
Helal I, Noel LH, Legendre C. High sirolimus levels may induce focal
segmental glomerulosclerosis de novo. Clin J Am Soc Nephrol 2: 326–
333, 2007
67. Jhaveri KD, Flombaum CD, Kroog G, Glezerman IG. Nephrotoxicities
associated with the use of tyrosine kinase inhibitors: A single-center
experience and review of the literature. Nephron Clin Pract 117: c312–
c319, 2011
68. Bollee G, Patey N, Cazajous G, Robert C, Goujon JM, Fakhouri F,
Bruneval P, Noel LH, Knebelmann B. Thrombotic microangiopathy
secondary to VEGF pathway inhibition by sunitinib. Nephrol Dialysis
Transplant 24: 682–685, 2009
69. Hayman SR, Leung N, Grande JP, Garovic VD. VEGF inhibition, hypertension, and renal toxicity. Curr Oncol Rep 14: 285–294, 2012
70. Kelly RJ, Billemont B, Rixe O. Renal toxicity of targeted therapies.
Target Oncol 4: 121–133, 2009
71. Izzedine H, Escudier B, Lhomme C, Pautier P, Rouvier P, Gueutin V,
Baumelou A, Derosa L, Bahleda R, Hollebecque A. Kidney diseases associated with anti-vascular endothelial growth factor (VEGF): An 8-year
observational study at a single center. Medicine 93: 333–339, 2014
72. Kintzel PE, Visser JA, Campbell AD. Clofarabine-associated acute
kidney injury and proteinuria. Pharmacotherapy 31: 923, 2011
73. Jhaveri KD, Chidella S, Allen SL, Fishbane S. Clofarabine-induced
kidney toxicity. J Oncol Pharmacy Pract 20: 305–308, 2013
74. Fadel F, El Karoui K, Knebelmann B. Anti-CTLA4 antibody-induced
lupus nephritis. New Engl J Med 361: 211–212, 2009
75. Izzedine H, Gueutin V, Gharbi C, Mateus C, Robert C, Routier E, Thomas
M, Baumelou A, Rouvier P. Kidney injuries related to ipilimumab. Invest
New Drugs 32: 769–773, 2014
76. Mohamed N, Goldstein J, Schiff J, John R. Collapsing glomerulopathy
following anthracycline therapy. Am J Kidney Dis 61: 778–781, 2013
American Society of Nephrology
REVIEW QUESTIONS
1. Which of the following statements regarding glomerular
diseases seen with cancer is true?
a. The most common glomerular pathology seen with
solid tumors is minimal change disease
b. The most common glomerular pathology seen with hematologic malignancies is membranous nephropathy
c. The most common associated glomerular disease with
GVHD is membranous nephropathy
d. Thymoma has not been associated glomerular diseases
Answer: c is correct. The most common glomerular pathology seen with solid tumors is membranous nephropathy
(MN). Minimal change disease (MCD) is commonly seen
with hematologic malignancies such as Hodgkin lymphoma.
MN accounts for the majority of the cases of HSCT-associated
glomerular disease. Thymoma has been associated with MCD
(lymphocyte predominant) and MN (epithelial origin).
2. A primary care physician refers a 60-year-old white
woman for evaluation of nephrotic range proteinuria. She
presented with a 1-month history of worsening bilateral
lower extremity edema. She has a history of refractory malignant melanoma. She was recently started on ipilimumab
after failing standard chemotherapy. Her melanoma has
responded to therapy.
On physical examination, her BP was normal at 120/80 mmHg,
and there was 31 pitting edema of his lower extremities.
The rest of the examination was unremarkable. At the time
of presentation, serum creatinine was 0.9 mg/dL, serum
albumin was 2.8 g/dL, total cholesterol was 290 mg/dL,
and low-density lipoprotein (LDL) cholesterol was 197 mg/dL.
Liver function tests and complete blood count were
normal. A 24-hour urine collection revealed 8.5 g protein. A
workup for secondary causes of nephrotic syndrome revealed
normal complement levels. Hepatitis B surface antigen,
hepatitis C antibody, antinuclear antibody, cryoglobulins,
and human immunodeficiency virus (HIV) antibody were
negative. Serum free light chains were within the normal ratio.
Sonogram revealed normal-sized kidneys. A kidney biopsy
reveals a proliferative glomerular disease with immunofluorescence suggestive of a full house pattern and electron microscopy showing mesangial and subendothelial deposits.
What is the most likely diagnosis?
American Society of Nephrology
a.
b.
c.
d.
Melanoma-induced proliferative glomerulonephritis
Ipilimumab-induced lupus-like nephritis
De novo seronegative lupus nephritis
Membranoproliferative glomerulonephritis
Answer: b is correct. Ipilimumab is a monoclonal antibody
against human cytotoxic T-lymphocyte antigen 4, which is
FDA approved for unresectable or metastatic melanoma. Nephrotic range proteinuria with a lupus nephritis–like picture
on renal biopsy has been reported.
3. A 62-year-old white man with a long-standing history of
hypertension and recent history of CLL was referred by his
oncologist for evaluation of proteinuria and elevated serum
creatinine. He denied any history of diabetes, hepatitis, or
blood transfusion. There was no recent infection or travel
history. Review of systems was significant for bilateral intermittent lower extremity swelling over the last 4 months.
He denied fever, chills, dyspnea, gross hematuria, arthralgia, or rash. His current medication included amlodipine
for hypertension management.
On physical examination, his BP was elevated at 160/94 mmHg.
There was mild edema of his lower extremities. The rest
of the examination was unremarkable. At the time of presentation, serum creatinine was 1.5 mg/dL, and serum albumin was 3.5 g/dL. Complete blood count, liver function
tests, and a lipid profile were normal. Urinalysis was significant for 10–20 RBC/hpf and 21 proteinuria. A 24-hour
urine collection revealed 1.8 g protein. A workup for secondary causes of proteinuria revealed low C3 and C4 levels.
Hepatitis B surface antigen, hepatitis C antibody, antinuclear
antibody, cryoglobulins, and human immunodeficiency
virus (HIV) antibody were negative. Serum and urine
immunofixation did not reveal any monoclonal immunoglobulin. Sonogram revealed normal-sized kidneys. A
kidney biopsy was subsequently performed.
What is the most likely kidney biopsy diagnosis?
a.
b.
c.
d.
Membranous nephropathy
Membranoproliferative glomerulonephritis
Focal segmental glomerulosclerosis
Acute interstitial nephritis
Answer: b is correct. A membranoproliferative glomerulonephritis (MPGN) pattern of injury on renal biopsy has been
most commonly associated with CLL, followed by MN.
Onco-Nephrology Curriculum
9
Chapter 7: Hematologic Disorders and Kidney Disease
Ala Abudayyeh, MD,* and Kevin Finkel, MD, FACP, FASN, FCCM*†
*Division of General Internal Medicine, Section of Nephrology, University of Texas MD Anderson Cancer Center,
Houston, Texas; and †UTHealth Science Center at Houston Medical School, Department of Medicine, Division of
Renal Diseases and Hypertension, Houston, Texas
MULIPLE MYELOMA
Pathogenesis
Multiple myeloma (MM) is a hematologic malignancy involving the pathologic proliferation of
terminally differentiated plasma cells. It is the
second most common hematologic malignancy
behind non-Hodgkin lymphoma, with an annual
incidence of 4–7 cases per 100,000 in the United
States. Clinical symptoms are due to osteolysis of
the bone, suppression of normal hematopoiesis,
and the overproduction of monoclonal immunoglobulins that deposit in organ tissues. Clinical
symptoms include bone pain and fractures, anemia,
infections, hypercalcemia, edema, heart failure, and
renal disease.
Kidney involvement and pathology
More than one-half of patients with MM initially
present with varying degrees of AKI. Nearly 20%
of patients present with a serum creatinine .2.0
mg/dL, and 10% of patients require dialysis on
presentation (1). AKI is associated with higher
mortality, but this may be reflective of patients
with more advanced disease (2).
The major diseases in the spectrum of myelomarelated kidney disease include cast nephropathy, light
chain deposition disease (LCDD), and AL-amyloidosis.
Renal biopsy demonstrates the presence of monotypic light chains on immunofluorescence exam, as
well as characteristic ultrastructural features of
deposits on electron microscopy. Less common
forms of renal injury include light chain–induced
Fanconi syndrome, cryoglobulinemia, proliferative glomerulonephritis, heavy chain deposition
disease, and immunotactoid glomerulonephritis
(Table 1) (3).3
Cast nephropathy
Cast nephropathy has been diagnosed in 41% of
patients with MM and renal disease (4). Excess light
American Society of Nephrology
chains precipitate with Tamm-Horsfall protein
(THP) secreted by the thick ascending limb of the
loop of Henle and produce casts in the distal tubule.
Decreased GFR may increase the concentration of
light chains in the distal tubule and enhance the
formation of casts. Therefore, hypercalcemia, volume depletion, diuretics, and nonsteroidal antiinflammatory drugs can exacerbate renal injury.
In some cases of AKI associated with MM, cast
formation is rare on renal biopsy. Instead, renal
injury is attributed to the direct toxic effects of
urinary free light chains (FLCs) on proximal tubule
cells (5,6). After reabsorption, lysosomal degradation of FLCs can activate the NF-kB pathway leading to oxidative stress with an inflammatory
response, apoptosis, and fibrosis. This lesion is
characterized histologically by loss of brush border
and cell vacuolization and necrosis (7).
The classic presentation is an elderly patient with
unexplained renal failure, anemia, and bone pain
or fractures. Proteinuria, when quantitatively measured with a 24-hour urine collection, is usually
subnephrotic and primarily composed of monoclonal light chains (Bence-Jones proteins). The
qualitative measurement of proteinuria using a
urine test strip, which mainly detects albumin, is
generally minimally reactive.
Most patients with myeloma cast nephropathy
are diagnosed without kidney biopsy using serum
and urine immunofixation and serum FLC analysis
(see Chapter 9). When biopsied, casts are eosin
positive, fractured, and waxy in appearance on light
microscopy (Figure 1) (8). Multinucleated giant
cells may surround casts, and an interstitial
Correspondence: Kevin W. Finkel, MD, FACP, FASN, FCCM,
Division of Renal Diseases & Hypertension, UTHealth Science
Center at Houston-McGovern Medical School, 6431 Fannin St.,
Houston, Texas 77030. Email: kevin.w.fi[email protected]
Copyright © 2016 by the American Society of Nephrology
Onco-Nephrology Curriculum
1
Table 1. Kidney manifestations of multiple myeloma
Myeloma cast nephropathy
AL amyloidosis
Light chain deposition disease
Heavy chain deposition disease
Immunotactoid glomerulopathy
Fibrillary glomerulopathy
Light chain Fanconi syndrome
Plasma cell infiltration
Cryoglobulinemia
Membranoproliferative glomerulonephritis
Membranous glomerulonephritis
inflammatory infiltrate composed of lymphocytes and monocytes may also be present. Widespread tubular atrophy and
interstitial fibrosis eventually develops. Immunofluorescence
staining generally demonstrates light chain restriction within
the casts, although patterns may be mixed or nondiagnostic as
well. Casts have a lattice-like appearance and may contain
needle-shaped crystals on electron microscopy. The glomeruli
and vessels appear normal, unless LCDD is concurrently present.
Light chain deposition disease
LCDD has been diagnosed at autopsy in 19% of patients with
MM and renal disease (4). The renal manifestations are most
apparent clinically, whereas light chain deposits within the
heart, liver, spleen, and peripheral nervous system may remain
asymptomatic. The hallmark of the disease is the development
of mesangial nodules from mesangial matrix expansion
secondary to the up-regulation of platelet derived growth
factor-b and transforming growth factor-b. The nodules can
occasionally be confused with the Kimmestiel-Wilson lesion
of diabetic nephropathy.
Clinically, patients present with proteinuria, renal insufficiency, and a nodular sclerosing glomerulopathy. Several
retrospective reviews have reported on the clinical characteristics of these patients (9,10). The mean age was 58 years with
no significant preference with respect to sex. Marked renal
insufficiency was common on presentation, with a median
serum creatinine .4 mg/dL, and renal function rapidly declined thereafter. Nephrotic range proteinuria was detected
in 26%–40% of patients and correlated with the degree of
glomerular involvement. Hypertension and microscopic hematuria were also present in the majority of patients.
Light chain deposition stimulates mesangial and matrix
expansion leading to nodule formation. On light microscopy,
mesangial nodules are more uniform in distribution and size in
LCDD compared with diabetic nephropathy (Figure 2) (8).
Irregular thickening and double contours of the glomerular
basement membrane may also be present. Eosin-positive deposits may be seen diffusely throughout the tubular basement
membranes. Immunofluorescence demonstrates a characteristic linear staining of basement membranes with monotypic
Figure 1. Pathology of myeloma cast nephropathy. (A) Large atypical casts are seen within distal tubular lumina. The casts appear
hypereosinophilic with fractured texture and sharp edges. They are associated with giant cell (arrows) and mononuclear cell reaction,
acute proximal tubular cell injury, and interstitial inflammation (hematoxylin and eosin, 3,2003). (B) The casts characteristically are
periodic acid–Schiff (PAS) negative (3,2003). (C) Myeloma casts almost always stain for just k or l light chain. This figure shows bright
staining of casts for k (immunofluorescence, 3,4003). The casts were negative for l (data not shown). (D) Occasionally, myeloma casts
are composed of small rod- or needle-shaped crystals that fill the distal tubular lumina and, as shown, appear highly electron dense on
electron microscopy (31,8503). Reprinted from reference 8, with permission of the Elsevier Science and Technology Journals.
2
Onco-Nephrology Curriculum
American Society of Nephrology
Figure 2. Pathology of light-chain proximal tubulopathy. (A) Large rod- and rhomboid-shaped hypereosinophilic crystals are seen
within proximal tubular cells. (hematoxylin and eosin, 3,6003). (B) In this patient with smoldering myeloma who has 20% k-restricted
plasma cells in the bone marrow, the proximal tubular crystals stain strongly for k (as shown) with negative l (not shown) by immunofluorescence performed on pronase-digested, paraffin-embedded tissue. The intracellular crystals failed to stain for k or l on
standard immunofluorescence on frozen tissue (not shown; 3,4003). (C) Ultrastructurally, the proximal tubular cells are filled with
electron-dense light-chain crystals with rod, rhomboid, or rectangular shapes. The proximal tubule brush border appears preserved
(31,8503). (D) In this case of light-chain proximal tubulopathy, the proximal tubular cells are loaded with large election-dense
phagolysosomes without crystals (electron microscopy, 32,5003). Reprinted from reference 8, with permission of the Elsevier Science
and Technology Journals.
light chains, which are most commonly k restricted. On electron microscopy, granular-powdery deposits are distributed
within the mesangium and midportion of the glomerular,
tubular, and vessel wall basement membranes.
Therapy of LCDD is directed at the underlying myeloma.
Cytotoxic chemotherapy followed by hematopoietic stem cell
transplantation (HCT) has met with good success (11,12).
AL amyloidosis
AL amyloidosis occurs when pathogenic light chains unfold
and deposit as insoluble fibrils extracellularly within tissues. It
is found in up to 15% of patients with MM on autopsy (13,14).
In 40% of patients with AL amyloidosis, the bone marrow will
have .10% plasma cells, although only 10% will meet other
criteria for MM (15). Amyloid fibrils may deposit within any
organ, but most commonly affect the kidneys, heart, liver, and
peripheral nervous system.
Patients often present with fatigue, weight loss, and nephrotic
syndrome. The clinical characteristics of patients with biopsyproven renal amyloidosis were described in a retrospective review
of 84 patients at the Mayo Clinic (15). The median age at diagnosis was 61 years, and 62% were male. The median serum
creatinine on presentation was 1.1 mg/dL. The majority of patients had nephrotic syndrome (86%) with a median 24-hour
American Society of Nephrology
protein loss of 7 g/day. RRT was eventually required in 42% of
patients, and median survival after starting dialysis was less than
1 year. In general, cardiac involvement occurs in nearly one-third
of patients and portends a poor prognosis.
AL amyloid presents as an amorphic hyaline substance
within the mesangium, glomerular basement membranes, and
vessel walls. Mesangial involvement may be diffuse or nodular.
Amyloid stains positive for Congo red and reveals a characteristic apple-green birefringence under polarized light. Immunofluorescence staining reveals the underlying monotypic
light chain, which has a l: k ratio of 6:1. Electron microscopy
demonstrates nonbranching randomly oriented 8- to 10-nm
fibrils (Figure 3) (8). Amyloid deposits may appear as subepithelial spikes along the basement membrane similar to
membranous nephropathy.
Treatment with high-dose melphalan followed by HCT
increases hematologic response and overall median survival
(16). Improvement in renal function highly correlates with
increased survival (17).
Treatment of cast nephropathy
General measures
Volume resuscitation to assure optimum hemodynamic
support and adequate urine output (;3 L/day) are of critical
Onco-Nephrology Curriculum
3
Figure 3. Pathology of kidney AL amyloidosis. (A) There is extensive global mesangial and segmental glomerular capillary wall
deposition of acellular, PAS-negative amyloid deposits. For comparison, the hyaline casts depicted are PAS positive (3,4003). (B) By
definition, amyloid deposits should be Congo red positive (i.e., stain red). The figure shows global glomerular and extensive interstitial
Congo red–positive amyloid. (3,2003). (C) Congophilic amyloid deposits characteristically show an apple-green birefringence when
viewed under polarized light (3,2003). (D) On immunofluorescence, amyloid deposits appear smudgy and only stain for one of the light
chains. The figure shows global mesangial and segmental glomerular capillary wall staining for l (3,4003). Staining for k was negative
(not shown). (E) A distinctive feature of kidney AL amyloidosis is glomerular amyloid spicules, which result from parallel alignment of
amyloid fibrils in the subepithelial space perpendicular to the glomerular basement membrane (electron microscopy, 320,0003). (F) On
high magnification, amyloid fibrils appear haphazardly oriented and measure between 7 and 12 nm in diameter (electron microscopy,
326,0003). AL, immunoglobulin light chain; PAS, periodic acid–Schiff. Reprinted from reference 8, with permission of the Elsevier
Science and Technology Journals.
importance in the initial management. Based on experimental evidence that furosemide promotes intratubular cast
formation by increasing sodium delivery to the distal tubule,
the use of loop diuretics should be avoided unless there is
volume overload. Hypercalcemia should be aggressively
treated because it can lead to renal vasoconstriction, volume
depletion, and enhanced cast formation. It has been suggested that urinary alkalinization decreases cast formation by
reducing the net positive charge of FLCs and the interaction
with THP (18). However, there is no clinical data supporting
this approach. Given the risk of causing renal calcium precipitation in the setting of hypercalcemia, urinar y
4
Onco-Nephrology Curriculum
alkalinization cannot be recommended. Colchicine was
shown to reduce cast formation through decreasing THP
secretion and binding in rats, but human studies have
been disappointing (19,20).
Chemotherapy and stem cell transplantation
The key to treating myeloma cast nephropathy is rapid
reduction in FLC concentrations. An early decrease in FLC
levels is associated with the highest rate of renal recovery. In
severe AKI due to cast nephropathy, a 60% reduction in FLC
levels by day 21 after diagnosis is associated with renal recovery
in 80% of cases (21). Previous studies with conventional
American Society of Nephrology
chemotherapy protocols demonstrated that high-dose dexamethasone rapidly reduced FLCs. Newer, novel agents such as
thalidomide and the proteasome inhibitor, bortezomib, also
rapidly lower FLC concentrations; this has been referred to as
“renoprotective chemotherapy.”
Significant improvement in renal dysfunction has been
reported for MM patients treated with bortezomib-based
regimens (22–24). Reversal of renal dysfunction with bortezomib may be more frequent and rapid than with other agents,
based on observational analysis. No dose adjustment for renal
function is necessary for bortezomib.
Thalidomide and lenalidomide are two related chemotherapeutic agents commonly used in the treatment of MM.
Lenalidomide dose must be adjusted for renal dysfunction
(25). Thalidomide is not dependent on renal function for
clearance and does not require dose adjustment for renal
function; however, it may predispose to hyperkalemia in the
setting of renal failure (26,27). Regimens with thalidomide or
lenalidomide have shown superior effectiveness to traditional
therapy with alkylating agents in terms of reversing renal dysfunction in MM; these agents may be nearly as effective as
bortezomib regimens (28). Their effects are likely due to rapid
lowering of serum FLC levels.
Hematopoietic stem cell transplantation is an important
and potentially curative therapy in MM; however, patient
selection criteria are stringent, and significant renal dysfunction has traditionally excluded patients from transplantation.
Recent studies have shown that HCT may be safe and effective
in highly selected patients with renal failure (29).
Extracorporeal removal of free light chains
Light chains are small-molecular-weight proteins. k light
chains usually circulate as monomers with a molecular weight
of 22.5 kDa, whereas l light chains are typically dimeric with a
molecular weight of 45 kDa (30). Because of their size, there
has been a keen interest in the use of extracorporeal therapy
as a means of FLC removal.
A. Therapeutic plasma exchange (TPE):
Several small trials initially suggested that TPE was effective in rapidly
lowering FLC concentrations and improving renal function. However, these studies were small, single center, and underpowered. The
largest randomized controlled trial of TPE did not demonstrate any
benefit in patients with cast nephropathy (31). This study assessed the
benefit of five to seven TPE sessions in 104 patients (30% were dialysis requiring) with presumed cast nephropathy (not all patients
had biopsy confirmation). There was no difference in the two groups
with respect to the composite outcome of death, dialysis, or reduced
renal function at 6 months. This lack of benefit may be related to the
volume of distribution of FLCs. Based on their molecular weights,
85% of light chains are confined to the extravascular space (32).
Therefore, a traditional 2-hour TPE session would be ineffective in
removing significant amounts of FLCs because of the excessive rebound effect. Most of the previous trials were performed prior to the
availability of bortezomib-containing regimens. In a recent study of
American Society of Nephrology
14 patients with presumed myeloma kidney treated with bortezomib
and TPE, 12 had complete or partial renal response by 6 months;
however, there were no control patients (33). Although there is still
interest in TPE as a therapy for cast nephropathy, its routine use
cannot be recommended based on the current evidence.
B. High cutoff hemodialysis (HCO-HD):
More recently interest has developed for another method of extracorporeal removal of FLCs: HCO-HD. In this technique, a
hemofilter with a large pore size (45 kDa) is used for extended
periods of time to remove FLCs.
In the largest study of dialysis dependent renal failure secondary to
MM, 67 patients were treated with HCO-HD and chemotherapy
(34). Only 57% of patients had a renal biopsy, of which 87% had
cast nephropathy. Most patients (85%) received combination
chemotherapy with dexamethasone and either bortezomib or
thalidomide. The median number of HCO-HD sessions was 11,
and all patients had extended (.4 hour) treatments. Overall, 63%
of the patients became dialysis independent. The factors that
predicted renal recovery were the degree of FLC reduction at days
12 and 21 and the time to initiating HCO-HD. Unfortunately, this
trial did not have a control group to assess the benefit of HCO-HD
compared with renoprotective chemotherapy alone.
It is not known whether HCO-HD offers any additional benefit
over current chemotherapeutic regimens. Randomized controlled
trials proving the benefit of adding HCO-HD to patients with cast
nephropathy treated with current chemotherapy will be necessary
before its routine use can be recommended.
LEUKEMIA AND LYMPHOMA
The development of kidney disease is common in patients with
lymphoma and leukemia. As with all hospitalized patients,
those with lymphoma and leukemia are at risk for developing
AKI from hypotension, sepsis, or administration of radiocontrast, antifungal, and antibacterial agents. With the presence of cancer, renal injury can also result from chemotherapy,
immunosuppressive drugs, hematopoietic stem cell transplantation, or tumor lysis syndrome. Furthermore, patients
are at risk for renal syndromes specific to the presence of
lymphoma or leukemia including various forms of paraneoplastic glomerulopathies, electrolyte disorders, urinary tract
obstruction, lysozymuria, leukostasis, and infiltration of renal
parenchyma (Table 2). Several of these manifestations are
discussed elsewhere in the Curriculum.
Lymphomatous infiltration
Background
Renal involvement in lymphoma is often clinically silent so
patients can present with slowly progressive CKD attributed to
other etiologies. Therefore, a high index of suspicion is needed
to make a diagnosis. Patients may present with AKI, but this is
rare and is most commonly seen in highly malignant and
Onco-Nephrology Curriculum
5
Table 2. Kidney manifestations of lymphoma and leukemia
Comorbid factors
Sepsis and shock
Hypotension
Volume depletion
Radiocontrast administration
Chemotherapeutic agents
Anti-infective agents
Drug- induced tubulointerstitial nephritis
Hematopoietic stem cell transplantation
Radiotherapy
Disease-related factors
Tumor lysis syndrome
Thrombotic microangiopathy
Malignancy-associated hypercalcemia
Paraneoplastic glomerulopathies
Urinary tract obstruction
Parenchymal infiltration
Lysozymuria
Leukostasis
disseminated disease (35–38). Other presentations include
proteinuria in both the nephrotic and nonnephrotic range,
as well as a variety of glomerular lesions including pauciimmune crescentic glomerulonephritis (39). Patients may also
present with flank pain and hematuria.
Although a variety of cancers can metastasize to the kidneys
and invade the parenchyma, the most common malignancies to
do so are lymphomas and leukemia. The true incidence of renal
involvement is unknown because it is usually a silent disease and
only occasionally causes renal impairment. Autopsy studies
suggest renal involvement occurs in 90% of patients with
lymphoma, whereas radiographic evidence is significantly lower.
The cause of impaired renal function from lymphomatous
infiltration is poorly understood. Based on biopsy series, patients who present with AKI have predominantly bilateral
interstitial infiltration of the kidneys with lymphoma cells and
uniformly have increased renal size on radiographic imaging
(40). These findings suggest that increased interstitial pressure
results in reduced intrarenal blood flow with subsequent renal
tubular injury. Patients who present with proteinuria, on the
other hand, often have intraglomerular infiltration with lymphoma (40). It is not known how proteinuria develops in these
cases, but the local release of permeability factors and cytokines has been suggested (41,42).
Diagnosis
The diagnosis of lymphomatous infiltration is necessarily one
of exclusion because more common explanations are often
present. Renal ultrasonography or computed tomography
(CT) scan may reveal diffusely enlarged kidneys sometimes
with multiple focal lesions (Figure 4) (43). However, many
times, radiology will be unrevealing. In a study of 668 consecutive patients with lymphoproliferative disease who underwent diagnostic imaging with a CT scan, only 3% with
6
Onco-Nephrology Curriculum
Figure 4. Computed tomography scan of abdomen and pelvis
without radiocontrast showing bilateral kidney enlargement.
Reprinted from reference 43, with permission of the Elsevier
Science and Technology Journals.
non-Hodgkin lymphoma were found to have kidney abnormalities (44). Both diffuse enlargement and solitary lesions
were detected. This discrepancy between radiologic and autopsy/
histopathologic results may due to the fact that renal involvement is often indolent and only detectable in histopathologic
examination (Figure 5) (43). Due to increased metabolic activity
within lymphomatous deposits, positron emission tomography
may be a more sensitive imaging technique (45). Although definitive diagnosis depends on renal biopsy, this procedure often
is impossible because of contraindications. In such cases, the
following criteria support the diagnosis of kidney disease due
to lymphomatous infiltration: 1) renal enlargement without obstruction; 2) absence of other causes of kidney disease; and 3)
rapid improvement of kidney function after radiotherapy or
systemic chemotherapy.
Treatment
The treatment of lymphomatous involvement of the kidney is
directed at the underlying malignancy. There are numerous
case reports of improvement in renal function after initiation of
antitumor therapy. In indolent malignant disease that is usually
treated by observation alone, kidney involvement is an indication for starting systemic therapy.
Leukemic infiltration
Background
Leukemia cells can infiltrate any organ, and the kidneys are the
most frequent extramedullary site of infiltration. Autopsy
studies reveal that 60%–90% of patients have renal involvement (46). On biopsy, cells are usually located in the renal
interstitium, although occasional glomerular lesions are noted
(47). Increased interstitial pressure leads to vascular and tubular compression and subsequent tubular injury. Occasional
nodular lesions are found, but this is more common with
lymphoma.
American Society of Nephrology
Figure 5. (A) Kidney biopsy stained with hematoxylin and eosin at 6033 magnification. (B) Tissue stained with B cell–specific
marker anti-CD38 at 20033 magnification. Reprinted from reference 43, with permission of the Elsevier Science and Technology
Journals.
Clinical Features
Leukemic infiltration of the kidneys is often an indolent and
clinically silent disease. Most often it is incidentally noted after
autopsy or by detection of renal enlargement on ultrasound or
CTscan. Although uncommon, many cases of AKI attributable
to leukemic infiltration have been described (48–50). Patients
may also experience hematuria or proteinuria. Occasionally
renal enlargement is accompanied by flank pain or fullness.
There are also reports of patients with chronic lymphocytic
leukemia who develop AKI from leukemic infiltration and are
infected with polyomavirus (BK) (51). Urine from patients
demonstrates viral inclusions in tubular cells (“decoy” cells)
and blood is positive for BK viral DNA. Therefore, in leukemia
patients with AKI considered due to leukemic infiltration,
evidence for coexisting BK virus infection should be sought.
Diagnosis
The diagnosis of leukemia infiltration as a cause of AKI
requires a high level of vigilance because it is often clinically
silent, and leukemic patients usually have multiple alternative
explanations for renal injury. A presumptive diagnosis can be
made if there is no other obvious cause of AKI, bilateral renal
enlargement is demonstrated radiographically, and there is
prompt improvement in renal function after chemotherapy.
Screening for leukemic infiltration with radiographic imaging
is not revealing. In a study of 668 consecutive patients with
lymphoproliferative disease who underwent diagnostic imaging with a CT scan, only 5% with leukemia were found to have
kidney abnormalities (45). As with lymphoma, this discrepancy between radiologic and autopsy/histopathologic results
may be due to the fact that renal involvement is often indolent
and only detectable during histopathologic examination.
Treatment
Treatment is directed by the type of leukemia. Although some
patients do not recover, in the majority of cases, renal function
does improve as the leukemia responds to systemic treatment.
American Society of Nephrology
Lysozymuria
Lysozyme is a cationic protein produced by macrophages and
monocytes and released in response to bacterial infection. It is
freely filtered by the glomerulus and reabsorbed by the
proximal tubule. In certain leukemias, clonal expansion leads
to an excessive production of lysozyme and subsequent
proximal tubular injury and AKI (52). Damage to the proximal tubule reduces reabsorption and can result in Fanconi
syndrome and nephrotic range proteinuria. The presence of
lysozymuria can be confirmed by detection of an increased g
globulin level on serum and urine protein electrophoresis with
immunofixation negative for monoclonal gammopathy (53).
Treatment is directed at the underlying malignancy.
Leukostasis
Patients with myeloid leukemia and exceedingly high white
blood cell counts (usually in excess of 100,000 cells/mm3) can
develop organ dysfunction due to intravascular aggregation of
leukemic cells. The pulmonary and cerebral circulations are
the most severely affected, although there are case reports of
patients developing AKI (54,55). Leukemic cells occlude the
peritubular and glomerular capillaries, thereby reducing GFR.
Patients may be oliguric, but their renal function often improves with therapeutic leukopheresis or chemotherapy.
Leukostasis is thought to result from the abnormal morphology
of blast cells and the hyperviscocity of the serum. It has been
described in both acute and chronic leukemia. Treatment is
directed at treatment of the malignancy with appropriate chemotherapeutic regimens; in severe cases, therapeutic leukopheresis can rapidly lower cell counts.
CONCLUSION
Numerous kidney diseases are associated with hematologic
malignancies unique to this population (Tables 1 and 2). The
most common cause of kidney injury in MM patients is cast
nephropathy (myeloma kidney). Rapid reduction of
Onco-Nephrology Curriculum
7
circulating free light chains with newer “renoprotective chemotherapy” can reverse renal failure in the majority of cases.
Although infiltration of the kidneys by leukemia or lymphoma
is almost universal histologically, clinical renal disease is uncommon. Given the myriad of potential kidney insults in these
patients, a high index of suspicion is necessary to diagnose
infiltration as the cause of renal dysfunction.
13.
14.
TAKE HOME POINTS
15.
c Cast nephropathy is the most common cause of renal failure in patients
with multiple myeloma.
16.
c Rapid lowering of free light chains with “renoprotective” chemotherapy
can reverse renal failure from cast nephropathy in the majority of cases.
c Based on current evidence, extracorporeal removal of free light chains
with either therapeutic plasmapheresis or HCO hemodialysis cannot be
17.
recommended.
c Leukemic and lymphomatous infiltration of the kidneys is common on
autopsy, although it is usually not clinically apparent.
c Enlarged kidneys on imaging and resolution of AKI after therapy with
18.
systemic chemotherapy or radiation support the diagnosis of kidney
infiltration.
REFERENCES
1. Kyle RA, Gertz MA, Witzig TE, Lust JA, Lacy MQ, Dispenzieri A, Fonseca
R, Rajkumar SV, Offord JR, Larson DR, Plevak ME, Therneau TM, Greipp
PR. Review of 1027 patients with newly diagnosed multiple myeloma.
Mayo Clinic Proc 78: 21–33, 2003
2. Eleutherakis-Papaiakovou V, Bamias A, Gika D, et al. Renal failure in
multiple myeloma: Incidence, correlations, and prognostic significance. Leukemia Lymphoma 48: 337–341, 2007
3. Markowitz GS. Dysproteinemia and the kidney. Adv Anatomic Pathol
11: 49–63, 2004
4. Montseny JJ, Kleinknecht D, Meyrier A, Vanhille P, Simon P, Pruna A,
Eladari D. Long-term outcome according to renal histological lesions
in 118 patients with monoclonal gammopathies. Nephrol Dialysis
Transplant 13:1438–1445, 1998
5. Sanders PW. Mechanisms of light chain injury along the tubular nephron. J Am Soc Nephrol 23: 1777–1781, 2012
6. Batuman V. Proximal tubular injury in myeloma. Contribut Nephrol 153:
87–104, 2007
7. Sanders PW, Herrera GA, Lott RL, Galla JH. Morphologic alterations of
the proximal tubules in light chain-related renal disease. Kidney Int 33:
881–889, 1988
8. Leung N, Nasr SH. Myeloma-related kidney disease. Adv Chronic
Kidney Dis 21: 36–47, 2014
9. Lin J, Markowitz GS, Valeri AM, Kambham N, Sherman WH, Appel GB,
D’Agati V D. Renal monoclonal immunoglobulin deposition disease:
The disease spectrum. J Am Soc Nephrol 12:1482–1492, 2001
10. Pozzi C, D’Amico M, Fogazzi GB, Curioni S, Ferrario F, Pasquali S,
Quattrocchio G, Rollino C, Segagni S, Locatelli F. Light chain deposition disease with renal involvement: Clinical characteristics and
prognostic factors. Am J Kidney Dis 42: 1154–1163, 2003
11. Girnius S, Seldin DC, Quillen K, Dember LM, Segal A, Sanchorawala V.
Long-term outcome of patients with monoclonal Ig deposition disease
treated with high-dose melphalan and stem cell transplantation. Bone
Marrow Transplant 46: 161–162, 2011
12. Hassoun H, Flombaum C, D’Agati VD, Rafferty BT, Cohen A, Klimek
VM, Boruchov A, Kewalramani T, Reich L, Nimer SD, Comenzo RL.
8
Onco-Nephrology Curriculum
19.
20.
21.
22.
23.
24.
25.
26.
27.
28.
High-dose melphalan and auto-SCT in patients with monoclonal Ig
deposition disease. Bone Marrow Transplant 42: 405–412, 2008
Ivanyi B. Frequency of light chain deposition nephropathy relative to
renal amyloidosis and Bence Jones cast nephropathy in a necropsy
study of patients with myeloma. Arch Pathol Lab Med 114: 986–987,
1990
Oshima K, Kanda Y, Nannya Y, Kaneko M, Hamak T, Suguro M,
Yamamoto R, Chizuka A, Matsuyama T, Takezako N, Miwa A, Togawa A,
Niino H, Nasu M, Saito K, Morita T. Clinical and pathologic findings
in 52 consecutively autopsied cases with multiple myeloma. Am J
Hematol 67: 1–5, 2001
Kyle RA, Gertz MA. Primary systemic amyloidosis: Clinical and laboratory features in 474 cases. Semin Hematol 32: 45–59, 1995
Cibeira MT, Sanchorawala V, Seldin DC, Quillen K, Berk JL, Dember
LM, Segal A, Ruberg F, Meier-Ewert H, Andrea NT, Sloan JM, Finn KT,
Doros G, Blade J, Skinner M. Outcome of AL amyloidosis after highdose melphalan and autologous stem cell transplantation: Long-term
results in a series of 421 patients. Blood 118: 4346–4352, 2011
Leung N, Dispenzieri A, Lacy MQ, Kumar SK, Hayman SR, Fervenza FC,
Cha SS, Gertz MA. Severity of baseline proteinuria predicts renal
response in immunoglobulin light chain-associated amyloidosis after
autologous stem cell transplantation. Clin J Am Soc Nephrol 2: 440–
444, 2007
Holland MD, Galla JH, Sanders PW, Luke RG. Effect of urinary pH and
diatrizoate on Bence Jones protein nephrotoxicity in the rat. Kidney Int
27: 46–50, 1985
Sanders PW, Booker BB, Bishop JB, Cheung HC. Mechanisms of intranephronal proteinaceous cast formation by low molecular weight
proteins. J Clin Invest 85: 570–576, 1990
Cairns HS, Dawnay A, Woolfson RG, Unwin RJ. Evaluation of therapy for
cast nephropathy: Failure of colchicine to alter urinary Tamm Horsfall
glycoprotein excretion in normal subjects. Exper Nephrol 2:257–258,
1994
Hutchison CA, Cockwell P, Stringer S, Bradwell A, Cook M, Gertz MA,
Dispenzieri A, Winters JL, Kumar S, Rajkumar SV, Kyle RA, Leung N.
Early reduction of serum-free light chains associates with renal recovery
in myeloma kidney. J Am Soc Nephrol 22: 1129–1136, 2011
Moreau P, Richardson PG, Cavo M, Orlowski RZ, San Miguel JF,
Palumbo A, Harousseau JL. Proteasome inhibitors in multiple myeloma:
10 years later. Blood 120: 947–959, 2012
Ludwig H, Adam Z, Hajek R, Greil R, Tothova E, Keil F, Autzinger EM,
Thaler J, Gisslinger H, Lang A, Egyed M, Womastek I, Zojer N. Light
chain-induced acute renal failure can be reversed by bortezomibdoxorubicin-dexamethasone in multiple myeloma: Results of a phase II
study. J Clin Oncol 28: 4635–4641, 2010
Dimopoulos MA, Roussou M, Gkotzamanidou M, Nikitas N, Psimenou E,
Mparmparoussi D, Matsouka C, Spyropoulou-Vlachou M, Terpos E,
Kastritis E. The role of novel agents on the reversibility of renal impairment
in newly diagnosed symptomatic patients with multiple myeloma.
Leukemia 27: 423–429, 2013
Chen N, Lau H, Kong L, Kumar G, Zeldis JB, Knight R, Laskin OL.
Pharmacokinetics of lenalidomide in subjects with various degrees of
renal impairment and in subjects on hemodialysis. J Clin Pharmacol 47:
1466–1475, 2007
Eriksson T, Hoglund P, Turesson I, Waage A, Don BR, Vu J, Scheffler M,
Kaysen GA. Pharmacokinetics of thalidomide in patients with impaired
renal function and while on and off dialysis. J Pharm Pharmacol 55:
1701–1706, 2003
Harris E, Behrens J, Samson D, Rahemtulla A, Russell NH, Byrne JL. Use
of thalidomide in patients with myeloma and renal failure may be associated with unexplained hyperkalaemia. Br J Haematol 122: 160–
161, 2003
Roussou M, Kastritis E, Christoulas D, Migkou M, Gavriatopoulou M,
Grapsa I, Psimenou E, Gika D, Terpos E, Dimopoulos MA. Reversibility
of renal failure in newly diagnosed patients with multiple myeloma and
the role of novel agents. Leukemia Res 34: 1395–1397, 2010
American Society of Nephrology
29. Parikh GC, Amjad AI, Saliba RM, Kazmi SM, Khan ZU, Lahoti A, Hosing
C, Mendoza F, Qureshi SR, Weber DM, Wang M, Popat U, Alousi
AM, Champlin RE, Giralt SA, Qazilbash MH. Autologous hematopoietic stem cell transplantation may reverse renal failure in patients with multiple myeloma. Biol Blood Marrow Transplant 15:
812–816, 2009
30. Pratt G. The evolving use of serum free light chain assays in haematology. Br J Haematol 141: 413–422, 2008
31. Clark WF, Stewart AK, Rock GA, Sternbach M, Sutton DM, Barrett BJ,
Heidenheim AP, Garg AX, Churchill DN, Canadian Apheresis G. Plasma
exchange when myeloma presents as acute renal failure: a randomized,
controlled trial. Ann Intern Med 143: 777–784, 2005
32. Hutchison CA, Cockwell P, Reid S, Chandler K, Mead GP, Harrison J,
Hattersley J, Evans ND, Chappell MJ, Cook M, Goehl H, Storr M,
Bradwell AR. Efficient removal of immunoglobulin free light chains by
hemodialysis for multiple myeloma: In vitro and in vivo studies. J Am
Soc Nephrol 18: 886–895, 2007
33. Burnette BL, Leung N, Rajkumar SV. Renal improvement in myeloma
with bortezomib plus plasma exchange. New Engl J Med 364: 2365–
2366, 2011
34. Hutchison CA, Heyne N, Airia P, Schindler R, Zickler D, Cook M,
Cockwell P, Grima D. Immunoglobulin free light chain levels and
recovery from myeloma kidney on treatment with chemotherapy and
high cut-off haemodialysis. Nephrol Dialysis Transplant 27: 3823–
3828, 2012
35. Kanfer A, Vandewalle A, Morel-Maroger L, Feintuch MJ, Sraer JD,
Roland J. Acute renal insufficiency due to lymphomatous infiltration of
the kidneys: Report of six cases. Cancer 38: 2588–2592, 1976
36. Koolen MI, Schipper P, v Liebergen FJ, Kurstjens RM, v Unnik AJ,
Bogman MJ. Non-Hodgkin lymphoma with unique localization in the
kidneys presenting with acute renal failure. Clin Nephrol 29: 41–46,
1988
37. Malbrain ML, Lambrecht GL, Daelemans R, Lins RL, Hermans P, Zachee
P. Acute renal failure due to bilateral lymphomatous infiltrates. Primary
extranodal non-Hodgkin’s lymphoma (p-EN-NHL) of the kidneys: Does
it really exist? Clin Nephrol 42: 163–169, 1994
38. Miyake JS, Fitterer S, Houghton DC. Diagnosis and characterization of
non-Hodgkin’s lymphoma in a patient with acute renal failure. Am J
Kidney Dis 16: 262–263, 1990
39. Henriksen KJ, Hong RB, Sobrero MI, Chang A. Rare association of
chronic lymphocytic leukemia/small lymphocytic lymphoma, ANCAs,
and pauci-immune crescentic glomerulonephritis. Am J Kidney Dis 57:
170–174, 2011
40. Tornroth T, Heiro M, Marcussen N, Franssila K. Lymphomas
diagnosed by percutaneous kidney biopsy. Am J Kidney Dis 42:
960–971, 2003
American Society of Nephrology
41. D’Agati V, Sablay LB, Knowles DM, Walter L. Angiotropic large cell
lymphoma (intravascular malignant lymphomatosis) of the kidney:
Presentation as minimal change disease. Human Pathol 20: 263–268,
1989
42. Agar JW, Gates PC, Vaughan SL, Machet D. Renal biopsy in angiotropic
large cell lymphoma. Am J Kidney Dis 24: 92–96, 1994
43. Luciano RL, Brewster UC. Kidney involvement in leukemia and lymphoma. Adv Chronic Kidney Dis 21: 27–35, 2014
44. Bach AG, Behrmann C, Holzhausen HJ, Katzer M, Arnold D, Spielmann
RP, Surov A. Prevalence and patterns of renal involvement in imaging of
malignant lymphoproliferative diseases. Acta Radiol 53: 343–348,
2012
45. Sheth S, Ali S, Fishman E. Imaging of renal lymphoma: patterns of
disease with pathologic correlation. Radiographics 26: 1151–1168,
2006
46. Suh WM, Wainberg ZA, de Vos S, Cohen AH, Kurtz I, Nguyen MK. Acute
lymphoblastic leukemia presenting as acute renal failure. Nature Clin
Pract Nephrol 3: 106–110, 2007
47. Kowalewska J, Nicosia RF, Smith KD, Kats A, Alpers CE. Patterns of
glomerular injury in kidneys infiltrated by lymphoplasmacytic neoplasms. Human Pathol 42: 896–903, 2001
48. Comerma-Coma MI, Sans-Boix A, Tuset-Andujar E, Andreu-Navarro J,
Perez-Ruiz A, Naval-Marcos I. Reversible renal failure due to specific
infiltration of the kidney in chronic lymphocytic leukaemia. Nephrol
Dialysis Transplant 13: 1550–1552, 1998
49. Pagniez DC, Fenaux P, Delvallez L, Dequiedt P, Gosselin B, Tacquet A.
Reversible renal failure due to specific infiltration in chronic lymphocytic leukemia. Am J Med 85: 579–580, 1988
50. Phillips JK, Bass PS, Majumdar G, Davies DR, Jones NF, Pearson TC.
Renal failure caused by leukaemic infiltration in chronic lymphocytic
leukaemia. J Clin Pathol 46: 1131–1133, 1993
51. Boudville N, Latham B, Cordingly F, Warr K. Renal failure in a patient
with leukaemic infiltration of the kidney and polyomavirus infection.
Nephrol Dialysis Transplant 16: 1059–1061, 2001
52. Patel TV, Rennke HG, Sloan JM, DeAngelo DJ, Charytan DM. A forgotten cause of kidney injury in chronic myelomonocytic leukemia. Am
J Kidney Dis 54: 159–164, 2009
53. Levinson SS, Elin RJ, Yam L. Light chain proteinuria and lysozymuria in a
patient with acute monocytic leukemia. Clin Chem 48: 1131–1132,
2002
54. McKee LC Jr, Collins RD. Intravascular leukocyte thrombi and aggregates as a cause of morbidity and mortality in leukemia. Medicine 53:
463–478, 1974
55. Dietrich PY, Pedraza E, Casiraghi O, Bayle C, Hayat M, Pico JL. Cardiac
arrest due to leucostasis in a case of prolymphocytic leukaemia. Br J
Haematol 78: 122–123, 1991
Onco-Nephrology Curriculum
9
REVIEW QUESTIONS
1. An 80-year-old patient with multiple myeloma presents
with serum uric acid of 12.0 mg/dL, serum calcium of 11.0
mg/dL, and a phosphate of 8.1 mg/dL. Serum creatinine is
4.0 mg/dL, and free l light chains are 3,700 mg/L. Which
ONE of the following therapies is contraindicated in this
patient?
a.
b.
c.
d.
Urinary alkalinization
Intravenous saline
Rasburicase
Allopurinol
Answer: a is correct. There is a lack of evidence to support
the use of urinary alkalinization to decrease cast formation by
reducing the net positive charge of FLCs. In addition, there is
an increased risk of renal calcium precipitation with the presence of hypercalcemia.
2. A 57-year-old man with no medical history is diagnosed
with chronic lymphocytic leukemia with Richter’s transformation and presents to the emergency room with creatinine of 3.61 mg/dL and white blood cell count of 20,000/
mm3. The patient has undergone a renal ultrasound indicating slightly enlarged kidneys 13 cm bilaterally. On
examination he was noted to have hepatosplenomegaly.
Urinalysis was negative for proteinuria and hematuria. He
is planned for chemotherapy pending a renal consult for
his rise in creatinine. Which one of the following likely
explains his renal injury?
a. Urinary obstruction related to retroperitoneal lymphadenopathy
b. Tumor lysis syndrome
c. Leukemic infiltration of the kidney
d. Membranoproliferative glomerulonephritis with C3
and monoclonal immunoglobulin deposition
Answer: c is correct. It is likely leukemic infiltration in
the setting of enlarged kidney and hepatosplenomegaly with
the lack of other possible etiologies for the patient’s renal
dysfunction.
3. A 74-year-old man with chronic myelomonocytic leukemia with a long-standing history of hypertension and
hyperlipidemia is seen in the clinic. Peripheral blood
monocyte count is .1,000/mm3, and splenomegaly is
noted. His creatinine has started to rise in the last 6 months
to 2.0 mg/dL from a baseline of 1.3 mg/dL. Urinalysis indicated 31 protein and 31 glucose. His labs included the
following: white blood cells, 50,000/mm3; hemoglobin, 9.0
g/dL; platelets, 60,000/mm3; serum potassium, 3.0 mEq/L;
calcium, 8.0 mg/dL; phosphorus, 1.8 mg/dL; serum glucose, 80 mg/dL. Renal ultrasound showed normal size
kidneys and no hydronephrosis. Which ONE of the following is the likely cause of this patient’s renal dysfunction?
10
Onco-Nephrology Curriculum
a.
b.
c.
d.
Infiltrative disease secondary to underlying leukemia
Membranous glomerulopathy secondary to neoplasm
Renal vein thrombosis
Lysozyme-induced kidney injury
Answer: d is correct. Lysozyme-induced kidney injury has
been underdiagnosed and underrecognized. Uncontrolled
production of lysozyme secondary to underlying malignancy
predisposes the patient to the damage to the proximal tubule
reduces reabsorption of electrolytes and can result in Fanconi
syndrome and nephrotic range proteinuria. A clue to Fanconi
syndrome is glycosuria with normal serum glucose level.
On electron microscopy, the kidney biopsies have shown an
increase in number and size of lysosomes in the proximal
tubules.
4. A 56-year-old woman with a history of diabetes and hypertension (HTN) presented to the hospital with AKI,
calcium of 12.0 mg/dL, hemoglobin of 10.0 g/dL, and serum albumin of 3.5 g/dL. Urinalysis revealed a specific
gravity 1.015 with trace protein on dipstick examination
and occasional granular cast on microscopic examination.
Renal ultrasonography revealed normal-sized kidneys
without hydronephrosis. Creatinine was noted to be at 5
mg/dL, and blood urea nitrogen was 82 mg/dL with a
normal baseline 6 months ago. Because there was a high
suspicion for multiple myeloma, the patient had a serum
free light chain (FLC) assay to determine monoclonality,
and the free serum l light chains were 1,500 mg/L. Which
of the following treatments is indicated for AKI due to cast
nephropathy?
a. Chemotherapy
b. Hemodialysis using dialyzers that have a high-molecularweight cutoff
c. Plasma exchange therapy
d. All of the above
Answer: a is correct. Based on the current evidence, performing plasma exchange and high cutoff hemodialysis to
treat cast nephropathy cannot be recommended. Significant
and rapid reductions in FLC concentrations using “renoprotective chemotherapy” have been shown of benefit in preserving renal function.
5. A 63-year-old man with a medical history of well-controlled
diabetes and hypertension presents with a creatinine of 2.0
mg/dL, weight loss, fatigue, edema, and worsening peripheral neuropathy. He was noted to have a serum albumin level
of 3.0 g/dL, and spot urine protein to creatinine ratio of 2 g,
with no red blood cells in the urinalysis. Liver function
studies were normal. He had a negative skeletal survey, and a
bone marrow biopsy showed normal cellularity with 1%
plasma cells. Serum protein electrophoresis indicated IgA l
M-protein. Due to the possibility of amyloidosis, the patient
underwent a kidney biopsy which showed a mesangial area
expanded by amyloid fibrils. If the patient had AL-type
American Society of Nephrology
amyloidosis, what is the most appropriate intervention(s)
that would improve overall survival?
e. Chemotherapy plus extracorporeal removal of the Ig
FLC
a. Chemotherapy to reduce monoclonal protein overproduction
b. Plasma exchange to reduce circulating FLC levels
c. High-dose melphalan followed by autologous hematopoietic stem cell transplant
d. Hemodialysis using dialyzers with high-molecularweight cutoff
Answer: c is correct. Based on retrospective studies, patients with AL amyloidosis whom underwent stem cell transplant have been shown to have improved overall survival and
improved quality of life compared with those undergoing
chemotherapy alone. There is no role for plasma exchange
or hemodialysis with a high cutoff filter in the treatment of
amyloidosis.
American Society of Nephrology
Onco-Nephrology Curriculum
11
Chapter 8: Clinical Tests for Monoclonal Proteins
Nelson Leung, MD
Division of Nephrology and Hypertension, Hematology, Mayo Clinic, Rochester, Minnesotta
INTRODUCTION
SERUM PROTEIN ELECTROPHORESIS
Monoclonal gammopathy is a hallmark of plasma
cell dyscrasias and some B-cell lymphoproliferative disorders (1). They cover a wide spectrum of diseases from the premalignant condition
monoclonal gammopathy of undetermined significance (MGUS) to symptomatic multiple
myeloma, malignant lymphomas, and chronic
lymphocytic leukemia (CLL). The monoclonal
(M) proteins can be the entire immunoglobulin,
light chain only, or, rarely, heavy chain only. Their
ability to cause kidney disease is another characteristic they have in common. In a disease
such as multiple myeloma, the risk of AKI correlates with the severity of disease and can be as high
as 50% (2,3). In one study, 87% of patients with
AKI had the most advanced stage (III) of disease
according to the Durie Salmon classification (4).
In fact, only 44% of the patients with stage III
disease had normal renal function. Other less
common causes of AKI in this population include
interstitial nephritis and acute tubular necrosis
(5–7).
A number of glomerular and tubular lesions have
also been described in myeloma patients; however, these lesions are actually more common in
patients where the diagnostic criteria for multiple
myeloma or lymphoma have not been met and are
diagnosed with monoclonal gammopathy of renal
significance (MGRS) (8). Patients with MGRSrelated kidney disease are more likely to present
with proteinuria, hematuria, and mild renal impairment than rapid-onset AKI as in cast nephropathy. In either situation, the identification of a
monoclonal protein changes the diagnosis, pathophysiology, and prognosis and directs the clinician toward a hematologic evaluation (9).
Monoclonal protein testing should be a part of
any workup of AKI, as well as proteinuria with
mild reduction of renal function in adults. This
article will review the current available tests for
monoclonal proteins.
Serum protein electrophoresis (SPEP) is the most
commonly used laboratory test for the detection of
monoclonal proteins. Serum proteins are loaded
on to a gel or a capillary tube and are separated by
an electrical current based on charge and size. The
proteins are then stained for visualization. The
proteins migrate into five zones or fractions. These
are albumin, a1, a2, b, and g. The b fraction often
has two peaks. Albumin is the most abundant protein
in the serum and should make up the largest peak in
normal serum. When a monoclonal (M) protein is
present, a sharp band appears often in the g region.
However, M proteins can migrate to the b or even the
a fractions. This often occurs when the M protein is
comprised of an IgA or free light chain (FLC). In
some cases, no band is detected but instead there
is a decrease in the g peak (10). The hypogammaglobulinemia is due to the monoclonal gammopathy.
Currently, SPEP is the most commonly used test
for M proteins globally because of its ease of use and
relatively low cost. Fully automated systems are
available for both the agarose gel and capillary tube
methods, which have increased reproducibility and
efficiency. Because SPEP is quantitative, it is used in
both diagnostic and response criteria in multiple
myeloma (11,12). Despite its utility, its detection limit
is not sensitive enough as a single screening test, especially in low-burden diseases like MGRS. The detection
limit for an M protein is 0.3–0.5 g/dL in the g region
and up to 0.7 g/dL in the a or b region (13). SPEP is
positive in 87.6% of multiple myelomas but only
73.8% of immunoglobulin light chain (AL) amyloidosis
and 55.6% of light chain deposition disease (LCDD)
(14). In addition, the M-band on the SPEP only indicates an M protein but it does not distinguish the isotypes. To find out the type, immunofixation is required.
American Society of Nephrology
Correspondence: Nelson Leung, Mayo Clinic, Rochester, Minnesotta: Q:1
Email: [email protected]
Copyright © 2016 by the American Society of Nephrology
Onco-Nephrology Curriculum
1
Table 1. Screening panel
Condition
SPEP
SIFE
SFLC
UPEP/IFE
MGRS
AL/MIDD
WM
Multiple myeloma
X
X
X
X
X
X
X
X
X
X
X
X
Immunofixation should be performed if a screening test is positive to help
type the monoclonal protein. SPEP, serum protein electrophoresis; SIFE,
serum immunofixation; SFLC, serum free light chain; U, urine; MGRS,
monoclonal gammopathy of renal significance; AL, AL amyloidosis; MIDD,
monoclonal immunoglobulin deposition disease; WM, Waldenström macroglobulinemia.
URINE PROTEIN ELECTROPHORESIS
The first person to recognize that monoclonal proteins have
special properties in the urine was Dr. William MacIntyre (15).
Urine of myeloma patients turns opaque when boiled, clears
with the addition of acid, and turns opaque again as it cools.
The property was attributed to a protein that was named after
the physician who misidentified it: Dr. Henry Bence Jones. It
was not until later that Bence-Jones protein was identified as
immunoglobulin FLC. Urine protein electrophoresis (UPEP)
uses the same principle as SPEP and has the same advantages
and disadvantages. The urine M-spike is used for diagnosis
and response determination in multiple myeloma (16). However, because M proteins are not always present in the urine of
patients with monoclonal gammopathy, the sensitivity of
UPEP is the lowest of all the tests and should never be used
alone for screening. In a study with 2,799 patients of which
4.4% had a plasma cell dyscrasia, SPEP was positive in 94.4%
of patients, whereas only 37.7% had a positive UPEP (17). This
has led some to suggest that the serum FLC assay should replace
the UPEP for screening (see below). However, despite the low
sensitivity, UPEP does provide additional information. In patients with renal impairment, the presence of an M protein and
low albumin concentration on the UPEP is highly suggestive of
cast nephropathy, whereas a high albumin concentration is more
likely the results of LCDD or AL amyloidosis (18). In addition,
the presence of a monoclonal light chain in the urine significantly increases the risk of renal injury in myeloma patients (19).
Finally, urine M-spike is still used in response determination in
multiple myeloma (11). Therefore, UPEP remains a useful supplemental test in patients with paraproteinemia.
Figure 1. Serum protein electrophoresis (PEL) and immunofixation (IFE). A monoclonal (M) spike was detected in the b
region of the protein electrophoresis. The immunofixation identified a band in the IgA and k lanes that corresponded to the
band in the b region of the protein electrophoresis. The M protein in this case is a monoclonal IgA k. The blurry smudge in IgG
lane indicated that the IgG was polyclonal.
monoclonal protein. In multiple myeloma, serum IFE increased the detection rate from 87.6% to 94.4%. Similarly,
the sensitivity increases from 65.9% to 73.8% in AL amyloidosis. In a mixed group of patients, the sensitivity
increases from 79% to 87%. Not only is the sensitivity improved with IFE, but the type of the monoclonal protein can
be identified. IFE is qualitative and not quantitative. Therefore, for response determination in multiple myeloma and
AL amyloidosis, it is only used for assessment of complete
response. The extra reagents add significantly to the cost,
making IFE less affordable.
SERUM FLC ASSAY
SERUM AND URINE IMMUNOFIXATION
Samples are electrophoresed in parallel lanes in immunofixation (IFE). Antibodies against the heavy and light chains of the
immunoglobulin are then applied to each lane separately. A
reaction forming a sharp band would suggest the presence of
monoclonal immunoglobulin component (Figure 1). The M
protein composed of the entire monoclonal immunoglobulin
would be positive for both a heavy chain and a light chain.
The sensitivity of IFE has a detection limit of ;0.1 g/dL of
2
Onco-Nephrology Curriculum
In the early 2000s, a new assay was introduced to aid in the detection
of monoclonal proteins. Using antibodies against epitopes that are
normally hidden in the intact immunoglobulin, the assay detects
both k and l FLCs. It is quantitative and automated. The assay is
not specific for monoclonal light chains but instead infers monoclonality when an abnormal ratio between the k and l FLCs is
detected. The normal ratio is between 0.26 and 1.65. A high ratio
suggests a k clone, whereas a ratio ,0.26 suggest a l clone.
The addition of the serum FLC assay to SPEP and serum IFE
has significantly increased the sensitivity of monoclonal protein
American Society of Nephrology
testing. Prior to the introduction of the FLC assay, up to 5% of
multiple myelomas were thought to be nonsecretory. Using the
FLC assay, 19 of 28 nonsecretory myeloma patients were found to
have abnormal k to l ratios, and 4 had suppression of one or
both FLCs (20). The FLC assay was able to identify abnormalities
in 82% of patients classified as nonsecretory by serum and urine
PEP and IFN. Most of these were light chain–only myeloma.
Recently, the serum FLC ratio was added to the diagnostic criteria
of multiple myeloma (12). In AL amyloidosis, the serum FLC
assay increases the detection rate from 69% (with serum IFE
alone) to 99% (21). The addition of UPEP did not identify any
additional patients. Another study found the combination of
serum IFE and serum FLC detected 100% of the patients with
multiple myeloma, Waldenström macroglobulinemia, and
smoldering multiple myeloma (14). The addition of UPEP did
not increase the sensitivity for the above diagnoses but did assist
in the identification of MGUS, extramedullary plasmacytoma,
AL amyloidosis, and LCDD.
In addition to diagnostic evaluation, serum FLC is also used
in disease monitoring and assessment of response. The degree
and speed of serum FLC reduction have been found to be the
most important predictors of renal recovery in cast nephropathy (22). In AL amyloidosis, the reduction of serum FLC has
been shown to be a better predictor of outcomes than the
M-spike. To achieve stringent complete response in multiple
myeloma and complete response in AL amyloidosis, the serum
FLC ratio has to be normalized (16,23).
Although the serum FLC assay increases the detection rate
of monoclonal gammopathy, clinicians should be aware of
its limitations. First, the assay does not distinguish between
polyclonal FLCs and monoclonal FLCs. The higher (or lower)
the k to l ratio, the more likely a monoclonal gammopathy
exists. However, several conditions are known to cause minor
abnormalities. The most common is renal insufficiency. Because FLCs are mostly cleared by the kidney, a reduction in
glomerular filtration rate will cause a rise in the FLC levels.
This rise, however, is not symmetric because k FLCs are
cleared more readily than l. Thus, the asymmetric increase
in FLCs results in an increase in the ratio. In patients with
severe renal impairment, a renal reference range for the k to
l ratio (0.37 to 3.1) has been recommended (24). Patients with
autoimmune diseases can also have mildly abnormality k to l
ratios. Finally, look for a biclonal gammopathy if there is elevation in both FLCs but the renal function is normal and
autoimmune disorders have been ruled out.
URINARY FLC
The measurement and quantification of FLC can also be performed in the urine. An elevated k to l ratio suggests a k clone,
and a depressed ratio suggests a l clone (25). Studies suggest
that the urinary FLC can correlate with the serum FLC in an
individual patient (26). However, urinary FLC excretion does
not always increase even in patients with elevated serum FLC
American Society of Nephrology
levels (27). Thus, urinary FLC levels do not appear to contribute to diagnostic sensitivity of the current monoclonal testing
regimen.
SCREENING AND MONITORING
The screening test(s) for any disease must have sufficient sensitivity to identify as many patients as possible but also cost
effective enough to apply to the general population. Currently, no single test is capable of accomplishing these goals in
monoclonal gammopathy. This is especially true for diseases
with very low levels of monoclonal protein (28). However, in
diseases such as multiple myeloma and Waldenström macroglobulinemia, where the monoclonal protein is usually abundant, the combination of serum IFE and serum FLC has been
shown to be nearly 100% sensitive (14). In diseases with lower
levels of monoclonal gammopathy, urine IFE can increase the
sensitivity but at a higher cost (Table 1).
For monitoring, the goals are different. Typically, the type
of monoclonal protein is no longer important so IFE is not
routinely required. The response is typically based on the
reduction of the M protein. Depending on the M protein, this
is done with SPEP, UPEP, and/or serum FLC assay. IFE is required when the M-spike is no longer detectable to evaluate
for complete response. In multiple myeloma, FLC should be
followed even when patient has a M-spike because of the
phenomenon light chain escape. This occurs in some heavily
treated patients where the clonal evolution produces a clone
that makes more FLC than the intact immunoglobulin (29). In
these patients, the M-spike would remain low, suggesting persistent response but the involved FLC will rise rapidly.
TAKE HOME POINTS
c Monoclonal protein testing is an important diagnostic tool for the
evaluation of AKI and proteinuria.
c The Serum free light chain assay significantly increases the detection
rate of monoclonal protein when added to serum immunofixation.
c Urine protein electrophoresis can help distinguish between tubular or
glomerular injury in patients with multiple myeloma.
REFERENCES
1. Kyle RA, Therneau TM, Rajkumar SV, Larson DR, Plevak MF, Offord JR,
Dispenzieri A, Katzmann JA, Melton LJ 3rd. Prevalence of monoclonal
gammopathy of undetermined significance. N Engl J Med 354: 1362–
1369, 2006
2. Knudsen LM, Hippe E, Hjorth M, Holmberg E, Westin J. Renal function
in newly diagnosed multiple myeloma: A demographic study of 1353
patients. The Nordic Myeloma Study Group. Eur J Haematol 53: 207–
212, 1994
3. Ivanyi B. Frequency of light chain deposition nephropathy relative to
renal amyloidosis and Bence Jones cast nephropathy in a necropsy
study of patients with myeloma. Arch Pathol Lab Med 114: 986–987,
1990
Onco-Nephrology Curriculum
3
4. Blade J, Fernandez-Llama P, Bosch F, Montoliu J, Lens XM, Montoto S,
Cases A, Darnell A, Rozman C, Montserrat E. Renal failure in multiple
myeloma: Presenting features and predictors of outcome in 94 patients
from a single institution. Arch Intern Med 158: 1889–1893, 1998
5. Nasr SH, Valeri AM, Sethi S, Fidler ME, Cornell LD, Gertz MA, Lacy M,
Dispenzieri A, Rajkumar SV, Kyle RA, Leung N. Clinicopathologic correlations in multiple myeloma: A case series of 190 patients with kidney
biopsies. Am J Kidney Dis 59: 786–794, 2012
6. Pasquali S, Zucchelli P, Casanova S, Cagnoli L, Confalonieri R, Pozzi C,
Banfi G, Lupo A, Bertani T. Renal histological lesions and clinical syndromes in multiple myeloma. Renal Immunopathology Group. Clin
Nephrol 27: 222–228, 1987
7. Rota S, Mougenot B, Baudouin B, De Meyer-Brasseur M, Lemaitre V, Michel
C, Mignon F, Rondeau E, Vanhille P, Verroust P, Ronco P. Multiple myeloma
and severe renal failure: A clinicopathologic study of outcome and prognosis in 34 patients. Medicine (Baltimore) 66: 126–137, 1987
8. Leung N, Bridoux F, Hutchison CA, Nasr SH, Cockwell P, Fermand JP,
Dispenzieri A, Song KW, Kyle RA. Monoclonal gammopathy of renal
significance: When MGUS is no longer undetermined or insignificant.
Blood 120: 4292–4295, 2012
9. Bridoux F, Leung N, Hutchison CA, Touchard G, Sethi S, Fermand JP,
Picken MM, Herrera GA, Kastritis E, Merlini G, Roussel M, Fervenza FC,
Dispenzieri A, Kyle RA, Nasr SH. Diagnosis of monoclonal gammopathy
of renal significance. Kidney Int 87: 698–671, 2015
10. O’Connell TX, Horita TJ, Kasravi B. Understanding and interpreting
serum protein electrophoresis. Am Fam Physician 71: 105–112, 2005
11. Durie BG, Harousseau JL, Miguel JS, Blade J, Barlogie B, Anderson K,
Gertz M, Dimopoulos M, Westin J, Sonneveld P, Ludwig H, Gahrton G,
Beksac M, Crowley J, Belch A, Boccadaro M, Cavo M, Turesson I,
Joshua D, Vesole D, Kyle R, Alexanian R, Tricot G, Attal M, Merlini G,
Powles R, Richardson P, Shimizu K, Tosi P, Morgan G, Rajkumar SV.
International uniform response criteria for multiple myeloma. Leukemia
20: 1467–1473, 2006
12. Rajkumar SV, Dimopoulos MA, Palumbo A, Blade J, Merlini G, Mateos
MV, Kumar S, Hillengass J, Kastritis E, Richardson P, Landgren O, Paiva
B, Dispenzieri A, Weiss B, LeLeu X, Zweegman S, Lonial S, Rosinol L,
Zamagni E, Jagannath S, Sezer O, Kristinsson SY, Caers J, Usmani SZ,
Lahuerta JJ, Johnsen HE, Beksac M, Cavo M, Goldschmidt H, Terpos E,
Kyle RA, Anderson KC, Durie BG, Miguel JF. International Myeloma
Working Group updated criteria for the diagnosis of multiple myeloma.
The Lancet Oncology 15: e538–e548, 2014
13. Gay-Bellile C, Bengoufa D, Houze P, Le Carrer D, Benlakehal M,
Bousquet B, Gourmel B, Le Bricon T. Automated multicapillary electrophoresis for analysis of human serum proteins. Clin Chem 49: 1909–
1915, 2003
14. Katzmann JA, Kyle RA, Benson J, Larson DR, Snyder MR, Lust JA,
Rajkumar SV, Dispenzieri A. Screening panels for detection of monoclonal gammopathies. Clin Chem 55: 1517–1522, 2009
15. Steensma DP, Kyle RA. A history of the kidney in plasma cell disorders.
Contrib Nephrol 153: 5–24, 2007
16. Durie BGM, Harousseau JL, Miguel JS, Blade J, Barlogie B, Anderson K,
Gertz M, Dimopoulos M, Westin J, Sonneveld P, Ludwig H, Gahrton G,
Beksac M, Crowley J, Belch A, Boccadaro M, Turesson I, Joshua D,
Vesole D, Kyle R, Alexanian R, Tricot G, Attal M, Merlini G, Powles R,
Richardson P, Shimizu K, Tosi P, Morgan G, Rajkumar SV, Grp IMW.
International uniform response criteria for multiple myeloma (vol. 20,
pg 1467, 2006). Leukemia 20: 2220–2220, 2006
4
Onco-Nephrology Curriculum
17. McTaggart MP, Lindsay J, Kearney EM. Replacing urine protein electrophoresis with serum free light chain analysis as a first-line test for detecting
plasma cell disorders offers increased diagnostic accuracy and potential
health benefit to patients. Am J Clin Pathol 140: 890–897, 2013
18. Leung N, Gertz M, Kyle RA, Fervenza FC, Irazabal MV, Eirin A, Kumar S,
Cha SS, Rajkumar SV, Lacy MQ, Zeldenrust SR, Buadi FK, Hayman SR,
Nasr SH, Sethi S, Ramirez-Alvarado M, Witzig TE, Herrmann SM,
Dispenzieri A. Urinary albumin excretion patterns of patients with cast
nephropathy and other monoclonal gammopathy-related kidney diseases. Clin J Am Soc Nephrol 7: 1964–1968, 2012
19. Drayson M, Begum G, Basu S, Makkuni S, Dunn J, Barth N, Child JA.
Effects of paraprotein heavy and light chain types and free light chain
load on survival in myeloma: An analysis of patients receiving conventional-dose chemotherapy in Medical Research Council UK multiple
myeloma trials. Blood 108: 2013–2019, 2006
20. Drayson M, Tang LX, Drew R, Mead GP, Carr-Smith H, Bradwell AR.
Serum free light-chain measurements for identifying and monitoring
patients with nonsecretory multiple myeloma. Blood 97: 2900–2902,
2001
21. Katzmann JA, Abraham RS, Dispenzieri A, Lust JA, Kyle RA. Diagnostic
performance of quantitative kappa and lambda free light chain assays
in clinical practice. Clin Chem 51: 878–881, 2005
22. Hutchison CA, Cockwell P, Stringer S, Bradwell A, Cook M, Gertz MA,
Dispenzieri A, Winters JL, Kumar S, Rajkumar SV, Kyle RA, Leung N.
Early reduction of serum-free light chains associates with renal recovery
in myeloma kidney. J Am Soc Nephrol 22: 1129–1136, 2011
23. Palladini G, Dispenzieri A, Gertz MA, Kumar S, Wechalekar A, Hawkins
PN, Schonland S, Hegenbart U, Comenzo R, Kastritis E, Dimopoulos
MA, Jaccard A, Klersy C, Merlini G. New criteria for response to treatment in immunoglobulin light chain amyloidosis based on free light
chain measurement and cardiac biomarkers: impact on survival outcomes. J Clin Oncol 30: 4541–4549, 2012
24. Hutchison CA, Plant T, Drayson M, Cockwell P, Kountouri M, Basnayake
K, Harding S, Bradwell AR, Mead G. Serum free light chain measurement aids the diagnosis of myeloma in patients with severe renal failure.
BMC Nephrol 9: 11, 2008
25. Nakano T, Nagata A, Takahashi H. Ratio of urinary free immunoglobulin
light chain kappa to lambda in the diagnosis of Bence Jones proteinuria. Clin Chem Lab Med 42: 429–434, 2004
26. Abraham RS, Clark RJ, Bryant SC, Lymp JF, Larson T, Kyle RA, Katzmann
JA: Correlation of serum immunoglobulin free light chain quantification
with urinary Bence Jones protein in light chain myeloma. Clin Chem 48:
655–657, 2002
27. Alyanakian MA, Abbas A, Delarue R, Arnulf B, Aucouturier P. Free immunoglobulin light-chain serum levels in the follow-up of patients with
monoclonal gammopathies: Correlation with 24-hr urinary light-chain
excretion. Am J Hematol 75: 246–248, 2004
28. Bhutani G, Nasr SH, Said SM, Sethi S, Fervenza FC, Morice WG, Kurtin
PJ, Buadi FK, Dingli D, Dispenzieri A, Gertz MA, Lacy MQ, Kapoor P,
Kumar S, Kyle RA, Rajkumar SV, Leung N. Hematologic characteristics
of proliferative glomerulonephritides with nonorganized monoclonal
immunoglobulin deposits. Mayo Clinic Proc 90: 587–596, 2015
29. Kuhnemund A, Liebisch P, Bauchmuller K, zur Hausen A, Veelken H,
Wasch R, Engelhardt M. ’Light-chain escape-multiple myeloma’-an
escape phenomenon from plateau phase: Report of the largest patient series using LC-monitoring. J Cancer Res Clin Oncol 135: 477–
484, 2009
American Society of Nephrology
REVIEW QUESTIONS
1. A 68-year-old man presents with a 1-year history of suddenonset nephrotic range proteinuria, easy bruising with
spontaneous “black eyes,” and more recent onset of
dyspnea on exertion. Which of the following tests should
be performed for screening?
a.
b.
c.
d.
e.
Serum protein electrophoresis
Urine protein electrophoresis
Serum immunofixation
Serum free light chain assay
All of the above
Answer: e is correct. The presentation is concerning for
amyloidosis. Because amyloidosis is usually caused by a low
burden plasma cell dyscrasia, all of the tests should be performed to avoid false negativity.
2. Which test is most important in the screening of nonsecretory multiple myeloma?
a.
b.
c.
d.
e.
Urine immunofixation
Complete blood count
Serum creatinine
Serum calcium
Serum free light chain assay
American Society of Nephrology
Answer: e is correct. Prior to the serum free light chain
assay, up to 5% of the multiple myeloma cases were thought
to be nonsecretory. The serum free light chain assay identifies
80% of these cases as light chain myeloma.
3. A 65-year-old man with hypertension, mild anemia, and
CKD stage IV presents with a mildly depressed serum k to
l free light chain ratio of 0.12 (normal 5 0.26–1.65). Serum and urine immunofixation are negative. What is the
most likely possible explanation for his abnormal k to l
free light chain ratio?
a.
b.
c.
d.
e.
CKD
Hypertension
Chronic myelogenous leukemia
Monoclonal gammopathy of undetermined significance
None of the above
Answer: d is correct. CKD and renal impairment result in a
mildly elevated k to l ratio. This is because the reduction of
glomerular filtration reduces the clearance of k free light chain
more than l. Hypertension does not affect free light chain
clearance. Chronic myelogenous leukemia is a myeloid disease
and does not produce monoclonal protein. With the history of
CKD, the low k to l ratio is likely to be due to the presence of a
monoclonal gammopathy.
Onco-Nephrology Curriculum
5
AUTHOR QUERIES
AUTHOR PLEASE ANSWER ALL QUERIES
Q1: Please provide mailing address for correspondence.
Chapter 9: Hematopoietic Stem Cell
Transplant–Related Kidney Disease
Joseph R. Angelo, MD,* and Sangeeta Hingorani, MD, MPH†‡
*Pediatric Nephrology and Hypertension, University of Texas Health Science Center at Houston, University of Texas
MD Anderson Cancer Center, Houston, Texas; †Division of Nephrology, Seattle Children’s Hospital, Seattle,
Washington; and ‡Department of Pediatrics, Division of Nephrology, University of Washington, Seatlle, Washington
INTRODUCTION
Acute and chronic kidney diseases are common
following hematopoietic cell transplantation (HCT)
and can lead to long-term effects. Additionally, the
occurrence of kidney disease in the setting of HCT
can negatively affect mortality and morbidity. Etiologies of HCT-associated kidney injury are often
multifactorial, including conditioning chemotherapy,
radiation, nephrotoxic medications, sepsis, sinusoidal obstruction syndrome (SOS), transplantationassociated thrombotic microangiopathy (TA-TMA),
and graft-versus-host disease (GVHD). Continued
improvement in survival following HCT highlights
the importance of monitoring renal function both
before and after transplant and continued follow-up
of patients with CKD.
AKI
Pretransplant evaluation of renal function
Serum creatinine (SCr) is the most widely used
marker of kidney function in patients undergoing
HCT (1). Measurement of SCr provides estimation
of renal function at the bedside and allows for following trends in renal function. GFR prediction
formulas, such as the Modification of Diet in Renal
Disease (MDRD) equation for adults and the
Schwartz formula in children, are available (2,3).
However, several shortcomings are inherent in the
properties of SCr as a functional biomarker of AKI.
These include the delay between the onset of kidney
injury and an increase in SCr, limiting its utility to
provide the earliest window for intervention (4). In
addition, SCr is affected by factors such as age,
muscle mass, and hydration status, issues particularly relevant for HCT patients. Even small changes
in SCr in this population can represent significant
American Society of Nephrology
decline in kidney function. Other methods of GFR
estimation include 24-hour urine creatinine clearance, inulin clearance, and use of radioactive isotopes
(Tc-DTPA or Cr-EDTA) or iodinated contrast agents
(iothalamate or iohexol).
Defining AKI
Current definitions of AKI are based on increases
in SCr and decreased urine output. Two scoring
systems, RIFLE (risk, injury, failure, loss, ESRD) and
Acute Kidney Injury Network (AKIN), have been
developed to standardize stratification of AKI
severity (5,6). RIFLE criteria include two additional
categories (loss, ESRD) describing two post-AKI
clinical outcomes. For children, a modified version
of RIFLE criteria, pRIFLE, has been developed (7).
Several studies have shown a correlation between
these scores and clinical outcomes (8). Recently, a
new staging criteria for AKI was created by KDIGO;
however, this newer definition has not been prospectively studied in the HCT population.
Epidemiology of AKI
The incidence of AKI varies, based on the definition
of AKI, type of HCT, and chemotherapeutic conditioning regimen. When AKI is defined as a doubling of
SCr during the first 100 days after stem cell infusion,
the prevalence ranges from 15% to 73% (9). Severity
of AKI also varies. In a study of pediatric and adult
allogeneic HCTrecipients, up to a third of all patients
doubled their SCr in the first 100 days, and 5% required acute dialysis (10). Severity of AKI is associated with increased risk of morbidity and mortality
(11–13).
Correspondence: Sangeeta Hingorani, Division of Nephrology,
Seattle Children’s Hospital, 4800 Sand Point Way NE, Seattle,
Washington 98105.
Copyright © 2016 by the American Society of Nephrology
Onco-Nephrology Curriculum
1
For those receiving high-dose conditioning regimens and
allogeneic HCT, the incidence of AKI is as high as 69%. It often
occurs before day 28, and risk factors include lung toxicity, hepatic toxicity, SOS, amphotericin exposure, and sepsis
(14,15).
For patients receiving reduced-intensity chemotherapy
(RIC) and allogeneic HCT, AKI occurs less frequently, later
after transplant, and less often results in the need for dialysis. A
retrospective cohort study found that 47% of RIC patients
developed AKI compared with 73% in the high-dose treatment
group, developing at a median of 26–60 days after transplant
in the RIC group. Fewer RIC patients required dialysis, and
mortality was significantly lower (11,16,17). Compared with
allogeneic HCT, AKI incidence is lower in autologous HCT,
occurring in approximately 21% of these patients (18).
Causes of AKI
Common risk factors and causes of AKI after HCT include
volume depletion, sepsis, nephrotoxic medication exposure,
SOS, and GVHD (Table 1). Owing to a propensity for increased gastrointestinal (GI) fluid losses and poor oral intake,
HCT patients are highly susceptible to volume depletion.
Close tracking of fluid intake, urine output, fluid losses via
the GI tract and insensible losses, and daily weight measurement are required. Additional measures that can discriminate
prerenal AKI from other types include BUN/Cr ratio, fractional excretion of sodium (FENa), and fractional excretion
of urea (FEurea) (Table 2).
Sepsis can result in decreased effective circulating volume
and hypotension and is a major risk factor for AKI. Sepsisinduced inflammation leads to increased capillary permeability and intravascular fluid leak, resulting in total body volume
overload while depleting effective circulating volume and end
organ perfusion (19).
GVHD is unique to HCT and likely causes tissue and
endothelial damage via T cell– and cytokine-mediated injury
(20). The GI mucosa is a common site of GVHD, contributing
to inadequate fluid intake and increased GI losses.
Table 1. Risk factors for AKI in HCT
Intravascular volume depletion
Vomiting and diarrhea associated with acute gut GVHD
Systemic vasodilatation
Sepsis
Renal vasoconstriction
Sinusoidal obstruction syndrome
Calcineurin inhibitors
Endothelial injury
Acute GVHD
Calcineurin inhibitors
Total body irradiation
Thrombotic microangiopathy
Tubular injury
Medications: amphotericin, vancomycin
Conditioning chemotherapy
2
Onco-Nephrology Curriculum
Table 2. Urinary indices for AKI classification
AKI classification
BUN/Cr ratio
Prerenal AKI
Intrinsic AKI
Obstructive AKI
.20
Variable
Variable
FENa*
FEurea†
,1%
.3%
Variable
,35%
.35%
Variable
*FENa 5 (urinary Na 3 SCr)/(serum Na 3 urinary Cr).
†
FEurea 5 (urinary urea 3 SCr)/(serum urea 3 urinary Cr).
SOS and hepatorenal syndrome (HRS) have been identified
as an independent risk factor for AKI. HRS results in decreased
resistance in the systemic and splanchnic vasculature, leading
to renal hypoperfusion and compensatory increase in renal salt
and water reabsorption. It presents as oligo-anuric prerenal
AKI with edema and low urinary sodium. Septic shock and
other causes of AKI must be ruled out. Defibroitide exhibits
antithrombotic and fibrinolytic properties and has been
studied for use in SOS (21).
Common medications related to AKI include vancomycin,
aminoglycosides, and amphotericin. Calcineurin inhibitors
(CNIs) can lead to renal arteriolar vasoconstriction and have
been associated with development of TA-TMA.
Management of AKI
The management of AKI is mainly supportive and specific to
the underlying cause. For situations of renal hypoperfusion,
prompt administration of intravenous fluids is required to
restore effective circulating volume. However, a critical point is
that fluid overload (FO) can itself be an independent predictor
of mortality in critically ill patients (22,23). Stem cell transplant recipients are a population that may be particularly sensitive to FO, with one study of critically ill children suggesting
that .10% FO in HCT patients correlates with decreased survival (%FO 5 [Fluid In – Fluid Out]/Intensive Care Unit
Admission Weight in kilograms) (24,25). Judicious use and
dose adjustment of antimicrobials should be used to decrease
risk of AKI from nephrotoxin exposure. For those not responsive to medical interventions, dialysis is used as supportive
therapy for management of AKI-related fluid and metabolic
derangements.
CKD AFTER HCT
CKD stage 3 is defined as a GFR of ,60 mL/min per 1.73 m2 for
$3 months. This is often the definition of CKD used in studies
of HCT patients. The five stages of CKD (Table 3) range from
mild to ESRD. The prevalence of CKD after HCT, using this
definition, is between 20% and 30% (26). Cohorts including
children and adults have shown a CKD prevalence of 19% at
1 year and 7% at least 2 years after HCT, with a mean estimated
GFR of 46 mL/min per 1.73 m2 (27,28). Other chronic kidney
disorders following HCT include albuminuria, hypertension,
and renal tubular dysfunction. Regardless of underlying cause,
CKD can progress to ESRD and increases mortality risk after
American Society of Nephrology
Table 3. Stages of CKD (67)
Stage
1
2
3a
3b
4
5
2
Description
GFR (mL/min per 1.73 m )
Kidney damage with normal GFR
Mild
Moderate
$90
60–89
30–44
45–59
15–29
,15
Severe
ESRD
HCT. For those who progress to ESRD, this mortality risk can
be as high as 90% (29). Monitoring changes in renal function
and management of any existing chronic renal disease are important to the long-term survival and quality of life for postHCT patients.
monitoring of levels rather than complete cessation of CNI
may be more appropriate (36,37). In patients not responsive to
these interventions, pharmacologic therapies include rituximab, defibrotide, and eculizumab (33,38,39). Eculizumab, a
monoclonal immunoglobulin that binds complement factor
5, has been used for treatment of TA-TMA. A retrospective
analysis of 12 patients with post-HCT TMA treated with eculizumab reported hematologic response of 50% and overall
survival of 33% (40). In some cases, there may be dysregulation of the complement system; elevated levels of C5b-9, the
membrane attack complex, have been identified; and eculizumab has been used in these patients with mixed results (41).
IDIOPATHIC OR GVHD-RELATED CKD
Clinical entities associated with CKD after HCT
Preexisting CKD before HCT
Preexisting CKD is not a contraindication to HCT. A study of
141 adult patients with leukemia and pretransplant kidney
dysfunction showed that, at 1 year, these patients did not have
worse survival than those who initially had normal kidney
function. In addition, for some cancer diagnoses, CKD is
related to the underlying disease process and HCT can slow the
progression of CKD.
Important points in managing patients with preexisting
CKD include accurate assessment of renal function prior to
HCT, appropriate changes to medication dosing, and avoidance of nephrotoxins.
Transplantation-associated TMA
TMA is defined by hemolytic anemia with erythrocyte
fragmentation, thrombocytopenia, and renal failure. It is
characterized by endothelial damage, leading to thickened
glomerular and arteriolar vessels, the presence of fragmented
red blood cells, thrombosis, and endothelial cell swelling (30).
Two consensus guidelines outline the clinical criteria for the
diagnosis of TA-TMA. Both require the presence of schistocytes on peripheral smear and an elevated lactate dehydrogenase. The BMT Clinical Trials Network also includes AKI
(doubling of serum creatinine), unexplained CNS dysfunction,
and a negative Coombs test. The International Guidelines from
the European Group for Blood and Marrow Transplantation
include thrombocytopenia, anemia, and decreased haptoglobin
(31–33).
In the setting of HCT, the incidence of TMA ranges from 2%
to 21% (31). The clinical course of TMA can be rapid with
severe AKI but commonly follows a more indolent course,
resulting in CKD and, possibly, progression to ESRD (34).
Risk factors for the development of TMA after HCT include
CNI use, total body irradiation, and GVHD (35).
The mainstay of TA-TMA management remains reduction
in dose or stoppage of CNI and therapeutic plasma exchange. A
response rate of 50%–63% has been reported using these two
interventions (33). Given that GVHD itself can be a risk factor
for the development of TA-TMA, an approach with close
American Society of Nephrology
Many HCT survivors will not present with a clear etiology for
CKD and are labeled as idiopathic CKD. Some data support a
label of GVHD-related CKD, with renal disease resulting from
T cell– and cytokine-mediated tissue damage related to the
chronic inflammatory state of GVHD (42,43). The presence
of albuminuria in this patient population may be a marker of
the renal involvement of GVHD either directly or indirectly as
described above. Clinically, albuminuria is monitored using
urine albumin to creatinine ratio (ACR) on a spot urine sample. Microalbuminuria is defined as an ACR of 30–300 mg
albumin/g creatinine, whereas macroalbuminuria is defined
by ACR $300 mg/g creatinine. Albuminuria is common after
HCT and can have long-term effects. In a cohort of 142 HCT
patients, 94% developed albuminuria within 100 days after
HCT, 50% developed it at 1 year, and 4% had an ACR demonstrating overt proteinuria. Microalbuminuria at day 100
was associated with a four times greater risk of CKD, and
macroalbuminuria was associated with a seven times greater
risk of nonrelapse mortality (28). In a more recent study, both
micro- and macroalbuminuria in the first 100 days after HCT
were associated with an increased risk of nonrelapse mortality
at 1 year (43). Albuminuria can also provide a readily available
indicator of other underlying pathologic processes, such as
TA-TMA (37).
Recent consensus guidelines recommend screening urinalyses and ACR as part of the day 180 post-HCTevaluation and
then yearly screening after HCT. If macroalbuminuria is present, more frequent monitoring every 3–6 months is indicated
(44). Given its utility as a marker of underlying pathology and
the association between macroalbuminuria and long-term
outcomes, renal biopsy is indicated in patients with persistent
macroalbuminuria.
Glomerular disorders
Glomerular lesions related to HCT are typically discussed in
association with chronic GVHD (cGVHD). HCT-related
glomerular disease results in albuminuria, ranging from
mild to nephrotic range proteinuria. Rarely, post-HCT glomerular disease presents as glomerulonephritis (45). In
Onco-Nephrology Curriculum
3
contrast to albuminuria, more severe glomerular diseases are
less common. Among these, membranous nephropathy (67%)
and minimal change disease (33%) are the two most common
pathologies (45,46). Both tend to occur fairly late after transplant, 8–14 months, and often within several months of development of GVHD or lowering of immunosuppression for
GVHD prophylaxis (27,47). Treatment for HCT-associated nephrotic syndrome is similar to that in other settings, with corticosteroids being most common, as well as resumption of
GVHD prophylaxis with CNIs (27,48). HCT patients with
macroalbuminuria are likely to benefit from antiproteinuric
therapy with angiotensin-converting enzyme inhibitors
(ACE-Is) or angiotensin receptor blockers (ARBs) (27,49).
Hypertension
Elevations in BP are a common complication of HCT. In a
retrospective analysis of a cohort of children and adults followed for a median of 16 years after HCT, the prevalence of
hypertension was 17% (50). Risk factors associated with the development of hypertension include prior AKI, total body irradiation, autologous transplant, TA-TMA, obesity, and diabetes.
Consensus recommendations are for BP measurement at
each clinic visit, with a maximum interval of yearly (51).
Thresholds for treating hypertension in the HCT population
follow those of the general population, as recommended in the
Report from the Panel Members of the Eighth Joint National
Committee. For those .60 years of age, treatment goals are
based on a threshold of $150/90 mmHg. For all other adults
($18 years old), including those with CKD, the threshold for
treatment initiation is $140/90 mmHg (52). In children, the
Fourth Report on the Diagnosis, Treatment, Evaluation, and
Treatment of High Blood Pressure in Children and Adolescents defines hypertension as a systolic or diastolic BP .95th
percentile, based on sex, age, and height (53).
Effective treatment of hypertension can decrease cardiovascular disease risk and slow the progression of CKD. Initial
interventions are lifestyle modifications, including dietary
sodium restriction and regular exercise. Due to antiproteinuric
and renoprotective properties, ACE-I or ARB therapy should
be first choice for pharmacologic treatment of hypertension.
For patients in which ACE-I/ARB use is contraindicated,
choice of antihypertensive should be individualized.
study in pediatric HCT patients reported a 22% prevalance of
hemorrhagic cystitis, developing at a median of 35 days after
HCT, which was associated with worse survival (55). Risks for
developing BK-related complications include unrelated donor, myeloablative conditioning, and GVHD (56).
Despite a reported prevalence of 10%–30% for BK viremia,
less is known about the relationship between BK viremia and
nephropathy in the HCT population. In a study of 124 adult
allogeneic HCT recipients, 65% developed viruria and 17%
developed viremia after a median follow-up of 454 days.
Only 2 of 21 patients had persistent viremia and biopsyproven nephropathy; the remaining cases of viremia were
mild and transient. BK viremia was an independent risk factor
for an increase in post-HCT creatinine (57). In children after
HCT, BK viremia has been reported as more predictive of poor
renal outcomes than viruria, supporting the use of plasma BK
polymerase chain reaction (PCR) levels when monitoring for
the development of nephropathy (58). For those with elevations in SCr suspected to be related to BK nephropathy, definitive diagnosis requires kidney biopsy.
Current treatment options for hemorrhagic cystitis include
pain control, continuous bladder irrigation, and urologic
intervention for clearance of clots causing obstruction. Pharmacologic interventions include cidofovir, leflunomide, and
fluoroquinolones (59). CMX100 is an oral formulation of cidofovir and may have less kidney toxicity (60). Another novel
therapy being investigated is the use of exogenous BK-specific
T cells and manipulation of immunosuppression to maximize
the patient’s own immune response (61,62).
ESRD AND KIDNEY TRANSPLANT AFTER HCT
There are limited data on the risk of ESRD after HCT. The
reported prevalence ranges from 0.4% to 4.4% (63). One study
calculated a risk of end-stage kidney disease as being 16 times
higher than the general population 20 years after HCT (64). For
those progressing to ESRD, dialysis and renal transplant remain
the treatment options. There are several reports of successful
kidney transplantation in both children and adults (65,66).
CONCLUSION
BK nephropathy
BK virus is a double-stranded DNA virus in the polyomavirus
family with a seroprevalence of 80% reported in healthy blood
donors (54). After infection, BK remains dormant in the urothelial cells without clinical effects in immunocompetent
individuals. In the immunosuppressed, BK virus has been associated with nephropathy after both kidney transplant HCT;
however, hemorrhagic cystitis is more common in the HCT
population. In post-HCT patients, BK virus–associated hemorrhagic cystitis occurs in 10%–25% of patients and can lead
to obstructive AKI, long-term urologic dysfunction, need for
invasive intervention, and increased mortality. A prospective
4
Onco-Nephrology Curriculum
HCT patients are a population clearly at risk for the development of kidney disease, necessitating close monitoring with a
multidisciplinary approach involving both oncologists and
nephrologists.
TAKE HOME POINTS
c Acute and chronic kidney problems are common following HCT and are
associated with an increased risk of nonrelapse mortality and a decrease
in overall survival.
American Society of Nephrology
c Micro- and macroalbuminuria are associated with an increased risk of mor-
tality and CKD at 1 year after HCT. It is unclear if the presence of albuminuria
is a marker of systemic or local inflammation or a marker of GVHD.
17.
c Hypertension should be managed with ACE-Is or ARBs.
c A multidisciplinary approach is needed to insure appropriate manage-
ment of renal issues occurring after HCT.
REFERENCES
1. Tichelli A, Royo A, Gratwohl A. Late pulmonary, cardiovascular, and
renal complications after hematopoietic stem cell transplantation and
recommended screening practices. Hematology Am Soc Hematol
Educ Program 2008: 125–133, 2008
2. Schwartz GJ, Brion LP, Spitzer A. The use of plasma creatinine concentration for estimating glomerular filtration rate in infants, children,
and adolescents. Pediatr Clin North Am 34: 571–590, 1987
3. Levey AS, Coresh J, Greene T, Stevens LA, Zhang YL, Hendriksen S,
Kusek JW, Van Lente F; Chronic Kidney Disease Epidemiology Collaboration. Using standardized serum creatinine values in the modification of diet in renal diseases study equation for estimating glomerular
filtration rate. Ann Intern Med 145: 247–254, 2006
4. Devarajan P. Emerging biomarkers of acute kidney injury. Contrib
Nephrol 156: 203–212, 2007
5. Mehta RL, Kellum JA, Shah SV, Molitoris BA, Ronco C, Warnock DG, Levin A;
Acute Kidney Injury Network. Acute Kidney Injury Network: Report of an
initiative to improve outcomes in acute kidney injury. Crit Care 11: R31, 2007
6. Ronco C, Levin A, Warnock DG, Mehta R, Kellum JA, Shah S, Molitoris
BA; AKIN Working Group. Improving outcomews from acute kidney
injury (AKI): Report on an initiative. Int J Arfit Organs 30: 373–376, 2007
7. Arkan-Arikan A, Zappitelli M, Loftis LL, Washburn KK, Jefferson LS,
Goldstein SL. Modified RIFLE criteria in critically ill children with acute
kidney injury. Kidney Int 71: 1028–1035, 2007
8. Hoste EA, Clermont G, Kersten A, Venkataraman R, Angus DC, De Bacquer
D, Kellum JA. RIFLE criteria for acute kidney injury are associated with hospital
mortality in critically ill patients: A cohort analysis. Crit Care 10: R73, 2006
9. Hingorani SR, Guthrie K, Batchelder A, Schoch G, Aboulhosn N, Manchion
J, McDonald GB. Acute renal failure after myeloablative hematopioetic
cell transplant: Incidence and risk factors. Kidney Int 67: 272–277, 2005
10. Gooley TA, Chien JW, Peregam SA, Hingorani S, Sorror ML, Boeckh M,
Martin PJ, Sandmaier BM, Marr KA, Appelbaum FR, Storb R, McDonald
GB. Reduced mortality after allogeneic hematopoietic-cell transplantation. N Engl J Med 363: 2091–2101, 2010
11. Parikh CR, Yarlagadda SG, Storer B, Sorror M, Storb R, Sandmaier B.
Impact of acute kidney injury on long-term mortality after nonmyeloablative hematopoietic cell transplantation. Biol Blood Marrow
Transplant 14: 309–315, 2008
12. Bao YS, Xie RJ, Wang M, Feng SZ, Han MZ. An evaluation of the RIFLE
criteria for acute kidney injury after myeloablative allogeneic haematopoietic stem cell transplantation. Swiss Med Wkly 141: w13225, 2011
13. Ando M, Mori J, Ohashi K, Akiyama H, Morito T, Tsuchiya K, Nitta K,
Sakamaki H. A comparative assessment of the RIFLE, AKIN and conventional criteria for acute kidney injury after hematopoietic SCT. Bone
Marrow Transplant 45: 1427–1434, 2010
14. Kersting S, Koomans HA, Hene RJ, Verdonck LF. Acute renal failure
after allogeneic myeloablative stem cell transplantation: Retrospective
analysis of incidence, risk factors and survival. Bone Marrow Transplant
39: 359–365, 2007
15. Parikh CR, Schrier RW, Storer B, Diaconescu R, Sorror ML, Maris MB,
Maloney DG, McSweeney P, Storb R, Sandmaier BM. Comparison of
ARF after myeloablative and nonmyeloablative hematopioetic cell
transplantation. Am J Kidney Dis 45: 502–509, 2005
16. Liu D, Yan C, Xu L, Wang Y, Han W, Zhang X, Liu K, Huang X. Diarrhea
during the conditioning regimen is correlated with the occurrence of
American Society of Nephrology
18.
19.
20.
21.
22.
23.
24.
25.
26.
27.
28.
29.
30.
31.
32.
33.
34.
35.
severe acute graft-versus-host disease through systemic release of inflammatory cytokines. Biol Blood Marrow Transplant 16: 1567–1575, 2010
Kersting S, Drop SV, Theobald M, Verdonck LF. Acute renal failure after
nonmyeloablative stem cell transplantation in adults. Biol Blood Marrow Transplant 14: 125–131, 2008
Lopes JA, Jorge S, Silva S, de Almeida E, Abreu F, Martins C, do Carmo
JA, Lacerda JF, Parta MM. Acute renal failure following myeloablative
autologous and allogeneic hematopoietic cell transplantation. Bone
Marrow Transplant 38: 707, 2006
Wan L, Bagshaw SM, Langenberg C, Saotome T, May C, Bellomo R.
Pathophysiology of septic acute kidney injury: What do we really know?
Crit Care Med 36[4 Suppl]: S198–S203, 2008
Parikh CR, Coca SG. Acute renal failure in hematopoietic cell transplantation. Kidney Int 69: 430–435, 2006
Keating GM. Defibrotide: A review of its use in severe hepatic venoocclusive disease following haematopoietic stem cell transplantation.
Clin Drug Investig 34: 895–904, 2014
Murphy CV, Schramm GE, Doherty JA, Reichley RM, Gajic O, Afessa B,
Micek ST, Kollef MH. The importance of fluid management in acute
lung injury secondary to septic shock. Chest 136: 102–109, 2009
Boyd JH, Forbes J, Nakada TA, Walley KR, Russell JA. Fluid resuscitation in septic shock: A positive fluid balance and elevated central
venous pressure are associated witih increased mortality. Crit Care
Med 39: 259–265, 2011
Michael M, Kuehnle I, Goldstein SL. Fluid overload and acute renal failure
in pediatric stem cell transplant patients. Pediatr Nephrol 19: 91–95, 2004
Goldstein SL, Currier H, Graf C, Cosio CC, Brewer ED, Sachdva R.
Outcome in children receiving continuous venovenous hemofiltration.
Pediatrics 107: 1309–1312, 2001
Choi M, Sun CL, Kurian S, Carter A, Francisco L, Forman SJ, Bhatia S.
Incidence and predictors of delayed chronic kidney disease in longterm survivors of hematopoietic cell transplantation. Cancer 113: 1580–
1587, 2008
Abboud I, Peraldi MN, Hingorani S. Chronic kidney diseases in long-term
survivors after allogeneic hematopoietic stem cell transplantation: Monitoring and management guidelines. Semin Hematol 49: 73–82, 2012
Hingorani SR, Seidel K, Lindner A, Aneja T, Schoch G, McDonald G.
Albuminuria in hematopoietic cell transplantation patients: Prevalence,
clinical associations, and impact on survival. Biol Blood Marrow
Transplant 14: 1365–1372, 2008
Cohen EP, Piering WF, Kabler-Babbit C, Moulder JE. End-stage renal
disease (ESRD) after bone marrow transplantation: poor survival compared to other causes of ESRD. Nephron 79: 408–412, 1998
Batts ED, Lazarus HM. Diagnosis and treatment of transplantationassociated thrombotic microangiopathy: Real progress or are we still
waiting? Bone Marrow Transplant 40: 709–719, 2007
Ho VT, Cutler C, Carter S, Martin P, Adams R, Horowitz M, Ferrara J,
Soiffer R, Giralt S. Blood and Marrow Transplant Clinical Trials Network
Toxicity Committee consensus summary: Thrombotic microangiopathy
after hematopoietic stem cell transplantation. Biol Blood Marrow
Transplant 11: 571–575, 2005
Ruutu T, Barosi G, Benjamin RJ, Clark RE, George JN, Gratwohl A, Holler E,
Iacobelli M, Kentouche K, Lämmle B, Moake JL, Richardson P, Socié G,
Zeigler Z, Niederwieser D, Barbui T; European Group for Blood and Marrow
Transplantation; European LeukemiaNet. Diagnostic criteria for hematopoietic stem cell transplant-associated microangiopathy: Results of a consensus
process by an international working group. Haematologica 92: 100, 2007
Kim SS, Patel M, Yum K, Keyzner A. Hematopoietic stem cell transplantassociated thrombotic microangiopathy: Review of pharmacologic
treatment options. Transfusion 55: 452–458, 2015
Van Why SK, Friedman AL, Wei LJ, Hong R. Renal insufficiency after bone
marrow transplantation in children. Bone Marrow Transplant 7: 383–388, 1991
Schriber JR, Herzig GP. Transplantation-associated thrombotic
thrombocytopenic purpura and hemolytic uremic syndrome. Semin
Hematol 34: 126–133, 1997
Onco-Nephrology Curriculum
5
36. Mii A, Shimizu A, Kaneko T, Fujita E, Fukui M, Fujino T, Utsumi K, Yamaguchi
H, Tajika K, Tsuchiya S, Iino Y, Katayama Y, Fukuda Y. Renal thrombotic microangiopathy associated with chronic graft-versus-host disease after allogeneic hematopoietic stem cell transplantation. Pathol Int 61: 518–527, 2011
37. Laskin BL, Goebel J, Davies SM, Jodele S. Small vessels, big trouble in the
kidneys and beyond: Hematopoietic stem cell transplantation-associated
thrombotic microangiopathy. Blood 118: 1452–1462, 2011
38. Au WY, Ma ES, Lee TL, Ha Sy, Fung AT, Lie AK, Kwong YL. Successful
treatment of thrombotic microangiopathy after haematopoietic stem
cell transplantation with rituximab. Br J Haematol 137: 475–478, 2007
39. Corti P, Uderzo C, Tagliabue A, Della Volpe A, Annaloro C, Tagliaferri E,
Balduzzi A. Defibrotide as a promising treatment for thrombotic
thrombocytopenic purpura in atient sundergoing bone marrow transplantation. Bone Marrow Transplant 29: 542–543, 2002
40. de Fontbrune FS, Galambrun C, Sirvent A, Huynh A, Faguer S, Nguyen
S, Bay JO, Neven B, Moussi J, Simon L, Xhaard A, Resche-Riggon M,
O’Meara A, Fremeaux-Bacchi V, Veyradier A, Socié G, Coppo P, de
Latour RP. Use of eculizumab in patients with allogeneic stem cell
transplant-associated thrombotic microangiopathy: A study from the
SFGM-TC. Transplantation 99: 1953–1959, 2015
41. Jodele S, Davies SM, Lane A, Khoury J, Dandoy C, Goebel J, Myers K,
Grimley M, Bleesing J, El-Bietar J, Wallace G, Chima RS, Paff Z, Laskin BL.
Diagnostic and risk criteria for HSCT-associated thrombotic microangiopathy:
A study in children and young adults. Blood 1214: 645–653, 2014
42. Hingorani S, Guthjrie KA, Schoch G, Weiss NS, McDonald GB. Chronic
kidney disease in long-term survivors of hematopoietic cell transplant.
Bone Marrow Transplant 39: 223–229, 2007
43. Hingorani S, Gooley T, Pao E, Sandmaier B, McDonald G. Urinary cytokines
after HCT: Evidence for renal inflammation in the pathogenesis of proteinuria and kidney disease. Bone Marrow Transplant 49: 403–409, 2014
44. Pulsipher MA, Skinner R, McDonald GB, Hingorani S, Armenian SH, Cooke
KR, Gracia C, Petryk A, Bhatia S, Bunin N, Nieder ML, Dvorak CC, Sung L,
Sanders JE, Kurtzberg J, Baker KS. National Cancer Institute, National Heart,
Lung and Blood Instsitute/Pediatric Blood and Marrow Transplantation Consortium First International consensus conference on late effects after pediatric
hematopoietic cell transplantation: The need for pediatric-specific long-term
followup guidelines. Biol Blood Marrow Transplant 18: 334–347, 2012
45. Chanswangphuwana C, Townamchai N, Intragumtornchai T, Bunworasate U.
Glomerular diseases associated with chronic graft-versus-host disease after
allogeneic peripheral blood stem cell transplantation: Case reports. Transplant Proc 46: 3616–3619, 2014
46. Hu SL. The role of graft-versus-host disease in haematopoietic cell
transplantation-associated glomerular disease. Nephrol Dial Transplant 26: 2025–2031, 2011
47. Brukamp K, Doyle AM, Bloom RD, Bunin N, Tomaszewski JE, Cizman B.
Nephrotic syndrome after hematopoietic cell transplantation: Do glomerular lesions represent renal graft-versus-host disease? Clin J Am
Soc Nephrol 1: 685–694, 2006
48. Niscola P, Tendas A, Luo XD, Catalano G, Scaramucci L, Cupelli L,
Giovannini M, Ferranini M, Bondanini F, Piccioni D, Dentamaro T, Palumbo
R, Perrotti AP, Liu QF, de Fabritiis P. The management of membranous
glomerulopathy in allogeneic stem cells transplantation: Updated literature. Cardiovasc Hematol Agents Med Chem 11: 67–76, 2013
49. Osugi Y, Yamada H, Hosoi G, Noma H, Ikemiya M, Ishii T, Sako M. Treatment with candesartan combined with angiotensin-converting enzyme
inhibitor for immunosuppressive treatment-resistant nephrotic syndrome
after allogeneic stem cell transplantation. Int J Hematol 83: 454–458, 2006
50. Hoffmeister PA, Hingorani SR, Storer BE, Baker KS, Sanders JE. Hypertension in long-term survivors of pediatric hematopoietic cell
transplantation. Biol Blood Marrow Transplant 16: 515–524, 2010
51. Rizzo JD, Wingard JR, Tichelli A, Lee SJ, Van Lint MT, Burns LJ, Davies
SM, Ferrara JL, Socié G. Recommended screening and preventive
practices for long-term survivors after hematopoietic cell transplantation: Joint recommendations of the European Group for Blood and
Marrow Transplantation, the Center for International blood and Marrow
6
Onco-Nephrology Curriculum
52.
53.
54.
55.
56.
57.
58.
59.
60.
61.
62.
63.
64.
65.
66.
67.
Transplant Research, and the American Society of Blood and Marrow
Transplantation. Biol Blood Marrow Transplant 12: 138–151, 2006
James PA, Oparil S, Carter BL, Cushman WC, Dennison-Himmelfarb C,
Handler J, Lackland DT, LeFevre ML, MacKenzie TD, Ogedegbe O,
Smith SC Jr, Svetkey LP, Taler SJ, Townsend RR, Wright JT Jr, Narva AS,
Ortiz E. 2014 evidence-based guideline for the management of high
blood pressure in adults: report from the panel members appointed to
the Eighth Joint National Committee. JAMA 311: 507–520, 2014
National High Blood Pressure Education Program Working Group on High
Blood Pressure in Children and Adolescents. The fourth report on the
diagnosis, evaluation, and treatment of high blood pressure in children
and adolescents. Pediatrics 114[2 Suppl 4th Report]: 555–576, 2004
Egli A, Infanti L, Dumoulin A, Buster A, Samaridis J, Stebler C, Gosert R,
Hirsch HH. Prevalence of polyomavirus Bk and JC infection and replication in 400 healthy blood donors. J Infect Dis 199: 837–846, 2009
Cesaro S, Facchin C, Tridello G, Messina C, Calore E, Biasolo MA, Pillon
M, Varotto S, Brugiolo A, Mengoli C, Palù G. A prospective study of
Bi-virus-associated haemorrhagic cystitis in paediatric patients undergoing allogeneic haematopoietic stem cell transplantation. Bone
Marrow Transplant 41: 363–370, 2008
Giraud G, Priftakis P, Bogdanovic G, Remberger M, Dubrulle M, Hau A,
Gutmark R, Mattson J, Svahn BM, Ringden O, Winiarski J, Ljungman P,
Dalianis T. BK-viruria and haemorrhagic cystitis are more frequent in
allogeneic haematopoietic stem cell transplant patients receiving full
conditioning and unrelated-HLA-mismatched grafts. Bone Marrow
Transplant 41: 737–742, 2008
O’Donnell PH, Swanson K, Josephson MA, Artz AS, Parsad SD,
Ramaprasad C, Pursell K, Rich E, Stock W, van Besien K. BK virus infection is associated with hematuria and renal impairment in recipients
of allogeneic hematopoetic stem cell transplants. Biol Blood Marrow
Transplant 15: 1038–1048, 2009
Haines HL, Laskin BL, Goebel J, Davies SM, Yin HJ, Lawrence J, Mehta
PA, Bleesing JJ, Filipovich AH, Marsh RA, Jodele S. Blood, and not
urine, BK viral load predicts renal outcome in children with hemorrhagic
cystitis following hematopoietic stem cell transplantation. Biol Blood
Marrow Transplant 17: 1512–1519, 2011
Ramos E, Drachenberg CB, Wali R, Hirsch HH. The decade of polyomavirus Bi-associated nephropathy: State of affairs. Transplantation
87: 621–630, 2009
Dropoulic LK, Cohen JI. Update on new antivirals under development
for the treatment of double-stranded DNA virus infections. Clin Pharmacol Ther 88: 610–619, 2010
Babel N, Volk HD, Reinke P. BK polyomavirus infection and nephropathy:
The virus-immune system interplay. Nat Rev Nephrol 7: 399–406, 2011
Mani J, Jin N, Schmitt M. Cellular immunotherapy for patients with
reactivation of JC and BK polyomaviruses after transplantation. Cytotherapy pii(S1465–3249(14)00559-3), 2014
Touzot M, Elie C, van Massenhove J, Maillard N, Buzyn A, Fakhouri F.
Long-term renal function after allogenic haematopoietic stem cell
transplantation in adult patients: A single-centre study. Nephrol Dial
Transplant 25: 624–627, 2010
Cohen EP, Drobyski WR, Moulder JE. Significant increase in end-stage
renal disease after hematopoietic stem cell transplantation. Bone
Marrow Transplant 39: 571–572, 2007
Hingorani S. Chronic kidney disease in long-term survivors of hematopoietic cell transplantation: Epidemiology, pathogenesis, and treatment. J Am Soc Nephrol 17: 1995–2005, 2006
Bunin N, Guzikowski V, Rand ER, Goldfarb S, Baluarte J, Meyers K,
Olthoff KM. Solid organ transplants following hematopoietic stem cell
transplant in children. Pediatr Transplant 14: 1030–1035, 2010
Levey AS, Coresh J, Balk E, Kausz AT, Levin A, Steffes MW, Hogg RJ,
Perrone RD, Lau J, Eknoyan G; National Kidney Foundation. National
Kidney Foundation practice guidelines for chronic kidney disease:
Evaluation, classificaiton, and stratification. Ann Intern Med 139: 137–
147, 2003
American Society of Nephrology
REVIEW QUESTIONS
1. Following hematopoietic stem cell transplant your patient
is having a progressive rise in serum creatinine. A recent
CBC and blood smear showed anemia, thrombocytopenia,
and the presence of schistocytes. Considering the possibility
of transplant-associated thrombotic microangiopathy, labs
are sent and show an elevated lactate dehydrogenase level,
an undetectable haptoglobin level, and the Coombs test is
negative. Of the following, the best first step(s) in the treatment of this patient is:
a.
b.
c.
d.
e.
Administration of rituximab
Therapeutic plasma exchange (TPE)
Red blood cell and platelet transfusion
Decreasing dose of calcineurin inhibitor therapy
Both b and d
Answer: e is correct. Calcineurin inhibitor therapy, used
for GVHD prophylaxis, has been associated with TA-TMA
in the HCT population. First-line therapy includes lowering of CNI dose or stoppage and TPE, with the majority
of patients responding to these two interventions. Other
therapies being studied include rituximab, defibrotide,
and eculizumab.
2. In septic HCT patients, the pathophysiologic mechanism
most likely to lead to total body volume overload and
edema is:
a. Increased capillary leak related to sepsis-induced inflammatory response
b. Heart failure
c. Administration of pressors
d. Endothelial damage related to high-dose antibiotics
American Society of Nephrology
e. Decreased venous return related to positive pressure
ventilation
Answer: a is correct. Sepsis results in a cytokine and complement-stimulated systemic inflammatory response causing
increased capillary leak, total body volume overload, and decreased effective circulating volume. Sepsis is a common cause
of AKI following HCT. Other common causes include volume
depletion, nephrotoxic medication exposure, sinusoidal obstruction syndrome, and GVHD. Volume overload has been
associated with increased morbidity and mortality in critically
ill patients and HCT specifically.
3. As part of annual screening after HCT, your patient is
noted to have a urine albumin/creatinine ratio of 500 mg/g
Cr. Your frequency of monitoring for proteinuria should be
changed to:
a.
b.
c.
d.
e.
Monthly
Weekly
Continue with annual monitoring
Every 3–6 months
You no longer need to check for proteinuria
Answer: d is correct. Albuminuria is a common long-term
consequence of HCTand has been associated with progression to
CKD and non–relapse-associated mortality. Microalbuminuria
is defined as an ACR of 30–300 mg albumin/g creatinine,
whereas macroalbuminuria is defined by ACR $ 300 mg/g creatinine. Recent consensus guidelines recommend screening ACR
as part of the day 180 post-HCT evaluation and then yearly
screening after HCT. If proteinuria is present, more frequent
monitoring every 3–6 months is indicated. Additionally, for
those with persistent macroalbuminuria, renal biopsy is indicated to make definitive diagnosis of the underlying etiology.
Onco-Nephrology Curriculum
7
Chapter 10: Radiation Nephropathy
Amaka Edeani, MBBS,* and Eric P. Cohen, MD†
*Kidney Diseases Branch, National Institute of Diabetes and Digestive and Kidney Diseases, National Institutes of
Health, Bethesda, Maryland; and †Nephrology Division, Department of Medicine, University of Maryland School of
Medicine, and Baltimore Veterans Affairs Medical Center, Baltimore, Maryland
INTRODUCTION
The occurrence of renal dysfunction as a consequence
of ionizing radiation has been known for more than
100 years (1,2). Initial reports termed this condition
“radiation nephritis,” but that is a misnomer, because
it is not an inflammatory condition. Renal radiation
injury may be avoided by the exclusion of an adequate volume of kidney exposure during radiation
therapy, but the kidneys’ central location can make
this difficult to impossible when tumors of the abdomen or retroperitoneum are treated, or during total
body irradiation (TBI) (3).
BACKGROUND/CLINICAL SIGNIFICANCE
Radiation nephropathy is renal injury and loss of
function caused by ionizing radiation; this will occur
after sufficient irradiation of both kidneys (4). Ionizing radiation of sufficient energy disrupts chemical
bonds and knocks electrons out of atoms. It generates
oxygen radicals that cause prompt DNA injury within
milliseconds of irradiation. This is the desired action
of therapeutic irradiation to cause death of cancer
cells. It is also the cause of injury to irradiated normal
tissues such as the kidney. The doses of diagnostic
X-ray are orders of magnitude less than those of therapeutic irradiation; whereas there may be a very small
effect of carcinogenesis from a diagnostic X-ray, there
is no risk of normal tissue injury.
The kidneys are the dose-limiting organs for
radiation therapy for gastrointestinal cancers, gynecologic cancers, lymphomas, and sarcomas of the
upper abdomen and during TBI (5). Damage to
normal tissues can be reduced by shielding of nontarget tissues, shaping radiation beams that focus the
high-dose radiation on the cancer and attempting to
avoid surrounding normal tissues, and fractionated
radiation dosing. Fractionation enables DNA repair
in normal cells between dosing.
American Society of Nephrology
Classical radiation nephropathy occurred after
external beam radiation for treatment of solid
cancers such as seminomas (6); the incidence has
declined with the advent of more effective chemotherapy. In recent years, radiation nephropathy has
occurred due to TBI used as part of chemo-irradiation
conditioning just before hematopoietic stem cell
transplantation (HSCT) and also from targeted radionuclide therapy used for instance in the treatment
of neuroendocrine malignancies. TBI may be myeloablative or nonmyeloablative, with myeloablative
regimens using radiation doses of 10–12 Gy to destroy or suppress the recipient’s bone marrow. These
doses are given in a single fraction or in nine fractions
over 3 days (4). In addition, TBI for bone marrow
transplantation (BMT) is preceded or accompanied
by cytotoxic chemotherapy, which potentiates the
effects of ionizing radiation (7).
CKD after HSCT occurs in 10%–30% of HSCT
survivors in pediatric and adult populations (8).
CKD following HSCT can have many causes including medication-induced nephrotoxicity and graftversus-host disease (9); the role of radiation exposure
at the time of the HSCT is also well established (4).
CKD has significant patient impact by predisposing
to hypertension, by altering medication pharmacokinetics, and by predisposing to ESRD.
Threshold dose
Luxton identified 23 Gy as the threshold dose for
radiation nephropathy (6), from radiation of both
kidneys when given in 20 fractions over 4 weeks. If
the total irradiated renal volume is ,30% of both
kidneys, CKD will not occur from irradiation alone
Correspondence: Amaka Edeani, Kidney Diseases Branch, National Institute of Diabetes and Digestive and Kidney Diseases,
National Institutes of Health, 10 Center Dr., Rm. 5-5744, Bethesda, Maryland 20892.
Copyright © 2016 by the American Society of Nephrology
Onco-Nephrology Curriculum
1
(10), although there may be injury to the small, irradiated
volume of kidneys that leads to hypertension. In the case of
radiation nephropathy after BMT, a 10-Gy TBI single dose of
X-rays can cause radiation nephropathy within a year after
irradiation, as may 14 Gy fractionated over 3 days (11).
Lower radiation doses may cause kidney injury after many
years of follow-up. Thus, in survivors of the HiroshimaNagasaki atomic bombs, estimated single fraction doses
of ,200 cGy were associated with CKD after many decades
(12).
Radiosensitivity
Radiation nephropathy occurs as a late phenomenon, usually
presenting months to years after the radiation exposure. This
latency is associated with slower cell turnover rates in renal
tissue compared to early-responding (rapidly proliferating)
tissue such as gastrointestinal epithelium or bone marrow (4).
Normal renal tissue has low mitotic rates, which correlate with
delayed expression of renal injury after radiation (4). Less than
half of subjects exposed to threshold or higher radiation doses
will develop radiation nephropathy. The determinants of individual radiosensitivity are not well known.
EXPERIMENTAL MODELS AND PATHOGENESIS
Radiation nephropathy has been reproduced in many animal
models, including in mice, rats, dogs, and nonhuman primates.
A recent rat model demonstrates injury at similar doses to those
relevant in humans, a similar latency period between time of
irradiation and manifestations of injury, expression of injury as
proteinuria, hypertension, and azotemia, and a similar histopathology (4,13). In this model, suppression of the renin–
angiotensin system is beneficial, and angiotensin II infusion
exacerbates injury (14). However, in the irradiated rat, there
is no evidence of activation of the renin–angiotensin system,
which suggests other countervailing mechanisms relevant in
pathogenesis. Other notable features in this model include a
lack for evidence of chronic oxidative stress (15). Multiple
aspects of pathogenesis have been tested for causality in the
experimental model of TBI-HSCT in which the major lethal
toxicity is renal failure (8). These include oxidative stress, cell
proliferation, transforming growth factor-b, glomerular permeability, fibrosis, renin–angiotensin system, and vascular injury. Of these, the roles of the renin–angiotensin system and
vascular injury have not been disproven to date; the others are
either absent or do not play a causal role (8). Krochak and Baker
(16) noted a sequence of events in which the initial reaction
to irradiation was an increase in endothelial permeability with
an increase of ultrafiltrate extruded from the capillaries in the
glomerular tuft, with an escape of increased protein and other
high-molecular-weight blood components; these events were
transient with normal fluid dynamics restored within a few
days, but these permeability changes did appear to contribute
to eventual early and late clinical syndromes.
2
Onco-Nephrology Curriculum
CLINICAL PRESENTATION
The clinical features of radiation nephropathy will vary
according to dose and volume of irradiation (17). Presentation
can be acute and irreversible or subtle, with a gradual progressive dysfunction over years (5). There is a latent period
that is clinically “silent” until a stage is reached when there are
clinical manifestations of disease (11). These were well described by Luxton in his observation of 137 men with seminoma who were irradiated with 2250–3250 cGy over 5 weeks
(3). The incidence of radiation nephropathy in his cohort was
about 20%, with four clinical syndromes as identified in Table
1. One may expect similar presentations after sufficient renal
irradiation from any source, external or internal.
Acute radiation nephritis
The onset of acute radiation nephritis is only acute relative to the
other variants. Patients present 6–12 months following radiation
exposure with fatigue, varying degrees of edema, shortness of
breath with exertion and headaches with azotemia, malignant
hypertension, and severe anemia disproportionate to degree of
renal failure (16,18). Malignant hypertension may present with
headaches, encephalopathy, and retinopathy. There may be associated congestive heart failure with anasarca, pleuropericardial
effusions, and pulmonary edema. Anemia is normochromic
and normocytic with a nonaplastic marrow (17). This has
been described as “bone marrow transplant nephropathy”
(4,19) when it occurs after HSCT. Some of these cases may
present as hemolytic uremic syndrome or thrombotic thrombocytopenic purpura, i.e., as full-blown thrombotic microangiopathies. Proteinuria is generally not severe, with an average
of 2 g/g urine creatinine. Prognosis has been variably related
to occurrence of malignant hypertension (6) or severity of
fluid retention, with oliguria being a terminal event. Patients
surviving this acute phase usually are left with progression to
CKD.
Chronic radiation nephritis
There are two variants of chronic nephritis (6): 1) primary
chronic radiation nephritis—initial presentation up to 2 years
or longer following irradiation with proteinuria and other
evidence of chronic nephritis; and 2) secondary chronic radiation nephritis—seen in patients who survived acute radiation
nephritis and continued with signs of chronic renal damage.
Signs and symptoms are indistinguishable from renal failure from any other cause, with hypertension, albuminuria,
anemia, azotemia, and small atrophic kidneys on imaging.
Table 1. Clinical syndromes following renal irradiation (3)
Type
Latent period
Acute radiation nephritis (nephropathy)
Chronic radiation nephritis
Malignant hypertension
Benign hypertension
6–12 months
$18 months
12–18 months
$18 months
American Society of Nephrology
Malignant hypertension
In Luxton’s series of patients, malignant hypertension developed either during the acute phase or later, 12–18 months
following irradiation (6,20), with some presenting many
years after exposure. Clinical features included hypertensive
encephalopathy, retinopathy, and seizures. Renal size is
variable.
Benign hypertension
Luxton showed that hypertension may occur as a manifestation
of renal radiation injury in the absence of renal failure, with
variable degrees of proteinuria. However, other studies have
shown close correlation between degree of azotemia and
prevalence of hypertension (1). This is also valid for the hypertensive CKD that occurs after HSCT (4,21).
Hypertension following unilateral renal irradiation
Hypertension alone may be the presenting feature if only a
single kidney is irradiated. This may occur because the
irradiated kidney shrinks and creates renin-dependent hypertension in the manner of a Page kidney (22). Such cases have
been successfully treated by unilateral nephrectomy (23–25).
Histopathology
There are early and late changes following renal irradiation
(3,26). Early changes include endothelial microvascular injury
with cell swelling, subendothelial expansion, and capillary
loop occlusion (Figure 1). There is often mesangiolysis and
variable tubular injury. Ultrastructural examination shows
amorphous material within the subendothelial space, which
appears to extend the lamina rara interna (Figure 2). Late
changes include sclerosis of interlobular and arcuate arteries,
with residual parenchymal damage with increased mesangial
matrix, glomerular scarring, tubular atrophy, and renal mass
reduction. Fibrosis may be extensive.
Figure 1. Photomicrograph by light microscopy of a renal biopsy
specimen in case of BMT nephropathy (4). There is mesangiolysis
(*) and extreme widening of the space between endothelium and
glomerular basement membrane (arrow). The tubular epithelium is
intact, but the tubules are separated by an expanded interstitium.
periodic acid-Schiff stain; magnification, 2503. Reproduced with
permission from reference 4.
trial was based on the hypothesis of mitigation: when a subject
has received sufficient irradiation to kidneys, use of an angiotensin-converting enzyme inhibitor after exposure but before
expression of injury may be beneficial in mitigating the later
injury.
Despite treatment, patients with radiation nephropathy may
evolve to ESRD and require chronic dialysis therapy or undergo
kidney transplant. When this occurs after HSCT, survival on
dialysis is poor (29). Such patients may be eligible for kidney
transplantation and could avoid the need for immunosuppression if the transplanted kidney was from the same donor as gave
them their hematopoietic stem cells (30,31).
RISK OF CANCER FROM IMAGING
TREATMENT
Because it is uncommon, there are no controlled trials to guide
the management of radiation nephropathy. Thus, treatment
of radiation nephropathy is guided by the same principles of
treatment of any hypertensive kidney disease, including blood
pressure control and correction of metabolic acidosis. Standard
management of anemia, secondary hyperparathyroidism,
and electrolyte disturbances may be useful. Experimental data
show that angiotensin converting enzyme inhibitors or angiotensin receptor blockers are preferentially beneficial in radiation
nephropathy (14). Supportive measures are beneficial including
treatment of peripheral and pulmonary edema and treatment of
anemia with blood transfusions and/or erythropoietin stimulating agents.
Prevention may be better than treatment. There are favorable trends for the benefit of captopril to mitigate the late
occurrence of CKD after TBI-based HSCT (28). This clinical
American Society of Nephrology
High-dose ionizing radiation can cause cancer. The threshold
dose at which there is increased excess risk after a single
exposure is 34 mSv (32). For reference, an absorbed dose of
1 mGy is equivalent to an effective dose of 1 mSv, the body
radiation dose of a chest X-ray is ,0.1 mSv, and that of a body
computed tomography (CT) scan is approximately 10 mSv.
Recent reports express concern about the possible cancer risks
of radiologil imaging in patients with kidney disease. Kinsella
et al. (33) reported estimated X-ray doses received by dialysis
patients, and Nguyen (34) reported X-ray exposure in subjects
undergoing workup for kidney transplantation. However,
rather than create real worry, these reports actually underline
the lack of risk, and neither one reports cancers caused by
radiation. The median cumulative doses were 22 and
29 mSv in each report, respectively. The single fraction equivalent doses are about half of those values, well below the
single fraction dose at which there is significant excess cancer
risk. Furthermore, because radiation-induced cancer takes
Onco-Nephrology Curriculum
3
Figure 2. Electron micrographs of glomerular capillary loop in a case of radiation nephropathy (27). (A) Wrinkled capillary basement
membrane (BM) with variable widening of lamina rara interna (arrows). Capillary lumen (CL) contains circulating red blood cells (RBCs) and
has intact endothelial cell lining. Endothelial cells (ENs) are occasionally swollen and contain prominent organelles. Original magnification,
6,3003; reduced by 31%. (B) Higher magnification of A reveals variable widening of lamina rara interna and deposits of fluffy material (*)
resembling that within the capillary lumen. Newly formed basement membrane material is also evident adjacent to the lining endothelium
(long arrows). Original magnification, 13,5003; reduced by 31%. M, mesangial cells; EP, epithelial cells; US, urinary space. Reproduced with
permission from reference 27.
5–10 years to develop, for the average dialysis patient, i.e.,
those starting dialysis at age 65 or more, radiation-induced
excess risk of cancer cannot be a major risk because the expected remaining lifetime is 4 years or less for a 65 year old
starting dialysis (35).
Although cancer after kidney transplant is a genuine
concern, one would expect a surfeit of leukemias if pretransplant radiation exposure was the culprit, and leukemia after
kidney transplant is rare. An unjustified fear of cancer should
not get in the way of essential radiologic imaging. However,
prevalent kidney transplant patients received cumulative
median doses of 17 mSv in another report, and 12% of that
cohort had cumulative exposures of 100 mSv (36). Thus,
there may be reason for concern in individual patients, perhaps especially in younger people with longer expected
lifetimes and therefore more possibility of late radiationinduced cancers. Finally, the concern about exposure to
ionizing radiation has affected practice in surveillance for
nephrolithiasis; ultrasound, rather than CT may be preferable
for serial imaging.
TAKE HOME POINTS
c Radiation nephropathy can result from external or internal radiation
exposures.
c Radiation nephropathy will not occur after diagnostic X-ray exposures.
c Accidental or belligerent radiation exposures may result in renal radia-
tion injury.
c Radiation nephropathy is associated with mesangiolysis and glomerular
capillary thrombosis on renal biopsy; it can lead to a full blown thrombotic microangiopathy.
c Mitigation of radiation nephropathy may be possible with angiotensin
converting enzyme inhibitors.
ACKNOWLEDGEMENTS
This work was supported (in part) by the intramural Research Program of the National Institutes of Health, the National Institute of
Diabetes and Digestive and Kidney Diseases, and Merit Review Awards
IO1 BX002256 from the US Department of Veterans Affairs Biomedical Laboratory Research and Development and IO1 CX000569
Clinical Sciences Research and Development (E.P.C., principal investigator, both grants).
DISCLOSURES
FUTURE CONCERNS
The apparent excess of CKD in some Hiroshima-Nagasaki
survivors emphasizes that wartime or terrorist radionuclear
events could cause significant renal injury in those exposed to
acutely survivable irradiation. Space exploration is another
potential risk for both cardiovascular and renal disease.
4
Onco-Nephrology Curriculum
None.
REFERENCES
Q:1
1. Baerman G, Linser P. Review of localized and general effects of radiation. Munch Med Wschr 7: 996, 1904
American Society of Nephrology
2. Edsall DL. The attitude of the clinician in regard to exposing patients to
the x-ray. JAMA 47: 1425–1429, 1906
3. Cassady JR. Clinical radiation nephropathy. Int J Radiat Oncol Biol Phys
31: 1249–1256, 1995
4. Cohen EP. Radiation nephropathy after bone marrow transplantation.
Kidney Int 58: 903–918, 2000
5. Dawson LA, Kavanagh BD, Paulino AC, Das SK, Miften M, Li XA, Pan C,
Ten Haken RK, Schultheiss TE. Radiation-associated kidney injury. Int J
Radiat Oncol Biol Phys 76: S108–S115, 2010
6. Luxton RW. Radiation nephritis: A long term study of 54 patients. Lancet
2: 1221–1224, 1961
7. Phillips TL, Wharam MD, Margolis LW. Modification of radiation injury to
normal tissues by chemotherapeutic agents. Cancer 35: 1678–1684, 1975
8. Cohen EP, Pais P, Moulder JE. Chronic kidney disease after hematopoietic stem cell transplantation. Semin Nephrol 30: 627–634, 2010
9. Abboud I, Peraldi M, Hingorani S. Chronic kidney diseases in long-term
survivors after allogeneic hematopoietic stem cell transplantation: Monitoring and management guidelines. Semin Hematol 49: 73–82, 2012
10. Dawson LA, Horgan A, Cohen EP. Kidney and ureter. In: ALERT c Adverse Late Effects of Cancer Treatment, Volume 2: Normal Tissue
Specific Sites and Systems, edited by Rubin P, Constine LS, Marks LB,
New York, Springer, 2014
11. Luxton RW. Radiation nephritis. Q J Med 22: 215–242, 1953
12. Sera N, Hida A, Imaizumi M, Nakashima E, Akahoshi M. The association
between chronic kidney disease and cardiovascular risk factors in
atomic bomb survivors. Radiat Res 179: 46–52, 2013
13. Moulder JE, Fish BL. Late toxicity of total body irradiation with bone
marrow transplantation in a rat model. Int J Radiat Oncol Biol Phys 16:
1501–1509, 1989
14. Cohen EP, Fish BL, Moulder JE. Mitigation of radiation injuries via
suppression of the renin-angiotensin system: Emphasis on radiation
nephropathy. Curr Drug Targets 11: 1423–1429, 2010
15. Cohen SR, Cohen EP. Chronic oxidative stress after irradiation: An
unproven hypothesis. Med Hypoth 80: 172–175, 2013
16. Krochak RJ, Baker DG. Radiation nephritis: Clinical manifestations and
pathophysiologic mechanisms. Urology 27: 389–393, 1986
17. Baldwin JN, Hagstrom JWC. Acute radiation nephritis. Calif Med 97:
359–362, 1962
18. Cohen EP, Lawton CA, Moulder JE, Becker CG, Ash RC. Clinical course
of late onset bone marrow transplant nephropathy. Nephron 64: 626–
635, 1993
19. Cohen EP, Robbins MEC. Radiation nephropathy. Semin Nephrol 23:
486–499, 2003
20. Fowler JF. Brief summary of radiobiological principles in fractionated
radiotherapy. Semin Radiat Oncol 2: 16–21, 1992
21. Kersting S, Hené RJ, Koomans HA, Verdonck LF. Chronic kidney disease after myelablative allogeneic hematopoietic stem cell transplantation. Biol Blood Marrow Transplant 13: 1169–1175, 2007
American Society of Nephrology
22. Cohen EP. Fibrosis causes progressive kidney failure. Med Hypoth 45:
459–462, 1995
23. Crummy Jr AB, Hellman S, Stansel Jr HC, Hukill PB. Renal hypertension
secondary to unilateral radiation damage relieved by nephrectomy.
Radiology 84: 108–111, 1965
24. Dhaliwal RS, Adelman RD, Turner E, Russo JC, Ruebner B. Radiation
nephritis with hypertension and hyperreninemia: cure by nephrectomy.
J Pediat 96: 68–70, 1980
25. Salvi S, Green DM, Brecher ML, Magoos I, Gamboa LN, Fisher JE,
Baliah T, Afshani E. Renal artery stenosis and hypertension after abdominal irradiation for Hodgkins disease: Successful treatment with
nephrectomy. Urology 21: 611–615, 1983
26. White DC. The histopathologic basis for functional decrement in late
radiation injury in diverse organs. Cancer 37: 1126–1143, 1976
27. Keane WF, Crosson JT, Staley NA, Anderson WR, Shapiro FL. Radiationinduced renal disease. Am J Med 60: 127–137, 1976
28. Cohen EP, Bedi M, Irving AA, Jacobs E, Tomic R, Klein J, Lawton CA,
Moulder JE. Mitigation of late renal and pulmonary injury after hematopoietic stem cell transplantation. Int J Radiat Oncol Biol Phys 83:
292–296, 2012
29. Cohen EP, Piering WF, Kabler-Babbitt C, Moulder JE. End-stage-renaldisease after bone marrow transplantation: Poor survival compared to
other causes of ESRD. Nephron 79: 408–412, 1998
30. Butcher JA, Hariharan S, Adams MB, Johnson CJ, Roza AM, Cohen EP.
Renal transplantation for end stage renal disease following bone marrow transplantation. Clin Transplant 13: 330–335, 1999
31. Sayegh MH, Fine NA, Smith JL, Rennke HG, Milford EL, Tilney NL.
Immunologic tolerance to renal allografts after bone marrow
transplants from the same donor. Ann Intern Med 114: 964–965,
1991
32. Brenner DJ, Doll R, Goodhead DT, Hall EJ, Land CE, Little JB, Lubin JH,
Preston DL, Preston RJ, Puskin JS, Ron E, Sachs RK, Samet JM, Setlow
RB, Zaider M. Cancer risks attributable to low doses of ionizing radiation: Assessing what we really know. Proc Natl Acad Sci U S A 100:
13761–13766, 2003
33. Kinsella SM, Coyle JP, Long EB, McWilliams SR, Maher MM, Clarkson
MR, Eustace JA. Maintenance hemodialysis patients have high cumulative radiation exposure. Kidney Int 78: 789–793, 2010
34. Nguyen KN, Patel AM, Weng FL. Ionizing radiation exposure
among kidney transplant recipients due to medical imaging during
the pretransplant evaluation. Clin J Am Soc Nephrol 8: 833–839,
2013
35. Collins AJ. USRDS 2013 Annual Data Report, Bethesda, MD, National
Institutes of Health, 2013
36. De Mauri A, Brambilla M, Izzo C, Matheoud R, Chiarinotti D, Carriero A,
Stratta P, De Leo M. Cumulative radiation dose from medical imaging
in kidney transplant patients. Nephrol Dial Transplant 27: 3645–3651,
2012
Onco-Nephrology Curriculum
5
REVIEW QUESTIONS
1. Which of the following statements is false?
a. Radiation nephropathy may result from radionuclide therapy
b. Acute radiation nephritis may be associated with thrombotic thrombocytopenic purpura
c. A single dose of 10 Gy of X-rays can lead to radiation nephropathy
d. CKD has been associated solely with myeloablative total
body irradiation regimens
e. Cytotoxic chemotherapy can potentiate the effects of
ionizing radiation.
Answer: d is correct. CKD can complicate both myeloablative
and nonmyeloablative regimens. The other statements are correct.
2. The pathologic features of radiation nephropathy include
the following except:
a.
b.
c.
d.
e.
6
Subendothelial expansion with amorphous material
Mesangiolysis
Neutrophilic infiltration of the mesangium
Arteriolar sclerosis
Varying degrees of tubular atrophy and interstitial fibrosis
Onco-Nephrology Curriculum
f. Endothelial cell swelling
Answer: c is correct. Pathologic features of radiation nephropathy include endothelial cell swelling with subendothelial expansion with amorphous material and mesangiolysis,
and late changes include arteriolar sclerosis and varying degrees of tubular atrophy and interstitial fibrosis; there is no
evidence of glomerular leucocytic infiltration.
3. Which of the following statements is false regarding hypertension associated with radiation nephropathy:
a. Hypertension following unilateral radiation may be treated
with nephrectomy of the affected kidney
b. Acute radiation nephritis is not associated with malignant
hypertension
c. Hypertension may occur in the absence of renal failure
d. ACE inhibitors and/or angiotensin receptor blockers have
been shown to be beneficial in management of hypertension
Answer: b is correct. Malignant hypertension in the setting
of radiation nephropathy may occur as part of the acute radiation nephritis complex, and it may also occur as an initial
presentation, 12–18 months after irradiation. The other statements are correct.
American Society of Nephrology
Chapter 11. Chemotherapy and Kidney Injury
Ilya G. Glezerman, MD,*† and Edgar A. Jaimes, MD*†
*Renal Service, Department of Medicine, Memorial Sloan Kettering Cancer Center, New York, New York; and
†
Department of Medicine, Weill Cornell Medical College, New York, New York
INTRODUCTION
By January 1, 2011, 4,594,732 people in the United
States carried the diagnosis of invasive malignancy.
On the other hand, with significant advances in
anticancer therapies, the 5-year survival for cancer
patients has increased from 48.9% in 1975–1979 to
68.5% in 2006 (1). These statistics show that a significant percentage of the population is likely to be
exposed to chemotherapy and suffer various shortterm and, in case of survivors, long-term adverse
effects of treatment.
Kidneys are vulnerable to the development of drug
toxicity due to their role in the metabolism and
excretion of toxic agents. The kidneys receive close to
25% of cardiac output, and the renal tubules and
proximal segment in particular have significant capacity for uptake of drugs via endocytosis or transporter
proteins. The high rate of delivery and uptake results
in high intracellular concentration of various substances that then undergo extensive metabolism,
leading to formation of potentially toxic metabolites
and reactive oxygen species (ROS) (2). Numerous
chemotherapy agents have been associated with
various renal toxicities including tubulointerstitial
damage, glomerular disease, electrolyte abnormalities, hypertension, and proteinuria (Table 1).
AGENTS WITH PREDOMONANLTY
TUBULAR TOXICITY
Platinum compounds
Cisplatin (cis-dichlorodiammineplatinum)–platinum
coordination complex is an effective chemotherapy
against a wide spectrum of tumors such as testicular,
head and neck, ovarian, lung, cervical, and bladder
cancers. Nephrotoxicity is the dose-limiting toxicity
of cisplatin. Cisplatin induces the production of ROS
and inhibits several antioxidant enzymes, leading to
oxidative stress injury. It increases renal expression of
tumor necrosis factor a, leading to increased tubular
American Society of Nephrology
cell apoptosis and production of ROS (3). Cisplatin is
excreted and concentrated in the kidneys entering renal tubular cells via organic cation transporter 2,
which is kidney specific (3).
Initial renal toxicity manifests as a decrease in renal
blood flow and subsequent decline in GFR within
3 hours of cisplatin administration. These changes are
probably due to increased vascular resistance secondary to tubulo-glomerular feedback and increased
sodium chloride delivery to macula densa. The
decline in GFR appears to be dose dependent. In
a group of patients who received four cycles of
100 mg/m2, the 51Cr-EDTA–measured GFR declined by 11.7%, whereas in patients who received
three cycles of 200 mg/m2, the mean decline was
35.7%. This effect was noted to be lasting, as GFR
was still 30% below baseline at 2 years (4). Acute
tubular toxicity of cisplatin causes mitochondrial
dysfunction, decreased ATPase activity, impaired
solute transport, and altered cation balance. As a result, sodium and water reabsorption is decreased,
and salt and water excretion is increased, leading
to polyuria (3). Cisplatin also causes dose-dependent
renal magnesium wasting.
Tubulointerstitial injury is a predominant finding on pathologic examination of human kidneys
affected by cisplatin toxicity. Both proximal and
distal tubules are affected, and in patients with AKI,
there is usually acute tubular necrosis. Long-term
cisplatin exposure may also cause cyst formation
and interstitial fibrosis (Figure 1) (3).
Patients with cisplatin toxicity typically present
with progressive azotemia in the setting of bland
urinalysis and minimal proteinuria. Although renal
function improves in most patients, a subgroup of
patients developed permanent renal impairment.
Hypomagnesemia is common and may be present in
Correspondence: Ilya G. Glezerman, Memorial Sloan Kettering
Cancer Center, 1275 York Ave., New York, New York 10024.
Copyright © 2016 by the American Society of Nephrology
Onco-Nephrology Curriculum
1
Table 1. Nephrotoxicity of common chemotherapy drugs
Agent
Drugs with tubular toxicity
Cisplatin
Ifosfamide
Methotrexate
Pemetrexed
Ipilimumab
Drugs with glomerular toxicity
Gemcitabine
Common pathologic finding
Clinical syndromes
ATN
Chronic interstitial fibrosis and cyst formation
ATN
AKI, hypomagnesemia, renal sodium
wasting, CKD
Fanconi syndrome (partial or complete)
AKI
ESRD
Nonoliguric AKI
AKI
CKD
Nephrogenic diabetes insipidus
AKI
Crystal nephropathy
ATN
AIN
Tubular atrophy and interstitial fibrosis
AIN
Thrombotic microangiopathy
Mitomycin
Thrombotic microangiopathy
Bevacizumab
Thrombotic microangiopathy
VEGFR mTKI
Sunitinib
Sorafenib
Axitinib
Pazopanib
Drugs causing electrolyte
abnormalities
EGFR antibody
Cetuximab
Panitumumab
Imatinib
Thrombotic microangiopathy
MCD/cFSGS
AKI
MAHA
Hypertension
Dose dependent: AKI, MAHA
Hypertension
Proteinuria
Hypertension
Less common: Nephrotic syndrome
AKI, MAHA
Proteinuria
Hypertension
Less common: Nephrotic syndrome
AKI, MAHA
Inhibition of TRMP6 in distal
convoluted tubule
Hypomagnesemia
Unknown
Hypophosphatemia
ATN, acute tubular necrosis; AIN, acute interstitial nephritis; MAHA, microangiopathic hemolytic anemia; VEGFR mTKI, vascular endothelial growth factor multitarget tyrosine kinase inhibitors; MCD/cFSGS, minimal change disease and/or collapsing-like focal segmental glomerulosclerosis; EGFR, epithelial growth factor
receptor; TRMP, transient receptor potential cation channel, subfamily M, member 6.
42%–100% of patients depending on total cisplatin dose and
length of exposure. Hypomagnesemia and renal magnesium
wasting may persist for up to 6 years after initial dose (5).
Renal salt wasting syndrome has been reported in up to 10%
of patients, manifesting as hyponatremia and severe orthostatic hypotension in the setting of high urinary sodium
concentration. This syndrome may present 2–4 months after
initiation of cisplatin therapy (6). Rare cases of thrombotic
microangiopathy have been reported in patients who were
also receiving bleomycin with cisplatin (4). Syndrome of inappropriate antidiuretic hormone secretion (SIADH) has
been documented in patients receiving vigorous hydration
(7) but is less common now as cisplatin-associated nausea is
treated with new-generation antiemetics, diminishing the
stimulus for antidiuretic hormone secretion.
Vigorous hydration has been shown to reduce the incidence
of AKI in patients receiving cisplatin. Both mannitol and loop
diuretics have also been used to ameliorate toxicity; however,
randomized studies have not shown a clear benefit (8).
2
Onco-Nephrology Curriculum
Numerous compounds have been studied to prevent cisplatin
nephrotoxicity but only amifostine is US Food and Drug Administration (FDA) approved for protection against cumulative
nephrotoxicity from cisplatin therapy. Amifostine is protective
by increasing the binding of ROS to thiol groups. Side effects,
cost, and concerns that it also diminishes antitumor effect have
limited its use in clinical practice (3). A recent study in a murine
model showed that magnesium supplementation during cisplatin therapy may attenuate renal damage; however, further
studies in humans are needed to validate these findings (9).
Carboplatin is also a platinum-based agent with a lower potential for nephrotoxicity compared with cisplatin but can be
nephrotoxic at myeloablative doses of .800 mg/m2 (10). Oxaliplatin, another platinum compound, has no nephrotoxic
potential.
Ifosfamide
Ifosfamide is an alkylating agent used in the treatment of a
variety of childhood and adult malignancies. Its use, however, is
American Society of Nephrology
Figure 1. Cisplatin-induced acute tubular injury and necrosis
(ATI/ATN). Light microscopy of the kidney biopsy specimen reveals dilated tubules with flattened epithelium. There is also apical
blebbing of tubular cells and drop out of tubular cells from the
basement membrane.
associated with a significant risk for nephrotoxicity. Because it
is commonly used in children, most of the data pertaining to
nephrotoxicity of ifosfamide have been obtained in pediatric
patients. It has been reported that GFR goes to ,90 mL/min
per 1.73 m2 in 50% of and ,60 mL/min per 1.73 m2 in 11% of
patients treated with ifosfamide, with an average reduction of
GFR of 35.1 mL/min per 1.73 m2 at a median of 6 months after
treatment (range, 1–47 months) (11). In adults, ifosfamide has
been shown to reduce mean GFR from 81.5 to 68.5 mL/min
per 1.73 m2 1 year after treatment in patients with prior exposure to cisplatin (12).
Fanconi syndrome characterized by proximal tubular dysfunction with variable degrees of glucosuria in the setting of
normoglycemia, renal phosphate and potassium wasting,
proximal tubular acidosis, hypouricemia, and aminoaciduria
has been reported in 5% of patients treated with ifosfamide (13).
Patients who receive cumulative dose ,60 g/m2 are at lower
risk of renal toxicity, whereas patients receiving .100 g/m2 are
at highest. Platinum combination therapy, renal irradiation,
nephrectomy, and hydronephrosis are additional risk factors
(14). Renal disease may progress even after ifosfamide is discontinued and may lead to ESRD (15). Although the precise
incidence of severe kidney dysfunction after ifosfamide exposure is unknown, recent review indicates that it appears to be a
sporadic complication without clear relationship to cumulative
dose (16).
Methotrexate
Methotrexate (MTX) is an antifolate agent that inhibits dihydrofolate reductase (DHFR), an important step in DNA synthesis. Fifty percent to 70% of the drug is bound to plasma proteins,
and 95% is found in the urine 30 hours after administration in
subjects with normal renal function (17). MTX is both filtered
American Society of Nephrology
and secreted by the kidneys. It is a weak organic acid and is
poorly soluble in acidic urine (18).
Although it is administered over a large therapeutic range,
only high-dose methotrexate (HDMTX) therapy of .1 g/m2
has the potential for nephrotoxicity. MTX renal toxicity is presumed to be due to direct precipitation of the drug, as well to
direct toxic effects on renal tubules. In a large clinical trial of
3,887 patients treated with HDMTX, renal dysfunction occurred
in 1.8% of the subjects and was associated with a 4.4% mortality
in this group (19). Affected patients usually develop nonoliguric
and in more severe cases oliguric AKI shortly after the administration of HDMTX. Urinalysis is generally bland and without
proteinuria. Because MTX is excreted in the urine, renal impairment affects the clearance of the drug. Prolonged exposure to toxic levels of MTX (.10 mmol/L at 24 hours; .1
mmol/L at 48 hours, and .0.1 mmol/L at 72 hours) may lead
to life-threatening nonrenal toxicities such as prolonged cytopenias, mucositis, neurotoxicity, and hepatic dysfunction.
MTX solubility is 10-fold higher in urine with a pH of 7.5
than in acidic urine, and therefore urinary alkalinization and
aggressive hydration (2.5–3.5 L/m 2 per 24 hours starting
12 hours prior to chemotherapy administration) are important
steps to establish brisk diuresis and prevent methotrexate precipitation in the tubules. Probenecid, penicillins, salicylates,
sulfisoxazole, and nonsteroid anti-inflammatory drugs may increase the risk of nephrotoxicity as they interfere with renal
tubular secretion of MTX and delay excretion. Leucovorin rescue is used in patients who develop nephrotoxicity and is aimed
at prevention of nonrenal complications. Leucovorin acts as an
antidote by bypassing blocked DHFR pathway. In patients who
have toxic levels of MTX, the leucovorin rescue dose is given
according to established nomograms (17) with doses of 100–
1,000 mg/m2 administered every 6 hours. Leucovorin rescue is
an effective sole therapy in patients with MTX toxicity (20).
Hemodialysis and hemoperfusion have been used in attempt
to remove MTX from circulation. Although both modalities
result in lower MTX plasma levels immediately after treatment,
there is significant rebound effect with levels reaching 90%–
100% of preprocedure MTX concentrations (19).
Glucarpidase (carboxypeptidase-G2), a recombinant bacterial enzyme that rapidly metabolizes MTX to inactive compounds, is able to decrease MTX plasma level .98% within
15 minutes after administration and is effective as a single
dose (21,22). Although a number of studies showed rapid rates
of MTX removal in patients with HDMTX nephrotoxicity, none
had a control group, and true clinical impact of glucarpidase is
difficult to assess (22). Time to renal recovery in most studies
was similar to that of the leucovorin rescue case series (20,22). In
one study, glucarpidase was associated with lower risk of grade
4 nonrenal toxicity if administered ,96 hours after HDMTX,
but in the same study, inadequate leucovorin rescue was predictive of nonrenal toxicities. Glucarpidase only affects extracellular levels of MTX, which may explain the delay in renal
recovery after MTX removal from circulation (23). The use of
glucarpidase is limited by its high cost (.$100,000/patient),
Onco-Nephrology Curriculum
3
other patients had AKI but did not undergo a kidney biopsy
(30). Most patients improved with prompt discontinuation of
ipilimumab and steroid therapy.
AGENTS WITH PREDOMINANTLY GLOMERULAR
TOXICITY
Gemcitabine
Pemetrexed is an antifolate agent that inhibits several enzymes
involved in DNA synthesis. This drug is not metabolized
significantly, and 70%–90% of the drug is excreted unchanged
in the urine within the first 24 hours after administration. The
half-life of pemetrexed is 3.5 hours in patients with normal renal
function but is increased in patients with renal insufficiency
resulting in higher exposure to the drug (24). Pemetrexed has
not been studied in patients with creatinine clearance (CrCl)
, 45 mL/min, but a fatality was reported in a patient with
CrCl of 19 mL/min who received this drug (25). Mild and reversible renal toxicity has been reported in patients who received
high-dose therapy ($600 mg/m2). Recently, several cases of
pemetrexed-induced tubular injury were reported (26–29) including interstitial nephritis and fibrosis, as well as diabetes insipidus.
After discontinuation of pemetrexed, the renal function stabilized
in these patients, but did not return to pretreatment baseline.
Gemcitabine is a pyrimidine analog used in the treatment of a
variety of solid tumors. Nephrotoxicity of this agent manifests as
thrombotic microangiopathy (TMA). During early clinical
experience, TMA was reported at a low rate of 0.015%; however,
as the drug became more widely used, the incidence was noted to
increase to as high as 2.2%. TMA presents as new-onset renal
insufficiency, various degrees of microangiopathic hemolytic
anemia (MAHA), and new or worsening hypertension (HTN).
In a single institution experience of 29 cases of gemcitabineinduced TMA, de novo renal dysfunction or worsening of preexisting CKD was noted in all patients (32). Kidney biopsies were
performed in four cases and showed thrombi in small blood
vessels, glomerular mesangiolysis, and widening of subendothelial space with detachment of endothelial cells from the glomerular basement membrane consistent with TMA (Figure 2). In
this study, the development of TMA was independent of cumulative dose, which ranged from 4 to 81 g/m2. After discontinuation of gemcitabine 28% of patients had complete recovery of
renal function, and 48% had partial recovery or stable renal
function. Although patients in this study did not undergo plasmapheresis, some authors advocate this treatment for patients
with TMA due to gemcitabine. Literature reviews show no difference in outcomes between patients treated with plasmapheresis and conservative management with drug withdrawal
(32,33).
Eculizumab, a monoclonal antibody directed against the complement protein C5 approved for treatment of atypical hemolytic
uremic syndrome, has been used to treat gemcitabine-induced
TMA (34–36). Of the six patients reported, two had complete
renal response, two had partial improvement in renal function,
and two patients showed no improvement. Given the response
rates similar to supportive care alone, the use of eculizumab
should be carefully weighed against its high cost.
Ipilimumab
Mitomycin
Ipilimumab is a novel immunotherapy agent that has shown
significant promise in the treatment of metastatic melanoma.
Ipilimumab is a fully human monoclonal antibody directed
against cytotoxic T-lymphocyte antigen-4 (CTLA-4), a key
negative regulator of T-cell activation. Because of its immunomodulatory effects, ipilimumab has been associated with a
number of immune-mediated side effects involving skin, liver,
gastrointestinal tract, and endocrine system (30). Renal involvement appears less common, but two cases of biopsy
proven granulomatous acute interstitial nephritis (AIN) and
one case of lupus nephritis have been reported (30,31). Three
Mitomycin is an antitumor antibiotic isolated from Streptomyces
caespitosus used for treatment of gastrointestinal and other solid
tumors. It has been associated with life-threatening TMA with
renal failure and MAHA. Mitomycin nephrotoxicity is dose dependent, with the risk of TMA being 1.6% with cumulative doses
#49 mg/m2 and as high as 30% at doses exceeding 70 mg/m2 (37).
Therefore, doses exceeding 40 mg/m2 are not recommended.
Figure 2. Gemcitabine-induced thrombotic microangiopathy.
Light microscopy (H&E stain) of the kidney biopsy shows intraarteriolar microthrombus. (Courtesy of Dr. Surya V. Seshan.)
and therefore its use should be considered only after standard
supportive measures are maximized (22). For most patients,
supportive care in the form of leucovorin rescue results in
recovery of renal function, and additional doses of HDMTX
may be given without untoward side effects (20).
Pemetrexed
4
Onco-Nephrology Curriculum
Antiangiogenic agents
In the last several years, a group of agents called antiangiogenic
therapies have been utilized in the treatment of a variety of solid
American Society of Nephrology
Figure 3. Vascular endothelial growth factor angiogenic
pathway inhibition. VEGF binds its receptor expressed on the
surface of endothelial cells and podocytes triggering intracellular
and extracellular processes resulting in vascular proliferation. Several drugs classes have been employed to inhibit the activation of
VEGFR and prevent angiogenesis, which is seminal for tumor
growth. VEGF, vascular endothelial growth factor; VEGFR, vascular
endothelial growth factor receptor; TK, intracellular VEGFR tyrosine
kinases; mTKI, multitarget tyrosine kinase inhibitors.
tumors. Angiogenesis is seminal for tumor growth and development of metastases, making it an attractive target for
therapeutic intervention. Vascular endothelial growth factor
(VEGF) is a proangiogenic factor that binds to a family of VEGF
receptors (VEGFRs), with tyrosine kinase activity (TKR). The
receptor binding triggers intracytoplasmic signaling pathways,
leading to proliferation of endothelial cells and pericytes,
recruitment of endothelial cell precursors, and growth of
capillaries (38). In the kidneys, VEGF is expressed in podocytes, signals glomerular endothelial cells, and regulates survival of podocytes via autocrine mechanisms. VEGF maintains
podocyte cytosolic calcium concentration and selective barrier
to macromolecules (39). In addition, VEGF influences BP by
up-regulating the synthesis of nitric oxide in the vascular endothelium and increasing the production of prostacycline resulting in vasodilatation (40).
Several classes of antiangiogenic therapies targeting VEGF
pathway are now available (Figure 3). Bevacizumab is a blocking humanized monoclonal antibody directed against VEGF.
Another class is represented by a group of drugs known as
small molecule multitarget tyrosine kinase inhibitors
(mTKIs). These agents inhibit VEGFR and a number of other
TKRs and include sunitinib, sorafenib, axitinib, and other
drugs. Ramucirumab is a recombinant human monoclonal
antibody directed against VEGFR.
The renal effects of VEGF inhibition have been studied in
murine models. When the VEGF gene is deleted only from
podocytes in mice, they become hypertensive and proteinuric.
Pathologic findings in the kidneys revealed typical features of
American Society of Nephrology
TMA with intracapillary thrombi, endotheliosis, and obliterated capillary loops (41). In humans, a similar spectrum of
disorders has been associated with VEGF inhibition. Hypertension, proteinuria, and TMA have all been reported after
VEGF inhibition.
The effects of anti-VEGF antibody therapy on blood pressure
were recently reviewed in a meta-analysis of seven randomized
clinical trials that included 1,850 patients treated with bevacizumab. In patients who received a low dose (3–7.5 mg/kg/dose) of the
drug, the relative risk (RR) of developing HTN was 3.0 (95% CI,
2.2–4.2; P,0.001). In a high-dose group (10–15 mg/kg/dose), the
RR was 7.5 (95% CI, 4.2–13.4; P,0.001). Grade III HTN (requiring therapy or more intense therapy) was observed in 8.7% of
patients in low-dose and 16.0% in high-dose groups. Proteinuria
was also more common in treated patients. In the low-dose group,
RR for proteinuria was 1.4 (95% CI, 1.1–1.7; P50.003), and in the
high-dose group, RR was 2.2 (95% CI, 1.6–2.9; P,0.001). Grade
III (.3.5 g/24 h) proteinuria was noted in 1.8% of patients in the
high-dose group vs. only 0.1% of controls (42).
TMA is the predominant glomerular lesion associated with
anti-VEGF antibody therapy. It has been reported after intravenous (41,43–45) and ntraocular administration (46).
However, concurrent mesangial IgA deposits, cryoglobulinemic glomerulonephritis (47), and immune complex–mediated
focal proliferative glomerulonephritis (48) have also been reported. In patients with kidney biopsy findings of TMA, the
clinical course varied from subnephrotic range proteinuria to
more fulminant disease with worsening renal function, hypertension, and microangiopathic anemia (41,43–45).
Hypertension is another major side effect of mTKI therapy.
In a meta-analysis of 13 clinical trials that included 4,999 patients
with renal cell carcinoma (RCC) and other malignancies treated
with sunitinib, the incidence of all-grade hypertension was 21.6%
and high grade was 6.8%. However, the RR was only statically
significant for patients with high-grade hypertension at 22.72
(95% CI, 4.48–115.3). Furthermore, when patients were analyzed by the type of malignancy, only those with RCC had a
statistically significant RR of developing both all-grade and
high-grade HTN. More pronounced effects of mTKI in RCC
may be due to higher VEGF levels in patients with RCC, resulting
in a more evident anti-VEGF effect. In addition, the majority of
patients with RCC also undergo nephrectomies, resulting in reduction in renal function and decreased excretion of sunitinib,
leading to prolonged exposure to the drug (49).
In phase 2 trials of axitinib proteinuria, all-grade proteinuria
was reported in 18%–36% of subjects and grade $3 was 0%–5%
(50). Proteinuria and nephrotic syndrome due to sunitinib or
sorafenib have been described in a number of case reports and
one case series (51–55). In these publications, proteinuria of up
to 20 g/24 h has been reported, usually in association with new or
worsening hypertension. These side effects generally resolved
after discontinuation of mTKI. Two patients had a kidney biopsy
that showed features of TMA in one patient and TMA and podocyte effacement in another. MAHA was not present in either
case (53,55).
Onco-Nephrology Curriculum
5
Several more fulminant cases of TMA with worsening renal
function, severe hypertension and MAHAwith low haptoglobin,
high lactate dehydrogenase levels, and schistocytosis have also
been reported (43,56–58). However, in a cohort study of 29
patients treated with mTKIs who developed proteinuria and
HTN and underwent a biopsy, minimal change disease and/or
collapsing-like focal segmental glomerulosclerosis (MCD/cFSGS)
was found in 20 cases (45). However, .55.5% of these patients had a history of nephrectomy, and hyperfiltration injury
as a cause of MCD/cFSGS could not be completely ruled out.
Because the putative mechanism of hypertension in patients
treated with anti-VEGF therapies is intricately related to the
antitumor action of these drugs, it has been proposed that the
development of HTN could be used as biomarker of response
(59). Two small retrospective studies showed that the development of hypertension was associated with improved oncologic
outcomes in patients with RCC treated with axitinib and
sunitinib (60,61). In a retrospective analysis of .500 patients
treated with sunitinib for RCC, the overall survival (OS) and
progression-free survival (PFS) was more than four-fold higher
in the group of patients who developed sunitinib-induced hypertension defined as a maximum systolic blood pressure of
$140 mmHg. However, hypertensive patients had more renal
adverse events (5% versus 3%, P 5 0.013) (62). OS and PFS were
also improved in patients with advanced non–small-cell lung
cancer treated with bevacizumab who developed treatmentrelated hypertension. In this study, hypertension was defined
as BP .150/100 mmHg or a $20-mmHg rise in diastolic blood
pressure (DBP) (63).
Nephrologists should be aware of these data as recommendations to discontinue anti-VEGF therapy due to the development of hypertension and proteinuria should be weighed against
the possible enhanced antitumor effects in this setting. An expert
panel from the National Institute of Cancer has issued guidelines
in management of anti-VEGF therapy–induced hypertension. It
recommends careful assessment of the patients prior to the initiation of therapy to identify those with cardiovascular risk factors, addressing preexisting hypertension prior to initiation of
anti-VEGF therapy, and frequent monitoring of BP particularly
during the first cycle. The patients should be treated if they develop BP .140/90 mmHg or DBP $20 mmHg higher than
baseline. The panel did not make any specific recommendations
about antihypertensive regimen due to lack of data and stated
that treatment should be individualized to fit the patient’s comorbid conditions and to minimize drug interaction (64). Other
considerations include concurrent development of proteinuria
as a complication of anti-VEGF therapy. In this setting, it may be
appropriate to use angiotensin converting enzyme inhibitors
(ACE-Is) or angiotensin receptor blockers (ARBs) for their antiproteinuric effect. Additionally, use of ACE-Is or ARBs in combination with anti-VEGF therapy may have a synergistic effect
on OS in patients with RCC (65). Although long-term effects of
anti-VEGF therapy–induced HTN and proteinuria are unknown, it is probably prudent to continue the anticancer therapy
if HTN and proteinuria are controlled with medical therapy.
6
Onco-Nephrology Curriculum
However, if complications such as nephrotic syndrome, HTN
with end-organ damage, renal insufficiency, or evidence of
MAHA develop, discontinuation of antiangiogenic therapy
should be considered promptly.
In addition to HTN, proteinuria, and TMA, both mTKI and
anti-VEGF antibody agents have been reported to cause acute AIN
(54,66–69). Although some cases have been confirmed by renal
biopsy, in others the diagnosis was made on clinical grounds
because biopsy was precluded by thrombocytopenia or the presence of a solitary kidney. These patients had eosinophilia, eosinophiluria, and kidney dysfunction, and renal function either
improved or stabilized after discontinuation of antiangiogenic
therapy. In two cases, mTKI was administered intermittently
(4 weeks on and 2 weeks off), and the patients exhibited “saw
tooth” fluctuations in eosinophilia and SCr levels, with both parameters improving just before the initiation of the next cycle (54).
AGENTS ASSOCIATED WITH ELECTROLYTE
ABNORMALITIES
Hypomagnesemia as a common complication of cisplatin
therapy and Fanconi syndrome due to ifosfamide treatment
have been addressed by this review already. However, a number
of targeted biological agents have been associated with electrolyte
imbalance.
Cetuximab
Cetuximab is a chimeric monoclonal antibody directed against
epithelial growth factor receptor (EGFR). The EGFR is overexpressed in several tumors of epithelial origin, and cetuximab is
often used in combination with chemotherapy for their treatment. Although in initial clinical trials hypomagnesemia was not
reported (70), numerous published reports have established a
link between low serum Mg21 and use of cetuximab.
Active Mg21 transport in the kidney occurs predominantly in
the distal convoluted tubule (DCT) and where EGFR is also expressed. TRPM 6 (transient receptor potential cation channel,
subfamily M, member 6) has been demonstrated to play a role in
this process. The epithelial growth factor (EGF) markedly increases the activity of TRMP 6, leading to the hypothesis that
EGFR activation is necessary for reabsorption of Mg21 and that
blockade of EGFR leads to renal Mg21 wasting (71) by blocking
the activity of TRMP6. In one of the earlier reports, 34 patients
on cetuximab had their Mg21 level measured at least once. Of
these patients, 23% had grade 3 (,0.9–0.7 mg/dL) and 6% had
grade 4 (,0.7 mg/dL) hypomagnesemia (72). In another report,
the incidence of grade 3/4 hypomagnesemia was 27% (73). The
severity of hypomagnesemia appears to correlate with duration
of exposure and is difficult to manage. Daily infusions of up to
6–10 g of MgSO4 were required to correct the deficit in one
cohort (73). Hypomagnesemia resolved in all cases after 4 weeks
of discontinuation of cetuximab.
Patients who develop clinically significant hypomagnesemia
are also hypocalcemic due to parathyroid hormone resistance,
American Society of Nephrology
which is often seen in the presence of hypomagnesemia and
resolves after Mg21 levels are normalized (72,73).
In more recent randomized trials, the incidence of grade 3/4
hypomagnesemia ranged between 1.8%–5.8% in the
cetuximab arm and 0%–0.4% in chemotherapy or best supportive care (BSC) arms. However, in these studies, the Mg21
levels were not routinely measured, which likely explain the
lower incidence of hypomagnesemia (74).
Panitumumab
Panitumumab is a fully human antibody directed at EGFR
and is used in treatment of metastatic colorectal cancer. In
randomized trials, it has also been shown to cause low serum
Mg21 levels, with an incidence of grade 3/4 hypomagnesemia
ranging between 3%–5% in the panitumumab arm and 0%–
,1% in chemotherapy or BSC arms (75,76).
therapies have been associated with kidney disease due to
interference with signaling pathways in nonmalignant cells.
TAKE HOME POINTS
c A high percentage of the US population undergoes chemotherapy
treatments and is at risk for renal complications because kidneys are a
major route of elimination of these drugs.
c Cisplatin and ifosfamide are major tubular toxins leading to both AKI
and CKD.
c Glomerular toxicity of chemotherapy most commonly manifests as
TMA, with gemcitabine and mitomycin as major offenders.
c VEGF antagonists such as anti-VEGF antibody and VEGFR TKI are associated with development of hypertension and proteinuria and in severe cases TMA. Most patients are managed with antihypertensive
drugs, with discontinuation of therapy only in patients who develop
nephrotic syndrome, malignant hypertension, or TMA.
Imatinib
Imatinib is a small molecule mTKI with specificity for BCRAbl, C-kit, and platelet-derived growth factor receptor
(PDGFR) and activity against tumors characterized by dysregulation of function of these enzymes. Use of imatinib has
been shown to cause hypophosphatemia. In the initial report,
hypophosphatemia developed in 25 (51%) of 49 patients who
had at least one measurement of serum phosphorus. Patients
with both low and normal serum phosphate levels were found
to have high urine fractional excretion of phosphate compared with controls, but only hypophosphatemic patients had
elevated parathyroid hormone (PTH) levels (77). In another
study, 14 (39%) of 36 patients treated with imatinib developed
hypophosphatemia and low PTH levels (78). Additionally, serum phosphate levels were measured routinely in two clinical
trials of 403 patients with chronic myeloid leukemia receiving
imatinib. Hypophosphatemia was observed in 50% of the patients, but hypophosphatemia as an adverse event was only
reported in 3% of the patients (79). The exact mechanism
by which imatinib causes hypophosphatemia is unknown,
but it has been proposed that it may inhibit bone resorption
via inhibition of PDGFR and lead to decreased calcium and
phosphate efflux from the bone. Lower calcium egress from
bone has been postulated to cause mild secondary hyperparathyroidism, which in turn leads to increased renal phosphate
losses (77).
CONCLUSIONS
Despite advances in diagnosis, treatment, and prevention of
chemotherapy-induced kidney injury, significant challenges
still remain. In many cases, the only therapeutic intervention
available is the discontinuation of the offending agent. Future
research should be directed toward development of antidote
agents that protect normal cells and allow continuation of
chemotherapy without compromising antitumor effects. In
addition to traditional cytotoxic agents, new targeted biological
American Society of Nephrology
REFERENCES
1. National Cancer Institute. Surveillance, Epidemiology and End Results
Program. Cancer Statistics. Available at: http://seer.cancer.gov/csr/
1975_2011/sections.html. Accessed March 9, 2015
2. Perazella MA. Renal vulnerability to drug toxicity. Clin J Am Soc
Nephrol 4: 1275–1283, 2009
3. Yao X, Panichpisal K, Kurtzman N, Nugent K. Cisplatin nephrotoxicity: A
review. Am J Med Sci 334: 115–124, 2007
4. Meyer KB, Madias NE. Cisplatin nephrotoxicity. Miner Electrolyte
Metab 20: 201–213, 1994
5. Lajer H, Daugaard G. Cisplatin and hypomagnesemia. Cancer Treat
Rev 25: 47–58 1999
6. Hutchison FN, Perez EA, Gandara DR, Lawrence HJ, Kaysen GA. Renal
salt wasting in patients treated with cisplatin. Ann Intern Med 108: 21–
25, 1988
7. Littlewood TJ, Smith AP. Syndrome of inappropriate antidiuretic hormone secretion due to treatment of lung cancer with cisplatin. Thorax
39: 636–637, 1984
8. Launay-Vacher V, Rey JB, Isnard-Bagnis C, Deray G, Daouphars M. Prevention of cisplatin nephrotoxicity: State of the art and recommendations
from the European Society of Clinical Pharmacy Special Interest Group on
Cancer Care. Cancer Chemother Pharmacol. 61: 903–909, 2008
9. Solanki MH, Chatterjee PK, Gupta M, Xue X, Plagov A, Metz MH,
Mintz R, Singhal PC, Metz C. Magnesium protects against cisplatininduced acute kidney injury by regulating platinum accumulation. Am J
Physiol Renal Physiol 307: F369–F384, 2014
10. Isnard-Bagnis C, Launay-Vacher V, Karie S, Deray G. Anticancer drugs.
In: Clinical Nephrotoxins Renal Injury from Drug and Chemicals, edited
by De Broe M, Porter G, Bennett W, Deray G, 3rd Ed., New York,
Springer Scientific, 511–535 2008
11. Skinner R, Cotterill SJ, Stevens MC. Risk factors for nephrotoxicity after
ifosfamide treatment in children: A UKCCSG Late Effects Group study.
United Kingdom Children’s Cancer Study Group. Br J Cancer 82: 1636–
1645, 2000
12. Farry JK, Flombaum CD, Latcha S. Long term renal toxicity of ifosfamide
in adult patients: 5 year data. Eur J Cancer 48: 1326–1331, 2012
13. Suarez A, McDowell H, Niaudet P, Comoy E, Flamant F. Long-term
follow-up of ifosfamide renal toxicity in children treated for malignant
mesenchymal tumors: An International Society of Pediatric Oncology
report. J Clin Oncol 9: 2177–2182, 1991
Onco-Nephrology Curriculum
7
14. Jones DP, Spunt SL, Green D, Springate JE. Renal late effects in patients treated for cancer in childhood: A report from the Children’s
Oncology Group. Pediatr Blood Cancer 51: 724–731, 2008
15. Berns JS, Haghighat A, Staddon A, Cohen RM, Schmidt R, Fisher S,
Rudnick MR, Tomaszewski JE. Severe, irreversible renal failure after
ifosfamide treatment. A clinicopathologic report of two patients.
Cancer 76:497–500, 1995
16. Akilesh S, Juaire N, Duffield JS, Smith KD. Chronic Ifosfamide toxicity:
Kidney pathology and pathophysiology. Am J Kidney Dis 63:843–850,
2014
17. Bleyer WA. The clinical pharmacology of methotrexate: new applications of an old drug. Cancer 41: 36–51, 1978
18. Smith SW, Nelson LS. Case files of the New York City Poison Control
Center: Antidotal strategies for the management of methotrexate
toxicity. J Med Toxicol 4: 132–140, 2008
19. Widemann BC, Adamson PC. Understanding and managing methotrexate nephrotoxicity. Oncologist 11: 694–703, 2006
20. Flombaum CD, Meyers PA. High-dose leucovorin as sole therapy for
methotrexate toxicity. J Clin Oncol 17: 1589–1594, 1999
21. Widemann BC, Balis FM, Kim A, Boron M, Jayaprakash N, Shalabi A,
O’Brien M, Eby M, Cole DE, Murphy RF, Fox E, Ivy P, Adamson PC.
Glucarpidase, leucovorin, and thymidine for high-dose methotrexateinduced renal dysfunction: Clinical and pharmacologic factors affecting
outcome. J Clin Oncol 28: 3979–3986, 2010
22. Cavone JL, Yang D, Wang A. Glucarpidase intervention for delayed
methotrexate clearance. Ann Pharmacother 48: 897–907, 2014
23. Meyers PA, Flombaum C. High-dose methotrexate-induced renal
dysfunction: Is glucarpidase necessary for rescue? J Clin Oncol 29:
e180, 2011
24. Villela LR, Stanford BL, Shah SR. Pemetrexed, a novel antifolate therapeutic alternative for cancer chemotherapy. Pharmacotherapy 26:
641–654, 2006
25. Mita AC, Sweeney CJ, Baker SD, Goetz A, Hammond LA, Patnaik A,
Tolcher AW, Villalona-Calero M, Sandler A, Chaudhuri T, Molpus K, Latz
JE, Simms L, Chaudhary AK, Johnson RD, Rowinsky EK, Takimoto CH.
Phase I and pharmacokinetic study of pemetrexed administered every
3 weeks to advanced cancer patients with normal and impaired renal
function. J Clin Oncol 24: 552–562, 2006
26. Glezerman IG, Pietanza MC, Miller V, Seshan SV. Kidney tubular toxicity
of maintenance pemetrexed therapy. Am J Kidney Dis 58: 817–820, 2011
27. Michels J, Spano JP, Brocheriou I, Deray G, Khayat D, Izzdine H. Acute
tubular necrosis and interstitial nephritits during pemetrexed Ttherapy.
Case Rep Oncol 2: 53–56, 2009
28. Stavroulopoulos A, Nakopoulou L, Xydakis AM, Aresti V, Nikolakopoulou
A, Klouvas G. Interstitial nephritis and nephrogenic diabetes insipidus in a
patient treated with pemetrexed. Ren Fail 32: 1000–1004, 2010
29. Vootukuru V, Liew YP, Nally JV Jr. Pemetrexed-induced acute renal
failure, nephrogenic diabetes insipidus, and renal tubular acidosis in a
patient with non-small cell lung cancer. Med Oncol 23: 419–422, 2006
30. Izzedine H, Gueutin V, Gharbi C, Mateus C, Robert C, Routier E, Thomas
M, Baumelou A, Rouvier P. Kidney injuries related to ipilimumab. Investig New Drugs 32: 769–773, 2014
31. Fadel F, El Karoui K, Knebelmann B. Anti-CTLA4 antibody-induced
lupus nephritis. N Engl J Med 361: 211–212, 2009
32. Glezerman I, Kris MG, Miller V, Seshan S, Flombaum CD. Gemcitabine
nephrotoxicity and hemolytic uremic syndrome: Report of 29 cases
from a single institution. Clin Nephrol 71: 130–139, 2009
33. Gore EM, Jones BS, Marques MB. Is therapeutic plasma exchange indicated for patients with gemcitabine-induced hemolytic uremic syndrome? J Clin Apheresis 24: 209–214, 2009
34. Starck M, Wendtner CM. Use of eculizumab in refractory gemcitabineinduced thrombotic microangiopathy. Br J Haematol 164: 894–896,
2014
35. Al Ustwani O, Lohr J, Dy G, Levea C, Connolly G, Arora P, Iyer R. Eculizumab therapy for gemcitabine induced hemolytic uremic syndrome:
Case series and concise review. J Gastrointestinal Oncol 5: E30–E33, 2014
8
Onco-Nephrology Curriculum
36. Tsai HM, Kuo E. Eculizumab therapy leads to rapid resolution of
thrombocytopenia in atypical hemolytic uremic syndrome. Adv Hematol 2014: 295323, 2014
37. Valavaara R, Nordman E. Renal complications of mitomycin C therapy
with special reference to the total dose. Cancer 55: 47–50, 1985
38. Mena AC, Pulido EG, Guillen-Ponce C. Understanding the molecularbased mechanism of action of the tyrosine kinase inhibitor: Sunitinib.
Anticancer Drugs 21(Suppl 1): S3–S11, 2010
39. Breen EC. VEGF in biological control. J Cell Biochem 102: 1358–1367,
2007
40. Yogi A, O’Connor SE, Callera GE, Tostes RC, Touyz RM. Receptor and
nonreceptor tyrosine kinases in vascular biology of hypertension. Curr
Opin Nephrol Hypertens 19: 169–176, 2010
41. Eremina V, Jefferson JA, Kowalewska J, Hochster H, Haas M, Weisstuch
J, Richardson C, Kopp JB, Kabir MG, Backx PH, Gerber HP, Ferrara N,
Barisoni L, Alpers CE, Quaggin SE. VEGF inhibition and renal thrombotic microangiopathy. N Engl J Med 358:1129–1136, 2008
42. Zhu X, Wu S, Dahut WL, Parikh CR. Risks of proteinuria and hypertension with bevacizumab, an antibody against vascular endothelial
growth factor: Systematic review and meta-analysis. Am J Kidney Dis
49: 186–193, 2007
43. Frangie C, Lefaucheur C, Medioni J, Jacquot C, Hill GS, Nochy D.
Renal thrombotic microangiopathy caused by anti-VEGF-antibody
treatment for metastatic renal-cell carcinoma. Lancet Oncol 8: 177–
178, 2007
44. Roncone D, Satoskar A, Nadasdy T, Monk JP, Rovin BH. Proteinuria in a
patient receiving anti-VEGF therapy for metastatic renal cell carcinoma.
Nat Clin Pract Nephrol 3: 287–293, 2007
45. Izzedine H, Escudier B, Lhomme C, Pautier P, Rouvier P, Gueutin V,
Baumelou A, Derosa L, Bahleda R, Hollebecgue A, Sahali D, Soria JC.
Kidney diseases associated with anti-vascular endothelial growth factor
(VEGF): An 8-year observational study at a single center. Medicine 93:
333–339, 2014
46. Pelle G, Shweke N, Van Huyen JP, Tricot L, Hessaine S, FremeauxBacchi V, Hiesse C, Delahousse M. Systemic and kidney toxicity of
intraocular administration of vascular endothelial growth factor inhibitors. Am J Kidney Dis 57: 756–759, 2011
47. Johnson DH, Fehrenbacher L, Novotny WF, Herbst RS, Nemunaitis JJ,
Jablons DM, Langer CJ, DeVore RF 3rd, Gaudreault J, Damico LA,
Holmgren E, Kabbinavar F. Randomized phase II trial comparing
bevacizumab plus carboplatin and paclitaxel with carboplatin and
paclitaxel alone in previously untreated locally advanced or metastatic
non-small-cell lung cancer. J Clin Oncol 22: 2184–2191, 2004
48. George BA, Zhou XJ, Toto R. Nephrotic syndrome after bevacizumab:
Case report and literature review. Am J Kidney Dis. 49: e23–e29, 2007
49. Zhu X, Stergiopoulos K, Wu S. Risk of hypertension and renal dysfunction with an angiogenesis inhibitor sunitinib: Systematic review and
meta-analysis. Acta Oncol 48: 9–17, 2009
50. Izzedine H, Massard C, Spano JP, Goldwasser F, Khayat D, Soria JC.
VEGF signalling inhibition-induced proteinuria: Mechanisms, significance and management. Eur J Cancer 46: 439–448, 2010
51. Patel TV, Morgan JA, Demetri GD, George S, Maki RG, Quigley M,
Humphreys BD. A preeclampsia-like syndrome characterized by reversible hypertension and proteinuria induced by the multitargeted
kinase inhibitors sunitinib and sorafenib. J Natl Cancer Inst 100: 282–
284, 2008
52. Obhrai JS, Patel TV, Humphreys BD. The case/progressive hypertension and proteinuria on anti-angiogenic therapy. Kidney Int 74: 685–
686, 2008
53. Overkleeft EN, Goldschmeding R, van Reekum F, Voest EE, Verheul
HM. Nephrotic syndrome caused by the angiogenesis inhibitor
sorafenib. Ann Oncol 21: 184–185, 2010
54. Jhaveri KD, Flombaum CD, Kroog G, Glezerman IG. Nephrotoxicities
associated with the use of tyrosine kinase inhibitors: A single-center
experience and review of the literature. Nephron Clin Pract 117: c312–
c319, 2011
American Society of Nephrology
55. Bollee G, Patey N, Cazajous G, Robert C, Goujon JM, Fakhouri F,
Bruneval P, Noel LH, Knebelmann B. Thrombotic microangiopathy
secondary to VEGF pathway inhibition by sunitinib. Nephrol Dial
Transplant 24: 682–685, 2009
56. Kapiteijn E, Brand A, Kroep J, Gelderblom H. Sunitinib induced hypertension, thrombotic microangiopathy and reversible posterior leukencephalopathy syndrome. Ann Oncol 18: 1745–1747, 2007
57. Choi MK, Hong JY, Jang JH, Lim HY. TTP-HUS associated with sunitinib.
Cancer Res Treat 40: 211–213, 2008
58. Levey SA, Bajwa RS, Picken MM, Clark JI, Baron K, Leehey DJ.
Thrombotic microangiopathy associated with sunutinib, a VEGF inhibitor, in a patient with factor V Leiden mutation. Nephrol Dialysis
Transplant Plus 3: 154–156, 2008
59. van Heeckeren WJ, Ortiz J, Cooney MM, Remick SC. Hypertension,
proteinuria, and antagonism of vascular endothelial growth factor signaling: Clinical toxicity, therapeutic target, or novel biomarker? J Clin
Oncol 25: 2993–2995, 2007
60. Rixe O, Billemont B, Izzedine H. Hypertension as a predictive factor of
Sunitinib activity. Ann Oncol 18: 1117, 2007
61. Rixe O, Dutcher JP, Motzer RJ, Wilding G, Stadler WM, Kim S, Tarazi J,
Motzer RJ. Association between diastolic blood pressure (DBP) $ 90 mm
Hg and efficacy in patients (pts) with metastatic renal cell carcinoma
(MRCC) receiving axitinib (AG-013736; AG). Ann Oncol 19(Suppl 8):
viii189, 2008
62. Rini BI, Cohen DP, Lu DR, Chen I, Hariharan S, Gore ME, Figlin RA,
Baum MS, Motzer RJ. Hypertension as a biomarker of efficacy in patients with metastatic renal cell carcinoma treated with sunitinib. J Natl
Cancer Inst 103: 763–773, 2011
63. Dahlberg SE, Sandler AB, Brahmer JR, Schiller JH, Johnson DH. Clinical
course of advanced non-small-cell lung cancer patients experiencing
hypertension during treatment with bevacizumab in combination with
carboplatin and paclitaxel on ECOG 4599. J Clin Oncol 28: 949–954,
2010
64. Maitland ML, Bakris GL, Black HR, Chen HX, Durand JB, Elliott WJ, Ivy
SP, Leier CV, Lindenfeld J, Liu G, Remick SC, Steingart R, Tang WH.
Initial assessment, surveillance, and management of blood pressure in
patients receiving vascular endothelial growth factor signaling pathway
inhibitors. J Natl Cancer Inst 102: 596–604, 2010
65. McKay RRRGE, Lin X, Simantov R, Choueiri TK. Impact of angiotensin
system inhibitors on outcomes in patients with metastatic renal cell
carcinoma: Results from a pooled clinical trials database. J Clin Oncol
32(4 Suppl): 437, 2014
66. Winn SK, Ellis S, Savage P, Sampson S, Marsh JE. Biopsy-proven
acute interstitial nephritis associated with the tyrosine kinase inhibitor
sunitinib: a class effect? Nephrol Dial Transplant 24: 673–675, 2009
American Society of Nephrology
67. Izzedine H, Brocheriou I, Rixe O, Deray G. Interstitial nephritis in a patient taking sorafenib. Nephrol Dial Transplant 22 :2411, 2007
68. Khurana A. Allergic interstitial nephritis possibly related to sunitinib
use. Am J Geriatr Pharmacother 5: 341–344, 2007
69. Barakat RK, Singh N, Lal R, Verani RR, Finkel KW, Foringer JR. Interstitial
nephritis secondary to bevacizumab treatment in metastatic leiomyosarcoma. Ann Pharmacother 41: 707–710, 2007
70. Cunningham D, Humblet Y, Siena S, Khayat D, Bleiberg H, Santoro A, bets
D, Mueser M, Harstrick A, Verslype C, Chau I, van Cutsen E. Cetuximab
monotherapy and cetuximab plus irinotecan in irinotecan-refractory
metastatic colorectal cancer. N Engl J Med 351: 337–345, 2004
71. Izzedine H, Bahleda R, Khayat D, Massard C, Magne N, Spano JP, Soria
JC. Electrolyte disorders related to EGFR-targeting drugs. Crit Rev
Oncol Hematol 73: 213–219, 2010
72. Schrag D, Chung KY, Flombaum C, Saltz L. Cetuximab therapy and
symptomatic hypomagnesemia. J Natl Cancer Inst 97: 1221–1224, 2005
73. Fakih MG, Wilding G, Lombardo J. Cetuximab-induced hypomagnesemia in patients with colorectal cancer. Clin Colorectal Cancer 6: 152–
156, 2006
74. Fakih M. Management of anti-EGFR-targeting monoclonal antibodyinduced hypomagnesemia. Oncology (Williston Park) 22: 74–76, 2008
75. Peeters M, Price TJ, Cervantes A, Sobrero AF, Ducreux M, Hotko Y,
Andre T, Chan E, Lordick F, Punt CJ, Strickland AH, Wilson G, Ciuleanu
TE, Roman L, Van Cutsen E, Tzekova V, Collins S, Oliner KS, Rong A,
Gansert J. Randomized phase III study of panitumumab with fluorouracil, leucovorin, and irinotecan (FOLFIRI) compared with FOLFIRI
alone as second-line treatment in patients with metastatic colorectal
cancer. J Clin Oncol 28: 4706–4713, 2010
76. Van Cutsem E, Peeters M, Siena S, Humblet Y, Hendlisz A, Neyns B,
Canon JL, Van Laethem JL, Maurel J, Richardson G, Wolf M, Amado
RG. Open-label phase III trial of panitumumab plus best supportive
care compared with best supportive care alone in patients with
chemotherapy-refractory metastatic colorectal cancer. J Clin Oncol
25: 1658–1664, 2007
77. Berman E, Nicolaides M, Maki RG, Fleisher M, Chanel S, Scheu K,
Wilson BA, Heller G, Sauter NP. Altered bone and mineral metabolism
in patients receiving imatinib mesylate. N Engl J Med 354:2006–2013,
2006
78. Osorio S, Noblejas AG, Duran A, Steegmann JL. Imatinib mesylate
induces hypophosphatemia in patients with chronic myeloid leukemia
in late chronic phase, and this effect is associated with response. Am J
Hematol 82:394–395, 2007
79. Owen S, Hatfield A, Letvak L. Imatinib and altered bone and mineral
metabolism. N Engl J Med 355: 627, 2006
Onco-Nephrology Curriculum
9
REVIEW QUESTIONS
1. The best treatment options for gemcitabine induced thrombotic microangiopathy are:
a. Plasmapheresis
b. Administration of eculizumab
c. Discontinuation of gemcitabine and best supportive care
d. All of the above
Answer: c is correct. Whereas both plasmapheresis and
eculizumab have been used in the treatment of gemcitabineinduced thrombotic microangiopathy, there is little evidence
that outcomes of these treatments are superior to supportive
care alone.
2. Which presentation is most consistent with cisplatin nephrotoxicity?
a. Elevated serum creatinine, minimal proteinuria, hypomagnesemia
b. Hypertension, elevated serum creatinine, low platelet
count
c. Nephrotic range proteinuria, hypertension, and edema
d. Elevated serum creatinine, hypophosphatemia, glucosuria
Answer: a is correct. Cisplatin toxicity involves damage to
the tubulo-interstitial compartment and manifests as AKI with
relatively normal urinalysis. Hypomagnesemia is a common
manifestation of cisplatin tubular toxicity. Cisplatin does not
10
Onco-Nephrology Curriculum
cause nephrotic syndrome, and thrombotic microangiopathy
and Fanconi syndrome are rare.
3. A 55-year-old man with a history of metastatic renal cell
carcinoma was begun on treatment with sunitinib (VEGFR
TKI). Two months after starting the treatment, he was
noted to have a BP of 154/90 mmHg, and his random
urinary protein to creatinine ratio was 2.3. The patient was
asymptomatic. His renal function remained normal, and
there was no evidence of hemolysis on his blood work. The
next step is:
a. Discontinue sunitinib and offer best supportive care
b. Begin antihypertensive therapy aimed at reducing his
blood pressure to ,140/90 mmHg and continue to monitor urinary protein to creatinine ratio closely
c. Reduce sunitinib dose
d. Switch the patient from sunitinib to sorafenib
Answer: b is correct. There are no evidence-based recommendations on management of proteinuria and hypertension induced by VEGF inhibitors. In practice, these
agents are generally continued unless patients develop nephrotic syndrome, malignant hypertension, or thrombotic
microangiopathy. Reduction of the sunitinib dose may be
attempted as a next step if hypertension is difficult to control (answer c). Sorafenib is likely to have a similar side
effect profile (answer d).
American Society of Nephrology
Chapter 12: Pharmacokinetics of Chemotherapeutic
Agents in Kidney Disease
Sheron Latcha, MD, FASN
Department of Medicine, Memorial Sloan Kettering Cancer Center, New York, New York
INTRODUCTION
The liver and kidneys serve as the major pathways for
drug metabolism and elimination, with much smaller
contributions from the fecal and reticuloendothelial
systems. As shown in Figure 1, some unique aspects of
renal physiology that make the kidneys particularly
susceptible to drug exposure and injury include 1) a
high blood flow rate and therefore high drug delivery
rate to the kidney (blood flow to the kidney approximates 25% of cardiac output); 2) the medulla’s considerable concentrating ability, which enhances local
drug tissue concentration; 3) the presence of organic
anion transporters within the tubules, which allow
nephrotoxic medications and their toxic metabolites
to become concentrated within the tubules; and 4)
the presence of renal enzymes, which can form toxic
metabolites and reactive oxygen species (CYP450 and
flavin-containing monooxygenase) (1,2).
Cancer drugs have been demonstrated to cause
nephrotoxicity via direct tubular injury, tubular obstruction, injury to the tubulointerstitium, and glomerular damage. Certainly, the prevalence of renal
insufficiency in cancer patients, and the kidney’s role in
drug metabolism, has implications for the choice and
dosing of chemotherapeutic agents. In this section,
recommendations for dose modifications for some
of the more frequently used chemotherapeutic agents,
which require adjustments in patients with various
levels of CKD, will be discussed. For a more complete
discussion of all chemotherapeutic agents that require
renal dosing, the reader is referred elsewhere (3–6).
Published guidelines for dose modification of
chemotherapy for cancer patients with CKD are
largely based on limited pharmacokinetic (PK) and
pharmacodynamic (PD) data and often on studies of
poor quality (physician-initiated postmarketing studies, small sample sizes). Historically, the Cockcroft
and Gault (CG) formula was most often used to
estimate GFR, and this equation has been shown to
overestimate GFR. Additionally, before the advent of
American Society of Nephrology
the isotope dilution mass spectroscopy (IDMS)
creatinine assay, the variability in creatinine assays
likely affected the PK and PD data from past studies,
and therefore, the resulting dosing recommendations
being used currently (7). What remains largely unaddressed in all PK and PD studies to date is that
CKD can significantly alter nonrenal clearance and
modify bioavailability of drugs predominantly metabolized by the liver and intestine. It has been shown
that CKD suppresses various liver metabolic enzymes
(CPT2C9, CYP2C19, and CYP3A4), and these effects
are clinically significant (8). GFR is the metric used to
guide dose adjustment, and as shown in Table 1 a
number of equations are available to calculate GFR.
In cancer patients, compared with Tc99mDPTA
clearance, the Martin and Wright formulae seem
to have the best concordance, followed by the
Chronic Kidney Disease Epidemiology Collaboration
(CKD-EPI) equation (60.2%, 56.5%, and 56.3%, respectively). However, there were similar levels concordance in dosage selection between the assorted
formulae and Tc99mDPTA clearance when selecting
carboplatin dose (9), so the variations in concordance may not appreciably change final dose recommendations.
CARBOPLATIN
Notwithstanding all of these limitations, carboplatin is one of the few chemotherapeutic agents with
good prospective PK and PD data in CKD patients
(10–12). It is the third most commonly prescribed
cytotoxic agent (3). About 70% of the administered
dose is eliminated by the kidneys, and the drug has
Correspondence: Department of Medicine, Memorial Sloan
Kettering Cancer Center, 1275 York Ave., Suite 1204b, New York,
New York 10065.
Copyright © 2016 by the American Society of Nephrology
Onco-Nephrology Curriculum
1
The drug can be administered to hemodialysis (HD)
patients to achieve an AUC of 4–6 such that carboplatin
dose (mg)=Target AUCx25. The drug is not avidly protein
bound shortly after administration, and approximately 70%
is cleared by HD if performed within several hours after infusion (15). Importantly, a majority of carboplatin becomes
protein bound after 24 hours (12), so carboplatin administration should be coordinated to perform dialysis within 24 hours
of dosing (16).
CISPLATIN
Figure 1. Susceptibility of the kidneys to drug exposure and
delivery.
only rarely been associated with AKI at high doses (1,600–
2,400 mg/m2) following bone marrow transplant (BMT) (13,14).
At the usual doses range from 400 to 600 mg/m2, the drug is
much less nephrotoxic. Neuropathy and myelosuppression
are its main toxicities. Calvert’s formula is used to calculate
the area under the curve (AUC): carboplatin dose (mg) 5
target AUC 3 (GFR 1 25).
Although they are members of the same chemical family, cisplatin is considered to be superior therapy for specific tumor
types compared with carboplatin. Unfortunately, nephrotoxicity
is the dose-limiting side effect of this very effective chemotherapeutic agent (17–19). In the earliest reports of renal toxicity,
incidence rates of 28%–36% were reported in patients
receiving a single dose of 50 mg/m2 (Bristol Meyer Packaging),
but the severity of renal toxicity decreased following institution
of vigorous hydration protocols with normal saline (NS). The
chloride in NS decreases the formation of toxic reactive platinum compounds (20). Initial declines in a GFR range from 12%
to 19% (21–23), but some patients have shown improvement in
renal function over time, implying that renal recovery is possible
in some cases (23,24). However, there is usually persistent subclinical renal injury following cisplatin exposure (20).
A detailed understanding of the mechanisms by which
cisplatin induces renal toxicity remains unclear. The kidney
Table 1. eGFR formulas in cancer patients
2
Onco-Nephrology Curriculum
American Society of Nephrology
selectively accumulates cisplatin and its analogues to a greater
extent than other organs(18). On microscopic examination,
cisplatin produces a tubulointerstitial lesion, with acute
tubular necrosis being the predominant lesion, whereas the
glomeruli are spared (20,25). Cisplatin-induced tubular damage affects the PK of subsequent cisplatin doses and causes a
decrease in the percentage of cisplatin excreted and an increase in the AUC following sequential doses of the drug
(26,27).
The drug is 90% protein bound within 2 hours after infusion,
and 30% is excreted in the urine within 24 hours. Dosing
guidelines for patients with CKD are empiric. Kintzel recommends for a creatinine clearance (CrCl) of 46–60 mL/min, give
75% of the usual dose; for a CrCl of 31–45 mL/min, give 50% of
the usual dose, and for a CrCl ,30 mL/min, the drug is not
recommended (4). Arnoff recommends for CrCl of 10–50
mL/min, give 75% of the usual dose, and for CrCl ,10 mL/min,
give 50% of the usual dose. The manufacturer recommends
withholding repeat administration of the drug until the serum
creatinine concentration is ,1.5 mg/dL. Cisplatin has been successfully administered to HD patients with similar tolerance as in
patients with normal renal function. Because the drug is highly
and irreversibly protein bound and because free cisplatin is well
dialyzed, drug that is dialyzed off cannot be replaced by bound
drug. As such, cisplatin must be given on a nondialysis day. For
HD patients, it is recommended that the initial dose be reduced
by 50%, or 25–50 mg/m2 every 3–6 weeks (16).
IFOSFAMIDE
Although hemorrhagic cystitis is the predominant toxicity of this
alkylating agent, renal toxicity can be dose limiting. A number of
histopathologic changes in the kidney have been observed with
ifosfamide and include segmental glomerular sclerosis, tubulointerstitial nephritis, tubular atrophy, and interstitial fibrosis
(28). Chloroacetaldehyde, a metabolite of ifosfamide, is toxic to
renal epithelial cells and may contribute to nephrotoxicity of the
parent drug (29,30). Clinical manifestations of renal tubular
toxicity include Fanconi syndrome, proximal and distal renal
tubular acidosis, hypophosphatemia, hypokalemia, and nephrogenic diabetes insipidus.
Up to 87% of ifosfamide and its metabolites are recovered in
the urine, and up to 41% of the dose is recovered in the urine as
alkylating activity (31). Central nervous system (CNS) toxicity
may be linked to the accumulation of the metabolite chloroacetaldehyde, and the risk of CNS toxicity is greater in patients with
abnormal renal function (32). Dosing guidelines are empiric and
vary widely. Kintzel recommends for CrCl of 46–60 mL/min,
give 80% of dose; for CrCl of 31–45 mL/min, give 75% of dose;
and for CrCl of ,30 mL/min, give 70% of dose. Aronoff recommends giving 75% of the usual dose for a GFR ,10 mL/min
(4,5). The drug has been used without significant myelosuppression or neurotoxicity in anuric and oliguric patients on HD at
starting doses of 1.5 g/m2 at 48- to 72-hour intervals, with HD to
American Society of Nephrology
follow in the range of 3–14 hours after drug administration.
Subsequent dose adjustments were based on signs of neurotoxicity or myelosuppression (33). For HD patients with urine
output, hydration and Mesna are needed to prevent hemorrhagic cystitis, and a hydration protocol is outlined elsewhere
(33,34).
CYCLOPHOSPHAMIDE
There are no clear guidelines for dose adjustment of cyclophosphamide in the setting of renal insufficiency. Although
cyclophosphamide is largely cleared by hepatic metabolism, up
to 60% of the total dose is eliminated by the kidney as the parent
drug or metabolites (35), and renal insufficiency is associated
in changes in the PK profile of the parent drug and its metabolites. Aronoff recommends giving 75% of the usual dose for
GFR ,10 mL/min. Because the AUC of cyclophosphamide is
increased in HD patients, it is recommended that the dose be
reduced by 25% in this group (36,37). The drug should be
given after HD because the drug and its metabolites are dialyzable.
METHOTREXATE (MTX)
Renal excretion of this antifolate agent is 60% and 94% when
the drug is administered over 6 and 24 hours, respectively.
Nephrotoxicity has been observed at doses exceeding 1 g/m2
(38) and results from intratubular precipitation of MTX and
its metabolites in the distal tubules, which causes an obstructive tubulopathy and decreased glomerular filtration (4,39).
Pretreatment with cisplatin has been reported to increase
MTX toxicity, possibly by decreasing renal clearance (40).
AKI prolongs extrarenal toxicity (myelosuppression and gastrointestinal toxicity) and is observed at plasma concentrations 5–20 mmol/L at 24 hours, 0.5–2 mmol/L at 48 hours,
and 0.05–0.1 mmol/L at 72 hours following drug administration. When serum drug levels reach toxic levels, intravenous
leucovorin is generally used to circumvent MTX’s inhibition of
dihydrofolate reductase to “rescue” normal cells from MTX
(38).
MTX is poorly soluble in acid urine, and decreased flow
rates increase its concentration in the renal tubules. Prevention
remains the best treatment for MTX toxicity and includes
measures to alkalinize and maximally dilute the urine with
bicarbonate containing intravenous fluids as needed to
achieve a urine pH . 7.0 and a urine output of $150 mL/h.
Because the drug is not lipophilic, it accumulates at high concentrations in ascites and pleural fluid, which can prolong
drug elimination and toxicity, especially in the setting of
AKI. Consequently, it is recommended that fluid collections
be drained prior to high-dose MTX.
When AKI occurs, it is reversible in 70%–100% of cases with
conservative medical management (38), and the drug can be
Onco-Nephrology Curriculum
3
readministered following recovery of renal function. In cases
where dialysis requiring AKI occurs and is associated with
prolonged myelosuppression and/or gastrointestinal (GI) ulceration, carboxypeptidase-G(2) can be obtained on a compassionate, albeit expensive basis, for management of severe
MTX intoxication (41–43). Carboxypeptidase-G(2) rapidly
hydrolyzes MTX into its inactive metabolites and decreases
median MTX concentrations by 98.7%.
Kintzel recommends for CrCl of 46–60 mL/min, give 65%
of dose; for CrCl of 31–45 mL/min, give 50% of dose, and for
CrCl ,30 mL/min, do not administer. Aronoff recommends a
dose reduction of 50% with a CrCl of .10–50 mL/min and
avoiding the drug for CrCl of ,10 mL/min (4,5). Although the
drug is removed by high-flux HD and charcoal hemoperfusion, because it is 50% protein bound, there is postdialysis
rebound in MTX concentrations of 90%–100% of the preprocedure levels. Therefore, patients may require daily or continuous renal replacement therapy to avoid rebound toxicity.
PEMETREXED
Pemetrexed is a derivative of MTX, and up to 90% of the drug
is excreted unchanged in the urine. The drug has been associated
with acute tubular necrosis and interstitial fibrosis (44,45).
The manufacturer recommends avoiding the drug for a CrCl
of ,45 mL/min and avoiding nonsteroidal anti-inflammatory
medications in the days before and after pemetrexed dosing in
patients with a CrCl of 45–79 mL/min (46). When nephrotoxicity occurs, it is probably best to avoid re-exposure.
MELPHALAN
Melphalan is effective therapy for multiple myeloma (MM) and
amyloidosis. Urinary excretion of melphalan ranges from 10%
to 34%, and the AUC for melphalan is inversely correlated with
the GFR (47). Bone marrow suppression, the limiting side
effect of melphalan, increases when the drug is given in
CKD patients (48). Kintzel recommends for CrCl of 46–60
mL/min, give 85% of dose; for CrCl of 31–45 mL/min, give
75% of dose; and for CrCl of ,30 mL/min, give 70% of dose.
Aronoff recommends giving 75% of the dose for a CrCl of
.10–50 mL/min and 50% for those with a CrCl of ,10
mL/min (4,5). Patients on dialysis have been safely and successfully treated with melphalan prior to stem cell transplant
at doses between 60 and 140 mg/m2 (49,50).
CKD patients (51,52). Rare cases of AKI due to acute and
chronic interstitial nephritis (AIN) have been reported with
lenalidomide (53–55). For MDS, the manufacturer recommends 5 mg daily for CrCl of ,59 mL/min, and increasing
the dosing interval to every 48 hours for CrCl of ,30 mL/min.
For patients on HD, a dose of 5 mg should be given three
times a week following each hemodialysis. For MM, the manufacturer recommends a 10-mg daily dose for CrCl of .30–59
mL/min, a 15-mg dose every 48 hours for CrCl of ,30
mL/min, and 5 mg daily for HD patients to be given after
HD on dialysis days (56).
CYTARABINE
Cytarabine is an antimetabolite that is extensively converted to
uridine-arabinoside (Ara-U), and 10%–30% of the parent
drug and 85% of the inactive metabolite are eliminated by
the kidneys (57). High-dose cytarabine (HDAC) can produce
significant and sometimes irreversible neurotoxicity. Renal
dysfunction is an independent risk factor for cerebellar (dysarthria, nystagmus, gait ataxia, dysdiadochokinesia) and noncerebellar (somnolence, seizures) neurotoxicity. The inactive
metabolite Ara-U inhibits cytidine deaminase activity, and in
the setting of renal dysfunction, further delays cytarabine
clearance and increases serum and cerebral spinal fluid levels
of the parent drug (58). For patients with CKD receiving
HDAC, Kintzel recommends for CrCl of 46–60 mL/min,
give 60% of the dose; for CrCl of 31–45 mL/min, give 50%
of the dose; and for CrCl of ,30 mL/min, do not administer
(4). Smith developed a dosing algorithm for HDAC based on
daily serum creatinine measurements while the patients were
on therapy. If the patient’s baseline serum creatinine concentration was between 1.5 and 1.9 mg/day, or if baseline serum
creatinine level increased by 0.5–1.2 mg/dL during treatment,
then the dose of Ara-C was reduced to 1 g/m2 per dose. If the
serum creatinine level was .2 or the change was .1.2 mg/dL,
then the dose of Ara-C was reduced to 0.1 g/m2/day (standard
dose) as a continuous infusion. (59). Cytarabine and Ara-U
are both cleared by HD. There are a few case reports of patients
on HD who have tolerated treatment with standard doses cytarabine (continuous infusion, 100 mg/m2 per day) when HD
was performed on day 1 of cytarabine infusion and then every
other day. In one case, HD was performed consecutively on
days 1 and 2 as well (60,61). HDAC has been safely used in a
patient with lymphoma on hemodialysis. The patient received
two doses of HDAC 1 g/m2, 24 hours apart, was dialyzed 6
hours after each dose, and then resumed his usual dialysis
schedule (62).
LENALIDOMIDE
Lenalidomide is a thalidomide analogue used to treat multiple myeloma (MM) and myelodysplastic syndrome (MDS).
Eighty-two percent of the drug is excreted unchanged in the
urine, and the AUC and risk for drug toxicity are increased in
4
Onco-Nephrology Curriculum
CAPECITABINE
Capecitabine is a pyrimidine analogue that is preferentially
converted to 5-fluorouracil (5-FU) within tumor cells.
American Society of Nephrology
Although neither capecitabine nor 5-FU is renally cleared, in
patients with CKD, there is retention of active metabolites
and a resultant increase in systemic toxicity (63). The manufacturer recommends a dose reduction of 75% from the starting dose of 1,250 mg/m2 for patients with a CrCl between 30
and 50 mL/min, and for patients with a CrCl of ,30 mL/min,
it is recommended that capecitabine be discontinued (64).
With careful monitoring and dose reduction, the drug has
been effectively and safely used in patients with advanced
CKD (GFR , 30 mL/min) and patients on hemodialysis
(65).
BLEOMYCIN
Bleomycin is not nephrotoxic, but urinary excretion accounts
for close to 70% of the intravenous dose of bleomycin (66,67),
and patients with renal dysfunction appeared to be at higher
risk for bleomycin pulmonary toxicity, the major dose-limiting
toxicity of this drug (48,68). Kintzel recommends for CrCl of
46–60 mL/min, give 70% of dose; for CrCl of 31–45 mL/min,
give 60% of dose; and for CrCl of ,30 mL/min, avoid the drug.
Aronoff recommends giving 75% of the dose for CrCl of .10–
50 mL/min and 50% for those with a CrCl of ,10 mL/min
(4,5). Because pulmonary toxicity can be cumulative in patients with CKD, if bleomycin is administered to patients
with CKD, repeat pulmonary function tests prior to each
drug administration may be prudent.
MOLECULARLY TARGETED AGENTS
Vascular endothelial growth factor (VEGF) pathway inhibitors (bevacizumab), tyrosine kinase inhibitors (sorafenib,
nilotinib, and dasatinib), and epithelial growth factor receptor
(EGFR) pathway inhibitors (erlotinib) have been described to
cause nephrotoxicity. Observed renal toxicities among these
relatively new agents include acute tubular necrosis, proteinuria, hypertension, thrombotic microangiopathy, acute
interstitial nephritis, tumor lysis syndrome, and glomerulonephritis (69–71).
Bevacizumab
As per the manufacturer’s guidelines, no studies were done to investigate the PK of bevacizumab in patients with CKD. For patients
with a CrCl between 20 and 39 mL/min, the manufacturer
recommends a 50% decrease in the usual starting dose with increases in subsequent doses as tolerated, but no .400 mg. For
patients with a CrCl of 40–59 mL/min, doses .600 mg are not
recommended (72). There is only one report on the PK of
bevacizumab in a dialysis-dependent patient with metastatic renal
cancer who received 5 mg/kg every 2 weeks. The drug was
not dialyzable, and its pharmacokinetic parameters were similar
to the reference values of patients with normal renal function. The
drug can be administered any time before or after hemodialysis (73).
American Society of Nephrology
Tyrosine kinase inhibitors
Erlotinib
The PK of erlotinib were never studied in patients with renal
insufficiency. There are case reports of patients with CrCl
between 25 and 41 mL/min who tolerated the usual dose of 150
mg/day without significant toxicity (74). There are no data on
its use in hemodialysis patients.
Imatinib
Although there is no significant renal excretion, the manufacturer recommends that patients with a CrCl of 20–30
mL/min receive 50% of the starting dose, with dose escalation
as tolerated but not to exceed 400 mg/day. For those with
CrCl of 40–59 mL/min, doses .600 mg are not recommended. Imatinib exposure can increase up to two-fold in
patients even with mild renal impairment (CrCl , 60 mL/min)
and that there is a significant correlation between decreased renal function and the incidence of serious adverse
events (75). PK data on one patient with ESRD on dialysis
that received 400 mg/day indicates that imatinib and its metabolite are unchanged in patients with ESRD on hemodialysis
(74).
Lenvatinib
The package insert recommends a dose reduction to 14 mg
daily for patients with CrCl is ,30 mL/min. There are no data
on the use of lenvatinib in patients on hemodialysis.
Sunitinib
Sunitinib use was studied in two ESRD patients. The PK parameters of sunitinib and its major metabolite were similar in
patients on HD and those with normal renal function. Furthermore, sunitinib is nondialyzable. Doses of 50 mg/day for 4 weeks
every 6 weeks were well tolerated (76).
Sorafenib
Although the manufacturer does not recommend any dose
adjustment for patients with any level of renal insufficiency
(77), based on dose-limiting toxicity in a phase 1 study, a
starting dose of 200 mg twice a day for CrCl of 20–39 mL/min
and 200 mg once daily for patients on hemodialysis was recommended. No recommendations could be made for those with a
CrCl ,20 mL/min and not on dialysis (78).
Vandetanib
The manufacturer recommends that the starting dose should
be reduced to 200 mg in patients with a CrCl of ,50 mL/min (79).
There are no data on use of this drug in hemodialysis patients.
BISPHOSPHONATES
Bisphosphonates are frequently administered to cancer patients for management of hypercalcemia of malignancy and
osteolytic bone lesions. Zoledronic acid has been associated
with acute tubular necrosis, especially after repeated dosing.
Onco-Nephrology Curriculum
5
Table 2. Recommended dose adjustments for selected chemotherapies for patients with CKD and ESRD on HD
Although pamidronate is more notoriously associated with
collapsing glomerulopathy, there are case reports of acute
tubular necrosis with this agent as well (80). Dehydration,
concomitant use of nephrotoxic medication, and overly frequent dosing of the bisphosphonates all increase susceptibility
to deterioration of renal function following bisphosphonate
exposure. The American Society of Clinical Oncology (ASCO)
recommends that his zoledronate be avoided in patients with
CrCl of ,30 mL/min and that the initial dose of 4 mg be
reduced to 3.5 mg for CrCl of 50–60 mL/min; 3.3 mg for
CrCl of 40–49 mL/min; and 3 mg for CrCl of 30–39 mL/
min. For pamidronate, the usual dose of 90 mg over 2–3 hours
and ASCO recommends giving 90 mg over 4–6 hours for CrCl
of ,59 mL/min. Neither agent should be used more frequently than every 3–4 weeks, the serum creatinine should
be checked prior to each administration, and the medication
should be held if the creatinine increases .0.5 mg/dL with
normal renal function or .1 mg/dL if there is abnormal renal
function at baseline (81).
Table 2 includes commonly used chemotherapeutic drugs
and their dose adjustment in the setting of CKD.
CONCLUSIONS
In summary, although the kidneys are a major pathway for drug
metabolism, unfortunately, the quality of PK and PD data on
6
Onco-Nephrology Curriculum
commonly prescribed chemotherapeutic agents for CKD patients requiring chemotherapy is quite poor. This chapter was an
effort to summarize the data that are available and to provide the
treating physician with some guidance when treating patients
with cancer and CKD.
TAKE HOME POINTS
c The liver and the kidneys serve the major pathways of drug metabolism
and elimination with much smaller contributions from the fecal and reticuloendothelial systems.
c Published guidelines for dose modification of chemotherapy for cancer
patients, with the exception of carboplatin, are largely based on limited
pharmacokinetic and pharmacodynamic data.
c In several cases, the parent drug and its metabolites are responsible for
systemic toxicity, and the presence of renal insufficiency can potentiate
toxicity of the parent drug and its metabolites.
REFERENCES
1. Perazella MA. Renal vulnerability to drug toxicity. Clin J Am Soc
Nephrol 4: 1275–1283, 2009
2. Aleksa K, Matsell D, Krausz K, Gelboin H, Ito S, Koren G. Cytochrome
P450 3A and 2B6 in the developing kidney: implications for ifosfamide
nephrotoxicity. Pediatr Nephrol 20: 872–885, 2005
American Society of Nephrology
3. Launay-Vacher V, Oudard S, Janus N, Gligorov J, Pourrat X, Rixe O,
Morere JF, Beuzeboc P, Deray G; Renal Insufficiency and Cancer
Medications (IRMA) Study Group. Prevalence of Renal Insufficiency in
cancer patients and implications for anticancer drug management: the
renal insufficiency and anticancer medications (IRMA) study. Cancer
110: 1376–1384, 2007
4. Kintzel PE, Dorr RT. Anticancer drug renal toxicity and elimination:
dosing guidelines for altered renal function. Cancer Treat Rev 21: 33–
64, 1995
5. Aronoff GR. Drug Prescribing in Renal Failure, Philadelphia, American
College of Physicians, 2007
6. Lichtman SM, Wildiers H, Launay-Vacher V, Steer C, Chatelut E, Aapro
M. International Society of Geriatric Oncology (SIOG) recommendations for the adjustment of dosing in elderly cancer patients with renal
insufficiency. Eur J Cancer 43: 14–34, 2007
7. Matzke GR, Aronoff GR, Atkinson AJ Jr, Bennett WM, Decker BS,
Eckardt KU, Golper T, Grabe DW, Kasiske B, Keller F, Kielstein JT,
Mehta R, Mueller BA, Pasko DA, Schaefer F, Sica DA, Inker LA, Umans
JG, Murray P. Drug dosing consideration in patients with acute and
chronic kidney disease: A clinical update from Kidney Disease: Improving Global Outcomes (KDIGO). Kidney Int 80: 1122–1137, 2011
8. Dreisbach AW, Lertora JJ. The effect of chronic renal failure on drug
metabolism and transport. Expert Opin Drug Metab Toxicol 4: 1065–
1074, 2008
9. Dooley MJ, Poole SG, Rischin D. Dosing of cytotoxic chemotherapy:
impact of renal function estimates on dose. Ann Oncol 24: 2746–2752,
2013
10. Chatelut E, Canal P, Brunner V, Chevreau C, Pujol A, Boneu A, Roché H,
Houin G, Bugat R. Prediction of carboplatin clearance from standard
morphological and biological patient characteristics. J Natl Cancer Inst
87: 573–580, 1995
11. Calvert AH, Newell DR, Gumbrell LA, O’Reilly S, Burnell M, Boxall FE,
Siddik ZH, Judson IR, Gore ME, Wiltshaw E: Carboplatin dosage: Prospective evaluation of a simple formula based on renal function. J Clin
Oncol 7: 1748–1756, 1989
12. Egorin MJ, Van Echo DA, Tipping SJ, Olman EA, Whitacre MY,
Thompson BW, Aisner J. Pharmacokinetics and dosage reduction of
cis-diammine(1,1-cyclobutanedicarboxylato)platinum in patients with
impaired renal function. Cancer Res 44: 5432–5438, 1984
13. Shea TC, Storniolo AM, Mason JR, Newton B, Mullen M, Taetle R,
Green MR. A dose-escalation study of carboplatin/cyclophosphamide/
etoposide along with autologous bone marrow or peripheral blood
stem cell rescue. Semin Oncol 19[Suppl 2]: 139–144, 1992
14. Beyer J, Rick O, Weinknecht S, Kingreen D, Lenz K, Siegert W. Nephrotoxicity after high-dose carboplatin, etoposide and ifosfamide in
germ-cell tumors: Incidence and implications for hematologic recovery
and clinical outcome. Bone Marrow Transplant 20: 813–819, 1997
15. Guddati AK, Joy PS, Marak CP. Dose adjustment of carboplatin in patients on hemodialysis. Med Oncol 31: 848, 2014
16. Janus N, Thariat J, Boulanger H, Deray G, Launay-Vacher V. Proposal
for dosage adjustment and timing of chemotherapy in hemodialyzed
patients. Ann Oncol 21: 1395–1403, 2010
17. Sastry J, Kellie SJ. Severe neurotoxicity, ototoxicity and nephrotoxicity
following high-dose cisplatin and amifostine. Pediatr Hematol Oncol
22: 441–445, 2005
18. Arany I, Safirstein RL. Cisplatin nephrotoxicity. Semin Nephrol 23: 460–
464, 2003
19. Boulikas T, Vougiouka M:. Cisplatin and platinum drugs at the molecular level. (Review). Oncol Rep 10: 1663–1682, 2003
20. Dentino M, Luft FC, Yum MN, Williams SD, Einhorn LH. Long term effect of cis-diamminedichloride platinum (CDDP) on renal function and
structure in man. Cancer 41: 1274–1281, 1978
21. Meijer S, Mulder NH, Sleijfer DT, Donker AJ, Sluiter WJ, de Jong PE,
Schraffordt Koops H, van der Hem GK. Influence of combination chemotherapy with cis-diamminedichloroplatinum on renal function:
Long-term effects. Oncology 40: 170–173, 1983
American Society of Nephrology
22. Hamilton CR, Bliss JM, Horwich A. The late effects of cis-platinum on
renal function. Eur J Cancer Clin Oncol 25: 185–189, 1989
23. Hansen SW, Groth S, Daugaard G, Rossing N, Rørth M. Long-term effects on renal function and blood pressure of treatment with cisplatin,
vinblastine, and bleomycin in patients with germ cell cancer. J Clin
Oncol 6: 1728–1731, 1988
24. Fjeldborg P, Sørensen J, Helkjaer PE. The long-term effect of cisplatin
on renal function. Cancer 58: 2214–2217, 1986
25. Madias NE, Harrington JT. Platinum nephrotoxicity. Am J Med 65: 307–
314, 1978
26. Erlichman C, Soldin SJ, Thiessen JJ, Sturgeon JF, Fine S. Disposition of
total and free cisplatin on two consecutive treatment cycles in patients
with ovarian cancer. Cancer Chemother Pharmacol 19: 75–79, 1987
27. Reece PA, Stafford I, Russell J, Gill PG. Reduced ability to clear ultrafilterable platinum with repeated courses of cisplatin. J Clin Oncol 4:
1392–1398, 1986
28. Morland BJ, Mann JR, Milford DV, Raafat F, Stevens MC. Ifosfamide
nephrotoxicity in children: Histopathological features in two cases.
Med Pediatr Oncol 27: 57–61, 1996
29. Zaki EL, Springate JE, Taub M. Comparative toxicity of ifosfamide
metabolites and protective effect of mesna and amifostine in cultured
renal tubule cells.Toxicol In Vitro 17: 397–402, 2003
30. Furlanut M, Franceschi L. Pharmacology of ifosfamide. Oncology 65
[Suppl 2]: 2–6, 2003
31. Allen LM, Creaven PJ. Pharmacokinetics of ifosfamide. Clin Pharmacol
Ther 17: 492–498, 1975
32. Meanwell CA, Blake AE, Kelly KA, Honigsberger L, Blackledge G.
Prediction of ifosfamide/mesna associated encephalopathy. Eur J
Cancer Clin Oncol 22: 815–819, 1986
33. Latcha S, Maki RG, Schwartz GK, Flombaum CD. Ifosfamide may be
safely used in patients with end stage renal disease on hemodialysis.
Sarcoma 2009: 575629, 2009
34. Carlson L, Goren MP, Bush DA, Griener JC, Quigley R, Tkaczewski I,
Kamen BA, Weitman SD. Toxicity, pharmacokinetics, and in vitro hemodialysis clearance of ifosfamide and metabolites in an anephric
pediatric patient with Wilms’ tumor. Cancer Chemother Pharmacol 41:
140–146, 1998
35. de Jonge ME, Huitema AD, Rodenhuis S, Beijnen JH. Clinical pharmacokinetics of cyclophosphamide. Clin Pharmacokinet 44: 1135–
1164, 2005
36. Haubitz M, Bohnenstengel F, Brunkhorst R, Schwab M, Hofmann U,
Busse D. Cyclophosphamide pharmacokinetics and dose requirements
in patients with renal insufficiency. Kidney Int 61: 1495–1501, 2002
37. Juma FD, Rogers HJ, Trounce JR. Effect of renal insufficiency on the
pharmacokinetics of cyclophosphamide and some of its metabolites.
Eur J Clin Pharmacol 19: 443–451, 1981
38. Flombaum CD, Meyers PA. High-dose leucovorin as sole therapy for
methotrexate toxicity. J Clin Oncol 17: 1589–1594, 1999
39. Howell SB, Carmody J. Changes in glomerular filtration rate associated
with high-dose methotrexate therapy in adults. Cancer Treat Rep 61:
1389–1391, 1977
40. Crom WR, Pratt CB, Green AA, Champion JE, Crom DB, Stewart CF,
Evans WE. The effect of prior cisplatin therapy on the pharmacokinetics
of high-dose methotrexate. J Clin Oncol 2: 655–661, 1984
41. Wall SM, Johansen MJ, Molony DA, DuBose TD Jr, Jaffe N, Madden T.
Effective clearance of methotrexate using high-flux hemodialysis
membranes. Am J Kidney Dis 28: 846–854, 1996
42. Relling MV, Stapleton FB, Ochs J, Jones DP, Meyer W, Wainer IW,
Crom WR, McKay CP, Evans WE. Removal of methotrexate, leucovorin,
and their metabolites by combined hemodialysis and hemoperfusion.
Cancer 62: 884–888, 1988
43. Widemann BC, Adamson PC. Understanding and managing methotrexate nephrotoxicity. Oncologist 11: 694–703, 2006
44. Chauvet S, Courbebaisse M, Ronco P, Plaisier E. Pemetrexed-induced
acute kidney injury leading to chronic kidney disease. Clin Nephrol 82:
402–406, 2014
Onco-Nephrology Curriculum
7
45. Glezerman IG, Pietanza MC, Miller V, Seshan SV. Kidney tubular toxicity
of maintenance pemetrexed therapy. Am J Kidney Dis 58: 817–820,
2011
46. Eli Lilly Canada Inc: Product Monograph Altima Pemetrexed Disodium
for Injection, 2013. Available at: http://www.lilly.ca/en/pdf/productmonograph/14_alimta-pm-10may2013.pdf. Accessed January 28, 2016
47. Alberts DS, Chang SY, Chen HS, Moon TE, Evans TL, Furner RL,
Himmelstein K, Gross JF. Kinetics of intravenous melphalan. Clin
Pharmacol Ther 26: 73–80, 1979
48. Cornwell GG 3rd, Pajak TF, McIntyre OR, Kochwa S, Dosik H. Influence
of renal failure on myelosuppressive effects of melphalan: Cancer and
Leukemia Group B experience. Cancer Treat Rep 66: 475–481, 1982
49. Badros A, Barlogie B, Siegel E, Roberts J, Langmaid C, Zangari M,
Desikan R, Shaver MJ, Fassas A, McConnell S, Muwalla F, Barri Y,
Anaissie E, Munshi N, Tricot G. Results of autologous stem cell transplant in multiple myeloma patients with renal failure. Br J Haematol
114: 822–829, 2001
50. Hamaki T, Katori H, Kami M, Yamato T, Yamakado H, Itoh T, Kusumi E,
Igarashi M, Ueyama J, Kanda Y, Miyakoshi S, Mineishi S, Morinaga S,
Mukai M, Hayashi M, Takaue Y, Hara S, Mutou Y. Successful allogeneic
blood stem cell transplantation for aplastic anemia in a patient with
renal insufficiency requiring dialysis. Bone Marrow Transplant 30: 195–
198, 2002
51. Chen N, Lau H, Kong L, Kumar G, Zeldis JB, Knight R, Laskin OL.
Pharmacokinetics of lenalidomide in subjects with various degrees of
renal impairment and in subjects on hemodialysis. J Clin Pharmacol 47:
1466–1475, 2007
52. Niesvizky R, Naib T, Christos PJ, Jayabalan D, Furst JR, Jalbrzikowski J,
Zafar F, Mark T, Lent R, Pearse RN, Ely S, Leonard JP, Mazumdar M,
Chen-Kiang S, Coleman M. Lenalidomide-induced myelosuppression
is associated with renal dysfunction: Adverse events evaluation of
treatment-naïve patients undergoing front-line lenalidomide and
dexamethasone therapy. Br J Haematol 138: 640–643, 2007
53. Shaaban H, Layne T, Guron G. A case of DRESS (drug reaction with
eosinophilia and systemic symptoms) with acute interstitial nephritis
secondary to lenalidomide. J Oncol Pharm Pract 20: 302–304, 2013
54. Batts ED, Sanchorawala V, Hegerfeldt Y, Lazarus HM. Azotemia associated with use of lenalidomide in plasma cell dyscrasias. Leuk Lymphoma 49: 1108–1115, 2008
55. Lipson EJ, Huff CA, Holanda DG, McDevitt MA, Fine DM. Lenalidomideinduced acute interstitial nephritis. Oncologist 15: 961–964, 2010
56. Available at: http://www.celgenecanada.net/pdfs/Revlimid%20
Product_Monograph_-_English_Version.pdf
57. Powis G. Effect of human renal and hepatic disease on the pharmacokinetics of anticancer drugs. Cancer Treat Rev 9: 85–124, 1982
58. Damon LE, Mass R, Linker CA. The association between high-dose
cytarabine neurotoxicity and renal insufficiency. J Clin Oncol 7: 1563–
1568, 1989
59. Smith GA, Damon LE, Rugo HS, Ries CA, Linker CA. High-dose cytarabine dose modification reduces the incidence of neurotoxicity in patients with renal insufficiency. J Clin Oncol 15: 833–839, 1997
60. Pöschl JM, Klaus G, Querfeld U, Ludwig R, Mehls O. Chemotherapy
with cytosine arabinoside in a child with Burkitt’s lymphoma on maintenance hemodialysis and hemofiltration. Ann Hematol 67: 37–39,
1993
61. Tsuchiya Y, Ubara Y, Suwabe T, Hoshino J, Sumida K, Hiramatsu R,
Hasegawa E, Yamanouchi M, Hayami N, Marui Y, Sawa N, Takemoto F,
Takaichi K. Successful treatment of acute promyelocytic leukemia in a
patient on hemodialysis. Clin Exp Nephrol 15: 434–437, 2011
62. Radeski D, Cull GM, Cain M, Hackett LP, Ilett KF. Effective clearance of
Ara-U the major metabolite of cytosine arabinoside (Ara-C) by hemodialysis in a patient with lymphoma and end-stage renal failure. Cancer
Chemother Pharmacol 67: 765–768, 2011
63. Poole C, Gardiner J, Twelves C, Johnston P, Harper P, Cassidy J,
Monkhouse J, Banken L, Weidekamm E, Reigner B. Effect of renal
8
Onco-Nephrology Curriculum
64.
65.
66.
67.
68.
69.
70.
71.
72.
73.
74.
75.
76.
77.
78.
79.
80.
81.
impairment on the pharmacokinetics and tolerability of capecitabine
(Xeloda) in cancer patients. Cancer Chemother Pharmacol 49: 225–
234, 2002
Hoffman-LaRoche Limited: Product Monograph Xeloda. Available
at: http://www.rochecanada.com/fmfiles/re7234008/Research/
ClinicalTrialsForms/Products/ConsumerInformation/Monographsand
PublicAdvisories/Xeloda/Xeloda_PM_E.pdf. Accessed January 28, 2016
Jhaveri KD, Flombaum C, Shah M, Latcha S. A retrospective observational study on the use of capecitabine in patients with severe renal
impairment (GFR ,30 mL/min) and end stage renal disease on hemodialysis. J Oncol Pharm Pract 18: 140–147, 2012
Alberts DS, Chen HS, Mayersohn M, Perrier D, Moon TE, Gross JF.
Bleomycin pharmacokinetics in man. II. Intracavitary administration.
Cancer Chemother Pharmacol 2: 127–132, 1979
Crooke ST, Comis RL, Einhorn LH, Strong JE, Broughton A, Prestayko
AW. Effects of variations in renal function on the clinical pharmacology
of bleomycin administered as an iv bolus. Cancer Treat Rep 61: 1631–
1636, 1977
Dalgleish AG, Woods RL, Levi JA. Bleomycin pulmonary toxicity: Its
relationship to renal dysfunction. Med Pediatr Oncol 12: 313–317, 1984
Gafter-Gvili A, Ram R, Gafter U, Shpilberg O, Raanani P. Renal failure
associated with tyrosine kinase inhibitors: Case report and review of the
literature. Leuk Res 34: 123–127, 2010
Kurita N, Mise N, Fujii A, Ikeda S, Sugimoto T. Crescentic glomerulonephritis in a patient with advanced lung cancer during erlotinib therapy. NDT Plus 2: 512–513, 2009
Usui J, Glezerman IG, Salvatore SP, Chandran CB, Flombaum CD,
Seshan SV. Clinicopathological spectrum of kidney diseases in cancer
patients treated with vascular endothelial growth factor inhibitors: A report of 5 cases and review of literature. Hum Pathol 45: 1918–1927, 2014
Hoffman-LaRoche Limited: Product Monograph Avastin. Available
at: http://rochecanada.com/fmfiles/re7234008/Research/Clinical
TrialsForms/Products/ConsumerInformation/MonographsandPublic
Advisories/Avastin/Avastin_PM_E.pdf. Accessed January 28, 2016
Garnier-Viougeat N, Rixe O, Paintaud G, Ternant D, Degenne D,
Mouawad R, Deray G, Izzedine H. Pharmacokinetics of bevacizumab in
haemodialysis. Nephrol Dial Transplant 22: 975, 2007
Pappas P, Karavasilis V, Briasoulis E, Pavlidis N, Marselos M. Pharmacokinetics of imatinib mesylate in end stage renal disease. A case study.
Cancer Chemother Pharmacol 56: 358–360, 2005
Novartis Pharmaceuticals Corporation: Highlights of Prescribing Information Gleevac, 2015. Available at: https://www.pharma.us.
novartis.com/product/pi/pdf/gleevec_tabs.pdf. Accessed January 28,
2016
Izzedine H, Etienne-Grimaldi MC, Renée N, Vignot S, Milano G. Pharmacokinetics of sunitinib in hemodialysis. Ann Oncol 20: 190–192,
2009
Baryer Inc: Product Monograph Nexavar, 2014. Available at: http://
www.bayer.ca/omr/online/nexavar-pm-en-17dec2014-178086.pdf.
Accessed January 28, 2016
Miller AA, Murry DJ, Owzar K, Hollis DR, Kennedy EB, Abou-Alfa G,
Desai A, Hwang J, Villalona-Calero MA, Dees EC, Lewis LD, Fakih MG,
Edelman MJ, Millard F, Frank RC, Hohl RJ, Ratain MJ. Phase I and
pharmacokinetic study of sorafenib in patients with hepatic or renal
dysfunction: CALGB 60301. J Clin Oncol 27: 1800–1805, 2009
AstraZeneca Pharmaceuticals Corporation: Highlights of Prescribing
Information Caprelsa. Available at: http://www1.astrazeneca-us.com/
pi/vandetanib.pdf. Accessed January 28, 2016
Perazella MA, Markowitz GS. Bisphosphonate nephrotoxicity. Kidney
Int 74: 1385–1393, 2008
Kyle RA, Yee GC, Somerfield MR, Flynn PJ, Halabi S, Jagannath S,
Orlowski RZ, Roodman DG, Twilde P, Anderson K; American Society of
Clinical Oncology. American Society of Clinical Oncology 2007 clinical
practice guideline update on the role of bisphosphonates in multiple
myeloma. J Clin Oncol 25: 2464–2472, 2007
American Society of Nephrology
REVIEW QUESTIONS
1. What is the best option for management of a patient on
cisplatin who has lost approximately 50% of the GFR following the first three cycles?
a. Continue cisplatin at the present dose
b. Continue cisplatin at a reduced dose
c. Consider alternative therapies
Answer: c is correct. Patients can have episodes of AKI
following each dose of cis-platinum. With each subsequent
episode of AKI, the baseline serum creatinine may not return to its pretreatment value. Up to 30% of patients may
have residual renal insufficiency due to cis-platinum nephrotoxicity. In this patient who has lost about 50% of his GFR
following his initial treatment, cisplatin is not recommended, and it may be best to consider alternative
therapies.
2. What is the dose of zoledronic acid recommended by
American Society of Clinical Oncology for the administration to patients with CrCl of 30–39 mL/min?
American Society of Nephrology
a.
b.
c.
d.
4 mg
3.5 mg
3 mg
Do not administer
Answer: c is correct. The American Society of Clinical Oncology recommends that zoledronate should be avoided in
patients with CrCl of ,30 mL/min and that the initial dose of
4 mg be reduced to 3.5 mg for CrCl of 50–60 mL/min; 3.3 mg for
CrCl of 40–49 mL/min; and 3 mg for CrCl of 30–39 mL/min.
3. What is the appropriate timing of cisplatin administration
in an ESRD patient receiving hemodialysis?
a.
b.
c.
d.
2 hours prior to hemodialysis
6 hours prior to hemodialysis
12 hours prior to hemodialysis
On the off-dialysis day
Answer: d is correct. Because the drug is highly and irreversibly protein bound and because free cisplatin is well dialyzed,
drug that is removed by dialysis cannot be replaced by bound
drug. As such, cisplatin must be given on a nondialysis day.
Onco-Nephrology Curriculum
9
Chapter 13: CKD as a Complication of Cancer
Laura Cosmai, MD,* Camillo Porta, MD,† and Maurizio Gallieni, MD, FASN‡
*Nephrology and Dialysis, Istituti Ospitalieri Cremona, Cremona, Italy; †Medical Oncology, I.R.C.C.S., San Matteo
University Hospital Foundation, Pavia, Italy; and ‡Nephrology and Dialysis, San Carlo Borromeo Hospital, University of
Milano, Milano, Italy
CKD AND CANCER: A “CIRCULAR”
RELATIONSHIP
CKD is recognized as a disease that may complicate
cancer and its therapy. This is in part related to the fact
that preexisting CKD is highly prevalent in oncologic
patients. Indeed, as observed in the Renal Insufficiency and Anticancer Medications (IRMA)-1 and -2
studies (1,2), more than half of patients with an active
malignancy present with an eGFR of ,90 mL/min
per 1.73 m2. Furthermore, the prevalence of more
severe CKD (i.e., stages 3–5), not requiring dialysis,
was 12.0% and 11.8%, respectively (1,2). Similar results have been reported in other series from different
countries, thereby confirming that CKD is a relatively
common occurrence in cancer patients, irrespective
of the type of malignancy. As a whole, causes potentially able to have a negative impact on kidney function are summarized in Table 1.
Interestingly, the relationship between the kidney
and cancer appears to be bidirectional (3). For example, preexisting CKD may impact the bioavailability
and/or safety profile of an anticancer drug, potentially leading to different and sometimes suboptimal
treatment choices. On the other hand, it is also possible that the renal effects of a novel anticancer drug
may lead to progressive kidney injury or to worsening
of preexisting CKD (3). In addition to the observed
increase in CKD prevalence in cancer patients, both
CKD and ESRD are risk factors for a number of malignancies (4). However, not all solid tumors appear
to be equally represented in this population.
A retrospective cohort study of 1,190,538 adults
assessed the association between eGFR level and the
risk of incident cancer (5). During 6,000,420 personyears of follow-up, 76,809 incident cancers were
identified in 72,875 subjects. After adjustment for
time-updated confounders, lower eGFR was associated with an increased risk of renal cancer, with an
adjusted hazard ratio (HR) of 2.28 (95% CI, 1.78–
2.92) for an eGFR ,30 mL/min per 1.73 m2 (5).
American Society of Nephrology
The authors also observed an increased risk of
urothelial cancer at an eGFR ,30 mL/min per
1.73 m2 but no significant associations between
eGFR and other cancers. Finally, CKD conferred an
increased cancer-specific mortality in patients with
kidney and urinary tract cancer (6). In ESRD patients
on dialysis, the observed increased risk for renal parenchymal cancer is related to the development of acquired renal cystic disease, which increases with time
on dialysis (7).
CKD and antineoplastic drugs
Acute and chronic kidney injury associated with
antineoplastic drug exposure is well described for the
classic cytotoxic agents that are used. In addition,
there is a large body of literature that describes dosing
of these drugs in patients with underlying renal
dysfunction and those on dialysis; however, little is
known about the appropriate use of the new targeted
agents in this population. This creates a complicated
issue for oncologists and nephrologists who care for
these patients and must provide both safe and effective
anticancer therapy. After decades of use of common
cytotoxic drugs, clinicians versed in cancer care and its
complications are well aware of the main toxicities of
these agents. The new, molecularly targeted, anticancer drugs that are entering clinical practice have a
wide array of previously unrecognized and ill-defined
adverse effects (8). Ultimately, these toxicities must
be readily recognized and managed by those providing care for patients exposed to these drugs. This
Correspondence: Laura Cosmai, Division of Nephrology and
Dialysis, Istituti Ospitalieri di Cremona, Largo Priori, 1, 26100,
Cremona, Italy. Email: [email protected]
All authors are members of the Joint Italian Association of Medical
Oncology (AIOM)/Italian Society of Nephrology (SIN) Working
Group on Onco-Nephrology.
L.C. and C.P. contributed equally to the preparation of this
chapter.
Copyright © 2016 by the American Society of Nephrology
Onco-Nephrology Curriculum
1
Table 1. Causes of kidney disease in cancer patients
Cause
Antineoplastic drugs
Traditional chemotherapeutic agents
Novel targeted therapies
Mechanism(s)
c Direct nephrotoxicity (e.g., cisplatin)
c Hypertension and/or proteinuria (e.g., VEGF[Rs]-targeted agents)
c TMA (e.g., VEGF-targeted agents)
c Interstitial nephritis and other glomerulonephritis
c Autoimmune nephropathies (e.g., anti-CTLA4 and anti-PD1/PDL1
antibodies)
c Indirect toxicities (e.g., nausea/vomiting, diarrhea, dysgeusia)
leading to dehydration/volume depletion
Other drugs used in cancer patients
Anti-pain drugs
Bisphosphonates
Radiation therapy
Contrast medium
Paraneoplastic renal syndromes
Nephrectomy
For cancer
For other causes
c Direct nephrotoxicity (e.g., NSAIDs, bisphosphonates)
Obstruction/compression
c Mechanical injury
Tumor Infiltration
c Kidney infiltration
Comorbid risk factors
c Hypertension
c Preexisting CKD
c Still ill defined
c Direct nephrotoxicity
c Autoimmune mechanism?
c Loss of nephrons
c AKI
c Diabetes mellitus
c AKI
c Previous use of nephrotoxic cancer therapies
VEGF(Rs), vascular endothelial growth factor (receptors); TMA, thrombotic microangiopathies; CTLA4, cytotoxic T-lymphocyte antigen 4; PD1, programmed cell
death 1; PDL1, programmed cell death ligand 1; NSAIDs, nonsteroidal anti-inflammatory drugs.
includes understanding risk factors for targeted drug-induced
kidney injury, appropriate drug dosing (if known) for the patient
with AKI, CKD, and those on dialysis, the clinical manifestations
of drug nephrotoxicity, and the optimal management of nephrotoxic complications (8).
Frequently, oncologists ask their nephrology colleagues to
assess the degree of kidney function impairment to provide
insight intodosageadjustmentofanticancer therapy.Toaccomplish
this, a thorough knowledge of the specific metabolism of anticancer
agents and of their pharmacokinetic and pharmacodynamic
properties is mandatory. The thought process includes deciding
“if” the drug should be administered, “when” it is appropriate to
dose the drugs, and to “what extent” dosage adjustment should be
used in the setting of underlying kidney disease (3). This approach
must be carefully done, as unnecessary treatment interruptions and
drug dose reductions may be associated with suboptimal cancer
therapy and hamper the clinical benefits of cancer therapy.
Optimal management of underlying CKD and its complications, which may be significantly ameliorated in many cases, as
well as prevention of further kidney damage from other
exogenous nephrotoxins (e.g., contrast medium, nonsteroidal
anti-inflammatory drugs, bisphosphonates) in cancer patients
with preexisting CKD, is also key to minimizing drug-related
complications. Unfortunately, patients with CDK and those on
dialysis are often undertreated for their neoplastic disease due
to the fear of drug-induced adverse effects.
2
Onco-Nephrology Curriculum
Although the relationship between kidney function and
cytotoxic agents will be covered in other chapters of the
curriculum, the pharmacokinetic properties and specific renal
toxicities of novel anticancer agents are summarized in Table 2.
The pharmacokinetics of these drugs are similar as most are
90%–98% bound to plasma proteins, and their excretion occurs predominantly via the feces (or the reticulo-endothelial
system), whereas urinary excretion is quite variable from one
drug to the other (8).
As stated in the drugs’ Summary of Product Characteristics
(SmPC), the pharmacokinetic properties of the majority of these
drugs are not influenced by kidney function (3). A population
pharmacokinetic model, which includes data from subjects with
baseline creatinine clearance ranging from 30 to 150 mL/min,
indicated that it is unlikely that renal insufficiency has a clinically
relevant effect on the pharmacokinetics of targeted therapies (9).
Thus, no dosage adjustment is recommended in patients
with a creatinine clearance .30 mL/min. To date, only patients
with adequate kidney function (serum creatinine #1.5 times the
upper limit of normal) have been included in registrative
randomized controlled trials. In patients with a creatinine
clearance ,30 mL/min, a patient population that is poorly
studied, caution is recommended (10). Interestingly, this conservative recommendation is not based on data, as drug exposure in patients with severe renal impairment was similar to
that observed in patients with normal kidney function (10).
American Society of Nephrology
Table 2. Renal toxicities of anticancer targeted agents (modified from reference 3)
Drug
Patients with renal
function impairment Renal
included in pivotal excretion
trial
VEGF/VEGFR-tageting
agents
Bevacizumab
Aflibercept
Sunitinib
Pazopanib
Axitinib
Other multikinase
inhibitors
Sorafenib
Dose reduction required?
Most-frequent
renal AEs
Patients with mild
to moderate CKD
Patients with
severe CKD
Patients
receiving
dialysis
No
No
No
No
No
No
No
16%
,4%
23%
Hypertension, proteinuria
Hypertension, proteinuria
Hypertension, proteinuria
Hypertension, proteinuria
Hypertension, (proteinuria)
No
No
No
No
No
No (no data)
No (no data)
No (no data)
No (no data)
No (no data)
No
No
No
No
No
No
19%
No
No (no data)
No
Regorafenib
No
19%
No
No (no data)
No data
Vandetanib
Imatinib
mTOR inhibitors
Everolimus
No
No
25%
13%
Hypertension, proteinuria,
hypophosphatemia
Hypertension, proteinuria,
electrolyte disorders
Hypertension, proteinuria, AKI
More renoprotective effects
No
No
Yes (few data)
No (no data)
No
No
No
2%
No
No
4.6%
No (no data);
suspend if AKI
No (no data);
suspend if AKI
No
Temsirolimus
Proteinuria, AKI, electrolyte
disorders
As for everolimus, but less
frequent
EGFR inhibitors
Gefitinib
Erlotinib
Afatinib
Cetuximab
No
No
No
No
,4%
,9%
,5%
No
No
No
No
No
No (no data)
No (no data)
No (no data)
No (no data)
No
No
No (no data)
No
Panitumumab
No
No
Electrolyte disorders
Electrolyte disorders
Electrolyte disorders
Hypomagnesemia, other
electrolyte disorders
Hypomagnesemia, other
electrolyte disorders
No
No (no data)
No (no data)
No
1%
AKI (tubular necrosis?)
No
No (no data)
Dabrafenib
No
23%
No
No (no data)
Possible
(risk of
arrhythmia)
No (no data)
Trametinib
No
No
No (no data)
No (no data)
No
No (no data)
No
No
No
No
No (no data)
No (no data)
No (no data)
No (no data)
No
No (no data)
No
No (no data)
No (no data)
B-Raf inhibitors and
MEK inhibitors
Vemurafenib
ERBB2-targeting agents
Trastuzumab
Hypophosphatemia,
(granulomatous nephritis?)
,20% Hypertension, hyponatremia
(with dabrafenib)
No
No
Hypertension, AKI
(with cisplatin)
No issues
No issues
Hypokalemia
Pertuzumab
Lapatinib
Trastuzumab emtansine
Antibodies against CTLA4
Ipilimumab
No
No
No
No
2
,5%
No
No
Autoimmune nephritis, (drug
reaction with eosinophilia
and systemic symptom
syndrome?)
Other agents
Crizotinib
No
No
No
No
Reduction of eGFR
(tubular necrosis?),
renal cysts
No issues
Catumaxomab
No
Possible, with
caution
No
No
Possible, with
No (no data)
caution (no data)
No (no data)
No (no data)
AE, adverse event.
American Society of Nephrology
Onco-Nephrology Curriculum
3
CKD postnephrectomy for kidney cancer
Surgical resection remains the gold standard treatment for localized renal cell carcinoma (RCC) and is also commonly performed
for synchronous metastatic disease. The type of surgical resection
utilized to treat RCC for the last several decades has been radical
nephrectomy, although, more recently, increasing emphasis has
been placed on the concept of nephron-sparing procedures (11).
Although radical nephrectomy and nephron-sparing surgery
do not appear to differ in terms of oncologic outcome, the two
different strategies differ in terms of incidence of postoperative
CKD and of cardiovascular complications (12). Less invasive surgical procedures for RCC are associated with improved outcomes
with less postoperative AKI and CKD and less cardiovascular complications (12). This is not trivial considering that 22% of patients
with renal tumors had a pre-nephrectomy stage 3 or greater CKD
(eGFR , 60 mL/min per 1.73 m2) (13). In patients 70 years of age
and older, the percentage approaches 40% (13). Overall, among
662 patients scheduled for partial or radical nephrectomy, the
prevalence of stage 3 or greater CKD was 26% (14). Furthermore,
patients that had postoperative AKI following radical nephrectomy
had a 4.24-fold higher risk of developing new-onset CKD (15).
Progression of underlying CKD was also noted in patients
undergoing a radical nephrectomy procedure for an RCC (15).
Accordingly, both the American Urological Association (16)
and the European Association of Urology (17) endorsed partial
nephrectomy as the novel standard of care for organ-confined
tumors #4 cm (T1a) and suggested that it be considered as a
viable option for patients with tumors .4 but #7 cm (T1b).
Evidence for a relationship between the extent of kidney tissue
removal and the risk of CKD comes from single-center retrospective studies, population-based studies, and a single randomized, controlled, phase 3 study (12). Among the population-based
studies available between 1990 and 2011, a meta-analysis of 36
studies, of which only one was prospective, examined 31,729
patients treated with radical nephrectomy and 9,281 patients
managed conservatively (18). The results demonstrated that partial nephrectomy was associated with a 19% reduction in the risk
of all-cause mortality (HR, 0.81; P ,0.0001), a 29% reduction in
cancer-specific mortality (HR, 0.71; P ,0.001), and a 61% reduction in the risk of severe CKD (HR, 0.39; P ,0.0001), supporting the findings observed in a number of smaller studies (12).
In contrast to these findings, however, the results of a highly
controversial randomized controlled trial, conducted by the
European Organization for the Research and Treatment of Cancer
(EORTC) (19), complicate the surgical approach to renal cell
cancer. In this trial, a more favorable outcome was observed in
patients treated with radical nephrectomy compared with those
treated conservatively. During a median follow-up of .9 years,
death occurred in 25% of patients treated by partial nephrectomy
and in 18.3% of those undergoing radical nephrectomy. Cardiovascular diseases were noted to be the most common cause of
death. The intention-to-treat analysis showed a 10-year overall
survival rate of 81.1% in the radical nephrectomy group compared with 75.5% in the partial nephrectomy group (HR, 1.5;
95% CI, 1.03–2.16). Interestingly, partial nephrectomy was
4
Onco-Nephrology Curriculum
associated with a 21% reduction in the absolute risk of developing moderate CKD (eGFR , 60 mL/min per 1.73 m2) over a
median follow-up of 6.7 years, whereas the difference in the
incidence of severe CKD (eGFR ,30 mL/min per 1.73 m2)
between the two groups was 3.7% (19).
These results have generated various hypotheses about the
effect of medical- versus surgical-associated CKD on patient
survival. It is speculated that medical disease–related CKD has
worse outcomes than CKD due to surgery (nephron loss),
likely related to other comorbidities and the primary renal
disease (i.e., diabetes mellitus). Indeed, a recent report suggested that patients with medical risk factors for CKD are at increased risk of progressive renal impairment, irrespective of
the use of partial nephrectomy (20).
The duration of renal ischemia during partial nephrectomy
may also play a key role in the development of postoperative CKD.
In a recent collaborative review (21), a strong association was
noted between the quality and quantity of renal tissue that is preserved after surgery and long-term kidney function, and also the
duration of ischemia proved to be an important modifiable predictor of postoperative kidney function. Prolonged warm ischemia time (WIT) proved to be significantly associated with adverse
postoperative kidney function (21). Available data suggest a renal
benefit of keeping WIT ,25 minutes (21). Conversely, cold ischemia appears to safely allow longer durations of ischemia (21).
Finally, patients with CKD (irrespective of its cause) within
the setting of a metastatic cancer usually tolerate cancer-targeted
agents poorly, experiencing higher-grade adverse events compared with patients with normal kidney function (3).
Tumor-induced CKD
End-stage cancer is often associated with malignant ureteral
obstruction (MUO), leading to obstructive nephropathy and
CKD. Direct ureteral infiltration by tumor or extrinsic ureter
compression by bulky tumor masses frequently cause these
sequelae (22). In general, cervical, bladder, and prostate cancers
are the most common culprits (22). However, urinary obstruction can also occur from retroperitoneal fibrosis due to surgery,
chemotherapy, and/or radiotherapy.
In the setting of malignant obstruction, percutaneous nephrostomy tube placement or retrograde ureteral double-J stent
placement should be urgently performed, recognizing the associated procedural complications (22). Recently, a prognostic
model for survival after palliative urinary diversion for malignant
urinary obstruction has been developed (23). Two risk factors (at
least four events related to malignancy and an Eastern Cooperative
Oncology Group [ECOG] index $2) were used to stratify patients
into three groups by survival type: favorable (no factors), intermediate (one factor), and unfavorable (two factors). The median
survival at 1, 6, and 12 months was 94.4%, 57.3%, and 44.9% in
the favorable group, 78.0%, 36.3%, and 15.5% in the intermediate
group, and 46.4%, 14.3%, and 7.1% in the unfavorable group,
respectively (23). This simple model could help to guide clinical
decisions when choosing which patients are reasonable candidates
for urinary diversion in a palliative setting.
American Society of Nephrology
CONCLUSION
CKD is highly prevalent in oncologic patients and appears to
be a risk factor for the development of cancer. Furthermore,
the use of antineoplastic drugs in patients with underlying
CKD raises several specific issues: 1) the direct nephrotoxicity
of several anticancer agents (especially novel molecularly
targeted agents); 2) the need to adjust antineoplastic doses
due to concomitant CKD; 3) the lack of prospective drug
dosing data in patients with advanced CKD or those on
dialysis; and 4) the nihilistic approach to the treatment of this
population of patients with CKD, leading to the frequent
undertreatment (or even absence of treatment) of these
patients, ultimately denying them potentially life-prolonging
options (3). Indeed, in our opinion, CKD should not be
regarded, in and of itself, as a reason to reduce or hold targeted therapies in the absence of other comorbidities or
medical indications (3).
TAKE HOME POINTS
c The relationship between CKD and cancer should be regarded as
bidirectional.
c The use of targeted agents in cancer patients with CKD is ill defined.
c CKD and ESRD requiring dialysis, per se, should not be regarded as
reasons not to administer anticancer treatments.
c Nephrectomy for kidney cancer is another common cause of CKD,
but the use of nephron-sparing surgical techniques have been developed to limit this issue.
c Medical disease–related CKD has worse outcomes than CKD due to
surgery.
c The tumor may cause CKD by causing urinary obstruction with obstructive nephropathy.
REFERENCES
1. Launay-Vacher V, Oudard S, Janus N, Gligorov J, Pourrat X, Rixe O,
Morere JF, Beuzeboc P, Deray G; Renal Insufficiency and Cancer
Medications (IRMA) Study Group. Prevalence of renal insufficiency in
cancer patients and implications for anticancer drug management: The
renal insufficiency and anticancer medications (IRMA) study. Cancer
110: 1376–1384, 2007
2. Launay-Vacher V. Epidemiology of chronic kidney disease in cancer
patients: Lessons from the IRMA study group. Semin Nephrol 30: 548–
556, 2010
3. Porta C, Cosmai L, Gallieni M, Pedrazzoli P, Malberti F. Renal effects of
targeted anticancer therapies. Nat Rev Nephrol 11: 354–370, 2015
4. Stengel B. Chronic kidney disease and cancer: A troubling connection.
J Nephrol 23: 253–262, 2010
5. Lowrance WT, Ordoñez J, Udaltsova N, Russo P, Go AS. CKD and the
risk of incident cancer. J Am Soc Nephrol 25: 2327–2334, 2014
6. Weng PH, Hung KY, Huang HL, Chen JH, Sung PK, Huang KC. Cancerspecific mortality in chronic kidney disease: Longitudinal follow-up of a
large cohort. Clin J Am Soc Nephrol 6: 1121–1128, 2011
7. Stewart JH, Buccianti G, Agodoa L, Gellert R, McCredie MR, Lowenfels
AB, Disney AP, Wolfe RA, Boyle P, Maisonneuve P. Cancers of the
kidney and urinary tract in patients on dialysis for end-stage renal disease: Analysis of data from the United States, Europe, and Australia and
New Zealand. J Am Soc Nephrol 14: 197–207, 2003
American Society of Nephrology
8. Porta C, Paglino C, Imarisio I, Bonomi L. Uncovering Pandora’s vase:
The growing problem of new toxicities from novel anticancer agents.
The case of sorafenib and sunitinib. Clin Exp Med 7: 127–134, 2007
9. Thomas SM, Grandis JR. Pharmacokinetic and pharmacodynamic
properties of EGFR inhibitors under clinical investigation. Cancer Treat
Rev 30: 255–268, 2004
10. Josephs D, Hutson TE, Cowey CL, Pickering LM, Larkin JM, Gore ME,
Van Hemelrijck M, McDermott DF, Powles T, Chowdhury P, Karapetis
C, Harper PG, Choueiri TK, Chowdhury S. Efficacy and toxicity of sunitinib in patients with metastatic renal cell carcinoma with severe renal
impairment or on haemodialysis. BJU Int 108: 1279–1283, 2011
11. Manikandan R, Srinivasan V, Rané A. Which is the real gold standard for
small-volume renal tumors? Radical nephrectomy versus nephronsparing surgery. J Endourol 18: 39–44, 2004
12. Li L, Lau WL, Rhee CM, Harley K, Kovesdy CP, Sim JJ, Jacobsen S,
Chang A, Landman J, Kalantar-Zadeh K. Risk of chronic kidney disease
after cancer nephrectomy. Nat Rev Nephrol 10: 135–145, 2014
13. Canter D, Kutikov A, Sirohi M, Street R, Viterbo R, Chen DY, Greenberg
RE, Uzzo RG. Prevalence of baseline chronic kidney disease in patients
presenting with solid renal tumors. Urology 77: 781–785, 2011
14. Huang WC, Levey AS, Serio AM, Snyder M, Vickers AJ, Raj GV, Scardino
PT, Russo P. Chronic kidney disease after nephrectomy in patients with
renal cortical tumours: a retrospective cohort study. Lancet Oncol 7:
735–740, 2006
15. Cho A, Lee JE, Kwon GY, Huh W, Lee HM, Kim YG, Kim DJ, Oh HY,
Choi HY. Post-operative acute kidney injury in patients with renal cell
carcinoma is a potent risk factor for new-onset chronic kidney disease
after radical nephrectomy. Nephrol Dial Transplant 26: 3496–3501,
2011
16. Campbell SC, Novick AC, Belldegrun A, Blute ML, Chow GK, Derweesh
IH, Faraday MM, Kaouk JH, Leveillee RJ, Matin SF, Russo P, Uzzo RG;
Practice Guidelines Committee of the American Urological Association. Guideline for management of the clinical T1 renal mass. J Urol
182: 1271–1279, 2009
17. Ljungberg B, Bensalah K, Canfield S, Dabestani S, Hofmann F, Hora M,
Kuczyk MA, Lam T, Marconi L, Merseburger AS, Mulders P, Powles T,
Staehler M, Volpe A, Bex A. EAU guidelines on renal cell carcinoma:
2014 update. Eur Urol 67: 913–924, 2015
18. Kim SP, Thompson RH, Boorjian SA, Weight CJ, Han LC, Murad MH,
Shippee ND, Erwin PJ, Costello BA, Chow GK, Leibovich BC. Comparative effectiveness for survival and renal function of partial and
radical nephrectomy for localized renal tumors: A systematic review
and meta-analysis. J Urol 188: 51–57, 2012
19. Van Poppel H, Da Pozzo L, Albrecht W, Matveev V, Bono A, Borkowski
A, Colombel M, Klotz L, Skinner E, Keane T, Marreaud S, Collette S,
Sylvester R. A prospective, randomised EORTC intergroup phase 3
study comparing the oncologic outcome of elective nephron-sparing
surgery and radical nephrectomy for low-stage renal cell carcinoma.
Eur Urol 59: 543–552, 2011
20. Satasivam P, Reeves F, Rao K, Ivey Z, Basto M, Yip M, Roth H, Grummet
J, Goad J, Moon D, Murphy D, Appu S, Lawrentschuk N, Bolton D,
Kearsley J, Costello A, Frydenberg M. Patients with medical risk factors
for chronic kidney disease are at increased risk of renal impairment
despite the use of nephron-sparing surgery. BJU Int 116: 590–595,
2015
21. Volpe A, Blute ML, Ficarra V, Gill IS, Kutikov A, Porpiglia F, Rogers C,
Touijer KA, Van Poppel H, Thompson RH. Renal ischemia and function
after partial nephrectomy: A collaborative review of the literature. Eur
Urol 68: 61–74, 2015
22. Russo P: Ureteral obstruction and stents: Still a difficult problem for
patients and urologists alike. J Urol 174: 2088, 2005
23. Cordeiro MD, Coelho RF, Chade DC, Pessoa RR, Chaib MS, ColomboJúnior JR, Pontes-Júnior J, Guglielmetti GB, Srougi M. A prognostic
model for survival after palliative urinary diversion for malignant
ureteric obstruction: A prospective study of 208 patients. BJU Int 117:
266–271, 2016
Onco-Nephrology Curriculum
5
REVIEW QUESTIONS
1. Which types of cancer are more commonly observed in
CKD patients?
a.
b.
c.
d.
Gastrointestinal cancers
Lung cancers
Breast cancers
Renal and urothelial cancers
Answer: d is correct. A large retrospective cohort study assessed the association between eGFR level and the risk of incident cancer. Lower eGFR was associated with an increased risk of
renal cancer with an adjusted HR of 2.28 (95% CI, 1.78–2.92) for
an eGFR ,30 mL/min per 1.73 m2. An increased risk of urothelial cancer at an eGFR ,30 mL/min per 1.73 m2 was also
evidenced. Finally, CKD conferred an increased cancer-specific
mortality in patients with kidney and urinary tract cancer.
2. Should the dose of a targeted anticancer agent be reduced
in a patient with mild to moderate CKD?
a.
b.
c.
d.
Yes, almost always
No, almost never
Yes, depending on the drug’s pharmacokinetic properties
The targeted anticancer agent should not be used in this
setting
Answer: b is correct. The pharmacokinetics of all targeted
anticancer agents are similar. Most are 90%–98% bound to
6
Onco-Nephrology Curriculum
plasma proteins, and their excretion occurs predominantly via
the feces (or the reticulo-endothelial system), whereas urinary
excretion is quite variable among the drugs. A population pharmacokinetic model indicated that it is unlikely that renal insufficiency has a clinically relevant effect on the pharmacokinetics
of targeted therapies. Thus, no dosage adjustment is recommended in patients with a creatinine clearance .30 mL/min.
3. Which of the following contributes to the development of
CKD in nephrectomized patients?
a.
b.
c.
d.
Nephrectomy itself (loss of nephrons)
Concomitant comorbidities
Ischemia time
All the above
Answer: d is correct. Radical nephrectomy and nephronsparing surgery greatly differ in terms of incidence of postoperative CKD, due to the different amount of nephron loss.
Less invasive surgical procedures for renal cell carcinoma are
associated with less postoperative AKI and CKD. Despite
this, a recent report suggested that patients with medical risk
factors for CKD are at increased risk of progressive renal impairment, irrespective of the use of partial nephrectomy, thus
highlighting the key role played by comorbidities in the development of post-nephrectomy CKD. The duration of ischemia
is another important predictor of postoperative kidney function. Prolonged warm ischemia time is significantly associated
with adverse postoperative kidney function.
American Society of Nephrology
Chapter 14: Hereditary Renal Cancer Syndromes
Katherine L. Nathanson, MD
Department of Medicine, Division of Translational Medicine and Human Genetics, and Cancer Control Program,
Abramson Cancer Center, Perelman School of Medicine at the University of Pennsylvania, Philadelphia, Pennsylvania
INTRODUCTION
Inherited forms of renal cancer are estimated to account for 2%–5% of all kidney cancer (1). Currently,
10 inherited cancer susceptibility syndromes are definitively associated with an increased risk of renal
cancer (Table 1) and are described in more detail below. Patients with these inherited syndromes develop
kidney cancer at an earlier age; furthermore, the lesions can be multifocal, bilateral, and heterogeneous.
Several, including Birt-Hogg-Dubé syndrome
(BHD), familial clear cell renal cancer due to chromosome 3 translocation, hereditary papillary renal cancer,
hereditary leiomyomatosis and renal cell cancer
(HLRCC), and von Hippel-Lindau disease (vHL)
have renal cancer as a primary feature, whereas in another inherited cancer susceptibility syndrome, such as
BAP1 mutation–associated disease, Lynch syndrome,
phosphatase and tensin homologue (PTEN) hamartoma syndrome, hereditary pheochromocytoma and
paraganglioma (due to SDHx mutations), and tuberous sclerosis complex, it is a secondary feature. Recently, mutations in CDKN2B and PBRM1 also have
been reported to predispose to clear cell renal cancer in
single case series (2,3) and, as such, need further validation. Many of the genes identified through the studies
of familial renal cancer have proven to play a critical role
in renal cancer development through somatic mutation, with vHL disease being the exemplar of this paradigm. The description of families with inherited cancer
susceptibility syndromes associated with an increased
risk of renal cancer has and will lead to the discovery
of mutated genes important in the pathogenesis of
renal cancer. Below, the features of the inherited cancer susceptibility syndromes associated with an increased risk of renal cancer, with a focus on the renal
manifestations and pathologic features, are reviewed.
VON-HIPPEL LINDAU DISEASE
Patients with this highly penetrant autosomal dominant cancer susceptibility syndrome can present with
American Society of Nephrology
a wide spectrum of hemangioblastomas of the brain,
spine, and retina, pancreatic cysts and neuroendocrine tumors, renal cysts and clear cell renal tumors,
endolympatic sac tumors, and pheochromocytomas. vHL disease is found across all ethnic groups,
with approximately one-quarter of the incidence due
to de novo mutations; genetic testing for mutations
in VHL detects nearly 100% of individuals with vHL
disease (6). Disease usually presents in the late teens
to early 20s, although an occasional individual may
be diagnosed in their mid-40s. The presentation of
renal disease is quite variable even within family
members, with some patients never developing renal
cancer, others having a few renal cysts, and others
with who have bilateral renal cancers and hundreds
of lesions within each kidney. In part, this variability
is due to a strong genotype–phenotype correlation
that is seen with a mutational type predictive of disease (7). Patients with type 1 (truncating) mutations
have a decreased incidence of pheochromocytoma
compared with those with type 2 (missense) mutations (8). Frameshift and nonsense mutations in VHL
are associated with a high penetrance of clear cell renal cancer, with a risk at age 50 of 70% (8). Full and
partial gene deletions of VHL confer a lower risk at
age 50 of 40%. Families with type 2 mutations have
either a low (type 2A) or high risk of clear cell renal cell
carcinoma (ccRCC) (type 2B); type 2C families develop pheochromocytoma only. Type 2A disease is
associated with the “Black Forest’ founder mutation
(Tyr98His) originating from southwestern Germany,
commonly found in the “Pennsylvania Dutch” population (9). Despite the variability in phenotype,
screening recommendations for vHL patients are
standardized, and in adult include annual CNS (brain,
spine) and abdominal/pelvic magnetic resonance
Correspondence: Katherine L. Nathanson, Perelman School of
Medicine at the University of Pennsylvania, 351 BRB 2/3, 421
Curie Blvd., Philadelphia, Pennsylvania 19104. Email: knathans@
exchange.upenn.edu
Copyright © 2016 by the American Society of Nephrology
Onco-Nephrology Curriculum
1
Table 1. Inherited cancer susceptibility syndromes associated with an increased risk of renal cancer
Syndrome
Gene
Protein
Predominant renal
Cancer Type
BAP1 mutation associated BAP1
cancer susceptibility
BRCA associated
protein
Clear cell
Birt-Hogg-Dubé syndrome FLCN
folliculin
Oncocytic,
chromophobe,
hybrid
Clear cell
Other Cancers
Non-Neoplastic Findings
Melanoma
Uveal melanoma
Mesothelioma
—
Epithelioid atypical
Spitz tumors
Fibrofolliculomas
Lung cysts,
pneumothorax
—
—
Familial clear cell renal
cancer due to
chromosome 3
translocation
Hereditary
Leiomyomatosis and
Renal Cell Cancer
Translocation
chr 3
FH
fumarate hydratase Papillary type 2
—
Hereditary Papillary
Renal Cancer
Hereditary
Pheochromocytoma
and Paraganglioma
MET
c-MET
Papillary type 1
—
SDHB
SDHC
SDHD
succinate
dehydrogenase
subunits B, C, D
Lynch Syndrome
MLH1
MSH2
Mismatch repair
proteins
MSH6
PMS2
PTEN
Clear cell
Paraganglioma
(distinct phenotype) Pheochromocytoma
Gastrointestinal
Stromal Tumor
Urothelial cancer
Colorectal cancer
(upper tract)
Endometrial (uterine)
cancer
Ovarian cancer
PTEN
Clear cell
Breast cancer
Thyroid cancer
TSC1
TSC2
hamartin
Tuberin
Angiomyolipoma
Epitheliod
angiomyolipoma
Angiomyolipomas
Subependymal giant
cell astrocytomas
VHL
pVHL
Clear cell
Clear cell papillary
CNS - hemangioblastoma
(brain, spine, retina)
Adrenal - pheochromocytoma
Inner ear - endolymphatic sac
tumors
Pancreas - neuroendocrine
tumors
PTEN Hamartoma
Syndrome
(Cowden syndrome)
Tuberous Sclerosis
Complex
Von Hippel Lindau
disease
(MR) imaging, ophthalmologic evaluation, plasma metanephrines, and consultation with vHL expert. The implementation
of screening guidelines has led to vast improvements in
survival (10).
Classically, vHL disease and mutations in VHL have been associated with clear cell renal cancers. However, clear cell papillary
renal cell carcinoma (CCPRCC), a relatively recently described entity with prominent papillary architecture, exclusive clear cell morphology, and a partially cystic appearance,
has been reported in patients with vHL disease (11). The exact
characteristics of CCPRCC in this context, and relationship to
sporadic disease are somewhat controversial, due to loss of
2
Onco-Nephrology Curriculum
Cutaneous
leiomyomas
Uterine
leiomyomas
—
—
Mucocutaneous papules,
hamartomas, lipomas,
macrocephaly
Facial angiofibroma
Hypomelanotic macule
Connective tissue nevus
Forehead plaque
Ungal and peri-ungal
fibromas
Pancreatic, renal cysts
chromosome 3p in these tumors and differing findings in regards to cytokeratin 7 staining (12). Based on multiple natural
history studies done at the National Cancer Institute, the standard
of care for timing of resection of renal cancer in patients with vHL
disease is when there is a solid component of 3 cm. In the initial
series, with a follow-up of .5 years, Walther et al. reported no
evidence of metastatic disease progression and no need for renal
transplantation or dialysis among 52 patients with tumors ,3 cm
at diagnosis. In contrast, distant metastases developed in 11 of 44
patients (25%) with lesions .3 cm in size, including 3 of 27
patients (11%) with lesions between 3 and 6 cm (13). Similar
results were obtained in a follow-up study 5 years later (14).
American Society of Nephrology
HEREDITARY PAPILLARY RENAL CELL CARCINOMA
(TYPE 1 PAPILLARY)
Hereditary papillary renal cell carcinoma (HPRCC) is an
autosomal dominant syndrome characterized by multifocal, bilateral type 1 papillary renal cell carcinomas, with
hundreds of tumors observed due to mutations in MET
(15). The tumors are indistinguishable pathologically
from sporadic type 1 papillary renal cancer. Mutations of
the MET gene on 7q31 have been causally associated with
HPRCC. Families with inherited mutations in MET leading to multifocal papillary renal cancer (type 1) are quite
rare, more so than other described inherited renal cancer
syndromes.
HEREDITARY LEIOMYOMATOSIS AND RENAL CELL
CANCER (TYPE 2 PAPILLARY)
Hereditary leiomyomatosis and renal cell cancer (HLRCC),
otherwise known as Reed’s syndrome, is an autosomal cancer
susceptibility syndrome characterized by the development of
cutaneous and uterine leiomyomas and renal cancer, due to
mutations in fumarate hydratase (FH) (16). The lifetime risk
of renal cancer is currently estimated to be 15% (17). The
pattern of renal cancer in HLRCC differs from other inherited
renal cancer susceptibility syndromes in that the tumors tend
to be solitary and unilateral and have a more aggressive course
of disease. Independent of underlying architecture, which is
most commonly described as a subtype of type 2 papillary
renal cancer, cells in the renal cancers associated with HLRCC
have a characteristic pathologic appearance with large nuclei,
with inclusion-like orangiophilic or eosinophilic nucleoi surrounded by a clear halo, which can be recognized by knowledgeable pathologists (18).
Recently, several studies have focused on using immunohistochemistry with S-(2-succinyl) cysteine (2SC) as a surrogate for FH deficiency, as an adjunct to pathologic features
to accurately diagnose HLRCC associated renal cancer. An
initial study from Bardella et al. suggested that positive staining with 2SC was sensitive and specific to detect renal cancers
associated with FH mutations (19). An independent followup study confirmed that 2SC demonstrated diffuse and strong
cytoplasmic staining in the confirmed HLRCC tumors compared with other tumor types (20). Thus, in addition to
family and personal history, histology and immunohistochemistry can be used to assist in the diagnosis of HLRCC
when the initial presenting manifestation is renal cancer.
The mean age of renal cancer diagnosis is 40 years, but
metastatic renal cancer can present in the teens. Given the
potential early age of renal cancer diagnosis, genetic testing
is recommended at age 8–10 for familial FH mutations with
annual MRI; however, the risk of renal cancers is relatively
low before age 20, so the drawbacks of screening should be
considered (17).
American Society of Nephrology
BIRT-HOGG-DUBÉ DISEASE
BHD disease is an autosomal dominant cancer susceptibility
syndrome characterized by the development of fibrofolliculomas
(dysplastic hair follicules), lung cysts and spontaneous pneumothorax, and renal cancer, and is due to mutations in folliculin
(FLCN) (21). The dermatologic features and lung disease are the
most common presenting features; BHD is underdiagnosed due
to its variable, often mild, presentation. Awide spectrum of renal
cancers (papillary RCC, ccRCC, mixed, and oncocytomas) has
been observed in patients with BHD, even within the same kidney (22). The renal parenchyma surrounding the renal tumor
can often contain multifocal oncocytosis. The most common
type of tumor is an unusual hybrid oncocytic tumor (mixed
oncocytoma and chromophobe). As a hybrid oncocytic tumor
is characteristic of BHD, any patient presenting with one should
be evaluated for BHD. Renal cancer is observed in approximately
30% of patients with a highly variable age of diagnosis ranging
from 20 to 83 years, with an average age of 46 years (23). Given
the low malignant potential of these tumors, it is generally recommended that screening with abdominal MRI take place every
2 years and that the tumors can be observed until they are 3 cm in
diameter prior to resection (24).
OTHER INHERITED SYNDROMES WITH AN
INCREASED RISK OF RENAL CANCER
BAP1 mutations and familial renal cancer
Mutations in BAP1 (BRCA-associated protein 1) were initially
identified through somatic sequencing of renal tumors. One of
the tumors in the initial study was found to carry a germ-line
BAP1 mutation, and subsequent studies suggested that BAP1
mutations predispose to familial clear cell renal cancer, along
with uveal and cutaneous melanoma and mesothelioma
(25,26).
Chromosome 3 translocations associated with clear cell
renal cancer
The first genetic changes identified as associated with inherited
risk of clear cell renal cancer were balanced translocations
involving chromosome 3; since then, multiple families have
been reported with multifocal bilateral disease (27). The
mechanism behind the increased risk of multifocal clear cell
renal cancer is thought to be loss of the rearranged chromosome during mitosis, which requires a quadrivalent (four
chromosomes coming together), leading to greater errors during chromosomal segregation. As multiple genes involved in
the pathogenesis of clear cell renal cancer are located on chromosome 3p, including VHL, PBRM1, BAP1, and SETD2 (28),
it is not surprising that a mechanism of increased loss of one
allele leads to an increased risk of clear cell renal cancer. Histologically, the clear cell renal cancers are indistinguishable
from those associated with VHL mutations, although the age
of onset tends to be later than in vHL disease.
Onco-Nephrology Curriculum
3
Lynch syndrome
Although Lynch syndrome (also known hereditary nonpolyposis colorectal cancer), due to mutations in the mismatch
repair genes (MMR) MLH1, MSH2, MSH6, and PMS2, is most
commonly associated with an increased risk of colorectal, endometrial, and ovarian cancers, upper tract urothelial cancers
are a well-recognized feature (29). The estimated risk ranges
from 2.6% to 11% (30,31). The risk appears to be highest in
patients with MSH2 and MLH1 mutations and presents at
earlier ages than sporadic disease (32). A recent review for
urologists suggests that evaluation for Lynch syndrome should
be done when a patient with upper tract urothelial cancer
presents before age 60 or the family meets Amsterdam I or II
criteria, which includes colorectal, small bowel, ureter, endometrial, and ovarian cancers (29). Screening for Lynch
syndrome can be done using immunohistochemistry for
the MMR proteins, in which loss of staining suggests the
diagnosis.
PTEN hamartoma tumor syndrome
Mutations in PTEN are associated with a pleomorphic syndrome
(PHTS, also known as Cowden disease and Bannayan-RileyRuvalcaba), which has a variety of disease manifestations ranging from cancer susceptibility to intellectual disability. Patients
are at increased risk of benign and malignant tumors of the
thyroid, breast, and endometrium; it has been suggested that
renal cancer is an underappreciated component of the cancer
spectrum (33). Recent estimates suggesting that 3%–5% of of
patients with PHTS have renal cancer, with a standardized
incidence ratio of 31.7 (95% CI, 15.4–58.4); however, these
estimates are based on small numbers and may be due to ascertainment basis (34). When centrally reviewed, the pathology
of the renal cancers was either papillary or chromophobe (34).
Screening for renal cancers is part of the standard surveillance
recommendations for patients with PHTS and is recommended biennially starting at age 40.
SDHx-associated paraganglioma/pheochromocytoma
Mutations in the five proteins (SDHA, SDHB, SDHC, SDHD,
and associated cofactor SDHAF2) that comprise the succinate
dehydrogenase complex, which participates in both the Krebs
cycle, converting fumarate to succinate, and as mitochondrial
respiratory chain complex II, have been associated with an
increased risk of pheochromocytomas, paragangliomas, gastrointestinal stromal tumors, and renal cancer (35). Renal cancer has been most commonly observed in association with
patients carrying SDHB mutations, which has the highest
risk of malignant disease, and these renal tumors have been
reported to be particularly aggressive (36). SDHB-associated
renal cancer displays a characteristic histopathology with solid
or focally cystic growth, uniform cytology with eosinophilic
flocculent cytoplasm, intracytoplasmic vacuolations and inclusions, and round to oval low-grade nuclei (37). Renal cancer can be the sentinel diagnosis in the family, so pathologists
should be aware of and alert to this potential diagnosis. SDHB
4
Onco-Nephrology Curriculum
immunohistochemistry (absence of staining) also can be used
to assist in making the diagnosis.
Tuberous sclerosis complex
Tuberous sclerosis complex (TSC) is an autosomal dominant
genetic disorder characterized by the formation of hamartomas in multiple organs, including brain, kidney, skin, and lung.
The formation of hamartomas leads to neurologic disorders,
including epilepsy, mental retardation, and autism, as well
as dermatologic manifestations such as facial angiofibromas,
renal angiomyolipomas, and pulmonary lymphangiomyomatosis (38). Inactivating mutations in TSC1 encoding hamartin,
or TSC2 encoding tuberin are responsible for the phenotype.
Patients with TSC2 mutations are more severely affected with
greater renal involvement among other features. Fifty percent to 80% of patients with TSC develop renal lesions including angiomyolipomas (AMLs), cysts, and oncocytomas;
renal cell cancer is estimated to occur in ,5% (with precise
estimates varying across studies) (39). A recent review of
TSC-associated renal cancer demonstrated young age at diagnosis, multifocal disease, an indolent clinical course, and
three morphologies: renal angiomyoadenomatous tumor,
chromophobe renal cancer, and a granular eosinophilicmacrocystic morphology (40). In patients with TSC2 mutations and multiple renal tumors, it has been shown that they
are due to a “shower” of second hits with different secondary
TSC2 mutations in each tumor (41). Everolimus is US Food
and Drug Administration (FDA) approved for treating AMLs
in the setting of TSC and also should be considered for TSCassociated renal cancer.
CONTROVERSIES IN GENETIC TESTING FOR
INHERITED SUSCEPTIBILITY TO RENAL CANCER
Two questions recently have arisen in relationship to genetic
testing for inherited susceptibility to renal cancer. Marston
Linehan and colleagues from the National Cancer Institute have
suggested that all patients with renal cancer diagnosed under the
age of 45 should have consideration of genetic counseling/germline mutation testing, even in the absence of a personal or family
medical history suggestive of an inherited syndrome (4). The
rates of renal cancer in patients under age 50 is steadily increasing and has doubled since 1995, going from 3 per 100,000 to 6
per 100,000 (5), presumably due to the increasing number of
incidental renal cancers detected on imaging studies. Although a
cutoff age of 45 for detecting cases of inherited renal cancer may
be quite sensitive, with the background rate increasing so dramatically, it would require testing of many individuals to identify
only a few with inherited disease. Population-based studies of
mutation testing in early-onset renal cancer are required to answer the question of utility of general genetic counseling and
testing in this setting. An earlier age cutoff, such as at 30, may
emerge as a more feasible alternative. As with many other genetic
diseases, multiplex gene panels using massively parallel
American Society of Nephrology
sequencing have emerged as an alternative for genetic testing
for renal cancer susceptibility syndromes (e.g., RenalNext from
Ambry Genetics). Given the usefulness of renal pathology and
extra-kidney manifestations to guide genetic testing, which is relatively unusual for other cancer types, the usefulness of panels
that include genes in which mutations predispose to vastly different diseases (e.g., vHL and HLRCC) is not immediately apparent.
However, many institutions do not always differentiate renal cancer pathologies to the degree needed for prioritization of genetic
testing studies, and even at experienced centers, discrimination
of renal cancer pathologies can be complex on occasion. Thus,
there has been uptake of massively parallel sequencing panels
for renal cancer–associated syndromes, but not to the same extent as for other cancer types (e.g., breast and ovarian cancers,
pheochromocytoma).
TAKE HOME POINTS
c Many different types of hereditary renal cancer exist, and in general,
each is associated with a histologic subtype.
c Renal cancer can either be a major or a minor feature of a cancer suscep-
tibility syndrome, but early age of onset, unusual or pathognomic pathology, and multiplicity of tumors all should be red flags, which prompt
questions about family history and consideration of inherited disease.
c Standard of care surveillance recommendations are available for essen-
tially all renal cancer susceptibility syndromes and should be followed.
REFERENCES
1. Cho E, Adami HO, Lindblad P. Epidemiology of renal cell cancer. Hematol Oncol Clin North Am 25: 651–665, 2011
2. Jafri M, Wake NC, Ascher DB, Pires DE, Gentle D, Morris MR,
Rattenberry E, Simpson MA, Trembath RC, Weber A, Woodward ER,
Donaldson A, Blundell TL, Latif F, Maher ER. Germline Mutations in the
CDKN2B tumor suppressor gene predispose to renal cell carcinoma.
Cancer Discov 5: 723–729, 2015
3. Benusiglio PR, Couvé S, Gilbert-Dussardier B, Deveaux S, Le Jeune H,
Da Costa M, Fromont G, Memeteau F, Yacoub M, Coupier I, Leroux D,
Méjean A, Escudier B, Giraud S, Gimenez-Roqueplo AP, Blondel C,
Frouin E, Teh BT, Ferlicot S, Bressac-de Paillerets B, Richard S, Gad S. A
germline mutation in PBRM1 predisposes to renal cell carcinoma. J
Med Genet 52: 426–430, 2015
4. Shuch B, Vourganti S, Ricketts CJ, Middleton L, Peterson J, Merino MJ,
Metwalli AR, Srinivasan R, Linehan WM. Defining early-onset kidney
cancer: implications for germline and somatic mutation testing and
clinical management. J Clin Oncol 32: 431–437, 2014
5. Howlader N, Noone AM, Krapcho M, Garshell J, Miller D, Altekruse SF,
Kosary CL, Yu M, Ruhl J, Tatalovich Z, Mariotto A, Lewis DR, Chen HS,
Feuer EJ, Cronin KA. SEER Cancer Statistics Review, 1975-2012, Bethesda, MD, National Cancer Institute, 2015
6. Crossey PA, Foster K, Richards FM, Phipps ME, Latif F, Tory K, Jones
MH, Bentley E, Kumar R, Lerman MI. Molecular genetic investigations of
the mechanism of tumourigenesis in von Hippel-Lindau disease: Analysis
of allele loss in VHL tumours. Hum Genet 93: 53–58, 1994
7. Maher ER, Webster AR, Richards FM, Green JS, Crossey PA, Payne SJ,
Moore AT. Phenotypic expression in von Hippel-Lindau disease: correlations with germline VHL gene mutations. J Med Genet 33: 328–332,
1996
American Society of Nephrology
8. Ong KR, Woodward ER, Killick P, Lim C, Macdonald F, Maher ER. Genotype-phenotype correlations in von Hippel-Lindau disease. Hum Mutat 28:
143–149, 2007
9. Brauch H, Kishida T, Glavac D, Chen F, Pausch F, Höfler H, Latif F,
Lerman MI, Zbar B, Neumann HP. Von Hippel-Lindau (VHL) disease with
pheochromocytoma in the Black Forest region of Germany: Evidence
for a founder effect. Hum Genet 95: 551–556, 1995
10. Schmid S, Gillessen S, Binet I, Brändle M, Engeler D, Greiner J, Hader C,
Heinimann K, Kloos P, Krek W, Krull I, Stoeckli SJ, Sulz MC, van Leyen K,
Weber J, Rothermundt C, Hundsberger T. Management of von hippellindau disease: an interdisciplinary review. Oncol Res Treat 37: 761–
771, 2014
11. Tickoo SK, dePeralta-Venturina MN, Harik LR, Worcester HD, Salama
ME, Young AN, Moch H, Amin MB. Spectrum of epithelial neoplasms in
end-stage renal disease: An experience from 66 tumor-bearing kidneys
with emphasis on histologic patterns distinct from those in sporadic
adult renal neoplasia. Am J Surg Pathol 30: 141–153, 2006
12. Rao P, Monzon F, Jonasch E, Matin SF, Tamboli P. Clear cell papillary
renal cell carcinoma in patients with von Hippel-Lindau syndrome:
Clinicopathological features and comparative genomic analysis of 3
cases. Hum Pathol 45: 1966–1972, 2014
13. Walther MM, Choyke PL, Glenn G, Lyne JC, Rayford W, Venzon D,
Linehan WM. Renal cancer in families with hereditary renal cancer:
Prospective analysis of a tumor size threshold for renal parenchymal
sparing surgery. J Urol 161: 1475–1479, 1999
14. Duffey BG, Choyke PL, Glenn G, Grubb RL, Venzon D, Linehan WM,
Walther MM. The relationship between renal tumor size and metastases in patients with von Hippel-Lindau disease. J Urol 172: 63–65, 2004
15. Schmidt L, Duh FM, Chen F, Kishida T, Glenn G, Choyke P, Scherer SW,
Zhuang Z, Lubensky I, Dean M, Allikmets R, Chidambaram A,
Bergerheim UR, Feltis JT, Casadevall C, Zamarron A, Bernues M,
Richard S, Lips CJ, Walther MM, Tsui LC, Geil L, Orcutt ML, Stackhouse
T, Lipan J, Slife L, Brauch H, Decker J, Niehans G, Hughson MD, Moch
H, Storkel S, Lerman MI, Linehan WM, Zbar B. Germline and somatic
mutations in the tyrosine kinase domain of the MET proto-oncogene in
papillary renal carcinomas. Nat Genet 16: 68–73, 1997
16. Tomlinson IP, Alam NA, Rowan AJ, Barclay E, Jaeger EE, Kelsell D,
Leigh I, Gorman P, Lamlum H, Rahman S, Roylance RR, Olpin S, Bevan
S, Barker K, Hearle N, Houlston RS, Kiuru M, Lehtonen R, Karhu A, Vilkki
S, Laiho P, Eklund C, Vierimaa O, Aittomäki K, Hietala M, Sistonen P,
Paetau A, Salovaara R, Herva R, Launonen V, Aaltonen LA; Multiple
Leiomyoma Consortium. Germline mutations in FH predispose to
dominantly inherited uterine fibroids, skin leiomyomata and papillary
renal cell cancer. Nat Genet 30: 406–410, 2002
17. Menko FH, Maher ER, Schmidt LS, Middelton LA, Aittomäki K,
Tomlinson I, Richard S, Linehan WM. Hereditary leiomyomatosis and
renal cell cancer (HLRCC): Renal cancer risk, surveillance and treatment.
Fam Cancer 13: 637–644, 2014
18. Merino MJ, Torres-Cabala C, Pinto P, Linehan WM. The morphologic
spectrum of kidney tumors in hereditary leiomyomatosis and renal cell
carcinoma (HLRCC) syndrome. Am J Surg Pathol 31: 1578–1585, 2007
19. Bardella C, El-Bahrawy M, Frizzell N, Adam J, Ternette N, Hatipoglu E,
Howarth K, O’Flaherty L, Roberts I, Turner G, Taylor J, Giaslakiotis K,
Macaulay VM, Harris AL, Chandra A, Lehtonen HJ, Launonen V,
Aaltonen LA, Pugh CW, Mihai R, Trudgian D, Kessler B, Baynes JW,
Ratcliffe PJ, Tomlinson IP, Pollard PJ. Aberrant succination of proteins
in fumarate hydratase-deficient mice and HLRCC patients is a robust
biomarker of mutation status. J Pathol 225: 4–11, 2011
20. Chen YB, Brannon AR, Toubaji A, Dudas ME, Won HH, Al-Ahmadie HA,
Fine SW, Gopalan A, Frizzell N, Voss MH, Russo P, Berger MF, Tickoo
SK, Reuter VE. Hereditary leiomyomatosis and renal cell carcinoma
syndrome-associated renal cancer: Recognition of the syndrome by
pathologic features and the utility of detecting aberrant succination by
immunohistochemistry. Am J Surg Pathol 38: 627–637, 2014
Onco-Nephrology Curriculum
5
21. Nickerson ML, Warren MB, Toro JR, Matrosova V, Glenn G, Turner ML,
Duray P, Merino M, Choyke P, Pavlovich CP, Sharma N, Walther M,
Munroe D, Hill R, Maher E, Greenberg C, Lerman MI, Linehan WM, Zbar
B, Schmidt LS. Mutations in a novel gene lead to kidney tumors, lung
wall defects, and benign tumors of the hair follicle in patients with the
Birt-Hogg-Dubé syndrome. Cancer Cell 2: 157–164, 2002
22. Pavlovich CP, Grubb RL 3rd, Hurley K, Glenn GM, Toro J, Schmidt LS,
Torres-Cabala C, Merino MJ, Zbar B, Choyke P, Walther MM, Linehan
WM. Evaluation and management of renal tumors in the Birt-HoggDubé syndrome. J Urol 173: 1482–1486, 2005
23. Benusiglio PR, Giraud S, Deveaux S, Méjean A, Correas JM, Joly D,
Timsit MO, Ferlicot S, Verkarre V, Abadie C, Chauveau D, Leroux D,
Avril MF, Cordier JF, Richard S; French National Cancer Institute Inherited Predisposition to Kidney Cancer Network. Renal cell tumour
characteristics in patients with the Birt-Hogg-Dubé cancer susceptibility syndrome: A retrospective, multicentre study. Orphanet J Rare
Dis 9: 163, 2014
24. Houweling AC, Gijezen LM, Jonker MA, van Doorn MB, Oldenburg RA,
van Spaendonck-Zwarts KY, Leter EM, van Os TA, van Grieken NC,
Jaspars EH, de Jong MM, Bongers EM, Johannesma PC, Postmus PE,
van Moorselaar RJ, van Waesberghe JH, Starink TM, van Steensel MA,
Gille JJ, Menko FH. Renal cancer and pneumothorax risk in Birt-HoggDubé syndrome; an analysis of 115 FLCN mutation carriers from 35
BHD families. Br J Cancer 105: 1912–1919, 2011
25. Farley MN, Schmidt LS, Mester JL, Peña-Llopis S, Pavia-Jimenez A,
Christie A, Vocke CD, Ricketts CJ, Peterson J, Middelton L, Kinch L,
Grishin N, Merino MJ, Metwalli AR, Xing C, Xie XJ, Dahia PL, Eng C,
Linehan WM, Brugarolas J. A novel germline mutation in BAP1 predisposes to familial clear-cell renal cell carcinoma. Mol Cancer Res 11:
1061–1071, 2013
26. Popova T, Hebert L, Jacquemin V, Gad S, Caux-Moncoutier V, Duboisd’Enghien C, Richaudeau B, Renaudin X, Sellers J, Nicolas A, SastreGarau X, Desjardins L, Gyapay G, Raynal V, Sinilnikova OM, Andrieu N,
Manié E, de Pauw A, Gesta P, Bonadona V, Maugard CM, Penet C, Avril
MF, Barillot E, Cabaret O, Delattre O, Richard S, Caron O, Benfodda M,
Hu HH, Soufir N, Bressac-de Paillerets B, Stoppa-Lyonnet D, Stern MH.
Germline BAP1 mutations predispose to renal cell carcinomas. Am J
Hum Genet 92: 974–980, 2013
27. Woodward ER, Skytte AB, Cruger DG, Maher ER. Population-based
survey of cancer risks in chromosome 3 translocation carriers. Genes
Chromosomes Cancer 49: 52–58, 2010
28. Cancer Genome Atlas Research Network. Comprehensive molecular
characterization of clear cell renal cell carcinoma. Nature 499: 43–49,
2013
29. Mork M, Hubosky SG, Rouprêt M, Margulis V, Raman J, Lotan Y, O’Brien
T, You N, Shariat SF, Matin SF. Lynch syndrome: A primer for urologists
and panel recommendations. J Urol 194: 21–29, 2015
30. Sijmons RH, Kiemeney LA, Witjes JA, Vasen HF. Urinary tract cancer
and hereditary nonpolyposis colorectal cancer: Risks and screening
options. J Urol 160: 466–470, 1998
6
Onco-Nephrology Curriculum
31. Win AK, Lindor NM, Winship I, Tucker KM, Buchanan DD, Young JP,
Rosty C, Leggett B, Giles GG, Goldblatt J, Macrae FA, Parry S, Kalady
MF, Baron JA, Ahnen DJ, Marchand LL, Gallinger S, Haile RW,
Newcomb PA, Hopper JL, Jenkins MA. Risks of colorectal and other
cancers after endometrial cancer for women with Lynch syndrome. J
Natl Cancer Inst 105: 274–279, 2013
32. Barrow PJ, Ingham S, O’Hara C, Green K, McIntyre I, Lalloo F, Hill J,
Evans DG. The spectrum of urological malignancy in Lynch syndrome.
Fam Cancer 12: 57–63, 2013
33. Shuch B, Ricketts CJ, Vocke CD, Komiya T, Middelton LA, Kauffman EC,
Merino MJ, Metwalli AR, Dennis P, Linehan WM. Germline PTEN mutation Cowden syndrome: An underappreciated form of hereditary
kidney cancer. J Urol 190: 1990–1998, 2013
34. Mester JL, Zhou M, Prescott N, Eng C. Papillary renal cell carcinoma is
associated with PTEN hamartoma tumor syndrome. Urology 79: 1187.
e1–1187.e7, 2012
35. Evenepoel L, Papathomas TG, Krol N, Korpershoek E, de Krijger RR,
Persu A, Dinjens WN. Toward an improved definition of the genetic and
tumor spectrum associated with SDH germ-line mutations. Genet Med
17: 610–620, 2015
36. Ricketts CJ, Shuch B, Vocke CD, Metwalli AR, Bratslavsky G,
Middelton L, Yang Y, Wei MH, Pautler SE, Peterson J, Stolle CA,
Zbar B, Merino MJ, Schmidt LS, Pinto PA, Srinivasan R, Pacak K,
Linehan WM. Succinate dehydrogenase kidney cancer: An aggressive example of the Warburg effect in cancer. J Urol 188:
2063–2071, 2012
37. Gill AJ, Hes O, Papathomas T, Sedivcová
M, Tan PH, Agaimy A,
Andresen PA, Kedziora A, Clarkson A, Toon CW, Sioson L, Watson N,
Chou A, Paik J, Clifton-Bligh RJ, Robinson BG, Benn DE, Hills K,
Maclean F, Niemeijer ND, Vlatkovic L, Hartmann A, Corssmit EP, van
Leenders GJ, Przybycin C, McKenney JK, Magi-Galluzzi C, Yilmaz A, Yu
D, Nicoll KD, Yong JL, Sibony M, Yakirevich E, Fleming S, Chow CW,
Miettinen M, Michal M, Trpkov K. Succinate dehydrogenase (SDH)deficient renal carcinoma: A morphologically distinct entity: a clinicopathologic series of 36 tumors from 27 patients. Am J Surg Pathol 38:
1588–1602, 2014
38. Crino PB, Nathanson KL, Henske EP. The tuberous sclerosis complex. N
Engl J Med 355: 1345–1356, 2006
39. Dixon BP, Hulbert JC, Bissler JJ. Tuberous sclerosis complex renal
disease. Nephron, Exp Nephrol 118: e15–e20, 2011
40. Guo J, Tretiakova MS, Troxell ML, Osunkoya AO, Fadare O, Sangoi AR,
Shen SS, Lopez-Beltran A, Mehra R, Heider A, Higgins JP, Harik LR,
Leroy X, Gill AJ, Trpkov K, Campbell SC, Przybycin C, Magi-Galluzzi C,
McKenney JK. Tuberous sclerosis-associated renal cell carcinoma: A
clinicopathologic study of 57 separate carcinomas in 18 patients. Am J
Surg Pathol 38: 1457–1467, 2014
41. Tyburczy ME, Jozwiak S, Malinowska IA, Chekaluk Y, Pugh TJ, Wu CL,
Nussbaum RL, Seepo S, Dzik T, Kotulska K, Kwiatkowski DJ. A shower of
second hit events as the cause of multifocal renal cell carcinoma in
tuberous sclerosis complex. Hum Mol Genet 24: 1836–1842, 2015
American Society of Nephrology
REVIEW QUESTIONS
1. What is the exception to the “3-cm rule” of tumor removal
in hereditary renal cancer syndromes?
a. von Hippel Lindau disease
b. Birt-Hogg-Dubé syndrome
c. Hereditary leiomyomatosis and renal cell cancer
Answer: c is correct. Hereditary leiomyomatosis and renal
cell cancer (HLRCC or Reed’s syndrome) is associated with a
very aggressive form of renal cancer, and tumors should be
removed as soon as they are detected.
2. A patient with renal cancer tells you that a sister had a
pheochromocytoma. What further evaluation might that
prompt?
a. Review of pathology of the renal cancer and potentially
additional immunohistochemistry
b. Further family history collection and potentially genetics
evaluation
c. Nothing, probably unrelated
d. a and b
Answer: d is correct. Renal cancer is associated with hereditary pheochromocytoma and paraganglioma syndromes caused by SDHx mutations. These renal cancers
have a characteristic pathology, which is well described.
The renal cancers associated with SDHB mutations also
American Society of Nephrology
have loss of staining on SDHB immunohistochemistry. Collection of additional history also could be useful, in that
further history of pheochromocytomas and paragangliomas in
the family supports the diagnosis of an SDHx-related tumor.
However, it is important to note that genetic testing is recommended for all patients with pheochromocytoma, so the family
should be referred to a cancer geneticist independent of family
history.
3. One of your patients has a hybrid chromophobe/oncocytic
renal tumor, but is aged 65 and has no family history of
renal cancer or other cancers. You vaguely remember
reading this chapter, and remember that tumor type is a red
flag,and deserves further evaluation, but can’t remember
for what?
a. BAP-1–associated renal cancer
b. Birt-Hogg-Dubé syndrome
c. Lynch syndrome
Answer: b is correct. The three major manifestations of
Birt-Hogg-Dubé (BHD) are fibrofolliculomas (facial), lung
cysts and pneumothorax, and renal cancer. A hybrid chromophobe/oncocytic renal tumor is considered characteristic of
BHD, and the finding on pathology should prompt further
evaluation. All three manifestations of BHD are incompletely
penetrant, particularly renal cancer. Thus, the lack of family
history of renal cancer and older age of diagnosis should not
preclude evaluation for BHD.
Onco-Nephrology Curriculum
7
Chapter 15: Workup and Management of “Small”
Renal Masses
Susie L. Hu, MD,* and Anthony Chang, MD†
*Division of Kidney Disease and Hypertension, Warren Alpert Medical School of Brown University, Rhode Island
Hospital, Providence, Rhode Island; and †Department of Pathology, The University of Chicago Medicine,
Chicago, Illinois
INTRODUCTION
Nephrologists are frequently asked by urology and
oncology colleagues to participate in the management of patients diagnosed with a renal mass. This is
especially the case when there is associated CKD,
hypertension, and/or other medically challenging
comorbidities. Renal masses are classified as large
and small. Small renal masses, defined as T1a (#4
cm) with no metastases or contralateral kidney involvement, have a 5-year survival rate approaching
100% in most studies. Therefore, as the vast majority of these patients are cured of their kidney cancer,
the maintenance of renal function is becoming the
major determinant of clinical outcomes. In this era
of almost indiscriminate use of diagnostic imaging,
.50% of renal masses are incidentally discovered
(1,2) and 16%–23% of these are benign (3,4). Given
the rise of incidental tumors, nephron-sparing procedures (partial nephrectomy, cryoablation, radiofrequency ablation, or thermal ablation) are increasingly
replacing traditional radical nephrectomy (RN),
and even diagnostic renal biopsies are a viable option
as the concept of tumor seeding along the needle
track has been largely unfounded (5).
Cancer-specific survival and overall survival
between radical nephrectomy and nephron-sparing
surgery are comparable (6). For poor or nonsurgical
candidates, greater consideration is being given to
ablative therapies or even active surveillance, given
that small renal masses grow very slowly at an average rate of 1.3 mm/yr (7). This population (with
small renal masses) who remain renal cancer free,
are dying of other causes, most frequently due to
cardiovascular events (8). With improving survival,
morbidity related to CKD from nephron mass loss,
as well comorbid disease–induced complications,
has become more relevant and ultimately impacts
survival (with increased risk for cardiovascular
American Society of Nephrology
death) (9). Consideration of preoperative kidney
function, comorbidities (10), nephron-sparing surgical methods, and tumor size (11) should be made
when determining the management plan for patients with small renal masses.
EPIDEMIOLOGY
Individuals with small renal tumors are older (average
age of 60 years), predominantly white men (12–14)
with notable comorbidity (13,14). In the Medicarelinked US Surveillance Epidemiology and End Results (SEER) database, .10,000 individuals with
small tumors #7 cm surveyed had an average tumor
size of 4 cm, had a higher median age of 74, were
predominantly white, 8.7% were African American,
and 65% were male. In this population with high
burden of comorbid diseases, almost half had diabetes mellitus (DM) or chronic obstructive pulmonary disease, a third had cerebrovascular disease,
and approximately 15% had peripheral vascular disease or preexisting CKD (14). The study population
of the sole prospective randomized controlled multicenter trial, European Organization for Research
and Treatment of Cancer (EORTC) trial, which included small renal masses (#5cm), had substantial
comorbid disease (36%), which was largely cardiovascular disease (8).
Overlapping risk factors between renal cell carcinoma and CKD may account for the high prevalence
for CKD and cardiovascular disease in this population (Figure 1). Risk factors for renal cell carcinoma
in the general population have traditionally
Correspondence: Susie L. Hu, Rhode Island Hospital, 593 Eddy
St., Providence, Rhode Island 02903. Email: [email protected]
Copyright © 2016 by the American Society of Nephrology
Onco-Nephrology Curriculum
1
Figure 1. Overlapping risk factors for renal cell carcinoma and
CKD in the general population and among those with small
renal masses. DM, diabetes mellitus; HLD, hyperlipidemia; HTN,
hypertension; Neph, nephrectomy.
included tobacco exposure, obesity, DM, and hypertension
(HTN) (15). Cystic disease and ESRD patients have a greater
predisposition to renal tumors (16). This risk is 100-fold for
ESRD patients (16) and a lesser but significant risk has been
identified for stage 3 and 4 CKD (17). Therefore, the risk factors for CKD seem to also predispose one to the development
of renal cell carcinoma.
With extension of survival, quality-of-life issues shaped by
postoperative chronic disease burden, primarily CKD, and
cardiovascular disease have increasingly become an important
factor in the care of these patients.
Preoperative CKD and comorbidity
Prevalence of CKD seems to vary, considerably ranging from
10% to 52% (18–21) for those with small renal masses, similar
to that of all-comers with all sizes of renal masses (11%–32%)
depending on the cutoff GFR and age of the general population (22–24). When examining older subgroups, CKD prevalence nearly doubled (19,23), consistent with the finding that
increasing age raises CKD risk. Of known CKD risk factors,
the most important two, DM and HTN, were also highly prevalent in this population, where 9%–22% had diabetes and
23%–59% were hypertensive (18,20,24–26). The extent of
DM and HTN was greater not only among those with a diagnosis of preexisting CKD, but also among those with renal
cell carcinoma. Not surprisingly, preexisting CKD patients
had more DM (26%) and HTN (60%) in a Korean cohort
with small renal masses (n 5 1,928) than those without
CKD (DM, 12.7%; HTN, 32%) (19). In a case-matched Taiwanese cohort of 26,460 patients, those diagnosed with renal cell
carcinoma had greater burden of DM (19.6%) and HTN
(30.6%) compared with case-matched controls (DM, 7.7%;
HTN, 14%) without renal cell carcinoma (10).
CKD risk factors
Predictive factors for new-onset CKD or progression of CKD
after therapeutic intervention include older age, male sex (6),
tobacco use (27), obesity (27,28), and concomitant DM or
2
Onco-Nephrology Curriculum
HTN, which are reflective of the predominant features defining
the renal mass cohort, and also include lower baseline estimated
GFR and larger tumor size (10,11,20). Hypoalbuminemia (19)
and postoperative AKI (29) are other likely determinants of GFR
decline (Table 1). With tumor resection, CKD prevalence increased anywhere from 10%–24% to 16%–52% after treatment
(18–20). Others reported a mean GFR decrease of 13 mL/min
per 1.73 m2, corresponding to a 30% drop in GFR after partial
nephrectomy (PN), and renal volume reduction seemed to be a
prognostic factor for GFR decline (30). Furthermore, among
diabetics, 60% developed CKD compared with only 43% of
the entire cohort; the 2-year probability of absence of CKD
was poor among patients with diabetes (47%) in contrast to
those without DM (76%, P 5 0.006) (20). Incidence of ESRD
was 5.6 times greater among renal cell carcinoma patients
(4.05%) than for a comparable control group (0.68%) (10). In
the US Renal Data System (USRDS), renal cell carcinoma has
been reported as a cause for ESRD in 0.5% of the 360,000 patients, with a higher mortality compared with other causes of
ESRD (28).
MANAGEMENT
Preoperative evaluation
Historically, small renal mass identification by imaging studies
nearly always led to surgical intervention with the possible
exception of an angiomyolipoma, which can be suspected when
there is a significant component of adipose tissue. However, active
surveillance and percutaneous kidney biopsies are viable options
that are increasingly utilized, as up to 23% of patients will have
benign small renal masses (oncocytoma or angiomyolipoma)
and can be spared any additional surgery (3,4). The diagnostic
rate of kidney biopsies approaches 80% in experienced centers
(31), and the concordance rate approaches 100% compared with
the surgical resection specimen (32).
Preoperative evaluation of potentially modifiable risk
factors including DM, HTN, and CKD may play a role in
the preservation of renal function. Optimizing glycemic and BP
control, as well as estimation of GFR and prevention of AKI,
may minimize risk for deterioration of GFR postoperatively.
Prevention of AKI can be achieved through proper medical
management to avoid nephrotoxic exposure and renal
Table 1. Risk factors for CKD
Older age (.65 years)
Male sex
Tobacco use
Comorbid diseases
Diabetes mellitus
Hypertension
Obesity
Lower baseline GFR
Larger tumor size
Surgical procedure: radical nephrectomy
Postoperative AKI
American Society of Nephrology
hypoperfusion (33). Renal nuclear scintigraphy has been used
to help determine proportional GFR of each kidney to better
assess the potential impact of renal resection (partial or radical
nephrectomy). Preoperative proportional GFR assessment
used to calculate the expected postoperative GFR tended to
underestimate the actual postoperative GFR by 12% in one
study (34), presumably due to compensatory hyperfiltration
and hypertrophy. Additionally, one study examining pre- and
postoperative differential function with nuclear scintigraphy
found that postsurgical differential function inversely correlated best with ischemia time and tumor size, which may be
more predictive of intraoperative renal damage (35). Evaluation of
differential function remains a useful tool in assessing operative
risk for CKD, providing we recognize these limitations of total
GFR underestimation (due to hyperfiltration by the preserved
kidney) and relative GFR overestimation in the surgical kidney.
Treatment of small renal masses and outcomes
RN has been the mainstay of therapy for generations and
remains an essential therapy for those with larger renal masses
or with lesions extending beyond the affected kidney. Small
renal masses have favorable prognosis and survival, which
do not necessitate radical nephrectomy. Partial nephrectomy
has equivalent/comparable oncologic and overall survival and
greater renal preservation (6,36,37).
Innumerable studies have emerged examining these outcomes over the last couple of decades with similar findings and
may be best illustrated in a meta-analysis performed by Kim
et al. Risk reduction with nephron-sparing surgery assessed
from 36 studies (40,000 patients; 31,000 RN and 9,300 PN)
was 19% for all-cause mortality, 29% for cancer specific mortality, and 61% for CKD (37). On the contrary, the EORTC
study, which was a clinical trial of 541 patients with solitary
unilateral small renal masses (#5 cm), found that overall
10-year survival was slightly higher for RN (81.1%) than
nephron-sparing surgery (75.7%), with a hazard ratio (HR)
of 1.5 (95% CI, 1.03–2.16). The small difference in mortality
was no longer present when only patients diagnosed with
renal cell carcinoma were considered. Progression of disease
and renal cell carcinoma death were no different between the
two therapies (8).
In the same EORTC study, risk of CKD was retrospectively
examined and demonstrated that partial nephrectomy (PN)
preserved GFR (baseline creatinine , 1.25 times the upper
normal range), particularly early on where the difference of
patients reaching GFR ,60 mL/min per 1.73 m2 between RN
(85.7%) and PN (64.7%) was 21%. However, over time, this
difference of progressive decline to lower GFRs ,30 (RN 10%,
PN 6.3%) and ,15 mL/min per 1.73 m2 (RN 1.5%, PN 1.6%)
became insignificant. The steeper GFR decline observed initially with RN was not associated with an increase in mortality
(36), potentially suggesting that GFR loss from nephrectomy
did not confer the same risk of death usually seen with GFR
decline from traditional causes of CKD such as DM or HTN
(9). The cohort studies, however, generally showed a survival
American Society of Nephrology
advantage in addition to GFR preservation with nephron-sparing
surgery (37), which is clearly limited by its retrospective design.
The increased mortality seen with CKD has been attributed
to high cardiovascular disease risk typically associated with
advanced GFR (9). Among 1,004 case-matched patients with
relatively small renal mass size (T1b, 4–7 cm), an association
of greater cardiovascular death and GFR decline was observed.
GFR loss was measured using the difference between extrapolated values of preoperative and (.3 weeks) postoperative
GFRs. The average GFR loss was less for those who had partial
nephrectomy (16.6 mL/min) as opposed to radical nephrectomy (23.5 mL/min, P ,0.0001). GFR decline generally occurred within 3 weeks and then stabilized thereafter. Each
excess loss of GFR of 7 mL/min per 1.73 m2 resulted in a
17% increase risk of death as well as a 25% greater cardiovascular disease risk. Cancer-specific survival was not different,
but overall survival was better for partial nephrectomy (85%
versus RN 78%, P 5 0.01) (38).
The hard end point of ESRD has also been examined in a
Taiwanese incident cohort with 10-year follow-up in a newly
diagnosed renal cell carcinoma group (n 5 2940) and a control
group (n 5 23,520). Progression to ESRD occurred in 4.05%
of the renal cell carcinoma group compared with only 0.68%
of the control group (HR, 5.63; 95% CI, 4.37–7.24) with the
same risk factors (DM, preexisting CKD) for CKD progression
(10). In the USRDS, the higher mortality noted with ESRD
from renal cell carcinoma compared with other causes was not
observed for those who had undergone nephrectomy, suggesting that even though progression to ESRD from renal
cell carcinoma could not be prevented, nephrectomy may still
confer a lower risk for mortality (28).
Alternative therapy for small renal masses in individuals
with high operative risk includes nonsurgical ablative therapies
such as radiofrequency ablation and cryoablation, which
are currently the two most common approaches used. Older
patients were more likely to receive radiofrequency ablation
(RFA) with fewer major complications (RFA 3.1 % versus PN
7.2%–7.9%, P ,0.001), but with higher local progression of
disease (RFA 4.6% versus PN 1.2%–1.9%, P ,0.001) than
seen with partial nephrectomy (39). In one series, however,
oncologic outcomes were no different when excluding those
with high risk for recurrence (40). Cryoablation was also utilized more frequently for older patients with higher operative
risk. In addition to fewer procedural complications, shorter
length of hospital stay was noted; however, this was also associated with higher local (relative risk [RR], 9.39; P ,0.0001)
and metastatic progression of disease (RR, 4.68; P 5 0.01)
after cryoablation compared with partial nephrectomy (both
performed laparoscopically) (41). Last, active surveillance
with judicious monitoring of tumor size and sometimes
with delayed surgical intervention resulted in acceptable outcomes particularly among those age .75, which was no worse
than surgical resection for select populations (14). Although
oncologic outcomes for ablative therapies may not be as favorable as surgical resection, they provide viable therapeutic options
Onco-Nephrology Curriculum
3
with less complications and likely greater renal parenchymal
(thus also GFR) preservation for nonoperative candidates.
c Older age, male sex, comorbid diseases including diabetes mellitus,
hypertension, preexisting CKD, and larger tumor size increase risk for
postoperative CKD.
c Nephron-sparing surgery (partial nephrectomy) among patients with
PATHOLOGIC EVALUATION OF TUMOR
NEPHRECTOMY
The evaluation of tumor nephrectomy specimens has always
centered around the renal mass, but careful assessment of the
nonneoplastic kidney parenchyma reveals the presence of
common yet undiagnosed nonneoplastic renal diseases.
Therefore, the synoptic reports established by the College of
American Pathologist required in 2010 that the nonneoplastic
parenchyma should be evaluated and reported for every renal
malignancy (42). Also, the Accreditation Council for Graduate
Medical Education will require that nephropathology be part
of the curriculum for all anatomic pathology residents effective July 1, 2015, as the vast majority of pathologists do not
receive any exposure to this subspecialty. Several large studies
found that diabetic nephropathy and hypertensive nephropathy (or arterionephrosclerosis) can be identified in 8%–20%
and 3%–14% of specimens, respectively (43–45), and 60%–
88% of these diagnoses were not identified during the initial
evaluation. With nearly 350,000 kidney cancer survivors in the
United States, the CKD burden will only increase, especially as
more attention is given to the nonneoplastic parenchyma examined by pathologists. In addition, there still remains much
room for improvement regarding the coordination of urologists,
pathologists, and especially nephrologists in the preoperative
and postoperative management of kidney cancer patients.
CONCLUSIONS
The majority of renal tumors are small renal tumors discovered
on routine imaging with excellent oncologic and overall survival. The prolonged survival with earlier discovery has
resulted in higher likelihood of nononcologic death, where
patients are saddled instead with CKD and associated increased
cardiovascular morbidity and mortality. Minimally invasive
techniques for diagnosis and nephron-sparing surgery have
minimized nephron mass and functional loss. Recognizing and
assessing modifiable risk factors for CKD such as HTN, DM,
and cardiovascular disease may potentially allow for greater
preservation of GFR and reduction of cardiovascular disease–
related death. To achieve this goal, communication and coordination of management is essential within the specialty
care team, which is comprised of nephrologists, oncologists,
urologists, and pathologists.
TAKE HOME POINTS
c Early diagnosis of small renal tumors is rising due to incidental discovery
with favorable prognosis.
c New-onset CKD is fairly common after nephrectomy with overlapping
risk factors for CKD and renal cell carcinoma.
4
Onco-Nephrology Curriculum
small renal masses has equivalent cancer-specific survival, practically
similar overall survival, and better GFR preservation compared with
radical nephrectomy.
REFERENCES
1. Patard JJ, Shvarts O, Lam JS, Pantuck AJ, Kim HL, Ficarra V, Cindolo L,
Han KR, De La Taille A, Tostain J, Artibani W, Abbou CC, Lobel B,
Chopin DK, Figlin RA, Mulders PF, Belldegrun AS. Safety and efficacy of
partial nephrectomy for all T1 tumors based on an international multicenter experience. J Urol 171: 2181–2185, quiz 2435, 2004
2. Chow WH, Devesa SS, Warren JL, Fraumeni JF Jr. Rising incidence of
renal cell cancer in the United States. JAMA 281: 1628–1631, 1999
3. McKiernan J, Yossepowitch O, Kattan MW, Simmons R, Motzer RJ,
Reuter VE, Russo P. Partial nephrectomy for renal cortical tumors: pathologic findings and impact on outcome. Urology 60: 1003–1009, 2002
4. Kutikov A, Fossett LK, Ramchandani P, Tomaszewski JE, Siegelman ES,
Banner MP, Van Arsdalen KN, Wein AJ, Malkowicz SB. Incidence of
benign pathologic findings at partial nephrectomy for solitary renal
mass presumed to be renal cell carcinoma on preoperative imaging.
Urology 68: 737–740, 2006
5. Volpe A, Kachura JR, Geddie WR, Evans AJ, Gharajeh A, Saravanan A,
Jewett MA. Techniques, safety and accuracy of sampling of renal tumors by fine needle aspiration and core biopsy. J Urol 178: 379–386,
2007
6. Li L, Lau WL, Rhee CM, Harley K, Kovesdy CP, Sim JJ, Jacobsen S,
Chang A, Landman J, Kalantar-Zadeh K. Risk of chronic kidney disease
after cancer nephrectomy. Nat Rev Nephrol 10: 135–145, 2014
7. Jewett MAS, Mattar K, Basiuk J, Morash CG, Pautler SE, Siemens DR,
Tanguay S, Rendon RA, Gleave ME, Drachenberg DE, Chow R, Chung H,
Chin JL, Fleshner NE, Evans AJ, Gallie BL, Haider MA, Kachura JR,
Kurban G, Fernandes K, Finelli A. Active surveillance of small renal
masses: progression patterns of early stage kidney cancer. Eur Urol 60:
39–44, 2011
8. Van Poppel H, Da Pozzo L, Albrecht W, Matveev V, Bono A, Borkowski
A, Colombel M, Klotz L, Skinner E, Keane T, Marreaud S, Collette S,
Sylvester R. A prospective, randomised EORTC intergroup phase 3
study comparing the oncologic outcome of elective nephron-sparing
surgery and radical nephrectomy for low-stage renal cell carcinoma. Eur
Urol 59: 543–552, 2011
9. Go AS, Chertow GM, Fan D, McCulloch CE, Hsu CY. Chronic kidney
disease and the risks of death, cardiovascular events, and hospitalization. N Engl J Med 351: 1296–1305, 2004
10. Hung PH, Tsai HB, Hung KY, Muo CH, Chung MC, Chang CH, Chung
CJ. Increased risk of end-stage renal disease in patients with renal cell
carcinoma: A 12-year nationwide follow-up study. Medicine (Baltimore)
93: e52, 2014
11. Jeon HG, Choo SH, Sung HH, Jeong BC, Seo SI, Jeon SS, Choi HY, Lee
HM. Small tumour size is associated with new-onset chronic kidney
disease after radical nephrectomy in patients with renal cell carcinoma.
Eur J Cancer 50: 64–69, 2014
12. Crépel M, Jeldres C, Sun M, Lughezzani G, Isbarn H, Alasker A,
Capitanio U, Shariat SF, Arjane P, Widmer H, Graefen M, Montorsi F,
Perrotte P, Karakiewicz PI. A population-based comparison of cancercontrol rates between radical and partial nephrectomy for T1A renal
cell carcinoma. Urology 76: 883–888, 2010
13. Lane BR, Campbell SC, Gill IS. 10-year oncologic outcomes after laparoscopic and open partial nephrectomy. J Urol 190: 44–49, 2013
14. Sun M, Becker A, Tian Z, Roghmann F, Abdollah F, Larouche A,
Karakiewicz PI, Trinh QD. Management of localized kidney cancer:
American Society of Nephrology
15.
16.
17.
18.
19.
20.
21.
22.
23.
24.
25.
26.
27.
28.
29.
30.
Calculating cancer-specific mortality and competing risks of death for
surgery and nonsurgical management. Eur Urol 65: 235–241, 2014
Chow W-H, Dong LM, Devesa SS. Epidemiology and risk factors for
kidney cancer. Nat Rev Urol 7: 245–257, 2010
Denton MD, Magee CC, Ovuworie C, Mauiyyedi S, Pascual M, Colvin
RB, Cosimi AB, Tolkoff-Rubin N. Prevalence of renal cell carcinoma in
patients with ESRD pre-transplantation: A pathologic analysis. Kidney
Int 61: 2201–2209, 2002
Lowrance WT, Ordoñez J, Udaltsova N, Russo P, Go AS. CKD and the
risk of incident cancer. J Am Soc Nephrol 25: 2327–2334, 2014
Barlow LJ, Korets R, Laudano M, Benson M, McKiernan J. Predicting
renal functional outcomes after surgery for renal cortical tumours: A
multifactorial analysis. BJU Int 106: 489–492, 2010
Kim SH, Lee SE, Hong SK, Jeong CW, Park YH, Kim YJ, Kang SH, Hong
SH, Choi WS, Byun SS. Incidence and risk factors of chronic kidney
disease in Korean patients with t1a renal cell carcinoma before and after
radical or partial nephrectomy. Jpn J Clin Oncol 43: 1243–1248, 2013
Jeon HG, Jeong IG, Lee JW, Lee SE, Lee E. Prognostic factors for
chronic kidney disease after curative surgery in patients with small renal
tumors. Urology 74: 1064–1068, 2009
Huang WC, Levey AS, Serio AM, Snyder M, Vickers AJ, Raj GV, Scardino
PT, Russo P. Chronic kidney disease after nephrectomy in patients with
renal cortical tumours: A retrospective cohort study. Lancet Oncol 7:
735–740, 2006
Kaushik D, Kim SP, Childs MA, Lohse CM, Costello BA, Cheville JC,
Boorjian SA, Leibovich BC, Thompson RH. Overall survival and development of stage IV chronic kidney disease in patients undergoing
partial and radical nephrectomy for benign renal tumors. Eur Urol 64:
600–606, 2013
Canter D, Kutikov A, Sirohi M, Street R, Viterbo R, Chen DY, Greenberg
RE, Uzzo RG. Prevalence of baseline chronic kidney disease in patients
presenting with solid renal tumors. Urology 77: 781–785, 2011
Clark MA, Shikanov S, Raman JD, Smith B, Kaag M, Russo P, Wheat JC,
Wolf JS Jr, Matin SF, Huang WC, Shalhav AL, Eggener SE. Chronic
kidney disease before and after partial nephrectomy. J Urol 185: 43–48,
2011
Choi YS, Park YH, Kim YJ, Kang SH, Byun SS, Hong SH. Predictive
factors for the development of chronic renal insufficiency after renal
surgery: A multicenter study. Int Urol Nephrol 46: 681–686, 2014
Takagi T, Kondo T, Iizuka J, Kobayashi H, Hashimoto Y, Nakazawa H, Ito F,
Tanabe K. Postoperative renal function after partial nephrectomy for renal
cell carcinoma in patients with pre-existing chronic kidney disease: A
comparison with radical nephrectomy. Int J Urol 18: 472–476, 2011
Malcolm JB, Bagrodia A, Derweesh IH, Mehrazin R, Diblasio CJ, Wake
RW, Wan JY, Patterson AL. Comparison of rates and risk factors for
developing chronic renal insufficiency, proteinuria and metabolic acidosis after radical or partial nephrectomy. BJU Int 104: 476–481, 2009
Stiles KP, Moffatt MJ, Agodoa LY, Swanson SJ, Abbott KC. Renal cell
carcinoma as a cause of end-stage renal disease in the United States:
Patient characteristics and survival. Kidney Int 64: 247–253, 2003
Cho A, Lee JE, Kwon G-Y, Huh W, Lee HM, Kim YG, Kim DJ, Oh HY,
Choi HY. Post-operative acute kidney injury in patients with renal cell
carcinoma is a potent risk factor for new-onset chronic kidney disease
after radical nephrectomy. Nephrol Dial Transplant 26: 3496–3501,
2011
Song C, Bang JK, Park HK, Ahn H. Factors influencing renal function reduction after partial nephrectomy. J Urol 181: 48–53, discussion 53–54, 2009
American Society of Nephrology
31. Volpe A, Finelli A, Gill IS, Jewett MA, Martignoni G, Polascik TJ, Remzi
M, Uzzo RG. Rationale for percutaneous biopsy and histologic characterisation of renal tumours. Eur Urol 62: 491–504, 2012
32. Volpe A, Mattar K, Finelli A, Kachura JR, Evans AJ, Geddie WR, Jewett
MA. Contemporary results of percutaneous biopsy of 100 small renal
masses: A single center experience. J Urol 180: 2333–2337, 2008
33. Thadhani R, Pascual M, Bonventre JV. Acute renal failure. N Engl J Med
334: 1448–1460, 1996
34. Bachrach L, Negron E, Liu JS, Su YK, Paparello JJ, Eggener S, Kundu
SD. Preoperative nuclear renal scan underestimates renal function after
radical nephrectomy. Urology 84: 1402–1406, 2014
35. Sankin A, Sfakianos JP, Schiff J, Sjoberg D, Coleman JA. Assessing
renal function after partial nephrectomy using renal nuclear scintigraphy and estimated glomerular filtration rate. Urology 80: 343–346,
2012
36. Scosyrev E, Messing EM, Sylvester R, Campbell S, Van Poppel H. Renal
function after nephron-sparing surgery versus radical nephrectomy:
Results from EORTC randomized trial 30904. Eur Urol 65: 372–377,
2014
37. Kim SP, Thompson RH, Boorjian SA, Weight CJ, Han LC, Murad MH,
Shippee ND, Erwin PJ, Costello BA, Chow GK, Leibovich BC. Comparative effectiveness for survival and renal function of partial and
radical nephrectomy for localized renal tumors: A systematic review
and meta-analysis. J Urol 188: 51–57, 2012
38. Weight CJ, Larson BT, Fergany AF, Gao T, Lane BR, Campbell SC,
Kaouk JH, Klein EA, Novick AC. Nephrectomy induced chronic renal
insufficiency is associated with increased risk of cardiovascular death
and death from any cause in patients with localized cT1b renal masses.
J Urol 183: 1317–1323, 2010
39. Wang S, Qin C, Peng Z, Cao Q, Li P, Shao P, Ju X, Meng X, Lu Q, Li J,
Wang M, Zhang Z, Gu M, Zhang W, Yin C. Radiofrequency ablation
versus partial nephrectomy for the treatment of clinical stage 1 renal
masses: A systematic review and meta-analysis. Chin Med J (Engl) 127:
2497–2503, 2014
40. Olweny EO, Park SK, Tan YK, Best SL, Trimmer C, Cadeddu JA. Radiofrequency ablation versus partial nephrectomy in patients with solitary clinical T1a renal cell carcinoma: Comparable oncologic outcomes
at a minimum of 5 years of follow-up. Eur Urol 61: 1156–1161, 2012
41. Klatte T, Shariat SF, Remzi M. Systematic review and meta-analysis of
perioperative and oncologic outcomes of laparoscopic cryoablation
versus laparoscopic partial nephrectomy for the treatment of small renal tumors. J Urol 191: 1209–1217, 2014
42. Srigley JR, Amin MB, Delahunt B, Campbell SC, Chang A, Grignon DJ,
Humphrey PA, Leibovich BC, Montironi R, Renshaw AA, Reuter VE;
Members of the Cancer Committee, College of American Pathologists.
Protocol for the examination of specimens from patients with invasive
carcinoma of renal tubular origin. Arch Pathol Lab Med 134: e25–e30,
2010
43. Bijol V, Mendez GP, Hurwitz S, Rennke HG, Nosé V. Evaluation of the
nonneoplastic pathology in tumor nephrectomy specimens: predicting
the risk of progressive renal failure. Am J Surg Pathol 30: 575–584, 2006
44. Salvatore SP, Cha EK, Rosoff JS, Seshan SV. Nonneoplastic renal cortical scarring at tumor nephrectomy predicts decline in kidney function.
Arch Pathol Lab Med 137: 531–540, 2013
45. Henriksen KJ, Meehan SM, Chang A. Non-neoplastic renal diseases are
often unrecognized in adult tumor nephrectomy specimens: A review
of 246 cases. Am J Surg Pathol 31: 1703–1708, 2007
Onco-Nephrology Curriculum
5
REVIEW QUESTIONS
1. What are the common risk factors for both CKD and renal
cell carcinoma (RCC)?
a.
b.
c.
d.
Obesity
Diabetes mellitus
Hypertension
All of the above
Answer: d is correct. Diabetes, hypertension, and obesity
are independent risk factors for renal cell carcinoma. Diabetes
and hypertension can be found in 25% and up to 60% of
kidney cancer patients, respectively. These are also the two
most common causes of ESRD. Given the link between CKD
and RCC, it is not surprising to find these common overlapping risk factors. Obesity is a lesser but also significant risk
factor common to both CKD and RCC.
2. Which of the following is the most common pathologic
finding in tumor nephrectomy specimens?
a.
b.
c.
d.
e.
Diabetic nephropathy
Membranous nephropathy
IgA nephropathy
Focal segmental glomerulosclerosis
Minimal change disease
Answer: a is correct. Diabetes is an independent risk factor
for RCC and is found in up to 25% of kidney cancer patients.
Therefore, diabetic nephropathy is common and can be identified in 8%–20% of tumor nephrectomy specimens (depending on the definition of diabetic nephropathy). IgA
6
Onco-Nephrology Curriculum
nephropathy occurs in ,2% of specimens. Amyloidosis occurred in 3% of specimens according to a study in the late
1980s, but is much less common due to the significant stage
migration that has occurred toward smaller neoplasms as a
result of early detection. Membranous nephropathy and
pauci-immune crescentic glomerulonephritis rarely occur in
the setting of kidney cancer, with only case reports being available in the literature.
3. Which of the following statements is true regarding outcomes in the management of small renal masses?
a. Cancer-specific survival and overall survival is superior
with radical nephrectomy, which should be considered
primarily in this population
b. Partial nephrectomy is associated with GFR preservation
and comparable cancer-specific survival and overall survival to that of radical nephrectomy
c. Ablative therapies are associated with increased treatmentrelated complication rates but less disease progression
d. Active surveillance is not indicated for management of
small renal masses
Answer: b is correct. Cancer-specific survival and overall
survival are equivalent between partial and radical nephrectomy. GFR preservation is greater with partial nephrectomy.
For small renal masses, nephron-sparing procedure should
be considered first. Ablative therapies have worse oncologic
outcomes but lower treatment-related complications. Active surveillance in select populations such as older poor
operative candidates have been shown to have comparable
outcomes.
American Society of Nephrology
Chapter 16: Cancer in Solid Organ Transplantation
Mona D. Doshi, MD
Department of Medicine, Wayne State University, Detroit, Michigan
INTRODUCTION
Solid organ transplantation provides lifesaving therapy for patients with end-organ disease. The Scientific Registry of Transplant Recipients (SRTR) report
announced that 17,654 kidney, 6455 liver, 1946
lung, 2554 heart, and 109 intestinal transplants were
performed in 2013, superseding the number of
transplants from prior years (1). It also reports continual improvement in death-censored graft survival (2,3). Success of field of transplantation is
reflected in the rising number and longevity of the
transplant organs. However, the same report also
noted an increase in recipient death with graft function, primarily due to increased infections and cancers associated with chronic immunosuppression.
Malignancy is the second leading cause of death
with graft function (4). The risk of cancer is twoto four-fold higher in transplant recipients than
age-, sex-, and race-matched individuals from similar
geographic areas (5,6). Not only are cancers common, but they tend to be more aggressive and are
associated with increased mortality among transplant
recipients than in the general population. The relative
risk varies by age. Children have the highest increase
(15–30 times), and older individuals (i.e., .65 years
of age) experience a two-fold increase. The magnitude of increase in risk for all cancer types is similar
across organ recipients; the incidence of specific
cancer varies by transplanted organ. Knowledge
of cancer types, including the magnitude of increased risk and its clinical course, can help develop prevention and early detection protocols
and prompt management (including adjustment of
immunosuppression) to minimize cancer related
deaths.
COMMON CANCER TYPES IN
TRANSPLANT RECIPIENTS
The incidence of cancer is highest for malignancies
related to viral infections, including non-Hodgkin
American Society of Nephrology
lymphoma (NHL; Epstein-Barr virus [EBV]), Kaposi sarcoma (human herpes virus-8 [HHV8]),
liver (hepatitis C virus [HCV] and hepatitis B virus
[HBV]), and anogenital cancers (human papilloma
virus [HPV]). The increased risk of these cancers is
believed to be related to impaired immune control
of these oncogenic viruses, which can be present in
the recipient prior to transplant or transmitted at
time of transplant via the donor organ. This notion
is supported by similar amplified risks of these
cancers in individuals with compromised immune
systems, i.e., HIV/AIDS, and reversal of this heightened risk on withdrawal of immunosuppression,
i.e., kidney graft failure and reinitiating of dialysis
(7). Independent of immunosuppressive effects,
the antirejection medications such as azathioprine
and cyclosporine directly enhance the carcinogenic
effects of ultraviolet (UV) radiation via inhibiting
DNA repair and resulting in apoptosis of keratinocyte (8,9). Certain malignancies with no obvious
infectious causes are also reported to be elevated,
i.e., colon, bladder, lip, kidney, and thyroid cancer.
These cancers have been reported to occur at a
greater frequency in patients with kidney disease
prior to receiving a kidney transplant but do not
occur frequently in people with HIV/AIDS, confirming the lack of a role of the immune system. Instead,
they may be occurring due to underlying renal
or bladder disease, loss of kidney function, and/
or malignant transformation of acquired cystic kidney disease (common in individuals with renal failure) (6). Traditional risk factors such as smoking
and alcohol intake leading to organ failure continue
to play a role in development of cancer, i.e., cancer
in the native lung for those receiving a solitary lung
transplant for chronic obstructive pulmonary disease, liver cancer in alcoholics, and colon cancer in
Correspondence: Department of Medicine, Wayne State University, Harper Professional Office Building, 4160 John R, Suite
908, Detroit, Michigan 48201.
Copyright © 2016 by the American Society of Nephrology
Onco-Nephrology Curriculum
1
patients with ulcerative colitis requiring a liver transplant due
to primary sclerosing cholangitis. There are a few case reports
of malignancy transmitted via donor organ. To summarize, both
immunosuppression and host-related factors play an important
role in the increased risk of cancer in transplant recipients. Table
1 lists the names of common malignancies with their incidence
rates after transplantation.
Of note, the cancer risk of nasopharynx, cervix, prostate,
breast, brain, and chronic lymphocytic leukemia are reported
to be lower in transplant recipients than in the general population (5). Similar observations have been made in people
with HIV infection, together suggesting that the immune system may not be primarily controlling the development and
growth of these cancers (10). Alternatively, this observation
can also be explained by aggressive screening for cervical,
prostate, and breast cancer in the transplant population, leading to prompt treatment of precancerous lesions prior to or after
transplant.
CLINICAL COURSE OF COMMON CANCERS
Non-melanoma skin cancer
Non-melanoma skin cancer is the most common malignancy
in adult white solid organ transplant recipients. Squamous and
basal cell carcinoma account for .90% of all skin cancers.
Unlike the general population, squamous cell carcinoma is
the most common skin cancer. It occurs 250 times as frequently as that seen in the general population, whereas the
risk of basal cell cancer is increased by 10-fold. Thus, the ratio
of squamous cell to basal cell cancer in patients without transplant (1:4) is reversed in transplant patients (4:1). A third of
patients will have both types of skin cancer.
The risk factors for developing skin cancer are as follows: 1)
recipient related: history of skin cancer prior to transplant, the
presence of premalignant skin lesions (warts or keratosis),
exposure to solar UV rays, location of residence (highest
incidence in Australia), older age, male sex, and fair skin
phenotype; 2) immunosuppression related: duration and type
(mainly azathioprine and cyclosporine); and 3) infection
related: keratinocytes of transplant recipients are more likely
to be infected with HPV than nontransplant people. The
Table 1. Common malignancies and incidence rates after
transplant in United States
Cancer type
Incidence/10,000
person-years
SIR (95% CI)
Skin cancer
Kaposi sarcoma
PTLD
Lung
Liver
Kidney
23.7
15.5
194.0
173.4
120
97
13.85 (11.92–16.00)
61.46 (50.95–73.49)
7.54 (7.17–7.93)
1.97 (1.86–2.08)
11.56 (10.83–12.33)
4.65 (432–4.99)
Data were obtained from reference 5. SIR, standardized incidence ratio (observed/
expected cases).
2
Onco-Nephrology Curriculum
appearance and distribution of cancer depends on recipient age.
The older recipients are likely to have their first lesion within 3
years of transplant and it will typically develop on their head,
whereas in younger folks, it occurs later (typically after 8 years),
and lesions are located on dorsum of their hands. Squamous
cell cancers in transplant recipients are also more aggressive,
especially when poorly differentiated on histology. The treatment of skin cancers depends on the type of lesion and its extent.
Superficial lesions can be managed with cryotherapy while deeper
lesions require excision with clean margins. Last, changing
immunosuppression to a mammalian target of rapamycin inhibitor (mTORi)-based regimen has been shown to decrease the risk
of recurrent cancers (11).
Kaposi sarcoma
Kaposi sarcoma occurs 80–500 times more frequently in
transplant recipients than in non-immunosuppressed populations. In addition, it tends to be more aggressive and multicentric with visceral involvement. Most cases occur due to
infection with HHV-8, and therefore, are commonly seen in
recipients from a high sero-prevalence area, i.e., Mediterranean
and African regions. It is also more common in men, with a
male to female ratio of 3:1, and often occurs within the first year
of transplant. Ninety percent of them present as cutaneous
lesions on the legs or mucosal angiomatous lesions. Visceral
involvement commonly occurs in heart and liver transplant
recipients. The mainstay of treatment is reduction of immunosuppression, but this may lead to graft dysfunction (12).
Non-Hodgkin lymphoma
NHL, which is more commonly referred to as post-transplant
lymphoproliferative disorders (PTLD), occurs seven to eight
times more frequently than in the general population (standardized incidence ratio [SIR], 7.54; 95% CI, 7.17–7.93). It
occurs more commonly in young (0–34 years) and older ($50
years) male recipients. The incidence is highest in lung and
heart recipients and lowest in kidney transplant recipients,
possibly due to varying transfer of lymphoid tissue during
organ transplant and intensity of immunosuppression. Its occurrence is associated with the use of T cell–depleting agents
and a mycophenolic acid–based antirejection regimen. Data
on the risk of PTLD with use of tacrolimus are equivocal. Use
of cyclosporine and azathioprine is not associated with increased risk of lymphoma. It commonly presents within the
first year of transplant or 5 years after transplant. Early-onset
lymphoma is related to primary EBV infection, and late-onset
lymphoma is independent of infection. Its pathology ranges
from benign hyperplasia to lymphoid malignancy. PTLD differs from lymphoma in the general population not only in
histopathologic findings, but it is also associated with increased extranodal involvement, predominant occurrence in
the transplanted organ, an aggressive clinical course, and poor
outcomes. The 5-year survival was 41% and did not vary based
on time of presentation. The mortality was higher in heart
transplant recipients than in kidney transplant recipients
American Society of Nephrology
due to inability to withdraw/cease immunosuppression. In
addition to conventional therapy, the mainstay of treatment
includes reduction in immunosuppression, especially antiproliferative agents, and use of antiviral agents in those with
primary EBV infections (13,14).
Lung cancer
Risk of lung cancer in transplant recipients is moderately
increased compared with that in the general population (SIR,
6.13; 95% CI, 5.18–7.21). It is common in older male transplant
recipients. It is more common in lung transplant recipients, with
the highest risk occurring in the first 6 months of transplant
(SIR, 11.17; 95% CI, 7.48–16.04) and falling to a five-fold greater
risk thereafter. The elevated early risk of cancer may be due to
cancer in the explanted lung (15). A novel study of single-lung
versus bilateral-lung transplant recipients matched for underlying disease, smoking history, and age reported a five-fold increase in cancer among those with single-lung transplants,
where the primary cancer was noted in the lung of the recipient
(16). In addition to smoking and chronic immunosuppression,
chronic inflammation and repeated infections may be playing a
role in development of lung cancer in native lung (17). Recipients of other organs had smaller elevations in their risk, and its
occurrence increased with time.
Liver cancer
Risk of liver cancer was strongly elevated in liver transplant
recipients compared with the general population (SIR, 43.83;
95% CI, 40.90–46.91). Ninety-five percent of liver cancers were
diagnosed within the first 6 months after transplant. Like the
lung, the increased incidence of cancer within 6 months of transplant may be due to delayed recognition of cancer in the explanted liver. Thereafter, the risk of liver cancer was two-fold higher
than the general population. These late-onset liver cancers may
be due to recurrent disease related to HCV or HBV. Liver cancer
risk is not increased among other organ recipients (5).
Kidney cancer
The risk of kidney cancer was highest in kidney transplant
recipients (SIR, 6.66; 95% CI, 1.57–3.04), but was also elevated
in liver and heart recipients. Among all recipients, kidney cancer occurs mainly in older men and had a bimodal pattern of
presentation. It occurred within the first 6 months, and a second peak was seen 4–15 years after transplant. Some of the
early cases can be explained by malignant transformation of
the cysts that develop in patients with ESRD prior to transplant (6). As mentioned previously, the risk of renal cancer is
already high among patients with CKD and continues to remain high after transplant (18).
for routine age-appropriate screening, as that in general population, is recommended for all (Table 2). Annual instead of
biannual pap testing is recommended to detect precancerous
lesions that may progress faster to cancer under influence of
immunosuppression. There are no data on vaccinating transplant recipients who are HPV naïve. Annual mammograms
are also recommended for all women over age 50. The patients
should be counseled about higher incidence of false-positive
findings (calcification and increased density of breast with
chronic steroid use), resulting in increased interventions. In
addition, recipients should also be screened for colorectal cancers with yearly fecal occult blood testing and flex sigmoidoscopy or colonoscopy every 5 years. Of note, most of these
practices have not been validated in a transplant cohort (19).
In the absence of evidence, an individualized approach to
screening should be used based on the individual’s cancer
risk, existing comorbidities, overall life expectancy, and preference for screening.
Skin cancer may be prevented by using sunscreen (SPF 115)
and sun hats, avoiding sun peak hours, and covering up the
exposed skin with long sleeves. Annual follow-up with an experienced dermatologist for total body skin examination is
also advocated for those at high risk. Systemic retinoid should
be avoided, and topical retinoid treatments can be tried to
treat dysplastic lesions but with caution due to fear of increased risk of rejection. Those with repeated precancerous
skin lesions can be counseled to switch to an mTOR
inhibitor-based immunosuppressive regimen. Routine
screening for renal cancer is not recommended (19).
In addition to traditional therapy, reduction in immunosuppression is often recommended. The underlying idea is that
this allows immune reconstitution and control of the malignancy by the recipient’s recovering immune system. If
Table 2. Common cancers and recommendations for
screening
Cancer type
Breast
Colorectal
Cervical
Prostate
Liver
Skin
SURVEILLANCE AND MANAGEMENT
Kidney
In view of the higher cancer incidence and poorer prognosis,
prevention and screening play an important role. Surveillance
American Society of Nephrology
Recommendations for screening
Annual or biennial mammography for all women
older than 50 years; for women between 40 and 49
years, no evidence for or against screening
Annual fecal occult blood testing and/or 5-year
flexible sigmoidoscopy or colonoscopy for
individuals .50 years
Annual pap and pelvic examination once sexually
active
Annual digital rectal examination and PSA in all
males after age 50 years
a-Fetoprotein and liver ultrasound every 6 months in
high-risk individuals, i.e., HBV or HCV infection,
but no firm data
Monthly self-examination and total body skin
examination every 6–12 months by an expert skin
physician
No firm recommendation, but some have suggested
regular ultrasound of native kidneys
PSA, prostate-specific antigen. Adapted from reference 24.
Onco-Nephrology Curriculum
3
immunosuppression is stopped or lowered, particularly early
after transplantation, graft monitoring at short intervals is
necessary. Successful reduction or cessation of immunosuppression was reported in transplanted patients who developed
NHL and Kaposi sarcoma (20). Use of mTORi has been shown
to reduce risk of new squamous cell cancer in patients with a
prior history of skin cancer (11,21) and are also very effective
in treating Kaposi sarcoma (22). However, tolerability of
mTORi is poor and is associated with a 35% discontinuation
rate. Although there are strong data favoring the use of an
mTORi-based regimen in those with skin cancers and Kaposi
sarcoma, there are insufficient data for solid organ cancers (23).
In summary, cancer remains a leading cause of morbidity
and mortality in transplant recipients. Routine surveillance
and early detection with prompt intervention directed at
cancer and immunosuppression are recommended to improve
the life of the recipient and their transplant organ.
TAKE HOME POINTS
8.
9.
10.
11.
12.
13.
14.
c Cancer risk is increased by two- to four-fold in transplant recipients and
tends to be more aggressive than age-, sex-, and race-matched individuals from the general population.
15.
c Skin cancer is the most common cancer, followed by PTLD and cancer of
the transplanted organ.
c Emphasis should be placed on adherence to recommended cancer
16.
surveillance protocols for early detection and prompt management.
c Management of cancer developing after transplant includes reduction
of immunosuppression and switching to an mTOR inhibitor–based
regimen for those with skin cancers.
REFERENCES
1. OPTN/SRTR. 2012 annual data report. Introduction. Am J Transplant
14(Suppl 1): 8–10, 2014
2. Matas AJ, Smith JM, Skeans MA, Thompson B, Gustafson SK, Stewart
DE, Cherikh WS, Wainright JL, Boyle G, Snyder JJ, Israni AK, Kasiske BL.
OPTN/SRTR 2013 annual data report: Kidney. Am J Transplant 15
(Suppl 2): 1–34, 2015
3. Kim WR, Lake JR, Smith JM, Skeans MA, Schladt DP, Edwards EB,
Harper AM, Wainright JL, Snyder JJ, Israni AK, Kasiske BL. OPTN/SRTR
2013 Annual Data Report: Liver. Am J Transplant 15(Suppl 2): 1–28, 2015
4. Watt KD, Pedersen RA, Kremers WK, Heimbach JK, Charlton MR. Evolution
of causes and risk factors for mortality post-liver transplant: Results of the
NIDDK long-term follow-up study. Am J Transplant 10: 1420–1427, 2010
5. Engels EA, Pfeiffer RM, Fraumeni JF Jr, Kasiske BL, Israni AK, Snyder JJ,
Wolfe RA, Goodrich NP, Bayakly AR, Clarke CA, Copeland G, Finch JL,
Fleissner ML, Goodman MT, Kahn A, Koch L, Lynch CF, Madeleine MM,
Pawlish K, Rao C, Williams MA, Castenson D, Curry M, Parsons R, Fant
G, Lin M. Spectrum of cancer risk among US solid organ transplant
recipients. JAMA 306: 1891–1901, 2011
6. Maisonneuve P, Agodoa L, Gellert R, Stewart JH, Buccianti G,
Lowenfels AB, Wolfe RA, Jones E, Disney AP, Briggs D, McCredie M,
Boyle P. Cancer in patients on dialysis for end-stage renal disease: An
international collaborative study. Lancet 354: 93–99, 1999
7. van Leeuwen MT, Grulich AE, McDonald SP, McCredie MR, Amin J,
Stewart JH, Webster AC, Chapman JR, Vajdic CM. Immunosuppression
4
Onco-Nephrology Curriculum
17.
18.
19.
20.
21.
22.
23.
24.
and other risk factors for lip cancer after kidney transplantation. Cancer
Epidemiol Biomarkers Prev 18: 561–569, 2009
Perrett CM, Walker SL, O’Donovan P, Warwick J, Harwood CA, Karran P,
McGregor JM. Azathioprine treatment photosensitizes human skin to
ultraviolet A radiation. Br J Dermatol 159: 198–204, 2008
Yarosh DB, Pena AV, Nay SL, Canning MT, Brown DA. Calcineurin inhibitors decrease DNA repair and apoptosis in human keratinocytes
following ultraviolet B irradiation. J Invest Dermatol 125: 1020–1025,
2005
Grulich AE, van Leeuwen MT, Falster MO, Vajdic CM. Incidence of
cancers in people with HIV/AIDS compared with immunosuppressed
transplant recipients: A meta-analysis. Lancet 370: 59–67, 2007
Campbell SB, Walker R, Tai SS, Jiang Q, Russ GR. Randomized controlled trial of sirolimus for renal transplant recipients at high risk for
nonmelanoma skin cancer. Am J Transplant 12: 1146–1156, 2012
Penn I. Kaposi’s sarcoma in transplant recipients. Transplantation 64:
669–673, 1997
Parker A, Bowles K, Bradley JA, Emery V, Featherstone C, Gupte G,
Marcus R, Parameshwar J, Ramsay A, Newstead C; Haemato-oncology
Task Force of the British Committee for Standards in Haematology and
British Transplantation Society. Management of post-transplant lymphoproliferative disorder in adult solid organ transplant recipients—
BCSH and BTS Guidelines. Br J Haematol 149: 693–705, 2010
Pirsch JD, Stratta RJ, Sollinger HW, Hafez GR, D’Alessandro AM,
Kalayoglu M, Belzer FO. Treatment of severe Epstein-Barr virusinduced lymphoproliferative syndrome with ganciclovir: Two cases
after solid organ transplantation. Am J Med 86: 241–244, 1989
Ritchie AJ, Mussa S, Sivasothy P, Stewart S. Single-lung transplant
complicated by unexpected explant carcinoma: A management dilemma. J Heart Lung Transplant 26: 1206–1208, 2007
Dickson RP, Davis RD, Rea JB, Palmer SM. High frequency of bronchogenic carcinoma after single-lung transplantation. J Heart Lung
Transplant 25: 1297–1301, 2006
Engels EA. Inflammation in the development of lung cancer: Epidemiological evidence. Expert Rev Anticancer Ther 8: 605–615, 2008
Vajdic CM, McDonald SP, McCredie MR, van Leeuwen MT, Stewart JH,
Law M, Chapman JR, Webster AC, Kaldor JM, Grulich AE. Cancer incidence before and after kidney transplantation. JAMA 296: 2823–
2831, 2006
Kasiske BL, Vazquez MA, Harmon WE, Brown RS, Danovitch GM,
Gaston RS, Roth D, Scandling JD, Singer GG; American Society of
Transplantation. Recommendations for the outpatient surveillance of
renal transplant recipients. J Am Soc Nephrol 11(Suppl 15): S1–S86,
2000
Penn I. Incidence and treatment of neoplasia after transplantation. J
Heart Lung Transplant 12: S328–S336, 1993
Euvrard S, Morelon E, Rostaing L, Goffin E, Brocard A, Tromme I,
Broeders N, del Marmol V, Chatelet V, Dompmartin A, Kessler M, Serra
AL, Hofbauer GF, Pouteil-Noble C, Campistol JM, Kanitakis J, Roux AS,
Decullier E, Dantal J; TUMORAPA Study Group. Sirolimus and secondary skin-cancer prevention in kidney transplantation. N Engl J Med
367: 329–339, 2012
Stallone G, Schena A, Infante B, Di Paolo S, Loverre A, Maggio G,
Ranieri E, Gesualdo L, Schena FP, Grandaliano G. Sirolimus for Kaposi’s
sarcoma in renal-transplant recipients. N Engl J Med 352: 1317–1323,
2005
Salgo R, Gossmann J, Schöfer H, Kachel HG, Kuck J, Geiger H,
Kaufmann R, Scheuermann EH. Switch to a sirolimus-based immunosuppression in long-term renal transplant recipients: Reduced rate of
(pre-)malignancies and nonmelanoma skin cancer in a prospective,
randomized, assessor-blinded, controlled clinical trial. Am J Transplant
10: 1385–1393, 2010
Wong G, Chapman JR, Craig JC. Cancer screening in renal transplant
recipients: What is the evidence? Clin J Am Soc Nephrol 3(Suppl 2):
S87–S100, 2008
American Society of Nephrology
REVIEW QUESTIONS
1. A 50-year-old white woman, status post–kidney transplant
10 years ago, was recently diagnosed with squamous cell
skin cancer on her nose. What should you advise her?
a. Continue regular follow-up with dermatology
b. Discontinue calcineurin inhibitor and switch to mTOR
inhibitor–based regimen
c. Apply sun screen
d. Avoid peak hours of sun exposure
e. All of the above
Answer: e is correct. The patient should follow-up with
dermatology for a complete skin examination as the risk of a
second skin cancer is high. Exposure to UV light is one of
the main risk factors for development of cancer, and therefore, avoiding sunlight and applying sunscreen may prevent
development of new skin cancers. Last, studies have shown
that switching to an mTOR inhibitor–based regimen reduces the risk of additional skin cancers.
2. Which of these factors lead to increased risk of cancer in the
transplant recipient?
a.
b.
c.
d.
e.
Viral infections
Immunosuppression
Chronic Infections
Smoking
All of the above
Answer: e is correct. The majority of the cancers in transplant recipients are due to viral infections such as HPV, HCV,
HBV, EBV, and HHV-8. However, smoking, chronic infections, and certain immunosuppressants are also reported to
increase risk of cancer.
American Society of Nephrology
3. A 12-year-old boy underwent kidney transplant 6 months
ago. He received thymoglobulin for induction and was maintained on a triple immunosuppressive regimen including
mycophenolic acid derivatives. He was EBV negative at the
time of transplant. He now presents with low-grade fever
and pain over the allograft. The biopsy reveals dense lymphocytic infiltrate with minimal tubulitis. The lymphocytes
stained positive for CD3 and CD20. SV-40 stain is negative.
What is the most likely diagnosis in this patient?
a.
b.
c.
d.
Rejection
Acute interstitial nephritis
Post-transplant lymphoproliferative disorder (PTLD)
BK virus nephropathy
Answer: c is correct. The boy has developed PTLD as suggested by a mixed lymphocytic population in the biopsy.
CD31 indicates the presence of T cells, and CD201 indicates
the presence of B cells. Rejection will have only CD31 or
T-lymphocytes. Lack of SV-40 staining rules out BK virus nephropathy. The presence of abundant polymorphic lymphocytes with minimal tubulitis should be a clue for PTLD. He had
several risk factors for development of PTLD including young
age, EBV naïve, use of a T cell–depleting agent for induction,
and use of mycophenolic acid derivatives.
4. How will you manage this patient?
a.
b.
c.
d.
Increase immunosuppression
Decrease immunosuppression
Discontinue bactrim
Start cidofovir
Answer: a is correct. Treatment includes reduction of immunosuppression in this patient. Increasing immunosuppression to treat possible rejection may be harmful.
Onco-Nephrology Curriculum
5
Chapter 17: Cancer Screening in ESRD
Jean L. Holley, MD
Department of Medicine, University of Illinois, Urbana-Champaign and Carle Physician Group, Urbana, Illinois
INTRODUCTION
The American Cancer Society recommends specific
age-related screening examinations for colorectal,
breast, and cervical cancer and suggests that individuals discuss their risk factors and screening for
prostate and lung cancer with their primary care
physician (Table 1) (1). Such recommendations are
incorporated into guidelines for periodic adult
health care for the general population. Cancer
screening for any individual is predicated on the
risk of developing cancer and the likelihood that
the screening test will detect the cancer. An individual’s expected survival is also an integral factor in
cancer screening. If expected survival is low, then
the cost-effectiveness of routine cancer screening in
average-risk individuals argues against screening
because the patient will probably die before cancer
develops and is detected. In the ESRD population,
therefore, when considering routine cancer screening, it is important to ask the following: 1) is there
an increased risk of cancer in this patient group?; 2)
are screening tests accurate in this population?; and
3) will the patient live long enough for cancer
screening to detect a life-threatening disease that
can be cured? These issues will be discussed to demonstrate that, because of the high mortality with
ESRD, routine cancer screening is not indicated
for most patients.
CANCER RISK IN ESRD
Table 2 shows a summary of the published literature
cancer incidence among ESRD patients. The standardized incidence ratio (SIR) is typically used to
assess cancer frequency. Viral-mediated cancers
like human papilloma virus (HPV)-associated cervical, uterine, and tongue cancer and hepatitis C–
and B–associated liver cancer are more common in
ESRD patients (2–8). Although there are no clinical
data, the SIR for cervical cancer in ESRD patients
suggests that young women (and men) with ESRD
American Society of Nephrology
should receive the HPV vaccine according to the
recommendations in the general population (9).
Bladder cancer is also more common in ESRD patients, likely in part due to medications associated
with the development or treatment of kidney disease (oral cyclophosphamide use, analgesic use
leading to chronic tubulointerstitial disease). These
cancers, as well as liver cancer, are more common
among Asian ESRD patients (6). Because of the development of acquired cystic disease in ESRD, renal
cell carcinoma is also more common among dialysis patients, albeit with a relatively low incidence in
most studies (5,7) (Table 2). Routine screening for
renal cell carcinoma in chronic dialysis patients remains somewhat controversial but most advocate
for individual patient-directed screening based on
cost-effectiveness (10,11). The relatively low incidence of renal cell carcinoma in the setting of
acquired cystic disease and the low expected patient
survival with ESRD argues against routine screening. However, for patients on transplant waiting
lists, screening may be advisable and required.
EFFICACY OF CANCER SCREENING TESTS
IN ESRD
Cancer screening is primarily based on imaging
techniques or laboratory and histopathologic examinations (Table 1). For most of these evaluations,
the positive and negative predictive value of the test
has not been assessed in ESRD patients. Due to the
presence of vascular calcifications, mammography
interpretation in women with ESRD may be more
difficult (12,13). The higher rates of gastrointestinal bleeding in ESRD may result in higher fecal
occult blood tests than the general population
(14). This may actually lead to higher rates of
Correspondence: Jean L. Holley, Nephrology, S2S2, 611 West
Park Street, Urbana, Illinois 61802.
Copyright © 2016 by the American Society of Nephrology
Onco-Nephrology Curriculum
1
Table 1. American Cancer Society Recommendations for
Routine Cancer Screening
Cancer
Recommended screening
Breast
Colorectal
Cervical
Prostate
Lung
Yearly mammogram beginning at age 40, continuing as
long as in good health
Clinical breast examination every 3 years from age 20 to
39 and then yearly for age .40
MRI for high-risk women
Beginning age 50: Flexible sigmoidoscopy every 5
years or colonoscopy every 10 years or double
contrast barium enema every 5 years or CT
colonography (virtual colonoscopy) every 5 years,
with yearly fecal occult blood test or fecal
immunochemical test or stool DNA test done every 3
years
Begin screening at age 21: 21–29 years: Pap every 3
years; no HPV unless Pap is abnormal; 30–65 years:
Pap 1 HPV every 5 years or Pap alone every 3 years;
.65 years: no screening
Age 50, discuss pros and cons with MD; age 45 if African
American or father or brother with prostate cancer
before the age of 65
No recommendation
High risk: consider screening age 55–74 in fairly good
health with at least 30–pack-years smoking history
and either still smoking or quit within the last 15 years
Individuals with risk factors for specific cancers may need alternate screening
protocols and should discuss with their physicians. Adapted from the
American Cancer Society website.
colonoscopies in ESRD patients, perhaps resulting in earlier
detection of colorectal cancer in ESRD patients compared with
the general population (15). There is no information on the
reliability of fecal immunochemical or stool DNA testing in
ESRD patients. An ongoing study on the performance of fecal
occult blood testing in CKD may help to clarify these issues (16).
Table 2. Cancer incidence in ESRD: Literature summary
Cancer
Renal cell
Bladder and
ureter
Tongue
Cervical and
uterine
Liver
Thyroid and
other
Endocrine
organs
Breast (women)
Lung/bronchus
Colon/rectum
Pancreas
Prostate
SIR
Risk factors in ESRD
3.6–24.1 Acquired cystic disease
1.5–16.4 Analgesic abuse,
Balkan nephropathy,
oral cyclophosphamide
1.2–1.9 Human papilloma virus
0.9
Human papilloma virus
2.7–4.31
1.4–4.5 Hepatitis B and C
2.2–2.3
2,3,6
6
2,3
2,3,5,6
2,3,6
0.8–1.42
0.5–1.28
1.0–1.27
1.08
0.5–1.08
3,6,8
3,6,8
3,6,8
8
3,6,8
Adapted from reference 25 with additional data and references.
2
References
Onco-Nephrology Curriculum
2–8
2,3,5,6,8
Tumor markers are sometimes used as cancer screening
tools and may be affected by ESRD. Total prostate specific
antigen (PSA) is probably valid in ESRD patients (17–19), but
free PSA and free/total PSA ratios are less useful, as free PSA
rises with hemoconcentration and high-flux dialysis membranes affect its clearance (18,19). For unclear reasons, prostate cancer is the only tumor diagnosed at a later stage in ESRD
patients compared with the general population (15). Prostate
cancer has generally not been more common in ESRD patients
(2–7). However, a recent study found an SIR of 1.06 for prostate cancer (8), raising the issue of an increasing incidence of
this cancer among ESRD patients. Controversy continues
about screening for prostate cancer in the general population
(Table 1). Most tumor markers are unreliable in ESRD patients; they are generally glycoproteins with high molecular
weight that are rarely removed by dialysis and rise with hemoconcentration, yielding false-positive results in ESRD. For
example, cancer antigen 125 (CA-125), a tumor marker for
ovarian cancer, is produced by mesothelial cells, and patients
with any serosal fluid (pleural effusion, ascites) will have
elevated levels. This is especially applicable to patients on
peritoneal dialysis, making CA-125 less useful in all ESRD patients, particularly those on peritoneal dialysis. b-human chorionic gonadotropin and a-fetoprotein, as well as total PSA, are
probably reliable in ESRD patients.
The recently published clinical trial on the cost-effectiveness
of computed tomography (CT) screening for lung cancer in
high-risk individuals reported a 20% reduction in mortality
over a 4-year period in patients undergoing three annual CT
exams at a cost of $81,000 per quality-adjusted life-year and
incremental cost-effectiveness ratios of $52,000 per life-year
gained (20). This study led the US Preventive Services Task
Force to assign a B rating to the recommendation that annual
low-dose CT scanning be performed as a screen for lung cancer in adults 55–80 years of age with a 30–pack-year smoking
history. However, remaining questions about the overall efficacy of this screening method prompted the American Cancer
Society (Table 1) to avoid endorsing CT scanning as a cancer
screen. The Centers for Medicare and Medicaid, despite initial
misgivings (21), have now endorsed lung cancer screening
with CT scans. Lung cancer has not traditionally been more
common in ESRD patients (2–6), and ESRD patients’ reduced
survival argues against the adoption of lung cancer screening
in this population. Recently, a 1.28 SIR for lung cancer in
ESRD patients (8), along with the benefits of CT screening
in at-risk individuals (20), suggests additional study may be
needed.
IS CANCER SCREENING APPROPRIATE IN ESRD
GIVEN EXPECTED SURVIVAL?
Although survival in ESRD may be improving slightly, it
remains poor (22). As noted above, a patient’s expected survival is an important factor to consider when weighing the
American Society of Nephrology
benefits of cancer screening. Hypothetical modeling done in
the 1990s suggested cancer screening in a dialysis patient
would on average provide a net gain of 5 days of survival
(23). This model was biased toward cancer screening, examining Papanicolaou smears for detecting cervical cancer,
mammography as a screen for breast cancer, flexible sigmoidoscopy for colorectal cancer, and digital examination with PSA
testing for prostate cancer assuming screening tests were perfectly sensitive and specific and that each detected cancer was
instantaneously treated and cured (23). Using this model, the
costs per unit of survival benefit provided by cancer screening
were 1.6–19.3 times higher among ESRD patients (23). Another study focusing on breast cancer screening in dialysis
patients found an absolute reduction in breast cancer mortality of 0.1% with a net gain in life expectancy of 1.3 days (24).
Even focusing on the dialysis patient with the best predicted
survival (a young black woman without diabetes mellitus) and
multiple risk factors for breast cancer, only 250 days of life
were estimated to be saved by screening with mammography
in another study (25). Such investigations led to the recommendation to perform cancer screening only on dialysis patients assumed to benefit; cancer screening in ESRD should be
based on the individual, considering his or her risk factors for
cancer, as well as expected survival with ESRD (23–30).
Transplant candidacy also needs to be considered when
contemplating cancer screening in ESRD patients. The evaluative process for kidney transplantation includes age- and
sex- appropriate cancer screening such as mammography,
Papanicolaou smears, and PSA testing. Thus, cancer screening for transplant candidates is generally required. However,
ESRD patients will need to be assessed on an individual basis,
considering cancer risk factors, transplant status, and, importantly, expected survival to proceed with cancer screening in a
cost-effective manner. Table 3 suggests an outline for cancer
screening in ESRD patients based on these factors.
Table 3. Suggested cancer screening in ESRD patients:
Individualized, considering expected survival, risk factors,
and transplant status
Cancer
Breast
Colorectal
Cervical
Prostate
Renal cell
Recommended screening
-Yearly mammogram beginning age 40 and on
transplant list
Clinical breast examination every 3 years for ages 20–39
and yearly for age .40
Beginning age 50: Yearly FIT or FOBT for those on
transplant lists and flexible sigmoidoscopy,
colonoscopy, double contrast barium enema, or
virtual colonoscopy per transplant evaluation
protocols
Positive FIT or FOBT will require additional evaluation
Begin screening at age 21: 21–65, yearly Pap for those
on transplant list; consider HPV DNA and HPV
vaccine in transplant candidates
Age 50, annual PSA and digital rectal examination for
men on transplant list
Age 45 if African American or father or brother had
prostate cancer before the age of 65
Yearly CT or MRI in patients on dialysis .3 years and on
transplant list
For all the above cancers, consider screening in high-risk patients with long
expected survival. FIT, fecal immunochemical test; FOBT, fecal occult blood
test. Adapted from references 22–29.
TAKE HOME POINTS
c Viral-associated cancers like hepatitis B– and C–associated liver cancer
and human papilloma virus–associated tongue and cervical cancer are
more common in ESRD patients.
c Because of acquired cystic disease, renal cell carcinoma is more common in
ESRD patients, and exposure to analgesic abuse and oral cyclophosphamide
result in an increased incidence of bladder cancer in ESRD patients.
c Due to poor expected survival with ESRD, cancer screening is not
appropriate for most dialysis patients. Patients with long expected
survival, those on transplant waiting lists, and those with increased
cancer risk factors are appropriate candidates for cancer screening.
SUMMARY AND CONCLUSIONS
Although bladder cancer and viral-mediated cancers like HPVassociated cervical cancer and hepatitis C– and B–associated liver
cancer are more common in ESRD patients, general routine
cancer screening in ESRD patients is not recommended. ESRD
patients in whom cancer screening should be considered are
those with good expected survival, candidates for kidney transplantation, and certain individuals with a high cancer risk and
good expected survival. Although acquired cystic kidney disease
is associated with an increased risk of renal cell carcinoma, the
same general rules apply; routine screening is not recommended
for most patients. The tendency may be to implement routine
cancer screening protocols in dialysis units, but individualized
patient assessment is required for appropriate cancer screening.
The emerging model of personalized cancer screening for the
general population is being discussed (31) and seems clearly
appropriate for those on dialysis.
American Society of Nephrology
REFERENCES
1. American Cancer Society: American Cancer Society Guidelines for the
Early Detection of Cancer, 2015. Available at: http://www.cancer.org/
healthy/findcancerearly/cancerscreeningguidelines/american-cancersociety-guidelines-for-the-early-detection-of-cancer. Accessed January 21, 2015
2. Buccianti G, Maisonneuve P, Ravasi B, Cresseri D, Locatelli F, Boyle P.
Cancer among patients on renal replacement therapy: a populationbased survey in Lombardy, Italy. Int J Cancer 66: 591–593, 1996
3. Maisonneuve P, Agodoa L, Gellert R, Stewart JH, Buccianti G,
Lowenfels AB, Wolfe RA, Jones E, Disney APS, Briggs D, McCredie M,
Boyle P. Cancer in patients on dialysis for end-stage renal disease: An
international collaborative study. Lancet 354: 93–99, 1999
4. Heidland A, Bahner U, Vamvakas S. Incidence and spectrum of dialysisassociated cancer in three continents. Am J Kidney Dis 35: 347–351,
discussion 352–353, 2000
5. Chen K-S, Lai M-K, Huang C-C, Chu S-H, Leu M-L. Urologic cancers in
uremic patients. Am J Kidney Dis 25: 694–700, 1995
Onco-Nephrology Curriculum
3
6. Lin H-F, Li Y-H, Wang C-H, Chou C-L, Kuo D-J, Fang TC. Increased risk
of cancer in chronic dialysis patients: A population-based cohort study
in Taiwan. Nephrol Dial Transplant 27: 1585–1590, 2012
7. Farivar-Mohseni H, Perlmutter AE, Wilson S, Shingleton WB, Bigler SA,
Fowler JE Jr. Renal cell carcinoma and end stage renal disease. J Urol
175: 2018–2020, discussion 2021, 2006
8. Butler AM, Olshan AF, Kshirsagar AV, Edwards JK, Nielsen ME, Wheeler
SB, Brookhart MA. Cancer incidence among US Medicare ESRD patients
receiving hemodialysis, 1996-2009. Am J Kidney Dis 65: 763–772, 2015
9. Centers for Disease Control and Prevention: Immunization Schedules,
2015. Available at: http://www.cdc.gov/vaccines/schedules/. Accessed
January 21, 2015
10. Sarasin FP, Wong JB, Levey AS, Meyer KB. Screening for acquired
cystic kidney disease: A decision analytic perspective. Kidney Int 48:
207–219, 1995
11. Brown EA. Renal tumours in dialysis patients: Who should we screen?
Nephron Clin Pract 97: c3–c4, 2004
12. Evans AJ, Cohen ME, Cohen GF. Patterns of breast calcification in
patients on renal dialysis. Clin Radiol 45: 343–344, 1992
13. Castellanos M, Varma S, Ahern K, Grosso SJ, Buchbinder S, D’Angelo
D, Raia C, Kleiner M, Elsayegh S. Increased breast calcifications in
women with ESRD on dialysis: Implications for breast cancer screening.
Am J Kidney Dis 48: 301–306, 2006
14. Akmal M, Sawelson S, Karubian F, Gadallah M: The prevalence and
significance of occult blood loss in patients with predialysis advanced
chronic renal failure (CRF), or receiving dialytic therapy. Clin Nephrol
42: 198–202, 1994
15. Taneja S, Mandayam S, Kayani ZZ, Kuo Y-F, Shahinian VB. Comparison
of stage at diagnosis of cancer in patients who are on dialysis versus
the general population. Clin J Am Soc Nephrol 2: 1008–1013, 2007
16. Wong G, Howard K, Chapman JR, Tong A, Bourke MJ, Hayen A,
Macaskill P, Hope RL, Williams N, Kieu A, Allen R, Chadban S,
Pollock C, Webster A, Roger SD, Craig JC. Test performance of
faecal occult blood testing for the detection of bowel cancer in people
with chronic kidney disease (DETECT) protocol. BMC Public Health 11:
516–522, 2011
17. Morton JJ, Howe SF, Lowell JA, Stratta RJ, Taylor RJ. Influence of endstage renal disease and renal transplantation on serum prostatespecific antigen. Br J Urol 75: 498–501, 1995
18. Djavan B, Shariat S, Ghawidel K, Güven-Marberger K, Remzi M, Kovarik
J, Hoerl WH, Marberger M. Impact of chronic dialysis on serum PSA,
free PSA, and free/total PSA ratio: Is prostate cancer detection
4
Onco-Nephrology Curriculum
19.
20.
21.
22.
23.
24.
25.
26.
27.
28.
29.
30.
31.
compromised in patients receiving long-term dialysis? Urology 53:
1169–1174, 1999
Bruun L, Björk T, Lilja H, Becker C, Gustafsson O, Christensson A.
Percent-free prostate specific antigen is elevated in men on haemodialysis or peritoneal dialysis treatment. Nephrol Dial Transplant 18:
598–603, 2003
Black WC, Gareen IF, Soneji SS, Sicks JD, Keeler EB, Aberle DR, Naeim
A, Church TR, Silvestri GA, Gorelick J, Gatsonis C; National Lung
Screening Trial Research Team. Cost-effectiveness of CT screening in
the National Lung Screening Trial. N Engl J Med 371: 1793–1802, 2014
Bindman A. JAMA Forum: Lung cancer screening and evidence-based
policy. JAMA 313: 17–18, 2015
US Renal Data System. USRDS 2014 Annual Data Report: Atlas of EndStage Renal Disease in the United States, Bethesda, MD, National Institutes of Health, National Institute of Diabetes and Digestive and
Kidney Diseases, 2014
Chertow GM, Paltiel AD, Owen WF Jr, Lazarus JM. Cost-effectiveness
of cancer screening in end-stage renal disease. Arch Intern Med 156:
1345–1350, 1996
Wong G, Howard K, Chapman JR, Craig JC. Cost-effectiveness of breast
cancer screening in women on dialysis. Am J Kidney Dis 52: 916–929, 2008
LeBrun CJ, Diehl LF, Abbott KC, Welch PG, Yuan CM. Life expectancy
benefits of cancer screening in the end-stage renal disease population.
Am J Kidney Dis 35: 237–243, 2000
Holley JL. Screening, diagnosis, and treatment of cancer in long-term
dialysis patients. Clin J Am Soc Nephrol 2: 604–610, 2007
Kajbaf S, Nichol G, Zimmerman D. Cancer screening and life expectancy of Canadian patients with kidney failure. Nephrol Dial Transplant
17: 1786–1789, 2002
Holley JL. Preventive medical screening is not appropriate for many
chronic dialysis patients. Semin Dial 13: 369–371, 2000
Williams AW, Dwyer AC, Eddy AA, Fink JC, Jaber BL, Linas SL, Michael
B, O’Hare AM, Schaefer HM, Shaffer RN, Trachtman H, Weiner DE, Falk
AR; American Society of Nephrology Quality, and Patient Safety Task
Force. Critical and honest conversations: the evidence behind the
“Choosing Wisely” campaign recommendations by the American
Society of Nephrology. Clin J Am Soc Nephrol 7: 1664–1672, 2012
Walter LC, Lindquist K, O’Hare AM, Johansen KL. Targeting screening
mammography according to life expectancy among women undergoing dialysis. Arch Intern Med 166: 1203–1208, 2006
Saini SD, van Hees F, Vijan S. Smarter screening for cancer: Possibilities
and challenges of personalization. JAMA 312: 2211–2212, 2014
American Society of Nephrology
REVIEW QUESTIONS
1. Is the cancer incidence in ESRD patients higher than in the
general population?
Answer: Yes, for certain cancers. Virus-associated cancers
(liver cancer, cervical cancer, and tongue cancer) and renal cell
and bladder cancer (because or risk factors) are more common
in ESRD patients. Breast, colorectal, and lung cancer are not
more common in ESRD patients.
2. What factors affect the efficacy or cost-effectiveness of
cancer screening in general?
cancer screening. Screening is predicated on the patient living
long enough to develop a specific cancer and the sensitivity
and specificity of the screening test to detect that cancer at a
stage when cure is possible.
3. Should routine screening protocols be in place in dialysis
units?
Answer: No, routine cancer screening is not cost-effective
for most dialysis patients because their expected survival is
short. An individualized approach to cancer screening is
most appropriate for ESRD patients, considering the patient’s
specific risk factors for cancer development, transplant status,
and expected survival.
Answer: The cancer risk, the effectiveness of the screening
test, and the patient’s expected survival all affect the efficacy of
American Society of Nephrology
Onco-Nephrology Curriculum
5
Chapter 18: Ethics of RRT, Initiation, and Withdrawal in
Cancer Patients
Michael J. Germain, MD
Department of Medicine, Tufts University, Springfield, Massachusetts
Malignancies are common in CKD patients, and the
incidence is higher than in the general age-matched
population. Because cardiovascular disease and infection are so prevalent in CKD, especially ESRD
patients, the mortality rate from cancer in ESRD
patients is lower than the age-matched general population due to these competing influences. Thus, the
relative risk of mortality from cancer is increased in
the younger ESRD population and then declines with
age (1,2).
Patients with cancer and a need for RRT present
very difficult scenarios for making clinical decisions,
and an approach grounded in medical ethical principles can be helpful (3–7). Medical ethics reflect the
culture and time that we are living in and also
include a religious perspective. This chapter will
focus on a US perspective that reflects the generally
accepted values of our society at the present time. The
United States has a wide representation of cultural
and religious values, with many patients who are
new immigrants from many countries. A discussion
of the different medical ethical approaches from these
societies is beyond the scope of this discussion, but
the clinician should always inquire from the patient
and family how they want prognosis, goals of care,
and end-of-life issues discussed with them. This discussion will rely heavily on the national clinical
practice guideline Shared Decision-Making in the Appropriate Initiation of and Withdrawal from Dialysis,
2nd Ed. (SDMG), in particular the section “Ethical
Considerations in Dialysis Decision-Making” (8). Six
ethical principles should be strongly considered for
patients with cancer when discussing RRT (Table 1).
Conflicts between respect for patient autonomy
and beneficence/nonmaleficence often can occur
with these patients. There are four scenarios where
the ethical issues of cancer and RRT intersect: 1)
patients with ESRD who develop a terminal malignancy; 2) patients with a terminal malignancy who
develop ESRD; 3) patients with a terminal malignancy
who develop AKI (AKI can be caused by the treatment
of the malignancy, obstruction or invasion of the
American Society of Nephrology
kidney by the malignancy, or surgical removal of the
kidney to remove the malignancy); and 4) a renal
transplant patient with a terminal malignancy.
In the first scenario, withdrawal of dialysis (9–11)
is often the ethical question. In the second and third
scenarios, withholding of dialysis or withdrawal may
be the ethical issue. An important ethical aspect is the
ethical imperative of the clinician to “first, do no
harm.” The clinician has the right and duty not to
order a treatment that will do more harm than good.
Nephrologists often find themselves in the position of
being asked to provide dialysis, by a patient, family, or
other clinicians, when dialysis may not be in the patient’s best interest. Many clinicians feel they are required to provide dialysis treatment when the patient
or health care provider (HCP) requests it. The SDMG
(recommendations 5 and 6) clearly state that the clinician has no such obligation. The clinician should
document these discussions and make it clear that the
patient/HCP has the right to transfer care to another
clinician. Clinicians should not fear medical–legal
concerns in this scenario; in reality, these rarely, if
ever, occur, especially if the SDMG is followed.
Instead, shared decision-making is the preferred
process where the clinician/care team (SDMG recommendation 1) and the patient/family/HCP
make a care plan for the patient. The first step in
this process is for the care team to ask, listen, and
understand the patient’s understanding of his or her
condition and values and goals in life. With the patient’s explicit permission, the care team then explains from their expert perspective the patient’s
condition, prognosis, and the risks and benefits of
the treatment options. Recent qualitative studies
have shown that CKD patients want to know their
prognosis. However, our experience is such that patients often do not want a numerical estimate, such
Correspondence: Michael J. Germain, Department of Medicine,
Tufts University, 100 Wason Avenue, Suite 200, Springfield,
Massachusetts 01107.
Copyright © 2016 by the American Society of Nephrology
Onco-Nephrology Curriculum
1
Table 1. Six medical ethics principles (http://en.wikipedia.
org/wiki/Medical_ethics#Values_in_medical_ethics)
1) Respect for autonomy: The patient has the right to refuse or choose
their treatment (voluntas aegroti suprema lex).
2) Beneficence: A practitioner should act in the best interest of the
patient (salus aegroti suprema lex).
3) Nonmaleficence: “First, do no harm” (primum non nocere).
4) Justice: Concerns the distribution of scarce health resources and the
decision of who gets what treatment (fairness and equality).
5) Respect for persons: The patient (and the person treating the patient)
has the right to be treated with dignity.
6) Truthfulness and honesty: The concept of informed consent
as how many months of life they may have remaining. Rather,
they prefer a general statement about overall prognosis (SDMG
recommendations 2 and 3).
Through the process of consensus building, a shared
decision and treatment plan is agreed on (SDMG recommendation 4). In a consensus, each party may not get the plan they
originally favored, but they may be convinced by hearing the
perspectives put forth by others that a different plan is preferred. Sometimes the party may not like the consensus plan
but agrees to accept it. I have seen this in situations where the
HCP may want to initiate or continue dialysis in a patient who
clearly is getting no benefit (or even suffering harm), but the
nephrologist has decided that he or she cannot ethically order
the dialysis treatment. After offering to transfer care to another
nephrologist, invariably the HCP will accept the clinician’s
decision. For this to work, the clinician must show respect
for the alternative point of view, listen carefully, and validate
the person’s views. If the views are based on religious objections, then it is sensible to involve a clergy person who may
help explain the religion’s positions. A time-limited trial of
RRT (7) can be undertaken in certain defined situations
such as when the benefit to be achieved by dialysis is uncertain
or a consensus about the benefit of dialysis cannot be reached
(SDMG recommendation 7).
When consensus cannot be reached, the SDMG suggests
conflict resolution (SDMG recommendation 8; “resolving
conflicts about what dialysis decision to make”; Box 1 and
Figure 1). The SDMG suggests a practical ethical approach
to decision-making. The patient’s case is analyzed from these
perspectives (Table 2).
Each perspective is viewed through the six ethical principles in
Table 1. The SDMG then recommends the following process for
ethical decision-making (Table 3). Although the SDMG recommends that patients with a terminal prognosis (,6 months)
should not receive dialysis, the guidelines recognize that “palliative dialysis” (12) is an option for those who require more time
to finish their life goals. Such goals include activities for significant events like a wedding, birth, or graduation. Palliative dialysis allows the patient to transition to a more comfort-oriented
care. The patient may shorten their dialysis treatment time, restrict further hospitalizations or procedures, and, when appropriate, receive hospice services (SDMG recommendation 9).
2
Onco-Nephrology Curriculum
Box 1. Suggested steps for implementing
recommendation 8 (reproduced with permission)
■
Extended conversation
2 Why does the patient or legal agent desire dialysis when it is not
recommended by the renal care team?
2 Why does the patient or legal agent refuse dialysis when it is
recommended by the renal care team?
2 Does the patient or legal agent misunderstand the diagnosis,
prognosis, and treatment alternatives?
2 Does the nephrologist misunderstand the patient’s or legal
agent’s reasons for requesting dialysis?
2 Does the nephrologist understand the psychosocial, cultural,
or spiritual concerns and values the patient or legal agent have?
2 Has the nephrologist consulted a psychologist, social worker, or
chaplain for assistance in fully understanding the concerns of the
patient or legal agent family?
■ Consultation with other physicians
2 Do other physicians agree or disagree with the attending
physician’s recommendation to withhold or withdraw
dialysis?
2 Is the request for dialysis by the patient or legal agent
medically appropriate?
■ Consultation with ethics committee or ethics consultants.
2 Has the patient or legal agent been informed that the purpose of
the ethics consult is to clarify issues of disagreement, and ideally,
to enable resolution?
2 Has the patient or legal agent met with the ethics committee or
ethics consultants to explain their perspective and reasoning
behind their request for dialysis?
2 Can the ethics committee identify the reasons why the patient or
legal agent is resistant to the physician’s recommendation to
forgo dialysis?
2 Can the ethics committee identify the reasons why the health
care provider is resistant to the patient’s or legal agent’s desire to
begin or continue dialysis?
2 Has the ethics committee explained in understandable terms to
the patient or legal agent its conclusions and the reasoning
behind them?
2 Can the impasse be resolved with accommodation, negotiation,
or mediation?
■ Documentation
2 The physician must document the medical facts and his/her
reasons for the recommendation to forgo dialysis and the
decision not to agree to the request by the patient or legal agent.
2 The consultants should also document their assessment of the
patient’s diagnosis, prognosis, and their recommendations in
the chart.
■ An attempt to transfer the patient’s care
2 If reconciliation is not achieved through the above procedure
and the physician in good conscience cannot agree to the
patient or legal agent’s request, the physician is ethically and
legally obligated to attempt to transfer the care of the patient to
another physician.
2 Another physician and/or institution may not be found who is
willing to accept the patient under the terms of the family’s
request. Physicians and institutions that refuse to accept the
patient in transfer and their reasons should also be documented
in the medical record.
2 Consider consultation with a mediator, extramural ethics
committee, or the ESRD Network in the region.
American Society of Nephrology
Table 2. Perspectives to consider in ethical decisionmaking*
1) Medical indications, the diagnosis, prognosis, and treatment
2) Patient preferences
3) Quality of life
4) Contextual features (social, economic, legal, and administrative)
*Adapted from reference 8 with permission from the Renal Physicians Association
Table 3. The seven-step process of ethical decision-making
in patient care*
1) What are the ethical questions
2) What are the clinically relevant facts
3) What are the values at stake
4) List options (what could you do)
5) What should you do (choice the best option from the ethical point of
view balancing all the above factors
6) Justify your choice based on the ethical principles
7) How could this ethical issue have been prevented
*Adapted from reference 8 with permission from the Renal Physicians Association.
Table 4. Systems approach to American College of
Physicians in nephrology practice and the dialysis unit
1) Normalize the conversation: start discussions of EOL issues early in
the patient’s interaction with the nephrology team.
2) Involve all members of the care team. In the office, this depends on
human resources available. In the dialysis unit, train and utilize the
dietician, technicians, social worker, and nurses.
3) Have a champion. Without this, likely there will be little buy-in or
progress. Although the nephrologist does not have to be the
champion, the nephrologist leader (i.e., medical director in the
dialysis unit) needs to show strong support.
4) Teach all staff members simple communication techniques.
5) Integrate ACP into the workflow.
6) Do continuous quality improvement on the process.
7) There are resources available to learn from established successful
programs .
Figure 1. Systematic approach to resolving conflict between
patient and renal care team.
Finally, an effective process depends on excellent clinician–
patient/family communication (13–15) (SDMG recommendation 10). To have these discussions, appropriate systems
must be in place in the nephrology practice and dialysis units
to facilitate the process (Table 4,) (16–18).
There are excellent resources to help the health care team to
accomplish these goals and tasks. Offering meticulous end-of-life
care, including hospice, is mandatory for all of our patients with a
,6-month prognosis (17–19). It is important for patients and
families to understand that palliative care and hospice do not
American Society of Nephrology
result in a shortened survival (20). Patients are not harmed, and
they appreciate honest communication of bad news (21). It is
important to recognize that our patients want to know their
prognosis, and there are validated tools available for the clinician
to utilize when having this discussion (22,23).
In the end, the goal of the communication between the
patient (and family or other preferred surrogate decisionmaker) and the kidney care team is shared decision-making.
Shared decision-making is the recognized preferred model
for medical decision-making because it addresses the ethical
need to fully inform patients about the risks and benefits of
treatments, as well as the need to ensure that patients’ values
and preferences play a prominent role (8). Shared decisionmaking has been referred to as the “pinnacle” of patient-centered
care (24). Patient-centered care has been one of the six specific aims for improvement for health care since the Institute
Onco-Nephrology Curriculum
3
Table 5. List of resources
1) RPA SDM Toolkit: https://itunes.apple.com/us/app/rpa-sdm-toolkit/
id843971920?mt58
2) ACP DECISIONS: http://www.acpdecisions.org
3) Coalition for Kidney Supportive Care: http://www.
kidneysupportivecare.org/Home.aspx
4) Supportive Care for the Renal Patient, edited by Chambers, Brown,
Germain, 2nd Ed., London, UK, Oxford Press, 2010
5) RPA SDM Guidelines, 2nd Ed., 2010: http://www.renalmd.org/
catalogue-item.aspx?id5682
6) Five Wishes: https://www.agingwithdignity.org
7) Alberta’s Conversations Matter: http://www.albertahealthservices.
ca/9098.asp
8) ASN Geriatric Curriculum: http://asn-online.org/education/
distancelearning/curricula/geriatrics/
9) KDIGO Renal Supportive Care Initiative: http://kdigo.org/home/
conferences/supportivecare/
10) Vital Talk communication techniques: http://vitaltalk.org/sites/
default/files/quick-guides/NURSEforVitaltalkV1.0.pdf
11) https://www.prepareforyourcare.org/
12) http://www.ihi.org/Engage/Initiatives/ConversationProject/
Pages/default.aspx
13) http://www.gundersenhealth.org/respecting-choices
14) Six-month mortality predictor: Under nephrology/HD, http://www.
qxmd.com/apps/calculate-by-qxmd
15) Breaking Bad News SPIKE: https://depts.washington.edu/bioethx/
topics/badnws.html
of Medicine (IOM) issued its 2001 report, Crossing the Quality
Chasm: A New Health System for the 21st Century (25). The
IOM noted that the US health care delivery system does not
provide consistent, high-quality medical care to all people. The
IOM defined patient-centered as “providing care that is respectful of and responsive to individual patient preferences,
needs, and values, and ensuring that patient values guide all
clinical decisions.” Since the publication of the IOM report,
there has been growing national interest in more individualized
patient-centered models of care. These models focus on what
matters most to individual patients and less on what might
matter to providers or health systems (26). A recent qualitative
study suggests that patients want to discuss ACP with the nephrologist (27). When effectively done, it can increase use of
hospice and provide a “good death” (28). A recent review emphasizes the ethical principles involved in these discussion with
the elderly CKD patient (29). This chapter has sought to explain
how high-quality, ethical care can be delivered to patients with
advanced kidney disease and cancer.
TAKE HOME POINTS
c The ethics of RRT in cancer balances the principles of respect for patient
autonomy with nonmaleficence.
c In some cases, palliative dialysis may be an option for these patients.
c Good communication skills are the key to shared decision-making and
patient-centered care.
4
Onco-Nephrology Curriculum
REFERENCES
1. Available at: http://www.usrds.org/2014/view/Default.aspx. Accessed
March 24, 2015
2. Oneschuk D, Fainsinger R. Medical and ethical dilemmas when an
advanced cancer patient discontinues dialysis. J Palliat Care 18: 123–
126, 2002
3. Del Vecchio L, Locatelli F. Ethical issues in the elderly with renal disease.
Clin Geriatr Med 25: 543–553, 2009
4. Davison SN. The ethics of end-of-life care for patients with ESRD. Clin J
Am Soc Nephrol 7: 2049–2057, 2012
5. Skold A, Lesandrini J, Gorbatkin S. Ethics and health policy of
dialyzing a patient in a persistent vegetative state. Clin J Am Soc
Nephrol 9: 366–370, 2014
6. Moss AH. Ethical principles and processes guiding dialysis decisionmaking. Clin J Am Soc Nephrol 6: 2313–2317, 2011
7. Rinehart A. Beyond the futility argument: the fair process approach and
time-limited trials for managing dialysis conflict. Clin J Am Soc Nephrol
8: 2000–2006, 2013
8. Renal Physicians Association. Shared Decision Making in the Appropriate Initiation of and Withdrawal from Dialysis, 2nd Ed., Rockville,
MD, Renal Physicians Association, 2010
9. Akbar S, Moss AH. The ethics of offering dialysis for AKI to the older patient: Time to re-evaluate? Clin J Am Soc Nephrol 9: 1652–1656, 2014
10. Brown EA. Non-dialysis therapy: A better policy than dialysis followed
by withdrawal? Semin Dial 25: 26–27, 2012
11. Murtagh F, Cohen LM, Germain MJ. Dialysis discontinuation: Quo vadis? Adv Chronic Kidney Dis 14: 379–401, 2007
12. Grubbs V, Moss AH, Cohen LM, Fischer MJ, Germain MJ, Jassal SV, Perl
J, Weiner DE, Mehrotra R; Dialysis Advisory Group of the American
Society of Nephrology. A palliative approach to dialysis care: A patientcentered transition to the end of life. Clin J Am Soc Nephrol 9: 2203–
2209, 2014
13. Schell JO, Cohen RA. A communication framework for dialysis
decision-making for frail elderly patients. Clin J Am Soc Nephrol 9:
2014–2021, 2014
14. Schell JO, Green JA, Tulsky JA, Arnold RM. Communication skills
training for dialysis decision-making and end-of-life care in nephrology.
Clin J Am Soc Nephrol 8: 675–680, 2013
15. Tamura MK, Tan JC, O’Hare AM. Optimizing renal replacement therapy in older adults: A framework for making individualized decisions.
Kidney Int 82: 261–269, 2012
16. Da Silva-Gane M, Cohen LM. Planning a supportive care programme
and it’s components. In: Supportive Care for the Renal Patient, edited
by Chambers J, Brown E, Germain MJ, 2nd Ed., Oxford, UK, Oxford
University Press, 2010, 39–48
17. Germain MJ, Kurella Tamura M, Davison SN. Palliative care in CKD: The
earlier the better. Am J Kidney Dis 57: 378–380, 2011
18. Wong SP, Kreuter W, O’Hare AM. Treatment intensity at the end of life
in older adults receiving long-term dialysis. Arch Intern Med 172: 661–
663, discussion 663–664, 2012
19. Farrington K, Chambers JE. Death and end-of-life care in advanced
kidney disease. In: Supportive Care for the Renal Patient, edited by
Chambers JE Brown E, Germain MJ, 2nd Ed., Oxford, UK, Oxford
University Press, 2010
20. Temel JS, Greer JA, Muzikansky A, Gallagher ER, Admane S, Jackson
VA, Dahlin CM, Blinderman CD, Jacobsen J, Pirl WF, Billings JA, Lynch
TJ. Early palliative care for patients with metastatic non-small-cell lung
cancer. N Engl J Med 363: 733–742, 2010
21. Wachterman MW, Marcantonio ER, Davis RB, Cohen RA, Waikar SS,
Phillips RS, McCarthy EP. Relationship between the prognostic expectations of seriously ill patients undergoing hemodialysis and their
nephrologists. JAMA Intern Med 173: 1206–1214, 2013
22. Germain MJ. How to integrate predictions in outcomes in planning
clinical care. Blood Purif 39: 65–69, 2015
American Society of Nephrology
23. Cohen LM, Ruthazer R, Moss AH, Germain MJ. Predicting six-month
mortality for patients who are on maintenance hemodialysis. Clin J Am
Soc Nephrol 5: 72–79, 2010
24. Barry MJ, Edgman-Levitan S. Shared decision making: Pinnacle of
patient-centered care. N Engl J Med 366: 780–781, 2012
25. Institute of Medicine. Crossing the Quality Chasm: A New Health System
for the 21st Century, Washington, DC, National Academies Press, 2001
26. Tinetti ME, Fried T. The end of the disease era. Am J Med 116: 179–
185, 2004
American Society of Nephrology
27. Goff SL, Eneanya ND, Feinberg R, Germain MJ, Marr L, Berzoff J,
Cohen LM, Unruh M. Advance care planning: A qualitative study of
dialysis patients and families. Clin J Am Soc Nephrol 10: 390–400, 2015
28. Cohen LM, Ruthazer R, Germain MJ. Increasing hospice services for
elderly patients maintained with hemodialysis. J Palliat Med 13: 847–
854, 2010
29. Thorsteinsdottir B, Swetz KM, Albright RC. The ethics of chronic dialysis
for the older patient: Time to reevaluate the norms. Clin J Am Soc
Nephrol 10: 2094–2099, 2015
Onco-Nephrology Curriculum
5
REVIEW QUESTIONS
1. A long-term dialysis patient presents with metastatic sarcoma that is not treatable, and the prognosis is poor. What
are the relevant medical ethical principles to consider in
this patient?
a.
b.
c.
d.
e.
f.
g.
Autonomy
Nonmaleficence
Beneficence
Respect for person
Truth and honesty
All of the above
None of the above
Answer: f is correct. Patient autonomy, nonmaleficence
(avoiding the harms of RRT), beneficence, respect for person,
truth, and honesty.
2. The family requests that you withhold the cancer diagnosis/prognosis information from this patient. What is
the ethical principle that would guide your decision?
a. Nonmaleficence: The information would be harmful to the
patient
b. Truth and honesty.
c. Beneficence
Answer: b is correct. In our society, it is not ethical to withhold this information. In some cultures, it is left to the doctor
to decide if it would be “harmful” to the patient to give them
bad news. If a family asks that you not give bad news to the
patient, it is acceptable to ask the patient if they prefer that
6
Onco-Nephrology Curriculum
these discussions take place with a family member or HCP
instead of with them; this is a common scenario for some
cultures in the United States (Native American, some Asian
cultures).
3. The patient’s health deteriorates rapidly, and she is in
pain whenever she is moved, such as transportation to
and from dialysis. She is very lethargic and not communicative. The clinician feels that dialysis is doing more
harm than good for the patient. When this is discussed
with the family, they insist that dialysis be continued.
They believe that in their religion withdrawing dialysis
is a sin. Attempts at shared decision making and involving
their pastor have not resulted in a resolution of the
conflict. The correct approach is to:
a. Continue dialysis
b. Seek a court order to withdraw dialysis
c. Explain to the family that you understand and respect their
point of view; explain that you have an ethical duty to do no
harm by the treatments that you order for your patients,
and at this point, dialysis is doing more harm than good;
and you will be discontinuing your order for dialysis and
the family can seek another clinician to take over care if
they wish
Answer: c is correct. After following an shared decision
making process and conflict resolution, if there is still no consensus, then the clinician has the right and ethical duty to not
order RRT if the principle of nonmaleficence and justice outweighs the principle of autonomy. The patient has the right to
refuse a treatment but not to demand a treatment.
American Society of Nephrology
Chapter 19: Palliative Care in Patients with Kidney
Disease and Cancer
Alvin H. Moss, MD, FACP, FAAHPM
Department of Medicine, Sections of Nephrology and Supportive Care, West Virginia University School of Medicine,
Morgantown, West Virginia
INTRODUCTION
RELEVANCE OF PALLIATIVE CARE
Many, if not most, cancer and kidney disease patients
have two things in common: they have a shortened life
expectancy and a high symptom burden. Both
populations benefit from early palliative care interventions. The goal of palliative care is to relieve
suffering and to support the best possible quality of life
for patients and their families, regardless of their stage
of disease or the need for other therapies, in accordance with their values and preferences. By Medicare
regulation, hospice care is limited to patients estimated to be in their last 6 months of life if their disease
follows the normal course. In patients with kidney
disease and cancer (hereafter kidney-cancer patients)
who have a higher symptom burden than patients
with either disease alone, the need for meticulous pain
and symptom management is even more important to
maintain quality of life. In addition, there is a unique
need for advance care planning for these patients,
most of whom have two life-limiting illnesses. As in
other populations, for kidney-cancer patients, pain is
one of the most common and severe symptoms.
Multiple studies in kidney patients show that pain is
undertreated. Treatment of pain in patients with stage
4 and 5 CKD and ESRD is more challenging because
of the failure of renal excretion of active metabolites
from some commonly used opioids, which leads to
opioid neurotoxicity. The nephrology community has
developed a clinical practice guideline that endorses
the process of shared decision-making in reaching
decisions about who should undergo dialysis. It
recognizes that the burdens of dialysis may substantially outweigh the benefits in some patients and
notes that nephrologists may want to recommend
forgoing dialysis to kidney-cancer patients who are
terminally ill from their cancer.
This chapter describes the growing interest in the
nephrology and oncology communities in incorporating palliative care into the standard treatment
of patients with CKD, ESRD, and cancer.
There is an increasing recognition that skills in
palliative and end-of-life care are required for physicians, nurses, and others who treat patients who have
CKD and ESRD (Table 1). The principal reasons are
as follows: first, they have a significantly shortened
life expectancy; just over half of dialysis patients
(52%) are still alive 3 years after the start of RRT (1).
Second, patients with CKD and ESRD have multiple
comorbidities and consequently many symptoms
such as pain, fatigue, itching, and difficulty with
sleep. In one study, the symptoms of CKD and
ESRD patients were found to be comparable (mean
of 10.7) and severity (2). Similarly, cancer patients
have been found to have a high symptom burden
compared with age-matched controls, and pain,
anxiety and depression, and insomnia were noted
as most prevalent in a population-based study of
1,904 cancer survivors (3). An interaction between
cancer status and comorbidity was found, resulting
in a higher symptom burden for patients with comorbidities such as CKD. Thus, it is reasonable to
conclude the CKD or ESRD patients with cancer
will have a higher symptom burden than patients
with either cancer or kidney disease alone (3).
Third, the dialysis population has been growing
progressively older. The incidence rates of ESRD are
highest in patients 75 years old and older, and they
continue to rise in this group (1). Older patients survive the shortest period of time on dialysis, and they
withdraw from dialysis significantly more often than
younger patients.
In consideration of the high symptom burden
and the low survival rate for dialysis patients, the
American Society of Nephrology
Correspondence: Alvin H. Moss, Department of Medicine,
Sections of Nephrology and Supportive Care, West Virginia
University School of Medicine, 1195 Health Sciences North,
Morgantown, West Virginia 26506-9022
Copyright © 2016 by the American Society of Nephrology
Onco-Nephrology Curriculum
1
Table 1. Palliative care for CKD/ESRD patients: Need for a
systematic approach
Table 2. Components of a dialysis facility palliative care
program
Pain and symptom assessment/management (11,21)
Shared decision-making for informed consent
Patient-specific estimates of prognosis using the surprise question
Timely discussions prompted by prognosis
Inclusion of family/legal agent in discussions
Completion of advance directives
Completion of physician orders for life-sustaining treatment (POLST)
paradigm form as appropriate
Immediately actionable medical orders
Transferrable throughout health care setting
Referral to hospice when indicated
1. Palliative care focus
a. Educational activities, including dialysis unit in-service trainings
b. Quality improvement activities, including morbidity and mortality
conferences
c. Use of the “Would you be surprised if this patient died within the next
year?” question to identify patients appropriate for palliative care
d. Collaboration with local hospice programs to coordinate a smooth
transition to end-of-life care
2. Pain and symptom assessment and management protocols
3. Systematized advanced care planning
4. Psychosocial and spiritual support to patients and families, including
the use of peer counselors
5. Terminal care protocols that include hospice referral
6. Bereavement programs for families that include memorial services
Adapted from reference 5.
American Society of Nephrology (ASN) and the Renal Physicians Association (RPA) have recommended that dialysis
facilities incorporate palliative care into their treatment of
patients (4,5). Nephrologists have been encouraged to obtain
education and skills in palliative care, and dialysis facilities
have been urged to developed protocols, policies, and programs to ensure that palliative care is provided to their patients
(Table 2) (5). Also, dialysis units have been urged to develop a
working relationship with local hospice programs, so patients
with ESRD who stop dialysis or patients undergoing dialysis
with a nonrenal terminal diagnosis may be referred for hospice. Similarly, the American Society of Clinical Oncology has
issued a provisional clinical opinion that early involvement of
palliative care when combined with standard cancer care leads
to better patient and caregiver outcomes, including improvement in symptoms, quality of life, and patient satisfaction and
reduced caregiver burden (6).
SYMPTOM MANAGEMENT
Pain
As in other patient populations, the burden of symptoms for
patients undergoing dialysis is inversely associated with their
reported quality of life (7). Pain is one of the most common
symptoms reported by patients undergoing dialysis, and several studies have found that approximately 50% of these patients report pain. For most patients undergoing dialysis, the
pain is musculoskeletal in origin. Smaller numbers of patients
have pain related to the dialysis procedure, peripheral neuropathy, peripheral vascular disease, or carpal tunnel syndrome.
Three studies have found that pain is undertreated in 75% of
patients undergoing dialysis (8–10). As in cancer patients, use
of the World Health Organization (WHO) three-step analgesic ladder has been found to be effective in the treatment of
pain in dialysis patients (9). Because the metabolites of some
of the opioids on the analgesic ladder are renally excreted and
active, these opioids, morphine, codeine, meperidine, and
propoxyphene, are not recommended for use in patients
with advanced kidney disease (Table 3) (11).
2
Onco-Nephrology Curriculum
Adapted from reference 5.
Morphine is the best studied of the opioids used for pain
management, and its most common metabolites (including
morphine-3-glucuronide, morphine-6-glucuronide, and normorphine) are excreted by the kidneys. The clearance of these
metabolites is particularly problematic in stage 4 and 5 CKD
and ESRD. Morphine-6-glucuronide is an active metabolite
with analgesic properties; it crosses the blood–brain barrier
and may have prolonged central nervous system effects. A
comprehensive review recommended that morphine not be
used in patients with kidney disease because it is so difficult
to manage the complicated adverse effects of the morphine
metabolites (12).
Studies of codeine pharmacokinetics suggest that codeine
metabolites would accumulate to toxic levels in a majority of
patients undergoing hemodialysis. Codeine use is not recommended because serious adverse effects have been reported in
patients with CKD (12).
Hydromorphone is metabolized in the liver largely to
hydromorphone-3-glucuronide. This metabolite accumulates
in patients with kidney disease and can cause opioid neurotoxicity. Some studies suggest that hydromorphone is removed
with dialysis. It is recommended that hydromorphone be used
cautiously, if at all, in patients stopping dialysis (12).
Use of oxycodone in patients with kidney disease has not
been well studied. The elimination half-life of oxycodone is
Table 3. Pain medications for use in advanced kidney failure
Recommended
Use with caution
Do not use
Fentanyl
Methadone
Hydromorphone
Acetaminophen
Gabapentin
Tramadol
Hydrocodone/oxycodone
Desipramine/nortriptyline
Morphine
Codeine
Meperidine
Propoxyphene
Adapted from reference 11. Seehttp://www.kidneysupportivecare.org/Files/
PainBrochure9-09.aspx for recommendations for use, dosage reductions,
and cautions in advanced kidney failure.
American Society of Nephrology
lengthened in dialysis patients, and excretion of metabolites is
impaired but almost all are inactive. Oxycodone can be used
with caution in patients with advanced CKD and ESRD (12).
The WHO analgesic ladder recommends the use of fentanyl
for severe pain. Fentanyl is metabolized in the liver primarily
to norfentanyl. There is no evidence that any fentanyl metabolites are active. Several studies have found that fentanyl can be
used safely in patients with CKD and ESRD. It has negligible
dialyzability. Fentanyl use is deemed to be one of the safest
opioids to use in patients with advanced CKD (12).
The WHO analgesic ladder recommends methadone for
severe pain. Studies in anuric patients have found that nearly all
of methadone and its metabolites doses are excreted in the
feces. No dose adjustments are recommended for patients
undergoing dialysis. The use of methadone appears safe in
patients with advanced CKD and ESRD (12).
Opioids are also often used to treat dyspnea at the end of life in
patients with CKD, ESRD, and cancer. In the setting of worsening renal function or withdrawal of dialysis, the clinician may
be challenged to distinguish uremic encephalopathy from opioid
neurotoxicity. Both can cause sedation, hallucinations, and
myoclonus. If respiratory depression is also present, it is advisable
to stop the opioid until the respiratory depression subsides. If
the patient’s respiratory rate is not compromised, the opioid can
usually be continued, and a benzodiazepine such as lorazepam
is added to control the myoclonus. Occasionally, a lorazepam
continuous intravenous infusion at 1 or 2 mg/h is necessary to
control the myoclonus.
Although nonsteroidal anti-inflammatory drugs are recommended for use in step 1 on the WHO analgesic ladder, the
use of these drugs in patients with CKD is contraindicated
because of their nephrotoxicity and in dialysis patients because
of the increased risk of upper gastrointestinal bleeding.
The Mid-Atlantic Renal Coalition and the Coalition for
Supportive Care of Kidney Patients assembled a panel of
international experts on pain management in CKD and developed an evidence-based algorithm for treating pain in dialysis
patients that is accessible online (11).
Other symptom management
Because of their comorbid illnesses, patients undergoing dialysis
are among the most symptomatic of any population with chronic
disease. In one study (13) of 162 patients undergoing dialysis
from three different dialysis units, the median number of symptoms reported by patients was 9.0. Pain, dyspnea, dry skin, and
fatigue were each reported by .50% of the patients. Of the 30
different symptoms reported by the patients, the 6 most bothersome (starting with the most severe first) were as follows: chest
pain, bone or joint pain, difficulty becoming sexually aroused,
trouble falling asleep, muscle cramps, and itching.
Pruritus, or itching, is one of the most frustrating symptoms
experienced by CKD and ESRD patients. Secondary hyperparathyroidism, increased calcium–phosphate deposition in
the skin, dry skin, inadequate dialysis, anemia, iron deficiency,
chronic inflammation, imbalance in endogenous opioids,
American Society of Nephrology
neuropathic processes, and low-grade hypersensitivity to
products used in the dialysis procedure have all been identified
as possible contributory factors. In addition to careful management of all these factors, the following interventions have
been tried for pruritus with some success: emollient skin
creams, phototherapy with ultraviolet B light three times
weekly, intravenous lidocaine during dialysis for refractory
itching, gabapentin, naltrexone, and thalidomide (14).
ADVANCE CARE PLANNING
Advance care planning is a process of communication among
patients, families, health care providers, and other important
individuals about the patient’s preferred decision-maker and
appropriate future medical care if and when a patient is unable
to make his or her own decisions. Advance care planning has
been recommended as a central tenet of CKD, ESRD, and cancer
patient care (15). It is especially appropriate because of the lifelimiting nature of these diseases. The “surprise” question—
would I be surprised if this patient died in the next year?—has
been validated in both the ESRD and cancer patient populations
as a reliable trigger to identify patients who are at increased risk
of death within 1 year and for whom palliative care consultation
including advance care planning is appropriate (16,17).
Researchers have developed an evidence-based robust integrated prognostic model with a C-statistic of 0.8 to estimate
dialysis patients’ 6- and 12-month survival (18), which is available online at www.touchcalc.com/calculators/sq. The American
Society of Nephrology and Renal Physicians Association have
recommended that advance care planning for CKD and ESRD
patients including a patient-specific estimate of prognosis and
shared decision-making occur prior to the initiation of dialysis
(5,19). Nephrologists are responsible for advance care planning,
although aspects of it can be delegated to other nephrology personnel (15). Advance care planning is important for kidneycancer patients because it can ensure that patients’ wishes for
end-of-life care are respected, that unwanted interventions are
avoided, and that patients and their families are satisfied with the
care provided (15). Although nephrologists are expected to possess primary palliative care skills, they are encouraged to consult
palliative care physicians for more complex cases (20). Table 2
presents a comprehensive approach to incorporating palliative
care into dialysis facility patient care.
CONCLUSIONS
There is a growing commitment among the leadership in the
nephrology and oncology communities to enhance palliative
care for advanced kidney disease and cancer patients. It is
highly likely that palliative care for these patients will be significantly improved over the next decade, as nephrologists,
oncologists, and palliative care consultants apply the knowledge and skills discussed in this chapter.
Onco-Nephrology Curriculum
3
TAKE HOME POINTS
c Early palliative care intervention is becoming the standard of care for
kidney-cancer patients. It improves patients’ quality of life and respects
their treatment wishes.
c CKD, ESRD, and cancer patients have shortened life expectancy. The 5-
year survival rate for incident dialysis patients is 40%, which is 30% less
than that of incident cancer patients (66%).
c Because they do not have active metabolites excreted by the kidneys,
fentanyl and methadone are the safest drugs to use for severe nociceptive pain in patients with advanced kidney disease.
c Palliative care consultation can help with complex pain and symptom
management and advance care planning, including shared decisionmaking about the goals of care. Collaboration with hospices can help
dialysis units implement a palliative care program and appropriately
refer patients for hospice care at the end of life.
REFERENCES
1. United States Renal Data System. 2013 USRDS Annual Data Report:
Atlas of Chronic Kidney Disease and End-Stage Renal Disease in the
United States, Bethesda, MD, National Institutes of Health, National
Institutes of Diabetes and Digestive and Kidney Diseases, 2013
2. Abdel-Kader K, Unruh ML, Weisbord SD. Symptom burden, depression, and quality of life in chronic and end-stage kidney disease.
Clin J Am Soc Nephrol 4: 1057–1064, 2009
3. Mao JJ, Armstrong K, Bowman MA, Xie SX, Kadakia R, Farrar JT.
Symptom burden among cancer survivors: impact of age and comorbidity. J Am Board Fam Med 20: 434–443, 2007
4. Renal Physicians Association and American Society of Nephrology.
Shared Decision-Making in the Appropriate Initiation of and Withdrawal
from Dialysis, Washington, DC, Renal Physicians Association, 2000
5. Renal Physicians Association. Shared Decision-Making in the Appropriate
Initiation of and Withdrawal from Dialysis, 2nd ed., Rockville, MD: Renal
Physicians Association, 2010
6. Smith TJ, Temin S, Alesi ER, Abernethy AP, Balboni TA, Basch EM,
Ferrell BR, Loscalzo M, Meier DE, Paice JA, Peppercorn JM, Somerfield
M, Stovall E, Von Roenn JH. American Society of Clinical Oncology
provisional clinical opinion: The integration of palliative care into
standard oncology care. J Clin Oncol 30: 880–887, 2012
7. Kimmel PL, Emont SL, Newmann JM, Danko H, Moss AH. ESRD patient
quality of life: Symptoms, spiritual beliefs, psychosocial factors, and
ethnicity. Am J Kidney Dis 42: 713–721, 2003
4
Onco-Nephrology Curriculum
8. Davison SN. Pain in hemodialysis patients: Prevalence, cause, severity,
and management. Am J Kidney Dis 42: 1239–1247, 2003
9. Barakzoy AS, Moss AH. Efficacy of the world health organization analgesic ladder to treat pain in end-stage renal disease. J Am Soc Nephrol
17: 3198–3203, 2006
10. Bailie GR, Mason NA, Bragg-Gresham JL, Gillespie BW, Young EW.
Analgesic prescription patterns among hemodialysis patients in the
DOPPS: Potential for underprescription. Kidney Int 65: 2419–2425,
2004
11. Coalition M-AR. Clinical algorithm and preferred medications to treat pain
in dialysis patients. Available at: http://www.kidneysupportivecare.org/
Files/PainBrochure9-09.aspx. Accessed March 25, 2015
12. Dean M. Opioids in renal failure and dialysis patients. J Pain Symptom
Manage 28: 497–504, 2004
13. Weisbord SD, Fried LF, Arnold RM, Fine MJ, Levenson DJ, Peterson RA,
Switzer GE. Prevalence, severity, and importance of physical and
emotional symptoms in chronic hemodialysis patients. J Am Soc
Nephrol 16: 2487–2494, 2005
14. Murtagh F, Weisbord S. Symptoms in renal disease: Their epidemiology, assessment, and management. In: Supportive Care for the Renal
Patient, edited by Chambers EJ, Germain M, Brown E, 2nd Ed., Oxford,
UK, Oxford University Press, 2010, pp 103–138
15. Holley JL, Davison SN. Advance care planning for patients with advanced CKD: A need to move forward. Clin J Am Soc Nephrol 10: 344–
346, 2015
16. Moss AH, Ganjoo J, Sharma S, Gansor J, Senft S, Weaner B, Dalton C,
MacKay K, Pellegrino B, Anantharaman P, Schmidt R. Utility of the
“surprise” question to identify dialysis patients with high mortality. Clin
J Am Soc Nephrol 3: 1379–1384, 2008
17. Moss AH, Lunney JR, Culp S, Auber M, Kurian S, Rogers J, Dower J,
Abraham J. Prognostic significance of the “surprise” question in cancer
patients. J Palliat Med 13: 837–840, 2010
18. Cohen LM, Ruthazer R, Moss AH, Germain MJ. Predicting six-month
mortality for patients who are on maintenance hemodialysis. Clin J Am
Soc Nephrol 5: 72–79, 2010
19. Williams AW, Dwyer AC, Eddy AA, Fink JC, Jaber BL, Linas SL, Michael
B, O’Hare AM, Schaefer HM, Shaffer RN, Trachtman H, Weiner DE,
Falk AR; American Society of Nephrology Quality, and Patient
Safety Task Force. Critical and honest conversations: The evidence
behind the “Choosing Wisely” campaign recommendations by the
American Society of Nephrology. Clin J Am Soc Nephrol 7: 1664–
1672, 2012
20. Quill TE, Abernethy AP. Generalist plus specialist palliative care:
Creating a more sustainable model. N Engl J Med 368: 1173–1175, 2013
21. Davison SN, Jhangri GS, Johnson JA. Cross-sectional validity of a
modified Edmonton symptom assessment system in dialysis patients: A
simple assessment of symptom burden. Kidney Int 69: 1621–1625,
2006
American Society of Nephrology
REVIEW QUESTIONS
1. Which one of the following is an advantage of the physician
orders for life-sustaining treatment (POLST) form compared with an advance directive for a patient with stage 5
kidney disease, terminal cancer, and loss of decisionmaking capacity?
a. It is legal in all fifty states
b. It is an immediately actionable medical order
c. Checklist format prevents contradictory orders from being
issued
d. It is appropriate for patients in all stages of CKD
Answer: b is correct. The POLST form or variant is endorsed in 22 states at present and being developed in another
23. Contradictory orders could be written between Sections A
and B such that the patient is to receive CPR in Section A and
comfort measures in Section B of the form. The form is only
appropriate for patients who are seriously ill and for whom the
physician would not be surprised if the patient died in the next
year.
Reference
Citko J, Moss AH, Carley M, Tolle SW. The National POLST
Paradigm Initiative, 2nd ed. FAST FACTS AND CONCEPTS.
Available at: http://www.mypcnow.org/#!blank/k2dh9. Accessed
January 28, 2016. See www.polst.org for more information.
2. Which one of the following medications would be the
preferred, recommended medication for a patient with
stage 5 CKD and lung cancer with painful metastases to the
bone who describes his pain as aching and 10/10?
a.
b.
c.
d.
Morphine
Acetaminophen
Codeine
Fentanyl
Answer: d is correct. Fentanyl is appropriate for severe nociceptive pain in patients with advanced kidney failure.
American Society of Nephrology
Fentanyl does not have active metabolites excreted by the kidneys. Morphine and codeine are contraindicated in advanced
kidney failure because of the toxic accumulation of active metabolites in kidney failure. Acetaminophen is not appropriate
for severe pain.
Reference
Dean M. Opioids in renal failure and dialysis patients. J Pain
Symptom Manage 28: 497–504, 2004
3. Which one of the following statements best summarizes
the role of shared decision-making for patients with advanced kidney disease and cancer approaching the need for
dialysis?
a. Shared decision-making is an outmoded concept from the
1980s
b. Shared decision-making fits well with a disease-oriented
approach to CKD patient treatment
c. Shared decision-making for CKD patients defaults to dialysis modality choices
d. Shared decision-making is the recognized preferred model
for medical decision-making
Answer: d is correct. Shared decision-making was introduced in the 1980s as a process to promote informed consent
and decisions that adequately take account of patients’ preferences. It fits well with an individualized, patient-centered
approach to decision-making and not a disease-oriented approach. Shared decision-making for CKD patients encompasses decisions about whether to start or stop dialysis and
not just dialysis modality.
References
Barry MJ, Edgman-Levitan S. Shared decision making: Pinnacle of patient-centered care. N Engl J Med 366: 780–781, 2012
Renal Physicians Association. Shared Decision-Making in the Appropriate Initiation of and Withdrawal from Dialysis, 2nd Ed.,
Rockville, MD, Renal Physicians Association, 2010
Onco-Nephrology Curriculum
5